Вы находитесь на странице: 1из 119

CONSTI2_Article IV_CITIZENSHIP (119 pages)

(1) (Co v. HRET, G.R. Nos. 92191-92, 92202-03, July 30, 1991)
(16) (In re: Yu v. Defensor-Santiago, G.R. No. 83882, January 24, 1989)
(18) (Frivaldo v. COMELEC, G.R. No. 87193, June 23, 1989)
(21) (Republic v. De La Rosa, G.R. No. 104654, 105715, 105735, June 06, 1994)
(26) (Labo, Jr. v. COMELEC, G.R. No. 86564, August 01, 1989)
(33) (Aznar v. COMELEC, G.R. No. 83820, May 25, 1990)
(36) (Mercado v. Manzano, G.R. No. 135083, May 26, 1999)
(44) (Re: Vicente D. Ching, B.M. No. 914 (Resolution), October 01, 1999)
(49) (Bengson III v. HRET, G.R. No. 142840, May 07, 2001)
(52) (Moy Ya Lim Yao v. Commissioner of Immigration, G.R. No. L-21289, October 04, 1971)
(91) (Tecson v. COMELEC, G.R. No. 161434, 161634, 161824, March 03, 2004)
(103) (Republic v. Lim, G.R. No. 153883, January 13, 2004)
(105) (Altarejos v. COMELEC, G.R. No. 163256, November 10, 2004)
(110) (Reyes v. COMELEC, G.R. No. 207264, June 25, 2013)
(116) (Republic v. Li Ching Chung, G.R. No. 197450, March 20, 2013)

(Co v. HRET, G.R. Nos. 92191-92, 92202-03,


July 30, 1991)
EN BANC
[G.R. Nos. 92191-92. July 30, 1991.]
ANTONIO Y. CO, petitioner, vs. ELECTORAL
TRIBUNAL OF THE HOUSE OF REPRESENTATIVES
and JOSE ONG, JR., respondents.
[G.R. Nos. 92202-03. July 30, 1991.]
SIXTO T. BALANQUIT, JR., petitioner,
ELECTORAL TRIBUNAL OF THE HOUSE
REPRESENTATIVES
and
JOSE
ONG,
respondents.

vs.
OF
JR.,

Hechanova & Associates for petitioner Co.


Brillantes, Nachura, Navarro and Arcilla Law
Offices for respondent Ong, Jr.
SYLLABUS
1. CONSTITUTIONAL LAW; ELECTORAL TRIBUNAL
OF THE HOUSE OF REPRESENTATIVES AND HOUSE
OF SENATE; SOLE JUDGES OF ALL CONTESTS
RELATING
TO
ELECTION,
RETURNS
AND
QUALIFICATIONS
OF
THEIR
RESPECTIVE
MEMBERS. The Constitution explicitly provides
that the House of Representatives Electoral
Tribunal (HRET) and the Senate Electoral Tribunal
(SET) shall be the sole judges of all contests
relating to the election, returns, and qualifications
of their respective members (See Article VI,
Section 17, Constitution). The authority conferred
upon the Electoral Tribunal is full, clear and
complete. The use of the word sole emphasizes
the exclusivity of the jurisdiction of these
Tribunals. The Supreme Court in the case of
Lazatin vs. HRET (168 SCRA 391 [1988]) stated
that under the 1987 Constitution, the jurisdiction
of the Electoral Tribunal is original and exclusive.
And that, " . . . so long as the Constitution grants
the HRET the power to be the sole judge of all
contests relating to election, returns and
qualifications of members of the House of
Representatives, any final action taken by the
HRET on a matter within its jurisdiction shall, as a
1|CONSTI2_Article IV_CITIZENSHIP

rule, not be reviewed by this Court . . . the power


granted to the Electoral Tribunal is full, clear and
complete and excludes the exercise of any
authority on the part of this Court that would in
any wise restrict it or curtail it or even affect the
same."
2. ID.; ID.; JUDGMENTS THEREOF AS A RULE
BEYOND JUDICIAL INTERFERENCE; EXCEPTION;
ARBITRARY AND IMPROVIDENT USE OF POWER
RESULTING TO DENIAL OF DUE PROCESS. In
the case of Robles vs. HRET (181 SCRA 780
[1980]) the Supreme Court stated that the
judgments of the Tribunal are beyond judicial
interference save only "in the exercise of this
Court's so-called extraordinary jurisdiction, . . .
upon a determination that the Tribunal's decision
or resolution was rendered without or in excess of
its jurisdiction, or with grave abuse of discretion
or paraphrasing Morrero, upon a clear showing of
such arbitrary and improvident use by the
Tribunal of its power as constitutes a denial of
due process of law, or upon a demonstration of a
very clear unmitigated ERROR, manifestly
constituting such GRAVE ABUSE OF DISCRETION
that there has to be a remedy for such abuse." In
the leading case of Morrero vs. Bocar (66 Phil.
429 [1938]) the Court ruled that the power of the
Electoral
Commission
"is
beyond
judicial
interference except, in any event, upon a clear
showing of such arbitrary and improvident use of
power as will constitute a denial of due process."
The Court does not venture into the perilous area
of trying to correct perceived errors of
independent branches of the Government. It
comes in only when it has to vindicate a denial of
due process or correct an abuse of discretion so
grave or glaring that no less than the Constitution
calls for remedial action.
3. ID.; ID.; ID.; APPLIED IN CASE AT BAR. In the
absence of a showing that the HRET has
committed grave abuse of discretion amounting
to lack of jurisdiction, there is no occasion for the
Court to exercise its corrective power; it will not
decide a matter which by its nature is for the
HRET alone to decide (See Marcos vs. Manglapus,
177 SCRA 668 [1989]). It has no power to look
into what it thinks is apparent error. As
constitutional creations invested with necessary

power, the Electoral Tribunals, although not


powers in the tripartite scheme of the
government, are, in the exercise of their functions
independent organs independent of Congress
and the Supreme Court. The power granted to
HRET by the Constitution is intended to be as
complete and unimpaired as if it had remained
originally in the legislature (Angara vs. Electoral
Commission, 63 Phil. 139 [1936]). In passing
upon petitions, the Court with its traditional and
careful regard for the balance of powers, must
permit this exclusive privilege of the Tribunals to
remain where the Sovereign authority has placed
it (See Veloso vs. Boards of Canvassers of Leyte
and Samar, 39 Phil. 886 [1919]).
4. ID.; SUPREME COURT; EXPANDED JURISDICTION
UNDER 1987 Constitution. The Supreme Court
under the 1987 Constitution, has been given an
expanded jurisdiction, so to speak, to review the
decisions of the other branches and agencies of
the government to determine whether or not they
have acted within bounds of the Constitution (See
Article VIII, Section 1, Constitution). Yet, in the
exercise thereof, the Court is to merely check
whether or not the government branch or agency
has gone beyond the Constitutional limits of its
jurisdiction, not that it erred or has a different
view.
5. ID.; CONSTITUTIONAL PROVISIONS; HOW
CONSTRUED; SPIRIT AND INTENDMENT MUST
PREVAIL. In construing the law, the Courts are
not always to be hedged in by the literal meaning
of its language. The spirit and intendment
thereof, must prevail over the letter, especially
where adherence to the latter would result in
absurdity and injustice (Casela vs. Court of
Appeals, 35 SCRA 279 [1970]). A Constitutional
provision should be construed so as to give it
effective operation and suppress the mischief at
which it is aimed, hence, it is the spirit of the
provision which should prevail over the letter
thereof (Jarrolt vs. Mabberly, 103 U.S. 580). In the
words of the Court in the case of J.M. Tuazon vs.
LTA (31 SCRA 413 [1970]); "To that primordial
intent, all else is subordinated. Our Constitution,
any constitution is not to be construed narrowly
or pedantically, for the prescriptions therein
contained, to paraphrase Justice Holmes, are not
mathematical formulas having their essence in
their form but are organic living institutions, the
significance of which is vital not formal . . . ."
6. ID.; CITIZENSHIP; SECTION 1, PARAGRAPH 3 OF
ARTICLE IV OF 1987 Constitution; CONSTRUED.
Article IV of the Constitution provides: "Section 1.
The following are citizens of the Philippines: . . .
(3) Those born before January 17, 1973, of Filipino
mothers, who elect Philippine citizenship upon
reaching the age of majority; and . . . Section 2.
Natural-born Citizens are those who are citizens
of the Philippines from birth without having to
perform any act to acquire or perfect their
citizenship. Those who elect Philippine citizenship
in accordance with paragraph 3 hereof shall be
deemed natural-born citizens." The Court
interprets Section 1, Paragraph 3 above as
applying not only to those who elect Philippine
2|CONSTI2_Article IV_CITIZENSHIP

citizenship after February 2, 1987 but also to


those who, having been born of Filipino mothers,
elected citizenship before that date. The provision
in Paragraph 3 was intended to correct an unfair
position which discriminates against Filipino
women. To make the provision prospective from
February 3, 1987 is to give a narrow
interpretation
resulting
in
an
inequitable
situation. It must also be retroactive. The
provision in question was enacted to correct the
anomalous situation where one born of a Filipino
father and an alien mother was automatically
granted the status of a natural-born citizen while
one born of a Filipino mother and an alien father
would still have to elect Philippine citizenship. If
one so elected, he was not, under earlier laws,
conferred the status of a natural-born. Under the
1973 Constitution, those born of Filipino fathers
and those born of Filipino mothers with an alien
father were placed in equal footing. They were
both considered as natural-born citizens. Hence,
the bestowment of the status of "natural-born"
cannot be made to depend on the fleeting
accident of time or result in two kinds of citizens
made up of essentially the same similarly
situated members. It is for this reason that the
amendments were enacted, that is, in order to
remedy this accidental anomaly, and, therefore,
treat equally all those born before the 1973
Constitution
and
who
elected
Philippine
citizenship either before or after the effectivity of
that Constitution.
7. ID.; ID.; SECTION 2 OF ARTICLE IV OF THE 1987
Constitution; ELECTION OF CITIZENSHIP; APPLIES
ONLY TO THOSE BORN OF FILIPINO MOTHER AND
ALIEN FATHER BUT NOT TO ONE WHOSE FATHER
HAS BEEN NATURALIZED WHEN MINOR WAS ONLY
NINE (9) YEARS OF AGE. There is no dispute
that respondent's mother was a natural born
Filipina at the time of her marriage. Crucial to this
case is the issue of whether or not the
respondent elected or chose to be a Filipino
citizen. Election becomes material because
Section 2 of Article IV of the Constitution accords
natural born status to children born of Filipino
mothers before January 17, 1973, if they elect
citizenship upon reaching the age of majority. To
expect the respondent to have formally or in
writing elected citizenship when he came of age
is to ask for the unnatural and unnecessary. The
reason is obvious. He was already a citizen. Not
only was his mother a natural born citizen but his
father had been naturalized when the respondent
was only nine (9) years old. He could not have
divined when he came of age that in 1973 and
1987 the Constitution would be amended to
require him to have filed a sworn statement in
1969 electing citizenship in spite of his already
having been a citizen since 1957. In 1969,
election through a sworn statement would have
been an unusual and unnecessary procedure for
one who had been a citizen since he was nine
years old.
8. ID.; ID.; ID.; ID.; CASE OF IN RE: FLORENCIO
MALLARE (59 SCRA 45 [1974]) APPLIES IN CASE
AT BAR. In the case of In Re: Florencio Mallare
(59 SCRA 45 [1974]), the Court held that the

exercise of the right of suffrage and the


participation in election exercises constitute a
positive act of election of Philippine citizenship. In
the exact pronouncement of the Court, we held:
"Esteban's exercise of the right of suffrage when
he came of age, constitutes a positive act of
election of Philippine citizenship." The private
respondent did more than merely exercise his
right of suffrage. He has established his life here
in the Philippines. For those in the peculiar
situation of the respondent who cannot be
expected to have elected citizenship as they were
already citizens, we apply the In Re Mallare rule.
The filing of a sworn statement or formal
declaration is a requirement for those who still
have to elect citizenship. For those already
Filipinos when the time to elect came up, there
are acts of deliberate choice which cannot be less
binding. Entering a profession open only to
Filipinos, serving in public office where citizenship
is a qualification, voting during election time,
running for public office, and other categorical
acts of similar nature are themselves formal
manifestations of choice for these persons.

9. ID.; ID.; AN ATTACK THERETO MAY ONLY BE


DONE THROUGH A DIRECT ACTION. The
petitioners argue that the respondent's father
was not, validly, a naturalized citizen because of
his premature taking of the oath of citizenship.
The Court cannot go into the collateral procedure
of stripping Mr. Ong's father of his citizenship
after his death and at this very late date just so
we can go after the son. The petitioners question
the citizenship of the father through a collateral
approach. This can not be done. In our
jurisprudence, an attack on a person's citizenship
may only be done through a direct action for its
nullity (See Queto vs. Catolico, 31 SCRA 52
[1970]).
10. ID.; ID.; TO DECLARE THE GRANT THEREOF AS
NULL AND VOID VIOLATIVE OF THE DUE PROCESS
CLAUSE WHERE PERSON INVOLVED HAS BEEN
LAID TO REST. To ask the Court to declare that
grant of Philippine citizenship to Jose Ong Chuan
as null and void would run against the principle of
due process. Jose Ong Chuan has already been
laid to rest. How can he be given a fair
opportunity to defend himself. A dead man
cannot speak. To quote the words of the HRET:
"Ong Chuan's lips have long been muted to
perpetuity by his demise and obviously he could
not rise beyond where his mortal remains now lie
to defend himself were this matter to be made a
central issue in this case."
11. ID.; ID.; ARTICLE 17 OF THE CIVIL CODE OF
SPAIN SUB-PARAGRAPH 4 THEREOF IN RELATION
TO SECTION 4 OF THE PHILIPPINE BILL OF 1902,
APPLIED IN CASE AT BAR. Article 17 of the Civil
Code of Spain enumerates those who were
considered Spanish Subjects, viz: "ARTICLE 17.
The following are Spaniards: . . . (4). Those
without such papers, who may have acquired
domicile in any town in the Monarchy." The
domicile of a natural person is the place of his
3|CONSTI2_Article IV_CITIZENSHIP

habitual
residence.
This
domicile,
once
established is considered to continue and will not
be deemed lost until a new one is established
(Article 50, NCC; Article 40, Civil Code of Spain;
Zuellig vs. Republic, 83 Phil. 768 [1949]). Ong Te
became a permanent resident of Laoang, Samar
around 1895. Correspondingly, a certificate of
residence was then issued to him by virtue of his
being a resident of Laoang, Samar. The domicile
that Ong Te established in 1895 continued until
April 11, 1899; it even went beyond the turn of
the 19th century. It is also in this place where Ong
Te set up his business and acquired his real
property. Ong Te falls within the meaning of subparagraph 4 of Article 17 of the Civil Code of
Spain. Although Ong Te made brief visits to China,
he, nevertheless, always returned to the
Philippines. The fact that he died in China, during
one of his visits in said country, was of no
moment. This will not change the fact that he
already had his domicile fixed in the Philippines
and pursuant to the Civil Code of Spain, he had
become a Spanish subject. If Ong Te became a
Spanish subject by virtue of having established
his domicile in a town under the Monarchy of
Spain, necessarily, Ong Te was also an inhabitant
of the Philippines for an inhabitant has been
defined as one who has actual fixed residence in
a place; one who has a domicile in a place
(Bouvier's Law Dictionary, Vol. II). A priori, there
can be no other logical conclusion but to educe
that Ong Te qualified as a Filipino citizen under
the provisions of Section 4 of the Philippine Bill of
1902.
12. ID.; ID.; "RESIDENCE"; MEANING THEREOF
UNDER THE CONSTITUTION. Under the
Constitution, the term "residence" has been
understood as synonymous with domicile not only
under the previous Constitutions but also under
the 1987 Constitution. The term "domicile"
denotes a fixed permanent residence to which
when absent for business or pleasure, one
intends to return (Ong Huan Tin vs. Republic, 19
SCRA 966 [1967]). The absence of a person from
said permanent residence, no matter how long,
notwithstanding, it continues to be the domicile
of that person. In other words, domicile is
characterized by animus revertendi (Ujano vs.
Republic, 17 SCRA 147 [1966]).
13. ID.; ID.; ID.; ESTABLISHMENT THEREOF;
OWNERSHIP OF A HOUSE NOT NECESSARY. The
petitioners' allegation that since the private
respondent owns no property in Laoang, Samar,
he cannot, therefore, be a resident of said place
is misplaced. The properties owned by the Ong
family are in the name of the private
respondent's parents. Upon the demise of his
parents, necessarily, the private respondent,
pursuant to the laws of succession, became the
co-owner thereof (as a co-heir), notwithstanding
the fact that these were still in the names of his
parents. Even assuming that the private
respondent does not own any property in Samar,
the Supreme Court in the case of De los Reyes vs.
Solidum (61 Phil. 893 [1935]) held that it is not
required that a person should have a house in
order to establish his residence and domicile. It is

enough that he should live in the municipality or


in a rented house or in that of a friend or relative.
14. ID.; ID.; ID.; TEMPORARY ABSENCE DOES NOT
NECESSARILY CONNOTE CHANGE THEREOF;
"ANIMUS REVERTENDI" ESTABLISHED IN CASE AT
BAR. It has also been settled that absence from
residence to pursue studies or practice a
profession or registration as a voter other than in
the place where one is elected, does not
constitute loss of residence (Faypon vs. Quirino,
96 Phil. 294 [1954]). The private respondent
stayed in Manila for the purpose of finishing his
studies and later to practice his profession. There
was no intention to abandon the residence in
Laoang, Samar. On the contrary, the periodical
journeys made to his home province reveal that
he always had the animus revertendi.
15.
ID.;
ID.;
PROSPECTIVE
JUDICIAL
RECOMMENDATION; MORE HUMANE AND LESS
TECHNICAL
APPROACH
TO
CITIZENSHIP
PROBLEMS. Our citizens no doubt constitute
the country's greatest wealth. Citizenship is a
special privilege which one must forever cherish.
However, in order to truly revere this treasure of
citizenship, we do not, on the basis of too harsh
an interpretation, have to unreasonably deny it to
those who qualify to share in its richness. Under
the overly strict jurisprudence surrounding our
antiquated naturalization laws only the very
affluent backed by influential patrons, who were
willing to suffer the indignities of a lengthy,
sometimes humiliating, and often corrupt process
of clearances by minor bureaucrats and whose
lawyers knew how to overcome so many
technical traps of the judicial process were able
to acquire citizenship. It is time for the
naturalization law to be revised to enable a more
positive, affirmative, and meaningful examination
of an applicant's suitability to be a Filipino. A
more humane, more indubitable and less
technical approach to citizenship problems is
essential.
16.
ID.;
HOUSE
OF
REPRESENTATIVE;
CANDIDATES; PROPERTY OWNERSHIP; NOT A
QUALIFICATION. To require the private
respondent to own property in order to be eligible
to run for Congress would be tantamount to a
property qualification. The Constitution only
requires that the candidate meet the age,
citizenship, voting and residence requirements.
Nowhere is it required by the Constitution that
the candidate should also own property in order
to be qualified to run (see Maquera vs. Borra, 122
Phil. 412 [1965]).
17. REMEDIAL LAW; BEST EVIDENCE RULE;
EXCEPTION;
ORIGINAL
HAS
BEEN
LOST;
REQUIREMENTS THEREOF TO BE ADMISSIBLE;
PROPERLY LAID IN CASE AT BAR. The
petitioners' sole ground in disputing that
respondent was a natural-born Filipino is that the
documents presented to prove it were not in
compliance with the best evidence rule. The
petitioners allege that the private respondent
failed to present the original of the documentary
evidence, testimonial evidence and of the
4|CONSTI2_Article IV_CITIZENSHIP

transcript of the proceedings of the body upon


which the resolution of the 1971 Constitutional
Convention was predicated. On the contrary, the
documents presented by the private respondent
fall under the exceptions to the best evidence
rule. It was established in the proceedings before
the HRET that the originals of the Committee
Report No. 12, the minutes of the plenary session
of the 1971 Constitutional Convention held on
November 28, 1972 cannot be found. This was
affirmed by Atty. Ricafrente, Assistant Secretary
of the 1971 Constitutional Convention; by Atty.
Nolledo, Delegate to the 1971 Constitutional
Convention; and by Atty. Antonio Santos, Chief
Librarian of the U.P. Law Center, in their
respective testimonies given before the HRET to
the effect that there is no governmental agency
which is the official custodian of the records of
the
1971
Constitutional
Convention.
The
execution of the originals was established by Atty.
Ricafrente, who as the Assistant Secretary of the
1971 Constitutional Convention was the proper
party to testify to such execution. The inability to
produce the originals before the HRET was also
testified to as aforestated by Atty. Ricafrente,
Atty. Nolledo, and Atty. Santos. In proving the
inability to produce, the law does not require the
degree of proof to be of sufficient certainty; it is
enough that it be shown that after a bona fide
diligent search, the same cannot be found (see
Government of P.I. vs. Martinez, 44 Phil. 817
[1918]). Since the execution of the document and
the inability to produce were adequately
established, the contents of the questioned
documents can be proven by a copy thereof or by
the recollection of witnesses.
PADILLA, J., dissenting:
1. CONSTITUTIONAL LAW; SUPREME COURT;
JURISDICTION THEREOF; EXPANDED UNDER THE
1987
Constitution;
DECISION
OF
HOUSE
ELECTORAL TRIBUNAL SUBJECT TO JUDICIAL
REVIEW. I believe that, contrary to the
respondents' contentions, the Court has the
jurisdiction and competence to review the
questioned decision of the House Electoral
Tribunal and to decide the present controversy.
Article VIII, Section 1 of the 1987 Constitution
provides that: "Judicial power includes the duty of
the courts of justice to settle actual controversies
involving rights which are legally demandable
and enforceable, and to determine whether or not
there has been a grave abuse of discretion
amounting to lack or excess of jurisdiction on the
part of any branch or instrumentality of the
government." The Constitution, it is true,
constitutes the tribunal as the sole judge of all
contests relating to the election, returns, and
qualifications of Members of the House of
Representatives. But as early as 1938, it was held
in Morrero vs. Bocar (66 Phil. 429), construing
Section 4, Article VI of the 1935 Constitution
which provided that " . . . The Electoral
Commission shall be the sole judge of all contests
relating to the election, returns and qualifications
of the Members of the National Assembly." that:
"The judgment rendered by the (electoral)
commission in the exercise of such and
acknowledged
power
is
beyond
judicial

interference, except, in any event, 'upon a clear


showing of such arbitrary and improvident use of
the power as will constitute a denial of due
process of law' (Barry vs. US ex rel. Cunningham,
279 US 597; 73 Law. ed., 867; Angara vs.
Electoral Commission, 35 Off. Gaz., 23)." And
then under the afore-quoted provisions of Article
VIII, Section 1 of the 1987 Constitution, this Court
is duty-bound to determine whether or not, in an
actual controversy, there has been a grave abuse
of discretion amounting to lack or excess of
jurisdiction on the part of any branch or
instrumentality of the government.

2. ID.; ID.; ID.; ID.; APPLIED IN CASE AT BAR.


The present controversy, involves more than
perceived irregularities in the conduct of a
congressional election or a disputed appreciation
of ballots, in which cases, it may be contended
with great legal force and persuasion that the
decision of the electoral tribunal should be final
and conclusive, for it is, by constitutional
directive, made the sole judge of contests
relating
to
such
matters.
The
present
controversy, however, involves no less than a
determination of whether the qualifications for
membership in the House of Representatives, as
prescribed by the Constitution, have been met.
Indeed, this Court would be unforgivably remiss
in the performance of its duties, as mandated by
the Constitution, were it to allow a person, not a
natural-born Filipino citizen, to continue to sit as a
Member of the House of Representatives, solely
because the House Electoral Tribunal has
declared him to be so. In such a case, the tribunal
would have acted with grave abuse of discretion
amounting to lack or excess of jurisdiction as to
require the exercise by this Court of its power of
judicial review. Besides, the citizenship and
residence qualifications of private respondent for
the office of Member of the House of
Representatives, are here controverted by
petitioners who, at the same time, claim that
they are entitled to the office illegally held by
private respondent. From this additional direction,
where one asserts and earnestly perceived right
that in turn is vigorously resisted by another,
there is clearly a justiciable controversy proper
for this Court to consider and decide.
3. ID.; ID.; ID.; EXERCISE OF JUDICIAL REVIEW
NOT
VIOLATIVE
OF
THE
PRINCIPLE
OF
SEPARATION OF POWERS. The Court, in
reviewing the decision of the tribunal, does not
assert supremacy over it in contravention of the
time-honored
principle
of
constitutional
separation of powers. The Court in this instance
simply performs a function entrusted and
assigned to it by the Constitution of interpreting,
in a justiciable controversy, the pertinent
provisions of the Constitution with finality. "It is
the role of the Judiciary to refine and, when
necessary,
correct
constitutional
(and/or
statutory) interpretation, in the context of the
interactions of the three branches of the
government, almost always in situations where
some agency of the State has engaged in action
5|CONSTI2_Article IV_CITIZENSHIP

that stems ultimately from some legitimate area


of governmental power (the Supreme Court in
Modern Role, C.B. Sevisher, 1958, p. 36)."
Moreover, it is decidedly a matter of great public
interest and concern to determine whether or not
private respondent is qualified to hold so
important and high a public office which is
specifically reserved by the Constitution only to
natural-born Filipino citizens.
4. ID.; CITIZENSHIP; NATURAL-BORN; REQUISITE;
NOT COMPLIED WITH IN CASE AT BAR. The
records show that private respondent was born
on 19 June 1948 to the spouses Jose Ong Chuan,
a Chinese citizen, and Agrifina E. Lao, a naturalborn Filipino citizen, in Laoang, Northern Samar.
In other words, at birth, private respondent was a
Chinese citizen (not a natural-born Filipino citizen)
because his father was then a Chinese citizen
(not a naturalized Filipino citizen). Under the 1935
Constitution which was enforced at the time of
private respondent's birth on 19 June 1948, only
those whose fathers were citizens of the
Philippines were considered Filipino citizens.
Those whose mothers were citizens of the
Philippines had to elect Philippine citizenship
upon reaching the age of majority, in order to be
considered Filipino citizens. Following the basic
definition in the 1987 Constitution of a naturalborn citizen, in relation to the 1935 Constitution,
private respondent is not a natural-born Filipino
citizen, having been born a Chinese citizen by
virtue of the Chinese citizenship of his father at
the time of his birth, although from birth, private
respondent had the right to elect Philippine
citizenship, the citizenship of his mother, but only
upon his reaching the age of majority.
5. ID.; ID.; ID.; SECTION 15 OF THE REVISED
NATURALIZATION LAW (C.A. 473); DID NOT
CONFER STATUS OF NATURAL-BORN IN CASE AT
BAR. While under Section 15 of the Revised
Naturalization Law (C.A. 473) minor children of a
naturalized citizen (father), who were born in the
Philippines prior to the naturalization of the
parent automatically become Filipino citizens, this
does not alter the fact that private respondent
was not born to a Filipino father, and the
operation of Section 15 of CA 473 did not confer
upon him the status of a natural-born citizen
merely because he did not have to perform any
act to acquire or perfect his status as a Filipino
citizen.
6. ID.; ID.; NATURALIZATION; NATURE THEREOF;
PRIVILEGE NOT A RIGHT. "Naturalization is not
a right, but a privilege of the most discriminating
as well as delicate and exacting nature, affecting
public interest of the highest order, and which
may be enjoyed only under the precise conditions
prescribed by law therefor."
7. ID.; ID.; ID.; PETITION; GRANT THEREOF;
APPEALABLE; OATH TAKEN BEFORE EXPIRATION
OF THE PERIOD OF APPEAL; IMPROPER. It is
settled that an order granting a petition to take
the requisite oath of allegiance of one who has
previously obtained a decision favorable to his
application for naturalization, is appealable. It is,

therefore, improper and illegal to authorize the


taking of said oath upon the issuance of said
order and before the expiration of the
reglementary period to perfect any appeal from
said order. In Cua Sun Ke vs. Republic (159 SCRA
477), this Court held that: "Administration of the
oath of allegiance on the same day as issuance of
order granting citizenship is irregular and makes
the proceedings so taken null and void (Republic
vs. Guy, 115 SCRA 244 [1982]; citing the case of
Ong So vs. Republic of the Philippines, 121 Phil.
1381)."
8.
ID.; ID.; NATURAL-BORN; DEFINED AND
INTERPRETED UNDER THE 1987 Constitution.
Article IV, Section 2 of the 1987 Constitution
defines natural-born (Filipino) citizens as:
"Natural-born citizens are those who are citizens
of the Philippines from birth without having to
perform any act to acquire or perfect their
Philippine citizenship. Those who elect Philippine
citizenship in accordance with paragraph (3),
Section 1 hereof shall be deemed natural-born
citizens." Article IV, Section 1, paragraph (3) of
the 1987 Constitution provides that: "Section 1.
The following are citizens of the Philippines: . . .
(3) Those born before January 17, 1973, of Filipino
mothers, who elect Philippine citizenship upon
reaching the age of majority." It would appear
then that the intent of the framers of the 1987
Constitution in defining a natural-born Filipino
citizen was to equalize the position of Filipino
fathers and Filipino mothers as to their children
becoming natural-born Filipino citizens. In other
words, after 17 January 1973, effectivity date of
the 1973 Constitution, all those born of Filipino
fathers (with alien spouse) or Filipino mothers
(with alien spouse) are natural-born Filipino
citizens. But those born to Filipino mothers prior
to 17 January 1973 must still elect Philippine
citizenship upon reaching the age of majority, in
order to be deemed natural-born Filipino citizens.
The election, which is related to the attainment of
the age of majority, may be made before or after
17 January 1973. This interpretation appears to
be in consonance with the fundamental purpose
of the Constitution which is to protect and
enhance the people's individual interests, and to
foster equality among them.
9. ID.; ID.; ELECTION THEREOF; MUST BE MADE
EXPRESSLY
AS
PROVIDED
FOR
UNDER
COMMONWEALTH ACT NO. 625. It is settled
doctrine in this jurisdiction that election of
Philippine citizenship must be made in
accordance with Commonwealth Act 625,
Sections 1 and 2 of the Act mandate that the
option to elect Philippine citizenship must be
effected expressly, not impliedly.
10. ID.; ID.; ID.; CASE OF IN RE: FLORENCIO
MALLARE (ADMINISTRATIVE CASE NO. 533,
SEPTEMBER 12, 1974, [59 SCRA 45]) NOT
APPLICABLE IN CASE AT BAR. The respondent
tribunal cites In re: Florencio Mallare which held
that Esteban Mallare's exercise of the right of
suffrage when he came of age, constituted a
positive act of election of Philippine citizenship.
Mallare, cited by respondent tribunal as authority
6|CONSTI2_Article IV_CITIZENSHIP

for the doctrine of implied election of Philippine


citizenship, is not applicable to the case at bar.
The respondent tribunal failed to consider that
Esteban Mallare reached the age of majority in
1924, or seventeen (17) years before CA 625 was
approved and, more importantly, eleven (11)
years before the 1935 Constitution (which
granted the right of election) took effect.
11.
ID.; ID.; ID.; REQUISITE PROVIDED FOR
UNDER COMMONWEALTH ACT NO. 625 NOT
COMPLIED WITH IN CASE AT BAR. The
respondent tribunal erred in ruling that by
operation of CA 473, the Revised Naturalization
Law,
providing
for
private
respondent's
acquisition of Filipino citizenship by reason of the
naturalization of his father, the law itself had
already elected Philippine citizenship for him. For,
assuming arguendo that the naturalization of
private respondent's father was valid, and that
there was no further need for private respondent
to elect Philippine citizenship (as he had
automatically become a Filipino citizen) yet, this
did not mean that the operation of the Revised
Naturalization Law amounted to an election by
him of Philippine citizenship as contemplated by
the Constitution. Besides, election of Philippine
citizenship derived from one's Filipino mother, is
made upon reaching the age of majority, not
during one's minority. There is no doubt in my
mind, therefore, that private respondent did not
elect Philippine citizenship upon reaching the age
of majority in 1969 or within a reasonable time
thereafter as required by CA 625. Consequently,
he cannot be deemed a natural-born Filipino
citizen under Sections 2 and 1 (3), Article IV of
the 1987 Constitution.

12. ID.; ELECTION PROTEST; QUESTIONING


ELIGIBILITY OF A CANDIDATE-ELECT; IN EFFECT A
QUO WARRANTO PROCEEDING; INELIGIBILITY OF
CANDIDATE-ELECT RESULTS IN NO-CHOICE.
Neither of the petitioners may take the place of
private
respondent
in
the
House
of
Representatives representing the second district
of Northern Samar. The ruling of this Court in
Ramon L. Labo, Jr. vs. The Commission on
Elections (COMELEC) EN BANC and Luis L.
Lardizabal (176 SCRA 1), is controlling. There we
held that Luis L. Lardizabal, who filed the quo
warranto petition, could not replace Ramon L.
Labo, Jr. as mayor of Baguio City for the simple
reason that as he obtained only the second
highest number of votes in the election, he was
obviously not the choice of the people of Baguio
City for mayor of that City. A petition alleging that
the candidate-elect is not qualified for the office
is, in effect, a quo warranto proceeding even if it
is labelled an election protest. It is a proceeding
to unseat the ineligible person from office but not
necessarily to install the protestant in his place.
The general rule is that the fact that a plurality or
a majority of the votes are cast for an ineligible
candidate in an election does not entitle the
candidate receiving the next highest number of
votes to be declared elected. In such a case, the

electors have failed to make a choice and the


election is a nullity.
13. ID.; ID.; PHILIPPINE BILL OF 1902;
REQUIREMENTS
PROVIDED
THEREIN;
NOT
COMPLIED WITH IN CASE AT BAR. The "test,"
following the premises of the 1971 Constitutional
Convention, is whether or not Ong Te, private
respondent's and Emil L. Ong's grandfather was
"an inhabitant of the Philippines who continued to
reside therein and was a Spanish subject on April
11, 1899." If he met these requirements of the
Philippine Bill of 1902, then, Ong Te was a Filipino
citizen; otherwise, he was not a Filipino citizen.
Petitioners (protestants) submitted and offered in
evidence before the House Electoral Tribunal
exhibits W, X, Y, Z, AA, BB, CC, DD and EE which
are copies of entries in the "Registro de Chinos"
from years 1896 to 1897 which show that Ong Te
was not listed as an inhabitant of Samar where he
is claimed to have been a resident. Petitioners
(protestants) also submitted and offered in
evidence before the House Electoral Tribunal
Exhibit V, a certification of the Chief of the
Archives Division, Records and Management and
Archives Office, stating that the name of Ong Te
does not appear in the "Registro de Chinos" for
the province of Samar for 1895. These exhibits
prove or at least, as petitioners validly argue,
tend to prove that Ong Te was NOT a resident of
Samar close to 11 April 1899 and, therefore,
could not continue residing in Samar, Philippines
after 11 April 1899, contrary to private
respondent's pretense. In the face of these proofs
or evidence, private respondent FAILED TO
PRESENT ANY REBUTTAL OR COUNTERVAILING
EVIDENCE.
14. ID.; ID.; RES JUDICATA; NOT APPLICABLE.
The decision of the 1971 Constitutional
Convention in the case of Emil L. Ong was a
decision of a political body, not a court of law.
And, even if we have to take such a decision as a
decision of a quasi-judicial body (i.e., a political
body exercising quasi-judicial functions), said
decision in the Emil L. Ong case can not have the
category or character of res judicata in the
present judicial controversy, because between
the two (2) cases, there is no identity of parties
(one involves Emil L. Ong, while the other
involves
private
respondent)
and,
more
importantly, there is no, identity of causes of
action because the first involves the 1935
Constitution while the second involves the 1987
Constitution. As held in Lee vs. Commissioners on
Immigration (G.R. No. L-23446, 20 December
1971, 42 SCRA 561): " . . . Everytime the
citizenship of a person is material or
indispensable in a judicial or administrative case,
whatever
the
corresponding
court
or
administrative authority decides therein as to
such citizenship is generally not considered as res
judicata, hence it has to be threshed out again
and again as the occasion may demand."
15. ID; SUPREMACY OF THE CONSTITUTION; MUST
BE ENFORCED. It is regrettable that one (as
private respondent) who unquestionably obtained
the highest number of votes for the elective
7|CONSTI2_Article IV_CITIZENSHIP

position of Representative (Congressman) to the


House of Representatives for the second district
of Northern Samar, would have to cease in office
by virtue of this Court's decision, if the full
membership of the Court had participated in this
case, with the result that the legislative district
would cease to have, in the interim, a
representative in the House of Representatives.
But the fundamental consideration in case of this
nature is the Constitution and only the
Constitution. It has to be assumed, therefore, that
when the electorate in the second legislative
district of Northern Samar cast the majority of
their votes for private respondent, they seemed
and believed that he was fully eligible and
qualified for the office because he is a naturalborn Filipino citizen. That erroneous assumption
and belief can not prevail over, but must yield to
the majesty of the Constitution.
SARMIENTO, J., concurring:
1. CONSTITUTIONAL LAW; ELECTORAL TRIBUNAL
OF THE HOUSE OF REPRESENTATIVES; AS SOLE
JUDGE OF ALL CONTEST RELATING TO MEMBERS
THEREOF; ISSUE OF CITIZENSHIP INCLUDED;
BEYOND JUDICIAL INTERVENTION. The question
of citizenship is a question of fact, and as a rule,
the Supreme Court leaves facts to the tribunal
that determined them. I am quite agreed that the
Electoral
Tribunal
of
the
House
of
Representatives, as the "sole judge" of all
contests relating to the membership in the
House, as follows: "Sec. 17. The Senate and the
House of Representatives shall each have an
Electoral Tribunal which shall be the sole judge of
all contests relating to the election, returns, and
qualifications of their respective Members. Each
Electoral Tribunal shall be composed of nine
Members, three of whom shall be Justices of the
Supreme Court to be designated by the Chief
Justice, and the remaining six shall be Members
of the Senate or the House of Representatives, as
the case may be, who shall be chosen on the
basis of proportional representation from the
political parties and the parties or organizations
registered
under
the
party-list
system
represented therein. The senior Justice in the
Electoral Tribunal shall be its Chairman." is the
best judge of facts and this Court can not
substitute its judgment because it thinks it knows
better.
2. ID.; SUPREME COURT; EXPANDED JURISDICTION
THEREOF; REVIEW OF FACTS NOT INCLUDED. In
the case of Aratuc vs. Commission on Elections
(88 SCRA 251), it was held that this Court can not
review the errors of the Commission on Elections
(then the "sole judge" of all election contests)
in the sense of reviewing facts and unearthing
mistakes and that this Court's jurisdiction is to
see simply whether or not it is guilty of a grave
abuse of discretion. It is true that the new
Constitution has conferred expanded powers on
the Court, but as the Charter states, our authority
is "to determine whether or not there has been a
grave abuse of discretion amounting to lack or
excess of jurisdiction on the part of any branch or
instrumentality of the government." It is not to
review facts.

3. ID.; ID.; ID.; "GRAVE ABUSE OF DISCRETION"


DEFINED. "Grave abuse of discretion" has been
defined as whimsical exercise of power
amounting to excess of jurisdiction, or otherwise,
to denial of due process of law.
DECISION
GUTIERREZ, JR., J p:
The petitioners come to this Court asking for the
setting aside and reversal of a decision of the
House of Representatives Electoral Tribunal
(HRET).
The HRET declared that respondent Jose Ong, Jr.
is a natural born Filipino citizen and a resident of
Laoang, Northern Samar for voting purposes. The
sole issue before us is whether or not, in making
that determination, the HRET acted with grave
abuse of discretion.
On May 11, 1987, the congressional election for
the second district of Northern Samar was held.
Among the candidates who vied for the position
of representative in the second legislative district
of Northern Samar are the petitioners, Sixto
Balinquit and Antonio Co and the private
respondent, Jose Ong, Jr.
Respondent Ong was proclaimed the duly elected
representative of the second district of Northern
Samar.
The petitioners filed election protests against the
private respondent premised on the following
grounds:
1) Jose Ong, Jr. is not a natural born citizen of the
Philippines; and
2) Jose Ong, Jr. is not a resident of the second
district of Northern Samar.
The HRET, in its decision dated November 6,
1989, found for the private respondent.
A motion for reconsideration was filed by the
petitioners on November 12, 1989. This was,
however, denied by the HRET, in its resolution
dated February 22, 1989.
Hence, these petitions for certiorari.
We treat the comments as answers and decide
the issues raised in the petitions.
ON THE ISSUE OF JURISDICTION
The first question which arises refers to our
jurisdiction.
The Constitution explicitly provides that the
House of Representatives Electoral Tribunal
(HRET) and the Senate Electoral Tribunal (SET)
shall be the sole judges of all contests relating to
the election, returns, and qualifications of their
8|CONSTI2_Article IV_CITIZENSHIP

respective members. (See Article VI, Section 17,


Constitution). prLL
The authority conferred upon the Electoral
Tribunal is full, clear and complete. The use of the
word sole emphasizes the exclusivity of the
jurisdiction of these Tribunals.
The Supreme Court in the case of Lazatin v. HRET
(168 SCRA 391 [1988]) stated that under the
1987 Constitution, the jurisdiction of the Electoral
Tribunal is original and exclusive, viz:
"The use of the word 'sole' emphasizes the
exclusive character of the jurisdiction conferred
(Angara v. Electoral Commission, supra at p.
162). The exercise of power by the Electoral
Commission under the 1935 Constitution has
been described as 'intended to be as complete
and unimpaired as if it had originally remained in
the legislature.' (id., at p. 175) Earlier this grant
of power to the legislature was characterized by
Justice Malcolm as 'full, clear and complete'.
(Veloso v. Board of Canvassers of Leyte and
Samar, 39 Phil. 886 [1919]) Under the amended
1935 Constitution, the power was unqualifiedly
reposed upon the Electoral Tribunal and it
remained as full, clear and complete as that
previously granted the Legislature and the
Electoral Commission, (Lachica v. Yap, 25 SCRA
140 [1968] The same may be said with regard to
the jurisdiction of the Electoral Tribunal under the
1987 Constitution." (p. 401).

The Court continued further, ". . . so long as the


Constitution grants the HRET the power to be the
sole judge of all contests relating to election,
returns and qualifications of members of the
House of Representatives, any final action taken
by the HRET on a matter within its jurisdiction
shall, as a rule, not be reviewed by this Court . . .
the power granted to the Electoral Tribunal is full,
clear and complete and excludes the exercise of
any authority on the part of this Court that would
in any wise restrict it or curtail it or even affect
the same." (pp. 403-404)
When may the Court inquire into acts of the
Electoral Tribunals under our constitutional grants
of power?
In the later case of Robles v. HRET (181 SCRA 780
[1990]) the Supreme Court stated that the
judgments of the Tribunal are beyond judicial
interference save only "in the exercise of this
Court's so-called extraordinary jurisdiction, . . .
upon a determination that the Tribunal's decision
or resolution was rendered without or in excess of
its jurisdiction, or with grave abuse of discretion
or paraphrasing Morrero, upon a clear showing of
such arbitrary and improvident use by the
Tribunal of its power as constitutes a denial of
due process of law, or upon a demonstration of a
very clear unmitigated ERROR, manifestly
constituting such GRAVE ABUSE OF DISCRETION
that there has to be a remedy for such abuse."
(at pp. 785-786)

In the leading case of Morrero v. Bocar (66 Phil.


429 [1938]) the Court ruled that the power of the
Electoral
Commission
"is
beyond
judicial
interference except, in any event, upon a clear
showing of such arbitrary and improvident use of
power as will constitute a denial of due process."
The Court does not venture into the perilous area
of trying to correct perceived errors of
independent branches of the Government. It
comes in only when it has to vindicate a denial of
due process or correct an abuse of discretion so
grave or glaring that no less than the Constitution
calls for remedial action. LLjur

In the case at bar, the Court finds no improvident


use of power, no denial of due process on the
part of the HRET which will necessitate the
exercise of the power of judicial review by the
Supreme Court.

The Supreme Court under the 1987 Constitution,


has been given an expanded jurisdiction, so to
speak, to review the decisions of the other
branches and agencies of the government to
determine whether or not they have acted within
the bounds of the Constitution. (See Article VIII,
Section 1, Constitution)

As a resident of Laoang, Ong Te was able to


obtain a certificate of residence from the then
Spanish colonial administration.

Yet, in the exercise thereof, the Court is to merely


check whether or not the governmental branch or
agency has gone beyond the Constitutional limits
of its jurisdiction, not that it erred or has a
different view. In the absence of a showing that
the HRET has committed grave abuse of
discretion amounting to lack of jurisdiction, there
is no occasion for the Court to exercise its
corrective power; it will not decide a matter which
by its nature is for the HRET alone to decide. (See
Marcos v. Manglapus, 177 SCRA 668 [1989]) It
has no power to look into what it thinks is
apparent error.

Jose Ong Chuan spent his childhood in the


province of Samar. In Laoang, he was able to
establish an enduring relationship with his
neighbors, resulting in his easy assimilation into
the community.

As
constitutional
creations
invested
with
necessary power, the Electoral Tribunals,
although not powers in the tripartite scheme of
the government, are, in the exercise of their
functions independent organs independent of
Congress and the Supreme Court. The power
granted to HRET by the Constitution is intended
to be as complete and unimpaired as if it had
remained originally in the legislature. (Angara v.
Electoral Commission, 63 Phil. 139 [1936])
In passing upon petitions, the Court with its
traditional and careful regard for the balance of
powers, must permit this exclusive privilege of
the Tribunals to remain where the Sovereign
authority has place it. (See Veloso v. Boards of
Canvassers of Leyte and Samar, 39 Phil. 886
[1919])
It has been argued that under Article VI, Section
17 of the present Constitution, the situation may
exist as it exists today where there is an
unhealthy one-sided political composition of the
two Electoral Tribunals. There is nothing in the
Constitution, however, that makes the HRET
because of its composition any less independent
from the Court or its constitutional functions any
less exclusive. The degree of judicial intervention
should not be made to depend on how many
legislative members of the HRET belong to this
party or that party. The test remains the same
manifest grave abuse of discretion.
9|CONSTI2_Article IV_CITIZENSHIP

ON THE ISSUE OF CITIZENSHIP


The records show that in the year 1895, the
private respondent's grandfather, Ong Te, arrived
in the Philippines from China. Ong Te established
his residence in the municipality of Laoang,
Samar on land which he bought from the fruits of
hard work.

The father of the private respondent, Jose Ong


Chuan was born in China in 1905. He was brought
by Ong Te to Samar in the year 1915.

As Jose Ong Chuan grew older in the rural and


seaside community of Laoang, he absorbed
Filipino cultural values and practices. He was
baptized into Christianity. As the years passed,
Jose Ong Chuan met a natural born-Filipina,
Agripina Lao. The two fell in love and, thereafter,
got married in 1932 according to Catholic faith
and practice.
The couple bore eight children, one of whom is
the private respondent who was born in 1948.
The private respondent's father never emigrated
from this country. He decided to put up a
hardware store and shared and survived the
vicissitudes of life in Samar.
The business prospered. Expansion became
inevitable. As a result, a branch was set-up in
Binondo, Manila. In the meantime, the father of
the private respondent, unsure of his legal status
and in an unequivocal affirmation of where he
cast his life and family, filed with the Court of
First Instance of Samar of application for
naturalization on February 15, 1954. LibLex
On April 28, 1955, the CFI of Samar, after trial,
declared Jose Ong Chuan a Filipino citizen.
On May 15, 1957, the Court of First Instance of
Samar issued an order declaring the decision of
April 28, 1955 as final and executory and that
Jose Ong Chuan may already take his Oath of
Allegiance.
Pursuant to said order, Jose Ong Chuan took his
Oath of Allegiance; correspondingly, a certificate
of naturalization was issued to him.

At the time Jose Ong Chuan took his oath, the


private respondent then a minor of nine years
was finishing his elementary education in the
province of Samar. There is nothing in the records
to differentiate him from other Filipinos insofar as
the customs and practices of the local populace
were concerned.
Fortunes changed. The house of the family of the
private respondent in Laoang, Samar was burned
to the ground.
Undaunted by the catastrophe, the private
respondent's family constructed another one in
place of their ruined house. Again, there is no
showing other than that Laoang was their abode
and home.
After completing his elementary education, the
private respondent, in search for better
education, went to Manila in order to acquire his
secondary and college education.
In the meantime, another misfortune was
suffered by the family in 1975 when a fire gutted
their second house in Laoang, Samar. The
respondent's family constructed still another
house, this time a 16-door apartment building,
two doors of which were reserved for the family.
The private respondent graduated from college,
and thereafter took and passed the CPA Board
Examinations.
Since employment opportunities were better in
Manila, the respondent looked for work here. He
found a job in the Central Bank of the Philippines
as an examiner. Later, however, he worked in the
hardware business of his family in Manila. In
1971, his elder brother, Emil, was elected as a
delegate to the 1971 Constitutional Convention.
His status as a natural born citizen was
challenged. Parenthetically, the Convention which
in drafting the Constitution removed the unequal
treatment given to derived citizenship on the
basis of the mother's citizenship formally and
solemnly declared Emil Ong, respondent's full
brother, as a natural born Filipino. The
Constitutional Convention had to be aware of the
meaning of natural born citizenship since it was
precisely amending the article on this subject.
cdll
The private respondent frequently went home to
Laoang, Samar, where he grew up and spent his
childhood days.
In 1984, the private respondent married a Filipina
named Desiree Lim.
For the elections of 1984 and 1986, Jose Ong, Jr.
registered himself as a voter of Laoang, Samar,
and correspondingly, voted there during those
elections.
The private respondent after being engaged for
several years in the management of their family
business decided to be of greater service to his
province and ran for public office. Hence, when
10 | C O N S T I 2 _ A r t i c l e I V _ C I T I Z E N S H I P

the opportunity came in 1987, he ran in the


elections for representative in the second district
of Northern Samar.
Mr. Ong was overwhelmingly voted by the people
of Northern Samar as their representative in
Congress. Even if the total votes of the two
petitioners are combined, Ong would still lead the
two by more than 7,000 votes.
The pertinent portions of the Constitution found
in Article IV read:.
"SECTION 1, the following are citizens of the
Philippines:
1. Those who are citizens of the Philippines at the
time of the adoption of the Constitution;
2. Those whose fathers or mothers are citizens of
the Philippines;
3. Those born before January 17, 1973, of Filipino
mothers, who elect Philippine citizenship upon
reaching the age of majority; and
4. Those who are naturalized in accordance with
law.
SECTION 2, Natural-born Citizens are those who
are citizens of the Philippines from birth without
having to perform any act to acquire or perfect
their citizenship. Those who elect Philippine
citizenship in accordance with paragraph 3 hereof
shall be deemed natural born citizens."

The Court interprets Section 1, Paragraph 3


above as applying not only to those who elect
Philippine citizenship after February 2, 1987 but
also to those who, having been born of Filipino
mothers, elected citizenship before that date.
The provision in Paragraph 3 was intended to
correct an unfair position which discriminates
against Filipino women. There is no ambiguity in
the
deliberations
of
the
Constitutional
Commission, viz:
"Mr. Azcuna:
With respect to the provision of section 4, would
this refer only to those who elect Philippine
citizenship after the effectivity of the 1973
Constitution or would it also cover those who
elected it under the 1973 Constitution?
Fr. Bernas:
It would apply to anybody who elected Philippine
citizenship by virtue of the provision of the 1935
Constitution whether the election was done
before or after January l7, 1973." (Records of the
Constitutional Commission, Vol. 1, p. 228;
Emphasis supplied).
xxx xxx xxx

"Mr. Trenas:
The Committee on Citizenship, Bill of Rights,
Political Rights and Obligations and Human Rights
has more or less decided to extend the
interpretation of who is a natural-born Citizen as
provided in section 4 of the 1973 Constitution by
adding that persons who have elected Philippine
Citizenship under the 1935 Constitution shall be
natural-born? Am I right Mr. Presiding Officer?
Fr. Bernas:
yes."
xxx xxx xxx
"Mr. Nolledo:
And I remember very well that in the Reverend
Father Bernas' well written book, he said that the
decision was designed merely to accommodate
former delegate Ernesto Ang and that the
definition on natural-born has no retroactive
effect. Now it seems that the Reverend Father
Bernas is going against this intention by
supporting the amendment?
Fr. Bernas:
As the Commissioner can see, there has been an
evolution in my thinking. (Records of the
Constitutional Commission, Vol. 1, p. 189)
xxx xxx xxx
"Mr. Rodrigo:
But this provision becomes very important
because his election of Philippine citizenship
makes him not only a Filipino citizen but a
natural-born Filipino citizen entitling him to run
for Congress . . .
Fr. Bernas:
Correct. We are quite aware of that and for that
reason we will leave it to the body to approve
that provision of section 4.
Mr. Rodrigo:
I think there is a good basis for the provision
because it strikes me as unfair that the Filipino
citizen who was born a day before January 17,
1973 cannot be a Filipino citizen or a natural born
citizen."
(Records
of
the
Constitutional
Commission, Vol. 1, p. 231)
xxx xxx xxx
"Mr. Rodrigo:
The purpose of that provision is to remedy an
inequitable situation. Between 1935 and 1973
when we were under the 1935 Constitution, those
born of Filipino fathers but alien mothers were
natural-born Filipinos. However, those born of
Filipino mothers but alien fathers would have to
11 | C O N S T I 2 _ A r t i c l e I V _ C I T I Z E N S H I P

elect Philippine citizenship upon reaching the age


of majority; and if they do elect, they become
Filipino citizens but not natural-born Filipino
citizens."
(Records
of
the
Constitutional
Commission, Vol. 1, p. 356)
The foregoing significantly reveals the intent of
the framers. To make the provision prospective
from February 3, 1987 is to give a narrow
interpretation
resulting
in
an
inequitable
situation. It must also be retroactive.
It should be noted that in construing the law, the
Courts are not always to be hedged in by the
literal meaning of its language. The spirit and
intendment thereof, must prevail over the letter,
especially where adherence to the latter would
result in absurdity and injustice. (Casela v. Court
of Appeals, 35 SCRA 279 [1970])
A Constitutional provision should be construed so
as to give it effective operation and suppress the
mischief at which it is aimed, hence, it is the spirit
of the provision which should prevail over the
letter thereof. (Jarrolt v. Mabberly, 103 U.S. 580)
In the words of the Court in the case of J.M.
Tuason v. LTA (31 SCRA 413 [1970]:
"To that primordial intent, all else is subordinated.
Our Constitution, any constitution is not to be
construed narrowly or pedantically, for the
prescriptions therein contained, to paraphrase
Justice Holmes, are not mathematical formulas
having their essence in their form but are organic
living institutions, the significance of which is
vital not formal . . ." (p. 427)
The provision in question was enacted to correct
the anomalous situation where one born of a
Filipino father and an alien mother was
automatically granted the status of a natural-born
citizen while one born of a Filipino mother and an
alien father would still have to elect Philippine
citizenship. If one so elected, he was not, under
earlier laws, conferred the status of a naturalborn.
Under the 1973 Constitution, those born of
Filipino fathers and those born of Filipino mothers
with an alien father were placed on equal footing.
They were both considered as natural-born
citizens.
Hence, the bestowment of the status of "naturalborn" cannot be made to depend on the fleeting
accident of time or result in two kinds of citizens
made up of essentially the same similarly
situated members.
It is for this reason that the amendments were
enacted, that is, in order to remedy this
accidental anomaly, and, therefore, treat equally
all those born before the 1973 Constitution and
who elected Philippine citizenship either before or
after the effectivity of that Constitution. Cdpr
The Constitutional provision in question is,
therefore curative in nature. The enactment was

meant to correct the inequitable and absurd


situation which then prevailed, and thus, render
those acts valid which would have been nil at the
time had it not been for the curative provisions.
(See Development Bank of the Philippines v.
Court of Appeals, 96 SCRA 342 [1980])
There is no dispute that the respondent's mother
was a natural born Filipina at the time of her
marriage. Crucial to this case is the issue of
whether or not the respondent elected or chose
to be a Filipino citizen.
Election becomes material because Section 2 of
Article IV of the Constitution accords natural born
status to children born of Filipino mothers before
January 17, 1973, if they elect citizenship upon
reaching the age of majority.
To expect the respondent to have formally or in
writing elected citizenship when he came of age
is to ask for the unnatural and unnecessary. The
reason is obvious. He was already a citizen. Not
only was his mother a natural born citizen but his
father had been naturalized when the respondent
was only nine (9) years old. He could not have
divined when he came of age that in 1973 and
1987 the Constitution would be amended to
require him to have filed a sworn statement in
1969 electing citizenship inspite of his already
having been a citizen since 1957. In 1969,
election through a sworn statement would have
been an unusual and unnecessary procedure for
one who had been a citizen since he was nine
years old.
We have jurisprudence that defines "election" as
both a formal and an informal process.
In the case of In Re: Florencio Mallare (59 SCRA
45 [1974]), the Court held that the exercise of the
right of suffrage and the participation in election
exercises constitute a positive act of election of
Philippine
citizenship.
In
the
exact
pronouncement of the Court, we held:
"Esteban's exercise of the right of suffrage when
he came of age, constitutes a positive act of
election of Philippine citizenship". (p. 52;
emphasis supplied)
The private respondent did more than merely
exercise his right of suffrage. He has established
his life here in the Philippines.
For those in the peculiar situation of the
respondent who cannot be expected to have
elected citizenship as they were already citizens,
we apply the In Re Mallare rule.
The respondent was born in an outlying rural
town of Samar where there are no alien enclaves
and no racial distinctions. The respondent has
lived the life of a Filipino since birth. His father
applied for naturalization when the child was still
a small boy. He is a Roman Catholic. He has
worked for a sensitive government agency. His
profession requires citizenship for taking the
examinations and getting a license. He has
12 | C O N S T I 2 _ A r t i c l e I V _ C I T I Z E N S H I P

participated in political exercises as a Filipino and


has always considered himself a Filipino citizen.
There is nothing in the records to show that he
does not embrace Philippine customs and values,
nothing to indicate any tinge of alien-ness, no
acts to show that this country is not his natural
homeland. The mass of voters of Northern Samar
are fully aware of Mr. Ong's parentage. They
should know him better than any member of this
Court will ever know him. They voted by
overwhelming numbers to have him represent
them in Congress. Because of his acts since
childhood, they have considered him as a Filipino.
The filing of sworn statement or formal
declaration is a requirement for those who still
have to elect citizenship. For those already
Filipinos when the time to elect came up, there
are acts of deliberate choice which cannot be less
binding. Entering a profession open only to
Filipinos, serving in public office where citizenship
is a qualification, voting during election time,
running for public office, and other categorical
acts of similar nature are themselves formal
manifestations of choice for these persons. LLjur
An election of Philippine citizenship presupposes
that the person electing is an alien. Or his status
is doubtful because he is a national of two
countries. There is no doubt in this case about Mr.
Ong's being a Filipino when he turned twenty-one
(21).
We repeat that any election of Philippine
citizenship on the part of the private respondent
would not only have been superfluous but it
would also have resulted in an absurdity. How can
a Filipino citizen elect Philippine citizenship?
The respondent HRET has an interesting view as
to how Mr. Ong elected citizenship. It observed
that "when protestee was only nine years of age,
his father, Jose Ong Chuan became a naturalized
Filipino. Section 15 of the Revised Naturalization
Act squarely applies its benefit to him for he was
then a minor residing in this country. Concededly,
it was the law itself that had already elected
Philippine citizenship for protestee by declaring
him as such." (Emphasis supplied)

The petitioners argue that the respondent's


father was not, validly, a naturalized citizen
because of his premature taking of the oath of
citizenship.
The Court cannot go into the collateral procedure
of stripping Mr. Ong's father of his citizenship
after his death and at this very late date just so
we can go after the son.
The petitioners question the citizenship of the
father through a collateral approach. This can not
be done. In our jurisdiction, an attack on a
person's citizenship may only be done through a
direct action for its nullity. (See Queto v. Catolico,
31 SCRA 52 [1970]).

To ask the Court to declare the grant of Philippine


citizenship to Jose Ong Chuan as null and void
would run against the principle of due process.
Jose Ong Chuan has already been laid to rest.
How can he be given a fair opportunity to defend
himself. A dead man cannot speak. To quote the
words of the HRET: "Ong Chuan's lips have long
been muted to perpetuity by his demise and
obviously he could not rise beyond where his
mortal remains now lie to defend himself were
this matter to be made a central issue in this
case."
The issue before us is not the nullification of the
grant of citizenship to Jose Ong Chuan. Our
function is to determine whether or not the HRET
committed abuse of authority in the exercise of
its powers. Moreover, the respondent traces his
natural born citizenship through his mother, not
through the citizenship of his father. The
citizenship of the father is relevant only to
determine whether or not the respondent "chose"
to be a Filipino when he came of age. At that time
and up to the present, both mother and father
were Filipinos. Respondent Ong could not have
elected any other citizenship unless he first
formally renounced Philippine citizenship in favor
of a foreign nationality. Unlike other persons
faced with a problem of election, there was no
foreign nationality of his father which he could
possibly have chosen.
There is another reason why we cannot declare
the HRET as having committed manifest grave
abuse of discretion. The same issue of naturalborn citizenship has already been decided by the
Constitutional Convention of 1971 and by the
Batasang Pambansa convened by authority of the
Constitution drafted by that Convention. Emil
Ong, full blood brother of the respondent, was
declared and accepted as a natural born citizen
by both bodies.
Assuming that our opinion is different from that of
the Constitutional Convention, the Batasang
Pambansa, and the respondent HRET, such a
difference could only be characterized as error.
There would be no basis to call the HRET decision
so arbitrary and whimsical as to amount to grave
abuse of discretion.
What was the basis for the Constitutional
Convention's declaring Emil Ong a natural born
citizen?
Under the Philippine Bill of 1902, inhabitants of
the Philippines who were Spanish subjects on the
11th day of April 1899 and then residing in said
islands and their children born subsequent
thereto were conferred the status of a Filipino
citizen.
Was the grandfather of the private respondent a
Spanish subject?
Article 17 of the Civil Code of Spain enumerates
those who were considered Spanish Subjects, viz:
"ARTICLE 17. The following are Spaniards:
13 | C O N S T I 2 _ A r t i c l e I V _ C I T I Z E N S H I P

1. Persons born in Spanish territory.


2. Children born of a Spanish father or mother,
even though they were born out of Spain.
3.
Foreigners
who
naturalization papers.

may

have

obtained

4. Those without such papers, who may have


acquired domicile in any town in the Monarchy."
(Emphasis supplied)
The domicile of a natural person is the place of
his habitual residence. This domicile, once
established is considered to continue and will not
be deemed lost until a new one is established.
(Article 50, NCC; Article 40, Civil Code of Spain;
Zuellig v. Republic, 83 Phil. 768 [1949])
As earlier stated, Ong Te became a permanent
resident of Laoang, Samar around 1895.
Correspondingly, a certificate of residence was
then issued to him by virtue of his being a
resident of Laoang, Samar. (Report of the
Committee on Election Protests and Credentials
of
the
1971
Constitutional
Convention,
September 7,1972, p. 3)
The domicile that Ong Te established m 1895
continued until April 11, 1899; it even went
beyond the turn of the 19th century. It is also in
this place were Ong Te set-up his business and
acquired his real property.
As concluded by the Constitutional Convention
Ong Te falls within the meaning of sub-paragraph
4 of Article 17 of the Civil Code of Spain.
Although Ong Te made brief visits to China, he,
nevertheless, always returned to the Philippines.
The fact that he died in China, during one of his
visits in said country, was of no moment. This will
not change the fact that he already had his
domicile fixed in the Philippines and pursuant to
the Civil Code of Spain, he had become a Spanish
subject. LibLex
If Ong Te became a Spanish subject by virtue of
having established his domicile in a town under
the Monarchy of Spain, necessarily, Ong Te was
also an inhabitant of the Philippines for an
inhabitant has been defined as one who has
actual fixed residence in a place; one who has a
domicile in a place. (Bouvier's Law Dictionary,
Vol. II) A priori, there can be no other logical
conclusion but to educe that Ong Te qualified as a
Filipino citizen under the provisions of section 4 of
the Philippine Bill of 1902.
The HRET itself found this fact of absolute verity
in concluding that the private respondent was a
natural-born Filipino.
The petitioners' sole ground in disputing this fact
is that the documents presented to prove it were
not in compliance with the best evidence rule.
The petitioners allege that the private respondent
failed to present the original of the documentary

evidence, testimonial evidence and of the


transcript of the proceedings of the body which
the
aforesaid
resolution
of
the
1971
Constitutional Convention was predicated.
On the contrary, the documents presented by the
private respondent fall under the exceptions to
the best evidence rule.
It was established in the proceedings before the
HRET that the originals of the Committee Report
No. 12, the minutes of the plenary session of
1971
Constitutional
Convention
held
on
November 28, 1972 cannot be found.
This was affirmed by Atty. Ricafrente, Assistant
Secretary of the 1971 Constitutional Convention;
by Atty. Nolledo, Delegate to the 1971
Constitutional Convention; and by Atty. Antonio
Santos, Chief Librarian of the U.P. Law Center, in
their respective testimonies given before the
HRET to the effect that there is no governmental
agency which is the official custodian of the
records of the 1971 Constitutional Convention.
(TSN, December 12, 1988, pp. 30-31; TSN,
January 17, 1989, pp. 34-35; TSN, February 1,
1989, p. 44; TSN, February 6, 1989, pp. 28-29)
The execution of the originals was established by
Atty. Ricafrente, who as the Assistant Secretary of
the 1971 Constitutional Convention was the
proper party to testify to such execution. (TSN,
December 12, 1989, pp. 11-24)
The inability to produce the originals before the
HRET was also testified to as aforestated by Atty.
Ricafrente, Atty. Nolledo, and Atty. Santos. In
proving the inability to produce, the law does not
require the degree of proof to be of sufficient
certainty; it is enough that it be shown that after
a bona fide diligent search, the same cannot be
found. (see Government of P.I. v. Martinez, 44
Phil. 817 [1918])
Since the execution of the document and the
inability to produce were adequately established,
the contents of the questioned documents can be
proven by a copy thereof or by the recollection of
witnesses.
Moreover, to erase all doubts as to the
authenticity of the documentary evidence cited in
the Committee Report, the former member of the
1971 Constitutional Convention, Atty. Nolledo,
when he was presented as a witness in the
hearing of the protest against the private
respondent, categorically stated that he saw the
disputed documents presented during the
hearing of the election protest against the brother
of the private respondent. (TSN, February 1,
1989, pp. 8-9)
In his concurring opinion, Mr. Justice Sarmiento, a
vice-president of the Constitutional Convention,
states that he was presiding officer of the plenary
session which deliberated on the report on the
election protest against Delegate Emil Ong. He
cites a long list of names of delegates present.
Among them are Mr. Chief Justice Fernan, and Mr.
14 | C O N S T I 2 _ A r t i c l e I V _ C I T I Z E N S H I P

Justice Davide, Jr. The petitioners could have


presented any one of the long list of delegates to
refute Mr. Ong's having been declared a naturalborn citizen. They did not do so. Nor did they
demur to the contents of the documents
presented by the private respondent. They
merely relied on the procedural objections
respecting the admissibility of the evidence
presented.
The Constitutional Convention was the sole judge
of the qualifications of Emil Ong to be a member
of that body. The HRET, by explicit mandate of
the Constitution, is the sole judge of the
qualifications of Jose Ong, Jr. to be a member of
Congress. Both bodies deliberated at length on
the controversies over which they were sole
judges. Decisions were arrived at only after a full
presentation of all relevant factors which the
parties wished to present. Even assuming that we
disagree with their conclusions, we cannot
declare their acts as committed with grave abuse
of discretion. We have to keep clear the line
between error and grave abuse.
ON THE ISSUE OF RESIDENCE
The
petitioners
question
the
qualification of respondent Ong.

residence

The petitioners lose sight of the meaning of


"residence" under the Constitution. The term
"residence" has been understood as synonymous
with domicile not only under the previous
Constitutions
but
also
under
the
1987
Constitution.
The
deliberations
of
the
Constitutional
Commission reveal that the meaning of residence
vis-a-vis the qualifications of a candidate for
Congress continues to remain the same as that of
domicile, to wit:

"Mr. Nolledo:
With respect to Section 5, I remember that in the
1971 Constitutional Convention, there was an
attempt to require residence in the place not less
than one year immediately preceding the day of
the elections. So my question is: What is the
Committee's concept of residence of a candidate
for the legislature? Is it actual residence or is it
the concept of domicile or constructive
residence?
Mr. Davide:
Madame President, insofar as the regular
members of the National Assembly are
concerned, the proposed section merely provides,
among others, 'and a resident thereof, that is, in
the district, for a period of not less than one year
preceding the day of the election'. This was in
effect lifted from the 1973 Constitution, the
interpretation given to it was domicile." (Records
of the 1987 Constitutional Convention, Vol. II, July
22, 1986, p. 87)

xxx xxx xxx


"Mrs. Rosario Braid:
The next question is on Section 7, page 2. I think
Commissioner Nolledo has raised the same point
that 'resident' has been interpreted at times as a
matter of intention rather than actual residence.
Mr. De los Reyes:
Domicile.
Ms. Rosario Braid:
Yes, So, would the gentlemen consider at the
proper time to go back to actual residence rather
than mere intention to reside?
Mr. De los Reyes:
But we might encounter some difficulty
especially considering that a provision in the
Constitution in the Article on Suffrage says that
Filipinos living abroad may vote as enacted by
law. So, we have to stick to the original concept
that it should be by domicile and not physical and
actual residence." (Records of the 1987
Constitutional Commission, Vol. II, July 22, 1986,
p. 110)
The framers of the Constitution adhered to the
earlier definition given to the word "residence"
which regarded it as having the same meaning as
domicile.
The term "domicile" denotes a fixed permanent
residence to which when absent for business or
pleasure, one intends to return. (Ong Huan Tin v.
Republic, 19 SCRA 966 [1967]) The absence of a
person from said permanent residence, no matter
how long, notwithstanding, it continues to be the
domicile of that person. In other words, domicile
is characterized by animus revertendi. (Ujano v.
Republic, 17 SCRA 147 [1966]) cdphil
The domicile of origin of the private respondent,
which was the domicile of his parents, is fixed at
Laoang, Samar. Contrary to the petitioners'
imputation, Jose Ong, Jr. never abandoned said
domicile; it remained fixed therein even up to the
present.
The private respondent, in the proceedings before
the HRET, sufficiently established that after the
fire that gutted their house in 1961, another one
was constructed.
Likewise, after the second fire which again
destroyed their house in 1975, a sixteen-door
apartment was built by their family, two doors of
which were reserved as their family residence.
(TSN, Jose Ong, Jr., November 18, 1988, p. 8)
The petitioners' allegation that since the private
respondent owns no property in Laoang, Samar,
he cannot, therefore, be a resident of said place
is misplaced.
15 | C O N S T I 2 _ A r t i c l e I V _ C I T I Z E N S H I P

The properties owned by the Ong Family are in


the name of the private respondent's parents.
Upon the demise of his parents, necessarily, the
private respondent, pursuant to the laws of
succession, became the co-owner thereof (as a
co-heir), notwithstanding the fact that these were
still in the names of his parents.
Even assuming that the private respondent does
not own any property in Samar, the Supreme
Court in the case of De los Reyes D. Solidum (61
Phil. 893 [1935]) held that it is not required that a
person should have a house in order to establish
his residence and domicile. It is enough that he
should live in the municipality or in a rented
house or in that of a friend or relative. (Emphasis
supplied)
To require the private respondent to own property
in order to be eligible to run for Congress would
be tantamount to a property qualification. The
Constitution only requires that the candidate
meet the age, citizenship, voting and residence
requirements. Nowhere is it required by the
Constitution that the candidate should also own
property in order to be qualified to run. (see
Maquera v. Borra, 122 Phil. 412 [1965])
It has also been settled that absence from
residence to pursue studies or practice a
profession or registration as a voter other than in
the place where one is elected, does not
constitute loss of residence. (Faypon v. Quirino,
96 Phil. 294 [1954])
As previously stated, the private respondent
stayed in Manila for the purpose of finishing his
studies and later to practice his profession. There
was no intention to abandon the residence in
Laoang, Samar. On the contrary, the periodical
journeys made to his home province reveal that
he always had the animus revertendi.
The Philippines is made up not only of a single
race; it has, rather, undergone an interracial
evolution. Throughout our history, there has been
a continuing influx of Malays, Chinese,
Americans, Japanese, Spaniards and other
nationalities. This racial diversity gives strength
to our country.
Many great Filipinos have not been whole-blooded
nationals, if there is such a person, for there is
none. To mention a few, the great Jose Rizal was
part Chinese, the late Chief Justice Claudio
Teehankee was part Chinese, and of course our
own President, Corazon Aquino is also part
Chinese. Verily, some Filipinos of whom we are
proud were ethnically more Chinese than the
private respondent.
Our citizens no doubt constitute the country's
greatest wealth. Citizenship is a special privilege
which one must forever cherish.
However, in order to truly revere this treasure of
citizenship, we do not, on the basis of too harsh
an interpretation, have to unreasonably deny it to
those who qualify to share in its richness.

Under the overly strict jurisprudence surrounding


our antiquated naturalization laws only the very
affluent backed by influential patrons, who were
willing to suffer the indignities of a lengthy,
sometimes humiliating, and often corrupt process
of clearances by minor bureaucrats and whose
lawyers knew how to overcome so many
technical traps of the judicial process were able
to acquire citizenship. It is time for the
naturalization law to be revised to enable a more
positive, affirmative, and meaningful examination
of an applicant's suitability to be a Filipino. A
more humane, more indubitable and less
technical approach to citizenship problems is
essential.
WHEREFORE,
the
petitions
are
hereby
DISMISSED. The questioned decision of the house
of Representatives Electoral Tribunal is AFFIRMED.
Respondent Jose Ong, Jr. is declared a naturalborn citizen of the Philippines and a resident of
Laoang, Northern Samar.
SO ORDERED.
Bidin, Grio-Aquino, Medialdea and Davide, Jr.,
JJ ., concur.
Fernan, C .J ., Melencio-Herrera, Cruz, Feliciano
and Gancayco, JJ ., took no part.
(In re: Yu v. Defensor-Santiago, G.R. No.
83882, January 24, 1989)
EN BANC
[G.R. No. 83882. January 24, 1989.]
IN RE PETITION FOR HABEAS CORPUS OF WILLIE
YU, WILLIE YU, petitioner, vs. MIRIAM DEFENSORSANTIAGO, BIENVENIDO P. ALANO, JR., MAJOR
PABALAN, DELEO HERNANDEZ, BLODDY
HERNANDEZ, BENNY REYES AND JUN ESPIRITU
SANTO, respondents.
Pelaez, Adriano and Gregorio and Bonifacio A.
Alentajan for petitioner.
Chavez, Hechanova & Lim Law Offices
collaborating counsel for petitioner.
Augusto Jose y. Arreza for respondents.
SYLLABUS
1. CONSTITUTIONAL LAW; CITIZENSHIP; EXPRESS;
RENUNCIATION; A RENUNCIATION MADE KNOWN
DISTINCTLY AND EXPLICITLY AND NOT LEFT TO
INFERENCE OR IMPLICATION. In Board of
Immigration Commissioners vs. Go Gallano,
express renunciation was held to mean a
renunciation that is made known distinctly and
explicitly and not left to inference or implication.
Petitioner, with full knowledge, and legal
capacity, after having renounced Portuguese
citizenship upon naturalization as a Philippine
citizen resumed or reacquired his prior status as a
16 | C O N S T I 2 _ A r t i c l e I V _ C I T I Z E N S H I P

Portuguese citizen, applied for a renewal of his


Portuguese passport and represented himself as
such in official documents even after he had
become a naturalized Philippine citizen. Such
resumption or reacquisition of Portuguese
citizenship is grossly inconsistent with his
maintenance of Philippine citizenship.
2. ID.; BILL OF RIGHTS; DUE PROCESS; DENIAL
THEREOF OBVIATED WHEN PETITIONER WAS
GIVEN BY COURT THE OPPORTUNITY TO SHOW
PROOF OF CONTINUED PHILIPPINE CITIZENSHIP.
Denial, if any, of due process was obviated
when petitioner was given by the Court the
opportunity to show proof of continued Philippine
citizenship, but he has failed.
3. ID.; CITIZENSHIP; PHILIPPINE CITIZENSHIP; NOT
A COMMODITY OR WARE TO BE DISPLAYED WHEN
REQUIRED AND SUPPRESSED WHEN CONVENIENT.
Philippine citizenship, it must be stressed, is
not a commodity or were to be displayed when
required and suppressed when convenient.
FERNAN, C.J., dissenting:
1. CONSTITUTIONAL LAW; BILL OF RIGHTS; DUE
PROCESS; POWERS OF REVIEW OF SUPREME
COURT CANNOT BE A SUBSTITUTE FOR DEMANDS
THEREOF SINCE SAID COURT IS NOT A TRIER OF
FACTS. The observation of Mr. Justice Hugo E.
Gutierrez, Jr. in his dissenting opinion that
"(c)onsidering the serious implications of deFilipinization, the correct procedures according to
law must be applied," is appropriate as it has
been held that "(i)f, however, in a deportation
proceeding, the alleged alien claims citizenship
and supports the claim by substantial evidence,
he is entitled to have his status finally determined
by a judicial, as distinguished from an executive,
tribunal" (3 Am Jur 2d 949 citing United States ex
rel. Bilokumsky v. Tod, 263 US 149, 68 L ed 221,
44 S Ct 54; Ng Fung Ho v. White, 259 US 276, 66
Led 938, 42 S Ct 492). By this, it means a full
blown trial under the more rigid rules of evidence
prescribed in court proceedings. And certainly,
the review powers being exercised by this Court
in this case fall short of this requirement. Said
powers of review cannot be a substitute for the
demands of due process, particularly in the light
of the well-recognized principle that this Court is
not a trier of facts.
DECISION
PADILLA, J p:
The present controversy originated with a petition
for habeas corpus filed with the Court on 4 July
1988 seeking the release from detention of herein
petitioner. 1 After manifestation and motion of
the Solicitor General of his decision to refrain
from filing a return of the writ on behalf of the
CID, respondent Commissioner thru counsel filed
the return. 2 Counsel for the parties were heard
in oral argument on 20 July 1988. The parties
were allowed to submit marked exhibits, and to
file memoranda. 3 An internal resolution of 7
November 1988 referred the case to the Court en
banc. In its 10 November 1988 resolution,

denying the petition for habeas corpus, the Court


disposed of the pending issues of (1) jurisdiction
of the CID over a naturalized Filipino citizen and
(2) validity of warrantless arrest and detention of
the same person.
Petitioner filed a motion for reconsideration with
prayer for restraining order dated 24 November
1988. 4 On 29 November 1988, the Court
resolved to deny with finality the aforesaid
motion for reconsideration, and further resolved
to deny the urgent motion for issuance of a
restraining order dated 28 November 1988. 5
Undaunted, petitioner filed a motion for
clarification with prayer for restraining order on 5
December 1988.
Acting on said motion, a temporary restraining
order was issued by the Court on 7 December
1988. 6 Respondent Commissioner filed a motion
to lift TRO on 13 December 1988, the basis of
which is a summary judgment of deportation
against Yu issued by the CID Board of
Commissioners on 2 December 1988. 7 Petitioner
also filed a motion to set case for oral argument
on 8 December 1988.
In the meantime, an urgent motion for release
from arbitrary detention 8 was filed by petitioner
on 13 December 1988. A memorandum in
furtherance of said motion for release dated 14
December 1988 was filed on 15 December 1988
together with a vigorous opposition to the lifting
of the TRO. cdrep
The lifting of the Temporary Restraining Order
issued by the Court on 7 December 1988 is
urgently sought by respondent Commissioner
who was ordered to cease and desist from
immediately deporting petitioner Yu pending the
conclusion of hearings before the Board of Special
Inquiry, CID. To finally dispose of the case, the
Court will likewise rule on petitioner's motion for
clarification with prayer for restraining order
dated 5 December 1988, 9 urgent motion for
release from arbitrary detention dated 13
December 1988, 10 the memorandum in
furtherance of said motion for release dated 14
December 1988, 11 motion to set case for oral
argument dated 8 December 1988. 12
Acting on the motion to lift the temporary
restraining order (issued on 7 December 1988)
dated 9 December 1988, 13 and the vigorous
opposition to lift restraining order dated 15
December 1988, 14 the Court resolved to give
petitioner Yu a non-extendible period of three (3)
days from notice within which to explain and
prove why he should still be considered a citizen
of the Philippines despite his acquisition and use
of a Portuguese passport. 15
Petitioner filed his compliance with the resolution
of 15 December 1988 on 20 December 1988 16
followed by an earnest request for temporary
release on 22 December 1988. Respondent filed
on 2 January 1989 her comment reiterating her
previous motion to lift temporary restraining
17 | C O N S T I 2 _ A r t i c l e I V _ C I T I Z E N S H I P

order. Petitioner filed a reply thereto on 6 January


1989.
Petitioner's own compliance reveals that he was
originally issued a Portuguese passport in 1971,
17 valid for five (5) years and renewed for the
same period upon presentment before the proper
Portuguese
consular
officer.
Despite
his
naturalization as a Philippine citizen on 10
February 1978, on 21 July 1981, petitioner
applied for and was issued Portuguese Passport
No. 35/81 serial N. 1517410 by the Consular
Section of the Portuguese Embassy in Tokyo. Said
Consular Office certifies that his Portuguese
passport expired on 20 duly 1986. 18 While still a
citizen of the Philippines who had renounced,
upon his naturalization, "absolutely and forever
all allegiance and fidelity to any foreign prince,
potentate, state or sovereignty" and pledged to
"maintain true faith and allegiance to the
Republic of the Philippines," 19 he declared his
nationality
as
Portuguese
in
commercial
documents he signed, specifically, the Companies
registry of Tai Shun Estate Ltd. 20 filed in
Hongkong sometime in April 1980.
To the mind of the Court, the foregoing acts
considered together constitute an express
renunciation of petitioner's Philippine citizenship
acquired through naturalization. In Board of
Immigration Commissioners vs. Go Gallano, 21
express renunciation was held to mean a
renunciation that is made known distinctly and
explicitly and not left to inference or implication.
Petitioner, with full knowledge, and legal
capacity, after having renounced Portuguese
citizenship upon naturalization as a Philippine
citizen 22 resumed or reacquired his prior status
as a Portuguese citizen, applied for a renewal of
his Portuguese passport 23 and represented
himself as such in official documents even after
he had become a naturalized Philippine citizen.
Such resumption or reacquisition of Portuguese
citizenship is grossly inconsistent with his
maintenance of Philippine citizenship. LexLib
This Court issued the aforementioned TRO
pending hearings with the Board of Special
Inquiry, CID. However, pleadings submitted
before this Court after the issuance of said TRO
have unequivocally shown that petitioner has
expressly renounced his Philippine citizenship.
The material facts are not only established by the
pleadings they are not disputed by petitioner.
A rehearing on this point with the CID would be
unnecessary and superfluous. Denial, if any, of
due process was obviated when petitioner was
given by the Court the opportunity to show proof
of continued Philippine citizenship, but he has
failed.
While normally the question of whether or not a
person has renounced his Philippine citizenship
should be heard before a trial court of law in
adversary
proceedings,
this
has
become
unnecessary as this Court, no less, upon the
insistence of petitioner, had to look into the facts
and satisfy itself on whether or not petitioner's

claim to continued
meritorious.

Philippine

citizenship

is

Philippine citizenship, it must be stressed, is not a


commodity or were to be displayed when
required and suppressed when convenient. This
then resolves adverse to the petitioner his motion
for clarification and other motions mentioned in
the second paragraph, page 3 of this Decision.
WHEREFORE, premises considered, petitioner's
motion for release from detention is DENIED.
Respondent's motion to lift the temporary
restraining order is GRANTED. This Decision is
immediately executory.

SO ORDERED.
Melencio-Herrera, Paras, Feliciano, Gancayco,
Bidin, Sarmiento, Grio-Aquino, Medialdea and
Regalado, JJ., concur.
Narvasa, J., in the result.
(Frivaldo v. COMELEC, G.R. No. 87193, June
23, 1989)
EN BANC
[G.R. No. 87193. June 23, 1989.]
JUAN GALLANOSA FRIVALDO, petitioner, vs.
COMMISSION ON ELECTIONS AND THE LEAGUE OF
MUNICIPALITIES, SORSOGON CHAPTER, HEREIN
REPRESENTED BY ITS PRESIDENT, SALVADOR NEE
ESTUYE, respondents.
J .L. Misa & Associates for petitioner.
Lladoc, Huab & Associates for private respondent.
DECISION
CRUZ, J p:
Petitioner Juan G. Frivaldo was proclaimed
governor-elect of the province of Sorsogon on
January 22, 1988, and assumed office in due
time. On October 27, 1988, the league of
Municipalities, Sorsogon Chapter (hereafter,
League), represented by its President, Salvador
Estuye, who was also suing in his personal
capacity, filed with the Commission on Elections a
petition for the annulment of Frivaldo's election
and proclamation on the ground that he was not
a Filipino citizen, having been naturalized in the
United States on January 20,1983. In his answer
dated May 22, 1988, Frivaldo admitted that he
was naturalized in the United States as alleged
but pleaded the special and affirmative defenses
that he had sought American citizenship only to
protect himself against President Marcos. His
naturalization, he said, was "merely forced upon
himself as a means of survival against the
unrelenting persecution by the Martial Law
Dictator's agents abroad." He added that he had
returned to the Philippines after the EDSA
18 | C O N S T I 2 _ A r t i c l e I V _ C I T I Z E N S H I P

revolution to help in the restoration of democracy.


He also argued that the challenge to his title
should be dismissed, being in reality a quo
warranto petition that should have been filed
within ten days from his proclamation, in
accordance with Section 253 of the Omhibus
Election Code. The League, moreover, was not a
proper party because it was not a voter and so
could not sue under the said section. prLL
Frivaldo moved for a preliminary hearing on his
affirmative
defenses
but
the
respondent
Commission on Elections decided instead by its
Order of January 20, 1988, to set the case for
hearing on the merits. His motion for
reconsideration was denied in another Order
dated February 21, 1988. He then came to this
Court in a petition for certiorari and prohibition to
ask that the said orders be set aside on the
ground that they had been rendered with grave
abuse of discretion. Pending resolution of the
petition, we issued a temporary order against the
hearing on the merits scheduled by the COMELEC
and at the same time required comments from
the respondents.
In their Comment, the private respondents
reiterated their assertion that Frivaldo was a
naturalized American citizen and had not
reacquired Philippine citizenship on the day of the
election on January 18, 1988. He was therefore
not qualified to run for and be elected governor.
They also argued that their petition in the
Commission on Elections was not really for quo
warranto under Section 253 of the Omnibus
Election Code. The ultimate purpose was to
prevent Frivaldo from continuing as governor, his
candidacy and election being null and void ab
initio because of his alienage. Even if their
petition were to be considered as one for quo
warranto, it could not have been filed within ten
days from Frivaldo's proclamation because it was
only in September 1988 that they received proof
of his naturalization. And assuming that the
League itself was not a proper party, Estuye
himself, who was suing not only for the League
but also in his personal capacity, could
nevertheless institute the suit by himself alone.
Speaking for the public respondent, the Solicitor
General supported the contention that Frivaldo
was not a citizen of the Philippines and had not
repatriated himself after his naturalization as an
American citizen. As an alien, he was disqualified
from public office in the Philippines. His election
did not cure this defect because the electorate of
Sorsogon could not amend the Constitution, the
Local Government Code, and the Omnibus
Election Code. He also joined in the private
respondent's argument that Section 253 of the
Omnibus Election Code was not applicable
because what the League and Estuye were
seeking was not only the annulment of the
proclamation and election of Frivaldo. He agreed
that they were also asking for the termination of
Frivaldo's incumbency as governor of Sorsogon
on the ground that he was not a Filipino.

In his Reply, Frivaldo insisted that he was a


citizen
of
the
Philippines
because
his
naturalization as an American citizen was not
"impressed with voluntariness." In support he
cited the Nottebohm Case, [1955 I.C.J. 4; 49
A.J.I.L. 396 (1955)] where a German national's
naturalization in Liechtenstein was not recognized
because it had been obtained for reasons of
convenience only. He said he could not have
repatriated himself before the 1988 elections
because the Special Committee on Naturalization
created for the purpose by LOI No. 270 had not
yet been organized then. His oath in his
certificate of candidacy that he was a naturalborn citizen should be a sufficient act of
repatriation. Additionally, his active participation
in the 1987 congressional elections had divested
him of American citizenship under the laws of the
United States, thus restoring his Philippine
citizenship. He ended by reiterating his prayer for
the rejection of the move to disqualify him for
being time-barred under Section 253 of the
Omnibus Election Code. LLpr
Considering the importance and urgency of the
question herein raised, the Court has decided to
resolve it directly instead of allowing the normal
circuitous route that will after all eventually end
with this Court, albeit only after a long delay. We
cannot permit this delay. Such delay will be
inimical to the public interest and the vital
principles of public office to be here applied.
It is true that the Commission on Elections has
the primary jurisdiction over this question as the
sole judge of all contests relating to the election,
returns and qualifications of the members of the
Congress and elective provincial and city officials.
However, the decision on Frivaldo's citizenship
has already been made by the COMELEC through
its
counsel,
the
Solicitor
General,
who
categorically claims that Frivaldo is a foreigner.
We assume this stance was taken by him after
consultation with the public respondent and with
its approval. It therefore represents the decision
of the COMELEC itself that we may now review.
Exercising our discretion to interpret the Rules of
Court and the Constitution, we shall consider the
present petition as having been filed in
accordance with Article IX-A, Section 7, of the
Constitution, to challenge the aforementioned
Orders of the COMELEC.
The basic question we must resolve is whether or
not Juan G. Frivaldo was a citizen of the
Philippines at the time of his election on January
18,1988, as provincial governor of Sorsogon. All
the other issues raised in this petition are merely
secondary to this basic question.
The reason for this inquiry is the provision in
Article XI, Section 9, of the Constitution that all
public officials and employees owe the State and
the Constitution "allegiance at all times" and the
specific requirement in Section 42 of the Local
Government Code that a candidate for local
elective office must be inter alia a citizen of the
Philippines and a qualified voter of the
constituency where he is running. Section 117 of
19 | C O N S T I 2 _ A r t i c l e I V _ C I T I Z E N S H I P

the Omnibus Election Code provides that a


qualified
voter
must
be,
among
other
qualifications, a citizen of the Philippines, this
being an indispensable requirement for suffrage
under Article V, Section 1, of the Constitution.
In the certificate of candidacy he filed on
November 19, 1987, Frivaldo described himself as
a "natural-born" citizen of the Philippines,
omitting mention of any subsequent loss of such
status. The evidence shows, however, that he
was naturalized as a citizen of the United States
in 1983 per the following certification from the
United States District Court, Northern District of
California, as duly authenticated by Vice Consul
Amado P. Cortez of the Philippine Consulate
General in San Francisco, California, U.S.A.
OFFICE OF THE CLERK
UNITED STATES DISTRICT COURT
NORTHERN DISTRICT OF CALIFORNIA
September 23, 1988
TO WHOM IT MAY CONCERN:
Our records show that JUAN GALLANOSA
FRIVALDO, born on October 20, 1915, was
naturalized in this Court on January 20, 1983, and
issued Certificate of Naturalization No. 11690178.
Petition No. 280225.
Alien Registration No. A23 079 270.
Very truly yours,
WILLIAM L. WHITTAKER
Clerk
by:
(Sgd.)
ARACELI V. BARENG
Deputy Clerk
This evidence is not denied by the petitioner. In
fact, he expressly admitted it in his answer.
Nevertheless, as earlier noted, he claims it was
"forced" on him as a measure of protection from
the persecution of the Marcos government
through his agents in the United States. cdll
The Court sees no reason not to believe that the
petitioner was one of the enemies of the Marcos
dictatorship. Even so, it cannot agree that as a
consequence thereof he was coerced into
embracing American citizenship. His feeble
suggestion that his naturalization was not the
result of his own free and voluntary choice is
totally unacceptable and must be rejected
outright.
There were many other Filipinos in the United
States similarly situated as Frivaldo, and some of
them subject to greater risk than he, who did not
find it necessary nor do they claim to have
been coerced to abandon their cherished
status as Filipinos. They did not take the oath of
allegiance to the United States, unlike the
petitioner who solemnly declared "on oath, that I
absolutely and entirely renounce and abjure all
allegiance and fidelity to any foreign prince,
potentate, state or sovereignty of whom or which

I have heretofore been a subject or citizen,"


meaning in his case the Republic of the
Philippines. The martyred Ninoy Aquino heads the
impressive list of those Filipinos in exile who,
unlike the petitioner, held fast to their Philippine
citizenship despite the perils of their resistance to
the Marcos regime.

The Nottebohm case cited by the petitioner


invoked the international law principle of effective
nationality which is clearly not applicable to the
case at bar. This principle is expressed in Article 5
of the Hague Convention of 1930 on the Conflict
of Nationality Laws as follows: prcd
Art. 5. Within a third State a person having more
than one nationality shall be treated as if he had
only one. Without prejudice to the application of
its law in matters of personal status and of any
convention in force, a third State shall, of the
nationalities which any such person possesses,
recognize exclusively in its territory either the
nationality of the country in which he is habitually
and principally resident or the nationality of the
country with which in the circumstances he
appears to be in fact most closely connected.
Nottebohm was a German by birth but a resident
of Guatemala for 34 years when he applied for
and acquired naturalization in Liechtenstein one
month before the outbreak of World War II. Many
members of his family and his business interests
were in Germany. In 1943, Guatemala, which had
declared war on Germany, arrested Nottebohm
and confiscated all his properties on the ground
that he was a German national. Liechtenstein
thereupon filed suit on his behalf, as its citizen,
against Guatemala. The International Court of
Justice held Nottebohm to be still a national of
Germany, with which he was more closely
connected than with Liechtenstein. cdphil
That case is not relevant to the petition before us
because it dealt with a conflict between the
nationality laws of two states as decided by a
third state. No third state is involved in the case
at bar; in fact, even the United States is not
actively claiming Frivaldo as its national. The sole
question presented to us is whether or not
Frivaldo is a citizen of the Philippines under our
own laws, regardless of other nationality laws. We
can decide this question alone as sovereign of
our own territory, conformably to Section 1 of the
said Convention providing that "it is for each
State to determine under its law who are its
nationals."
It is also worth noting that Nottebohm was
invoking his naturalization in Liechtenstein
whereas in the present case Frivaldo is rejecting
his naturalization in the United States.
If he really wanted to disavow his American
citizenship and reacquire Philippine citizenship,
the petitioner should have done so in accordance
with the laws of our country. Under CA No. 63 as
amended by CA No. 473 and PD No. 725,
20 | C O N S T I 2 _ A r t i c l e I V _ C I T I Z E N S H I P

Philippine citizenship may be reacquired by direct


act of Congress, by naturalization, or by
repatriation.
While Frivaldo does not invoke either of the first
two methods, he nevertheless claims he has
reacquired Philippine citizenship by virtue of a
valid repatriation. He claims that by actively
participating in the elections in this country, he
automatically forfeited American citizenship
under the laws of the United States. Such laws do
not concern us here. The alleged forfeiture is
between him and the United States as his
adopted country. It should be obvious that even if
he did lose his naturalized American citizenship,
such forfeiture did not and could not have the
effect of automatically restoring his citizenship in
the Philippines that he had earlier renounced. At
best, what might have happened as a result of
the loss of his naturalized citizenship was that he
became a stateless individual.
Frivaldo's contention that he could not have
repatriated himself under LOI 270 because the
Special Committee provided for therein had not
yet been constituted seems to suggest that the
lack of that body rendered his repatriation
unnecessary. That is far-fetched if not specious.
Such a conclusion would open the floodgates, as
it were. It would allow all Filipinos who have
renounced this country to claim back their
abandoned citizenship without formally rejecting
their adopted state and reaffirming their
allegiance to the Philippines.
It does not appear that Frivaldo has taken these
categorical acts. He contends that by simply filing
his certificate of candidacy he had, without more,
already
effectively
recovered
Philippine
citizenship. But that is hardly the formal
declaration the law envisions surely, Philippine
citizenship previously disowned is not that
cheaply recovered. If the Special Committee had
not yet been convened, what that meant simply
was that the petitioner had to wait until this was
done, or seek naturalization by legislative or
judicial proceedings.
The argument that the petition filed with the
Commission on Elections should be dismissed for
tardiness is not well-taken. The herein private
respondents are seeking to prevent Frivaldo from
continuing to discharge his office of governor
because he is disqualified from doing so as a
foreigner. Qualifications for public office are
continuing requirements and must be possessed
not only at the time of appointment or election or
assumption of office but during the officer's entire
tenure. Once any of the required qualifications is
lost, his title may be seasonably challenged. If,
say, a female legislator were to marry a foreigner
during her term and by her act or omission
acquires his nationality, would she have a right to
remain in office simply because the challenge to
her title may no longer be made within ten days
from her proclamation? It has been established,
and not even denied, that the evidence of
Frivaldo's naturalization was discovered only

eight months after his proclamation and his title


was challenged shortly thereafter.
This Court will not permit the anomaly of a person
sitting as provincial governor in this country while
owing exclusive allegiance to another country.
The fact that he was elected by the people of
Sorsogon does not excuse this patent violation of
the salutary rule limiting public office and
employment only to the citizens of this country.
The qualifications prescribed for elective office
cannot be erased by the electorate alone. The will
of the people as expressed through the ballot
cannot cure the vice of ineligibility, especially if
they mistakenly believed, as in this case, that the
candidate was qualified. Obviously, this rule
requires strict application when the deficiency is
lack of citizenship. If a person seeks to serve in
the Republic of the Philippines, he must owe his
total loyalty to this country only, abjuring and
renouncing all fealty and fidelity to any other
state. LexLib
It is true as the petitioner points out that the
status of the natural-born citizen is favored by the
Constitution and our laws, which is all the more
reason why it should be treasured like a pearl of
great price. But once it is surrendered and
renounced, the gift is gone and cannot be lightly
restored. This country of ours, for all its
difficulties and limitations, is like a jealous and
possessive mother. Once rejected, it is not quick
to welcome back with eager arms its prodigal if
repentant children. The returning renegade must
show, by an express and unequivocal act, the
renewal of his loyalty and love.
WHEREFORE, the petition is DISMISSED and
petitioner JUAN G. FRIVALDO is hereby declared
not a citizen of the Philippines and therefore
DISQUALIFIED from serving as Governor of the
Province of Sorsogon. Accordingly, he is ordered
to vacate his office and surrender the same to the
duly elected Vice-Governor of the said province
once this decision becomes final and executory.
The temporary restraining order dated March 9,
1989, is LIFTED.
SO ORDERED.
Fernan (C .J .), Narvasa, Melencio-Herrera, Paras,
Feliciano, Gancayco, Padilla, Bidin, Grio-Aquino,
Medialdea and Regalado, JJ ., concur.
Sarmiento, J ., took no part.
Cortes, J ., concurs in the result.
(Republic v. De La Rosa, G.R. No. 104654,
105715, 105735, June 06, 1994)
EN BANC
[G.R. No. 104654. June 6, 1994.]
REPUBLIC OF THE PHILIPPINES, petitioner, vs.
HON. ROSALIO G. DE LA ROSA, PRESIDING JUDGE
OF THE REGIONAL TRIAL COURT, BRANCH 28,
MANILA and JUAN G. FRIVALDO, respondents.
[G.R. No. 105715. June 6, 1994.]
21 | C O N S T I 2 _ A r t i c l e I V _ C I T I Z E N S H I P

RAUL R. LEE, petitioner, vs. COMMISSION ON


ELECTIONS and JUAN G. FRIVALDO, respondents.
[G.R. No. 105735. June 6, 1994.]
RAUL R. LEE, petitioner, vs. COMMISSION ON
ELECTIONS and JUAN G. FRIVALDO, respondents.
The Solicitor General for petitioner in G.R. No.
104654.
Yolando F. Lim counsel for private respondent.
SYLLABUS
1.
CONSTITUTIONAL
LAW;
REVISED
NATURALIZATION
LAW;
PROCEDURAL
REQUIREMENTS, JURISDICTIONAL; CASE AT BAR.
Private respondent, having opted to reacquire
Philippine citizenship thru naturalization under
the Revised Naturalization Law, is duty bound to
follow the procedure prescribed by the said law. It
is not for an applicant to decide for himself and to
select the requirement which he believes, even
sincerely, are applicable to his case and discard
those which be believes are inconvenient or
merely of nuisance value. The law does not
distinguish between an applicant who was
formerly a Filipino citizen and one who was never
such a citizen. It does not provide a special
procedure for the reacquisition of Philippine
citizenship by former Filipino citizens akin to the
repatriation of a woman who had lost her
Philippine citizenship by reason of her marriage to
an alien. The trial court never acquired
jurisdiction to hear the petition for naturalization
of
private
respondent.
The
proceedings
conducted, the decision rendered and the oath of
allegiance taken therein, are null and void or
failure to comply with the publication and posting
requirements under the Revised Naturalization
Law. Under Section 9 of the said law, both the
petition for naturalization and the order setting it
for hearing must be published once a week for
three consecutive weeks in the Official Gazette
and a newspaper of general circulation.
Compliance therewith is jurisdictional (Po Yi Bo v.
Republic, 205 SCRA 400 [1992]). Moreover, the
publication and posting of the petition and the
order must be in its full text for the court to
acquire jurisdiction (Sy v. Republic, 55 SCRA 724
[1974]). The petition for naturalization lacks
several allegations required by Sections 2 and 6
of the Revised Naturalization Law, particularly: (1)
that the petitioner is of good moral character; (2)
that he resided continuously in the Philippines for
at least ten years; (3) that he is able to speak and
write English and any one of the principal
dialects; (4) that he will reside continuously in the
Philippines from the date of the filing of the
petition until his admission to Philippine
citizenship; and (5) that he has filed a declaration
of intention or if he is excused from said filing,
the justification therefor. The absence of such
allegations is fatal to the petition (Po Yi Bi v.
Republic, 205 SCRA 400 [1992]). Likewise the
petition is not supported by the affidavit of at
least two credible persons who vouched for the

good moral character of private respondent as


required by Section 7 of the Revised
Naturalization Law. Private respondent also failed
to attach a copy of his certificate of arrival to the
petition as required by Section 7 of the said law.
The proceedings of the trial court was marred by
the following irregularities: (1) the hearing of the
petition was set ahead of the scheduled date of
hearing, without a publication of the order
advancing the date of hearing, and the petition
itself; (2) the petition was heard within six
months from the last publication of the petition;
(3) petitioner was allowed to take his oath of
allegiance before the finality of the judgment;
and (4) petitioner took his oath of allegiance
without observing the two-year waiting period.
Private respondent is declared NOT a citizen of
the Philippines and therefore DISQUALIFIED from
continuing to serve as GOVERNOR of the Province
of Sorsogon. He is ordered to VACATE his office
and to SURRENDER the same to the ViceGovernor of the Province of Sorsogon once this
decision becomes final and executory.
2. ID.; ID.; DECISION THEREON BECOMES FINAL
ONLY
AFTER
THIRTY
(30)
DAYS
FROM
PROMULGATION. A decision in a petition for
naturalization becomes final only after 30 days
from its promulgation and, insofar as the Solicitor
General is concerned, that period is counted from
the date of his receipt of the copy of the decision
(Republic v. Court of First Instance of Albay, 60
SCRA 195 [1974]).
3. ID.; ID.; DECISION GRANTING NATURALIZATION
SHALL BE EXECUTORY AFTER TWO (2) FROM
PROMULGATION; REASON. Section 1 of R.A.
NO. 530 provides that no decision granting
citizenship in naturalization proceedings shall be
executory until after two years from its
promulgation in order to be able to observe if: (1)
the applicant has left the country; (2) the
applicant has dedicated himself continously to a
lawful calling or profession; (3) the applicant has
not been convicted of any offense or violation of
government promulgated rules; and (4) the
applicant has committed any act prejudicial to
the interest of the country or contrary to
government announced policies.
4. REMEDIAL LAW; SPECIAL CIVIL ACTION; QUO
WARRANTO; PETITION NOT COVERED BY THE TEN
(10) DAY PERIOD FOR APPEAL PRESCRIBED IN
SECTION 253 OF THE OMNIBUS ELECTION CODE.
In Frivaldo v. Commission on Elections, 174
SCRA 245 (1989), we held that a petition for quo
warranto, questioning the respondent's title and
seeking to prevent him from holding office as
Governor for alienage, is not covered by the tenday period for appeal prescribed in Section 253 of
the Omnibus Election Code.
5.
POLITICAL
LAW;
PUBLIC
OFFICE;
QUALIFICATIONS THEREON ARE CONTINUING
REQUIREMENTS.

We
explained
that
"qualifications for public office are continuing
requirements and must be possessed not only at
the time of appointment or election or
assumption of office but during the officer's entire
22 | C O N S T I 2 _ A r t i c l e I V _ C I T I Z E N S H I P

tenure; once any of the required qualification is


lost, his title may be seasonably challenged."
6. ID.; ID.; ONLY FILIPINO CITIZENS CAN RUN AND
BE ELECTED THERETO. Petitioner's argument,
that to unseat him will frustrate the will of the
electorate, is untenable. Both the Local
Government Code and the Constitution require
that only Filipino citizens can run and be elected
to public office. We can only surmise that the
electorate, at the time they voted for private
respondent, was of the mistaken belief that he
had legally reacquired Filipino citizenship.
7. ID.; ELECTIONS; WHERE THE CANDIDATE WHO
OBTAINED THE HIGHEST NUMBER OF VOTES IS
DISQUALIFIED, THE CANDIDATE WHO GARNERED
THE SECOND HIGHEST NUMBER OF VOTES IS NOT
ENTITLED TO BE DECLARED WINNER. Petitioner
in G.R. No. 105715, prays that the votes cast in
favor of private respondent be considered stray
and that he, being the candidate obtaining the
second highest number of votes, be declared
winner. In Labo, Jr. v. COMELEC , 176 SCRA 1
(1989), we ruled that where the candidate who
obtained the highest number of votes is later
declared to be disqualified to hold the office to
which he was elected, the candidate who
garnered the second highest number of votes is
not entitled to be declared winner (See also
Geronimo v. Ramos, 136 SCRA 435 [1985];
Topacio v. Paredes, 23 Phil. 238 [1912]).
DECISION
QUIASON, J p:
In Frivaldo v. Commission on Elections, 174 SCRA
245 (1989), this Court declared private
respondent, Juan G. Frivaldo, an alien and
therefore disqualified from serving as Governor of
the Province of the Sorsogon.
Once more, the citizenship of private respondent
is put in issue in these petitions docketed as G.R.
No. 104654, G.R. No. 105715 and G.R. No.
105735. The petitions were consolidated since
they principally involved the same issues and
parties. LibLex
I
G.R. No. 104654
This is a petition for certiorari under Rule 45 of
the Revised Rules of Court in relation to R.A. No.
5440 and Section 25 of the Interim Rules, filed by
the Republic of the Philippines: (1) to annul the
Decision dated February 27, 1992 of the Regional
Trial Court, Branch 28, Manila, in SP Proc. No. 9158645, which re-admitted private respondent as a
Filipino citizen under the Revised Naturalization
Law (C.A. No. 63 as amended by C.A. No. 473);
and (2) to nullify the oath of allegiance taken by
private respondent on February 27, 1992.
On September 20, 1991, petitioner filed a petition
for naturalization captioned: "In the Matter of
Petition of Juan G. Frivaldo to be Re-admitted as a
Citizen of the Philippines under COMMONWEALTH
ACT NO. 63" (Rollo, pp. 17-23).

In an Order dated October 7, 1991 respondent


Judge set the petition for hearing on March 16,
1992, and directed the publication of the said
order and petition in the Official Gazette and a
newspaper of general circulation, for three
consecutive weeks, the last publication of which
should be at least six months before the said date
of hearing. The order further required the posting
of a copy thereof and the petition in a
conspicuous place in the Office of the Clerk of
Court of the Regional Trial Court, Manila (Rollo,
pp. 24- 26).
On January 14, 1992, private respondent filed a
"Motion to Set Hearing Ahead of Schedule,"
where he manifested his intention to run for
public office in the May 1992 elections. He
alleged that the deadline for filing the certificate
of candidacy was March 15, one day before the
scheduled hearing. He asked that the hearing set
on March 16 be cancelled and be moved to
January 24 (Rollo, pp. 27-28).
The Motion was granted in an Order dated
January 24, 1992, wherein the hearing of the
petition was moved to February 21, 1992. The
said order was not published nor a copy thereof
posted. cdrep
On February 21, the hearing proceeded with
private respondent as the sole witness. He
submitted the following documentary evidence:
(1) Affidavit of Publication of the Order dated
October 7, 1991 issued by the publisher of The
Philippine Star (Exh. "A"); (2) Certificate of
Publication of the order issued by the National
Printing Office (Exh. "B"); (3) Notice of Hearing of
Petition (Exh. "B-1"); (4) Photocopy of a Citation
issued by the National Press Club with private
respondent's picture (Exhs. "C" and "C-2"); (5)
Certificate of Appreciation issued by the Rotary
Club of Davao (Exh. "D"); (6) Photocopy of a
Plaque of Appreciation issued by the Republican
College, Quezon City (Exh. "E"); (7) Photocopy of
a Plaque of Appreciation issued by the DavaoBicol Association (Exh. "F"); (8) Certification
issued by the Records Management and Archives
Office that the record of birth of private
respondent was not on file (Exh. "G"); and (9)
Certificate of Naturalization issued by the United
States District Court (Exh. "H").

Six days later, on February 27, respondent Judge


rendered the assailed Decision, disposing as
follows:
"WHEREFORE, the petition is GRANTED, Petitioner
JUAN G. FRIVALDO, is re-admitted as a citizen of
the Republic of the Philippines by naturalization,
thereby vesting upon him, all the rights and
privileges of a natural born Filipino citizen" (Rollo,
p. 33).

23 | C O N S T I 2 _ A r t i c l e I V _ C I T I Z E N S H I P

On the same day, private respondent was allowed


to take his oath of allegiance before respondent
Judge (Rollo, p. 34).
On March 16, a "Motion for Leave of Court to
Intervene
and
to
Admit
Motion
for
Reconsideration" was filed by Quiterio H. Hermo.
He alleged that the proceedings were tainted with
jurisdictional defects, and prayed for a new trial
to conform with the requirements of the
Naturalization Law.
After receiving a copy of the Decision on March
18, 1992, the Solicitor General interposed a
timely appeal directly with the Supreme Court.
G.R. No. 105715
This is a petition for certiorari, mandamus with
injunction under Rule 65 of the Revised Rules of
Court in relation to Section 5(2) of Article VIII of
the Constitution with prayer for temporary
restraining order filed by Raul R. Lee against the
Commission on Elections (COMELEC) and private
respondent, to annul the en banc Resolution of
the COMELEC, which dismissed his petition
docketed as SPC Case No. 92-273. The said
petition sought to annul the proclamation of
private respondent as Governor-elect of the
Province of Sorsogon.
Petitioner was the official candidate of the Laban
ng Demokratikong Pilipino (LDP) for the position
of governor of the Province of Sorsogon in the
May 1992 elections. Private respondent was the
official candidate of the Lakas-National Union of
Christian Democrats (Lakas-NUCD) for the same
position.
Private respondent was proclaimed winner on
May 22, 1992.
On June 1, petitioner filed a petition with the
COMELEC to annul the proclamation of private
respondent as Governor-elect of the Province of
Sorsogon on the grounds: (1) that the
proceedings and composition of the Provincial
Board of Canvassers were not in accordance with
law; (2) that private respondent is an alien,
whose grant of Philippine citizenship is being
questioned by the State in G.R. No. 104654; and
(3) that private respondent is not a duly
registered voter. Petitioner further prayed that
the votes cast in favor of private respondent be
considered as stray votes, and that he, on the
basis of the remaining valid votes cast, be
proclaimed winner. llcd
On June 10, the COMELEC issued the questioned
en banc resolution which dismissed the petition
for having been filed out of time, citing Section
19 of R.A. No. 7166. Said section provides that
the period to appeal a ruling of the board of
canvassers on questions affecting its composition
or proceedings was three days.
In this petition, petitioner argues that the
COMELEC acted with grave abuse of discretion
when it ignored the fundamental issue of private

respondent's disqualification in the guise of


technicality.
Petitioner claims that the inclusion of private
respondent's name in the list of registered voters
in Sta. Magdalena, Sorsogon was invalid because
at the time he registered as a voter in 1987, he
was an American citizen.
Petitioner further claims that the grant of Filipino
citizenship to private respondent is not yet
conclusive because the case is still on appeal
before us.
Petitioner prays for: (1) the annulment of private
respondent's proclamation as Governor of the
Province of Sorsogon; (2) the deletion of private
respondent's name from the list of candidates for
the position of governor; (3) the proclamation of
the governor-elect based on the remaining votes,
after the exclusion of the votes for private
respondent; (4) the issuance of a temporary
restraining order to enjoin private respondent
from taking his oath and assuming office; and (5)
the issuance of a writ of mandamus to compel
the
COMELEC
to
resolve
the
pending
disqualification case docketed as SPA Case No.
92-016, against private respondent. LLphil
G.R. No. 105735
This is a petition for mandamus under Rule 65 of
the Revised Rules of Court in relation to Section
5(2) of Article VIII of the Constitution, with prayer
for temporary restraining order. The parties
herein are identical with the parties in G.R. No.
105715.
In substance, petitioner prays for the COMELEC's
immediate resolution of SPA Case No. 92-016,
which is a petition for the cancellation of private
respondent's certificate of candidacy filed on
March 23, 1992 by Quiterio H. Hermo, the
intervenor in G.R. No. 104654 (Rollo, p. 18).
The petition for cancellation alleged: (1) that
private respondent is an American citizen, and
therefore ineligible to run as candidate for the
position of governor of the Province of Sorsogon;
(2) that the trial court's decision re-admitting
private respondent as a Filipino citizen was
fraught with legal infirmities rendering it null and
void; (3) that assuming the decision to be valid,
private respondent's oath of allegiance, which
was taken on the same day the questioned
decision was promulgated, violated Republic Act
No. 530, which provides for a two-year waiting
period before the oath of allegiance can be taken
by the applicant; and (4) that the hearing of the
petition on February 27, 1992, was held less than
four months from the date of the last publication
of the order and petition. The petition prayed for
the
cancellation
of
private
respondent's
certificate of candidacy and the deletion of his
name from the list of registered voters in Sta.
Magdalena, Sorsogon.
In his answer to the petition for cancellation,
private respondent denied the allegations therein
24 | C O N S T I 2 _ A r t i c l e I V _ C I T I Z E N S H I P

and averred: (1) that Quiterio H. Hermo, not


being a candidate for the same office for which
private respondent was aspiring, had no standing
to file the petition; (2) that the decision readmitting him to Philippine citizenship was
presumed to be valid; and (3) that no case had
been filed to exclude his name as a registered
voter. LLjur
Raul R. Lee intervened in the petition for
cancellation of private respondent's certificate of
candidacy (Rollo, p. 37).
On May 13, 1992, said intervenor urged the
COMELEC to decide the petition for cancellation,
citing Section 78 of the Omnibus Election Code,
which provides that all petitions on matters
involving the cancellation of a certificate of
candidacy must be decided "not later than fifteen
days before election," and the case of Alonto v.
Commission on Elections, 22 SCRA 878 (1968),
which
ruled
that
all
pre-proclamation
controversies should be summarily decided
(Rollo, p. 50).
The COMELEC concedes that private respondent
has not yet reacquired his Filipino citizenship
because the decision granting him the same is
not yet final and executory (Rollo, p. 63).
However, it submits that the issue of
disqualification of a candidate is not among the
grounds
allowed
in
a
pre-proclamation
controversy, like SPC Case No. 92-273. Moreover,
the said petition was filed out of time.
The COMELEC contends that the preparation for
the elections occupied much of its time, thus its
failure to immediately resolve SPA Case No. 92016. It argues that under Section 5 of Rule 25 of
the COMELEC Rules of Procedure, it is excused
from deciding a disqualification case within the
period provided by law for reasons beyond its
control. It also assumed that the same action was
subsequently abandoned by petitioner when he
filed before it a petition for quo warranto
docketed as EPC No. 92-35. The quo warranto
proceedings
sought
private
respondent's
disqualification
because
of
his
American
citizenship. LLjur
II
G.R. No. 104654
We shall first resolve the issue concerning private
respondent's citizenship.
In his comment to the State's appeal of the
decision granting him Philippine citizenship in
G.R. No. 104654, private respondent alleges that
the precarious political atmosphere in the country
during Martial Law compelled him to seek
political asylum in the United States, and
eventually to renounce his Philippine citizenship.
He claims that his petition for naturalization was
his only available remedy for his reacquisition of
Philippine citizenship. He tried to reacquire his
Philippine citizenship through repatriation and
direct act of Congress. However, he was later
informed that repatriation proceedings were

limited to army deserters or Filipino women who


had lost their citizenship by reason of their
marriage to foreigners (Rollo, pp. 49-50). His
request to Congress for sponsorship of a bill
allowing him to reacquire his Philippine
citizenship failed to materialize, notwithstanding
the endorsement of several members of the
House of Representatives in his favor (Rollo, p.
51). He attributed this to the maneuvers of his
political rivals.
He also claims that the re-scheduling of the
hearing of the petition to an earlier date, without
publication, was made without objection from the
Office of the Solicitor General. He makes mention
that on the date of the hearing, the court was
jampacked. LLphil
It is private respondent's posture that there was
substantial compliance with the law and that the
public was well-informed of his petition for
naturalization due to the publicity given by the
media.
Anent the issue of the mandatory two-year
waiting period prior to the taking of the oath of
allegiance, private respondent theorizes that the
rationale of the law imposing the waiting period is
to grant the public an opportunity to investigate
the background of the applicant and to oppose
the grant of Philippine citizenship if there is basis
to do so. In his case, private respondent alleges
that such requirement may be dispensed with,
claiming that his life, both private and public, was
well-known.
Private
respondent
cites
his
achievements as a freedom fighter and a former
Governor of the Province of Sorsogon for six
terms.
The appeal of the Solicitor General in behalf of
the Republic of the Philippines is meritorious. The
naturalization proceedings in SP Proc. No. 9158645 was full of procedural flaws, rendering the
decision an anomaly. LLphil

Private respondent, having opted to reacquire


Philippine citizenship thru naturalization under
the Revised Naturalization Law, is duty bound to
follow the procedure prescribed by the said law. It
is not for an applicant to decide for himself and to
select the requirements which he believes, even
sincerely, are applicable to his case and discard
those which be believes are inconvenient or
merely of nuisance value. The law does not
distinguish between an applicant who was
formerly a Filipino citizen and one who was never
such a citizen. It does not provide a special
procedure for the reacquisition of Philippine
citizenship by former Filipino citizens akin to the
repatriation of a woman who had lost her
Philippine citizenship by reason of her marriage to
an alien.
The trial court never acquired jurisdiction to hear
the petition for naturalization of private
respondent. The proceedings conducted, the
decision rendered and the oath of allegiance
25 | C O N S T I 2 _ A r t i c l e I V _ C I T I Z E N S H I P

taken therein, are null and void for failure to


comply with the publication and posting
requirements under the Revised Naturalization
Law.
Under Section 9 of the said law, both the petition
for naturalization and the order setting it for
hearing must be published once a week for three
consecutive weeks in the Official Gazette and a
newspaper of general circulation. Compliance
therewith is jurisdictional (Po Yi Bo v. Republic,
205 SCRA 400 [1992]). Moreover, the publication
and posting of the petition and the order must be
in its full text for the court to acquire jurisdiction
(Sy v. Republic, 55 SCRA 724 [1974]).
The petition for naturalization lacks several
allegations required by Sections 2 and 6 of the
Revised Naturalization Law, particularly: (1) that
the petitioner is of good moral character; (2) that
he resided continuously in the Philippines for at
least ten years; (3) that he is able to speak and
write English and any one of the principal
dialects; (4) that he will reside continuously in the
Philippines from the date of the filing of the
petition until his admission to Philippine
citizenship; and (5) that he has filed a declaration
of intention or if he is excused from said filing,
the justification therefor.
The absence of such allegations is fatal to the
petition (Po Yi Bi v. Republic, 205 SCRA 400
[1992]).
Likewise the petition is not supported by the
affidavit of at least two credible persons who
vouched for the good moral character of private
respondent as required by Section 7 of the
Revised Naturalization Law. Private respondent
also failed to attach a copy of his certificate of
arrival to the petition as required by Section 7 of
the said law. LLphil
The proceedings of the trial court was marred by
the following irregularities: (1) the hearing of the
petition was set ahead of the scheduled date of
hearing, without a publication of the order
advancing the date of hearing, and the petition
itself; (2) the petition was heard within six
months from the last publication of the petition;
(3) petitioner was allowed to take his oath of
allegiance before the finality of the judgment;
and (4) petitioner took his oath of allegiance
without observing the two-year waiting period.
A decision in a petition for naturalization becomes
final only after 30 days from its promulgation
and, insofar as the Solicitor General is concerned,
that period is counted from the date of his receipt
of the copy of the decision (Republic v. Court of
First Instance of Albay, 60 SCRA 195 [1974]).
Section 1 of R.A. NO. 530 provides that no
decision granting citizenship in naturalization
proceedings shall be executory until after two
years from its promulgation in order to be able to
observe if: (1) the applicant has left the country;
(2) the applicant has dedicated himself
continously to a lawful calling or profession; (3)

the applicant has not been convicted of any


offense or violation of government promulgated
rules; and (4) the applicant has committed any
act prejudicial to the interest of the country or
contrary to government announced policies. prcd
Even discounting the provisions of R.A. NO. 530,
the courts cannot implement any decision
granting the petition for naturalization before its
finality.
G.R. No. 105715
In view of the finding in G.R. No. 104654 that
private respondent is not yet a Filipino citizen, we
have to grant the petition in G.R. No. 105715
after treating it as a petition for certiorari instead
of a petition for mandamus. Said petition assails
the en banc resolution of the COMELEC,
dismissing SPC Case No. 92-273, which in turn is
a petition to annul private respondent's
proclamation on three grounds: 1) that the
proceedings and composition of the Provincial
Board of Canvassers were not in accordance with
law; 2) that private respondent is an alien, whose
grant of Filipino citizenship is being questioned by
the State in G.R. No. 104654; and 3) that private
respondent is not a duly registered voter. The
COMELEC dismissed the petition on the grounds
that it was filed outside the three-day period for
questioning the proceedings and composition of
the Provincial Board of Canvassers under Section
19 of R.A. No. 7166. prcd
The COMELEC failed to resolve the more serious
issue the disqualification of private respondent
to be proclaimed Governor on grounds of lack of
Filipino citizenship. In this aspect, the petition is
one for quo warranto. In Frivaldo v. Commission
on Elections, 174 SCRA 245 (1989), we held that
a petition for quo warranto, questioning the
respondent's title and seeking to prevent him
from holding office as Governor for alienage, is
not covered by the ten-day period for appeal
prescribed in Section 253 of the Omnibus Election
Code.
Furthermore,
we
explained
that
"qualifications for public office are continuing
requirements and must be possessed not only at
the time of appointment or election or
assumption of office but during the officer's entire
tenure; once any of the required qualification is
lost, his title may be seasonably challenged."
Petitioner's argument, that to unseat him will
frustrate the will of the electorate, is untenable.
Both the Local Government Code and the
Constitution require that only Filipino citizens can
run and be elected to public office. We can only
surmise that the electorate, at the time they
voted for private respondent, was of the mistaken
belief that he had legally reacquired Filipino
citizenship.
Petitioner in G.R. No. 105715, prays that the
votes cast in favor of private respondent be
considered stray and that he, being the candidate
obtaining the second highest number of votes, be
declared winner. In Labo, Jr. v. COMELEC , 176
SCRA 1 (1989), we ruled that where the
candidate who obtained the highest number of
26 | C O N S T I 2 _ A r t i c l e I V _ C I T I Z E N S H I P

votes is later declared to be disqualified to hold


the office to which he was elected, the candidate
who garnered the second highest number of
votes is not entitled to be declared winner (See
also Geronimo v. Ramos, 136 SCRA 435 [1985];
Topacio v. Paredes, 23 Phil. 238 [1912]). prLL
G.R. No. 105735
In view of the discussions of G.R. No. 104654 and
G.R. No. 105715, we find the petition in G.R. No.
105735 moot and academic.
WHEREFORE, the petitions in G.R. No. 104654
and G.R. No. 105715 are both GRANTED while the
petition in G.R. No. 105735 is DISMISSED. Private
respondent is declared NOT a citizen of the
Philippines and therefore DISQUALIFIED from
continuing to serve as GOVERNOR of the Province
of Sorsogon. He is ordered to VACATE his office
and to SURRENDER the same to the ViceGovernor of the Province of Sorsogon once this
decision becomes final and executory. No
pronouncement as to costs. cdll
SO ORDERED.
Feliciano, Padilla, Bidin, Regalado, Davide, Jr.,
Romero, Bellosillo, Melo, Puno, Vitug and
Kapunan, JJ., concur.
Narvasa, C .J ., took no part. Related to a party.
Cruz, J., took no part. Related to one of the
counsel in the proceedings before the COMELEC.
(Labo, Jr. v. COMELEC, G.R. No. 86564,
August 01, 1989)
EN BANC
[G.R. No. 86564. August 1, 1989.]
RAMON L. LABO, JR., petitioner, vs. THE
COMMISSION ON ELECTIONS (COMELEC) EN BANC
AND LUIS LARDIZABAL, respondents.
Estelito P. Mendoza for petitioner.
Rillera and Quintana for private respondent.
SYLLABUS
1. SPECIAL CIVIL ACTION; QUO WARRANTO;
PETITION FILED TIMELY. The Court has
considered the arguments of the parties and
holds that the petition for quo warranto was filed
on time. We agree with the respondents that the
fee was paid during the ten-day period as
extended by the pendency of the petition when it
was treated by the COMELEC as a preproclamation proceeding which did not require
the payment of a filing fee. At that, we reach this
conclusion only on the assumption that the
requirement for the payment of the fees in quo
warranto proceedings was already effective.
There is no record that Res. No. 1450 was even
published; and as for Res. No. 1996, this took
effect only on March 3, 1988, seven days after its
publication in the February 25, 1988 issues of the
Manila Chronicle and the Philippine Daily Inquirer,
or after the petition was filed.

2. ID.; ID.; PAYMENT OF FILING FEES NECESSARY


FOR CONFERMENT OF JURISDICTION; COURT MAY
ALLOW PAYMENT WITHIN A REASONABLE TIME.
It is true that in the Manchester Case, we
required the timely payment of the filing fee as a
precondition for the timeliness of the filing of the
case itself. In Sun Insurance Office, Ltd. v.
Asuncion, however, this Court, taking into
account the special circumstances of that case,
reiterated the rule that the trial court acquires
jurisdiction over a case only upon the payment of
the prescribed filing fee. However, this court may
allow the payment of the said fee within a
reasonable time. In the event of non-compliance
therewith, the case shall be dismissed. The same
idea is expressed in Rule 42, Section 18, of the
COMELEC Rules of Procedure adopted on June 20,
1988.
3. CIVIL PROCEDURE; ACTION; RESOLUTION ON
THE MERITS INSTEAD OF REMANDING THE CASE
TO
THE
TRIAL
COURT
FOR
FURTHER
PROCEEDINGS; AS DEMANDED BY THE DICTATES
OF JUSTICE. Remand of the case to the lower
court for further reception of evidence is not
necessary where the court is in a position to
resolve the dispute based on the records before
it. On many occasions, the Court, in the public
interest and the expeditious administration of
justice, has resolved actions on the merits instead
of remanding them to the trial court for further
proceedings, such as where the ends of justice
would not be subserved by the remand of the
case or when public interest demands an early
disposition of the case or where the trial court
had already received all the evidence of the
parties.
4. ID.; ID.; DOCTRINE OF RES JUDICATA NOT
APPLICABLE TO QUESTIONS OF CITIZENSHIP;
DEFENSE TO BE SEASONABLY INVOKED. There
is also the claim that the decision can no longer
be reversed because of the doctrine of res
judicata, but this too must be dismissed. This
doctrine does not apply to questions of
citizenship, as the Court has ruled in several
cases. Moreover, it does not appear that it was
properly and seasonably pleaded, in a motion to
dismiss or in the answer, having been invoked
only when the petitioner filed his reply to the
private respondent's comment. Besides, one of
the requisites of res judicata, to wit, identity of
parties, is not present in this case.
5.
POLITICAL
LAW;
NATURALIZATION;
ACQUISITION
OF
CITIZENSHIP
BY
NATURALIZATION. The petitioner's contention
that his marriage to an Australian national in
1976 did not automatically divest him of
Philippine citizenship is irrelevant. There is no
claim or finding that he automatically ceased to
be a Filipino because of that marriage. He
became a citizen of Australia because he was
naturalized as such through a formal and positive
process, simplified in his case because he was
married to an Australian citizen. As a condition for
such naturalization, he formally took the Oath of
27 | C O N S T I 2 _ A r t i c l e I V _ C I T I Z E N S H I P

Allegiance and/or
Allegiance.

made

the

Affirmation

of

6. ID.; CITIZENSHIP; MODES OF LOSING


PHILIPPINE CITIZENSHIP. CA No. 63 enumerates
the modes by which Philippine citizenship may be
lost. Among these are: (1) naturalization in a
foreign country; (2) express renunciation of
citizenship; and (3) subscribing to an oath of
allegiance to support the Constitution or laws of a
foreign country, all of which are applicable to the
petitioner. It is also worth mentioning in this
connection that under Article IV, Section 5, of the
present Constitution, "Dual allegiance of citizens
is inimical to the national interest and shall be
dealt with by law."
7. ID.; ID.; ANNULMENT OF MARRIAGE TO A
FOREIGNER NOT AN AUTOMATIC RESTORATION
OF PHILIPPINE CITIZENSHIP. Even if it be
assumed that, as the petitioner asserts, his
naturalization in Australia was annulled after it
was found that his marriage to the Australian
citizen was bigamous, that circumstance alone
did not automatically restore his Philippine
citizenship.
His
divestiture
of
Australian
citizenship does not concern us here. That is a
matter between him and his adopted country.
What we must consider is the fact that he
voluntarily
and
freely
rejected
Philippine
citizenship and willingly and knowingly embraced
the citizenship of a foreign country. The possibility
that he may have been subsequently rejected by
Australia, as he claims, does not mean that he
has been automatically reinstated as a citizen of
the Philippines.
8. ID.; ID.; MEANS OF REACQUIRING PHILIPPINE
CITIZENSHIP. Under CA No. 63 as amended by
PD No. 725, Philippine citizenship may be
reacquired by direct act of Congress, by
naturalization, or by repatriation. It does not
appear in the record, nor does the petitioner
claim, that he has reacquired Philippine
citizenship by any of these methods.
9. ID.; ID.; LACK OF PHILIPPINE CITIZENSHIP ON
THE DAY OF LOCAL ELECTIONS; GROUND FOR
DISQUALIFICATION AS A CANDIDATE FOR MAYOR.
The petitioner is not now, nor was he on the
day of the local elections on January 18, 1988, a
citizen of the Philippines. In fact, he was not even
a qualified voter under the Constitution itself
because of his alienage. He was therefore
ineligible as a candidate for mayor of Baguio City
under Section 42 of the Local Government Code.
10.
ID.;
ELECTION;
QUALIFICATIONS
OF
CANDIDATE FOR PUBLIC OFFICE, CONTENDING
REQUIREMENTS. The probability that many of
those who voted for the petitioner may have
done so in the belief that he was qualified only
strengthens the conclusion that the results of the
election cannot nullify the qualifications for the
office now held by him. These qualifications are
continuing requirements; once any of them is lost
during incumbency, title to the office itself is
deemed forfeited. In the case at bar, the
citizenship and voting requirements were not

subsequently lost but were not possessed at all in


the first place on the day of the election. The
petitioner was disqualified from running as mayor
and, although elected, is not now qualified to
serve as such.
11. ID.; ELECTION; CANDIDATE OBTAINING THE
SECOND HIGHEST NUMBER OF VOTES; NOT
QUALIFIED TO REPLACE THE DISQUALIFIED
CANDIDATE; SANTOS RULING REVERSED.
Finally, there is the question of whether or not the
private respondent, who filed the quo warranto
petition, can replace the petitioner as mayor. He
cannot. The simple reason is that as he obtained
only the second highest number of votes in the
election, he was obviously not the choice of the
people of Baguio City. Re-examining Santos v.
Commission on Election, 137 SCRA 740 the Court
finds, and so holds, that it should be reversed in
favor of the earlier case of Geronimo v. Ramos,
which represents the more logical and democratic
rule. There the Court held it would be extremely
repugnant to the basic concept of the
constitutionally guaranteed right to suffrage if a
candidate who has not acquired the majority or
plurality of votes is proclaimed a winner and
imposed as the representative of a constituency,
the majority of which have positively declared
through their ballots that they do not choose him.
DECISION
CRUZ, J p:
The petitioner asks this Court to restrain the
Commission on Elections from looking into the
question of his citizenship as a qualification for
his office as Mayor of Baguio City. The allegation
that he is a foreigner, he says, is not the issue.
The issue is whether or not the public respondent
has jurisdiction to conduct any inquiry into this
matter, considering that the petition for quo
warranto against him was not filed on time. cdphil
It is noteworthy that this argument is based on
the alleged tardiness not of the petition itself but
of the payment of the filing fee, which the
petitioner contends was an indispensable
requirement. The fee is, curiously enough, all of
P300.00 only. This brings to mind the popular
verse that for want of a horse the kingdom was
lost. Still, if it is shown that the petition was
indeed filed beyond the reglementary' period,
there is no question that this petition must be
granted and the challenge abated.
The petitioner's position is simple. He was
proclaimed mayor-elect of Baguio City on January
20, 1988. The petition for quo warranto was filed
by the private respondent on January 26,1988,
but no filing fee was paid on that date. This fee
was finally paid on February 10, 1988, or twentyone days after his proclamation. As the petition
by itself alone was ineffectual without the filing
fee, it should be deemed filed only when the fee
was paid. This was done beyond the
reglementary period provided for under Section
253 of the Omnibus Election Code reading as
follows:
SEC. 253. Petition for quo warranto. Any voter
contesting the election of a Member of the
28 | C O N S T I 2 _ A r t i c l e I V _ C I T I Z E N S H I P

Batasang Pambansa, regional, provincial, or city


officer on the ground of ineligibility or of
disloyalty to the Republic of the Philippines shall
files sworn petition for quo warranto with the
Commission
within
ten
days
after
the
proclamation of the result of the election.

The petitioner adds that the payment of the filing


fee is required under Rule 36, Section 5, of the
Procedural Rules of the COMELEC providing that

Sec. 5. No petition for quo warranto shall be given


due course without the payment of a filing fee in
the amount of Three Hundred Pesos (P300.00)
and the legal research fee as required by law.
and stresses that there is abundant jurisprudence
holding that the payment of the filing fee is
essential to the timeliness of the filing of the
petition itself. He cites many rulings of the Court
to this effect, specifically Manchester v. Court of
Appeals. 1
For his part, the private respondent denies that
the filing fee was paid out of time. In fact, he
says, it was filed ahead of time. His point is that
when he filed his "Petition for Quo Warranto with
Prayer for Immediate Annulment of Proclamation
and Restraining Order or Injunction" on January
26, 1988, the COMELEC treated it as a preproclamation controversy and docketed it as SPC
Case No. 88-288. No docket fee was collected
although it was offered. It was only on February 8,
1988, that the COMELEC decided to treat his
petition as solely for quo warranto and redocketed it as EPC Case No. 88-19, serving him
notice on February 10, 1988. He immediately paid
the filing fee on that date.
The private respondent argues further that during
the period when the COMELEC regarded his
petition as a pre-proclamation controversy, the
time for filing an election protest or quo warranto
proceeding was deemed suspended under
Section 248 of the Omnibus Election Code. 2 At
any rate, he says, Rule 36, Section 5, of the
COMELEC Rules of Procedure cited by the
petitioner, became effective only on November
15, 1988, seven days after publication of the said
Rules in the Official Gazette pursuant to Section
4, Rule 44 thereof. 3 These rules could not
retroact to January 26,1988, when he filed his
petition with the COMELEC.
In his Reply, the petitioner argues that even if the
Omnibus Election Code did not require it, the
payment of filing fees was still necessary under
Res. No. 1996 and, before that, Res. No. 1450 of
the respondent COMELEC, promulgated on
January 12, 1988, and February 26, 1980,
respectively. To this, the private respondent
counters that the latter resolution was intended
for the local elections held on January 30, 1980,
and did not apply to the 1988 local elections,
which were supposed to be governed by the firstmentioned resolution. However, Res. No. 1996
took effect only on March 3, 1988, following the
lapse of seven days alter its publication as
required by RA No. 6646, otherwise known as the
Electoral Reform Law of 1987, which became

effective on January 5,1988. Its Section 30


provides in part:
Sec. 30. Effectivity of Regulations and Orders of
the Commission. The rules and regulations
promulgated by the Commission shall take effect
on the seventh day after their publication in the
Official Gazette or in at least (2) daily newspapers
of general circulation in the Philippines.
The Court has considered the arguments of the
parties and holds that the petition for quo
warranto was filed on time. We agree with the
respondents that the fee was paid during the tenday period as extended by the pendency of the
petition when it was treated by the COMELEC as a
pre-proclamation proceeding which did not
require the payment of a filing fee. At that, we
reach this conclusion only on the assumption that
the requirement for the payment of the fees in
quo warranto proceedings was already effective.
There is no record that Res. No. 1450 was even
published; and as for Res. No. 1996, this took
effect only on March 3, 1988, seven days after its
publication in the February 25, 1988 issues of the
Manila Chronicle and the Philippine Daily Inquirer,
or after the petition was filed. cdasia
The petitioner forgets Taada v. Tuvera 4 when he
argues that the resolutions became effective
"immediately upon approval" simply because it
was so provided therein. We held in that case that
publication was still necessary under the due
process clause despite such effectivity clause.
In any event, what is important is that the filing
fee was paid, and whatever delay there may have
been is not imputable to the private respondent's
fault or neglect. It is true that in the Manchester
Case, we required the timely payment of the
filing fee as a precondition for the timeliness of
the filing of the case itself. In Sun Insurance
Office, Ltd. v. Asuncion, 5 however, this Court,
taking into account the special circumstances of
that case, declared:
This Court reiterates the rule that the trial court
acquires jurisdiction over a case only upon the
payment of the prescribed filing fee. However,
the court may allow the payment of the said fee
within a reasonable time. In the event of noncompliance therewith, the case shall be
dismissed.
The same idea is expressed in Rule 42, Section
18, of the COMELEC Rules of Procedure adopted
on June 20, 1988, thus:
Sec. 18. Non-payment of prescribed fees. If the
fees above prescribed are not paid, the
Commission may refuse to take action thereon
until they are paid and may dismiss the action or
the proceeding. (Emphasis supplied.)
The Court notes that while arguing the technical
point that the petition for quo warranto should be
dismissed for failure to pay the filing fee on time,
the petitioner would at the same time minimize
his alleged lack of citizenship as "a futile
technicality." It is regrettable, to say the least,
that the requirement of citizenship as a
qualification for public office can be so
demeaned. what is worse is that it is regarded as
29 | C O N S T I 2 _ A r t i c l e I V _ C I T I Z E N S H I P

an even less important consideration than the


reglementary period the petitioner insists upon.
This matter should normally end here as the sole
issue originally raised by the petitioner is the
timeliness of the quo warranto proceedings
against him. However, as his citizenship is the
subject of that proceeding, and considering the
necessity for an early resolution of that more
important question clearly and urgently affecting
the public interest, we shall directly address it
now in this same action.
The Court has similarly acted in a notable number
of cases, thus:
From the foregoing brief statement of the nature
of the instant case, it would appear that our sole
function in this proceeding should be to resolve
the single issue of whether or not the Court of
Appeals erred in ruling that the motion for new
trial of the GSIS in question should indeed be
deemed pro forma. But going over the extended
pleadings of both parties, the Court is
immediately impressed that substantial justice
may not be timely achieved, if we should decide
this case upon such a technical ground alone. We
have carefully read all the allegations and
arguments of the parties, very ably and
comprehensively
expounded
by
evidently
knowledgeable and unusually competent counsel,
and we feel we can better serve the interests of
justice by broadening the scope of our inquiry, for
as the record before us stands, we see that there
is enough basis for us to end the basic
controversy between the parties here and now,
dispensing, however, with procedural steps which
would not anyway affect substantially the merits
of their respective claims. 6
xxx xxx xxx
While it is the fault of the petitioner for appealing
to the wrong court and thereby allowing the
period for appeal to lapse, the more correct
procedure was for the respondent court to
forward the case to the proper court which was
the Court of Appeals for appropriate action.
Considering, however, the length of time that this
case has been pending, we apply the rule in the
case of Del Castillo v. Jaymalin, (112 SCRA 629)
and follow the principle enunciated in Alger
Electric, Inc. v. Court of Appeals, (135 SCRA 37)
which states:
". . . it is a cherished rule of procedure for this
Court to always strive to settle the entire
controversy in a single proceeding leaving no root
or branch to bear the seeds of future litigation.
No useful purpose will be served if this case is
remanded to the trial court only to have its
decision raised again to the Intermediate
Appellate Court and from there to this Court." (p.
43)
Only recently in the case of Beautifont, Inc., et al.
v. Court of Appeals, et al. (G.R. No. 50141,
January 29,1988), we stated that:
". . . But all those relevant facts are now before
this Court. And those facts dictate the rendition of
a verdict in the petitioner's favor. There is

therefore no point in referring the case back to


the Court of Appeals. The facts and the legal
propositions involved will not change, nor should
the ultimate judgment. Considerable time has
already elapsed and, to serve the ends of justice,
it is time that the controversy is finally laid to
rest. (See Sotto v. Samson, 5 SCRA 733; Republic
v. Paredes, 108 Phil. 57; Lianga Lumber Co. v.
Lianga Timber Co., Inc., 76 SCRA 197; Erico v.
Heirs of Chigas, 98 SCRA 575; Francisco v. City of
Davao, 12 SCRA 628; Valencia v. Mabilangan, 105
Phil. 162). 'Sound practice seeks to accommodate
the theory which avoids waste of time, effort and
expense, both to the parties and the government,
not to speak of delay in the disposal of the case
(of: Fernandez v. Garcia, 92 Phil. 592, 597). A
marked characteristic of our judicial set-up is that
where the dictates of justice so demand . . . the
Supreme Court should act, and act with finality.'
(Li Siu Liat v. Republic, 21 SCRA 1039, 1046,
citing Samal v. CA, 99 Phil. 230 and U.S. v.
Gimenez, 34 Phil. 74). In this case, the dictates of
justice do demand that this Court act, and act
with finality." 7
xxx xxx xxx
Remand of the case to the lower court for further
reception of evidence is not necessary where the
court is in a position to resolve the dispute based
on the records before it. On many occasions, the
Court, in the public interest and the expeditious
administration of justice, has resolved actions on
the merits instead of remanding them to the trial
court for further proceedings, such as where the
ends of justice would not be subserved by the
remand of the case or when public interest
demands an early disposition of the case or
where the trial court had already received all the
evidence of the parties. 8

This course of action becomes all the more


justified in the present case where, to repeat for
stress, it is claimed that a foreigner is holding a
public office.
We also note in his Reply, the petitioner says:
In adopting private respondent's comment,
respondent COMELEC implicitly adopted as "its
own" private respondent's repeated assertion
that petitioner is no longer a Filipino citizen. In so
doing, has not respondent COMELEC effectively
disqualified itself, by reason of prejudgment, from
resolving the petition for quo warranto filed by
private respondent still pending before it? 9
This is still another reason why the Court has
seen fit to rule directly on the merits of this case.
Going over the record, we find that there are two
administrative decisions on the question of the
petitioner's citizenship. The first was rendered by
the Commission on Elections on May 12, 1982,
and found the petitioner to be a citizen of the
Philippines. 10 The second was rendered by the
Commission on Immigration and Deportation on
September 13, 1988, and held that the petitioner
was not a citizen of the Philippines. 11
30 | C O N S T I 2 _ A r t i c l e I V _ C I T I Z E N S H I P

The first decision was penned by then COMELEC


Chairman
Vicente
Santiago,
Jr.,
with
Commissioners Pabalate, Savellano and Opinion
concurring in full and Commissioner Bacungan
concurring in the dismissal of the petition
"without prejudice to the issue of the
respondent's citizenship being raised a new in a
proper case. "Commissioner Sagadraca reserved
his vote, while Commissioner Felipe was for
deferring decision until representations shall have
been made with the Australian Embassy for
official verification of the petitioner's alleged
naturalization as an Australian.
The second decision was unanimously rendered
by Chairman Miriam Defensor-Santiago and
Commissioners Alano and Geraldez of the
Commission on Immigration and Deportation.
prLL
It is important to observe that in the proceeding
before the COMELEC, there was no direct proof
that the herein petitioner had been formally
naturalized as a citizen of Australia. This
conjecture, which was eventually rejected, was
merely inferred from the fact that he had married
an Australian citizen, obtained an Australian
passport, and registered as en alien with the CID
upon his return to this country in 1980.
On the other hand, the decision of the CID took
into account the official statement of the
Australian Government dated August 12, 1984,
through its Consul in the Philippines, that the
petitioner was still an Australian citizen as of that
date by reason of his naturalization in 1976. That
statement 12 is reproduced in full as follows:
I, GRAHAM COLIN WEST, Consul of Australia in the
Philippines, by virtue of a certificate of
appointment signed and sealed by the Australian
Minister of State for Foreign Affairs on 19 October
1983, and recognized as such by Letter of Patent
signed and sealed by the Philippines Acting
Minister of Foreign Affairs on 23 November 1983,
do hereby provide the following statement in
response to the Subpoena Testificandum dated 9
April 1984 in regard to the Petition for
disqualification against RAMON LABO, JR. Y
LOZANO (SPC No. 84-73). and do hereby certify
that the statement is true and correct.
STATEMENT
A) RAMON LABO, JR. Y LOZANO, date of birth 23
December 1934, was married in the Philippines to
an Australian citizen. As the spouse of an
Australian citizen, he was not required to meet
normal requirements for the grant of citizenship
and was granted Australian citizenship by Sydney
on 28 July 1976.
B) Any person over the age of 16 years who is
granted Australian citizenship must take an oath
of allegiance or make an affirmation of allegiance.
The wording of the oath of affirmation is: "I. . . .,
renouncing all other allegiance . . . .," etc. This
need not necessarily have any effect on his
former nationality as this would depend on the
citizenship laws of his former country.
C) The marriage was declared void in the
Australian Federal Court in Sydney on 27 June

1980 on the ground that the marriage had been


bigamous.
D) According to our records LABO is still an
Australian citizen.
E) Should he return to Australia, LABO may face
court action in respect of Section 50 of Australian
Citizenship Act 1948 which relates to the giving of
false or misleading information of a material
nature in respect of an application for Australian
citizenship. If such a prosecution was successful,
he could be deprived of Australian citizenship
under Section 21 of the Act.
F) There are two further ways in which LABO
could divest himself of Australian citizenship:
(i) He could make a declaration of Renunciation of
Australian citizenship under Section 18 of the
Australian Citizenship Act, or
(ii) If he acquired another nationality, (for
example, Filipino) by a formal end voluntary act
other than marriage, then he would automatically
lose his Australian citizenship under Section 17 of
the Act.
IN WITNESS WHEREOF, I HAVE HEREUNTO SET
MAY HAND AND SEAL OF THE AUSTRALIAN
EMBASSY, MANILA, THIS 12TH DAY OF APRIL
1984.
DONE AT MANILA IN THE PHILIPPINES.
(Signed)
GRAHAM C. WEST
Consul
This was affirmed later by the letter of February
1, 1988, addressed to the private respondent by
the Department of Foreign Affairs reading as
follows: 13
Sir:
With reference to your letter dated 1 February
1988, I wish to inform you that inquiry made with
the Australian Government through the Embassy
of the Philippines in Canberra has elicited the
following information:
1) That Mr. Ramon L. Labo, Jr. acquired Australian
citizenship on 28 July 1976.
2) That prior to 17 July 1986, a candidate for
Australian citizenship had to either swear an oath
of allegiance or make an affirmation of allegiance
which carries a renunciation of "all other
allegiance."
Very truly yours,
For the Secretary of Foreign Affairs:
(SGD) RODOLFO SEVERINO, JR
Assistant Secretary
31 | C O N S T I 2 _ A r t i c l e I V _ C I T I Z E N S H I P

The decision also noted the oath of allegiance


taken by every naturalized Australian reading as
follows:
OATH OF ALLEGIANCE
I, A.B., renouncing all other allegiance, swear by
Almighty God that I will be faithful and bear true
allegiance to Her Majesty Elizabeth the Second,
Queen of Australia, Her heirs and successors
according to law, and that I will faithfully observe
the laws of Australia and fulfill my duties as an
Australian citizen. 14
and the Affirmation of Allegiance, which declares:
AFFIRMATION OF ALLEGIANCE
I, A.B., renouncing all other allegiance, solemnly
and sincerely promise and declare that I will be
faithful and bear true allegiance to Her Majesty
Elizabeth the Second, Queen of Australia, Her
heirs and successors according to law, and that I
will faithfully observe the Laws of Australia and
fulfill my duties as an Australian citizen. 15
The petitioner does not question the authenticity
of the above evidence. Neither does he deny that
he obtained Australian Passport No. 754705,
which he used in coming back to the Philippines
in 1980, when he declared before the
immigration authorities that he was an alien and
registered as such under Alien Certificate of
Registration No. B-323985. 16 He later asked for
the change of his status from immigrant to a
returning former Philippine citizen and was
granted Immigrant Certificate of Residence No.
223809. 17 He also categorically declared that he
was a citizen of Australia in a number of sworn
statements voluntarily made by him and. even
sought to avoid the jurisdiction of the barangay
court on the ground that he was a foreigner. 18
The decision of the COMELEC in 1982 quaintly
dismisses all these acts as "mistakes" that did not
divest the petitioner of his citizenship, although,
as earlier noted, not all the member joined in this
finding. We reject this ruling as totally baseless.
The petitioner is not an unlettered person who
was not aware of the consequences of his acts,
let alone the fact that he was assisted by counsel
when he performed these acts.
The private respondent questions the motives of
the COMELEC at that time and stresses Labo's
political affiliation with the party in power then,
but we need not go into that now.
There is also the claim that the decision can no
longer be reversed because of the doctrine of res
judicata, but this too must be dismissed. This
doctrine does not apply to questions of
citizenship, as the Court has ruled in several
cases. 19 Moreover, it does not appear that it was
properly and seasonably pleaded, in a motion to
dismiss or in the answer, having been invoked
only when the petitioner filed his reply 20 to the
private respondent's comment. Besides, one of
the requisites of res judicata, to wit, identity of
parties, is not present in this case.
The petitioner's contention that his marriage to
an Australian national in 1976 did not
automatically divest him of Philippine citizenship
is irrelevant. There is no claim or finding that he
automatically ceased to be a Filipino because of
that marriage. He became a citizen of Australia
because he was naturalized as such through a

formal and positive process, simplified in his case


because he was married to an Australian citizen.
As a condition for such naturalization, he formally
took the Oath of Allegiance and/or made the
Affirmation of Allegiance, both quoted above.
Renouncing all other allegiance, he swore "to be
faithful and bear true allegiance to Her Majesty
Elizabeth the Second, Queen of Australia. . . . ,
and to fulfill his duties as an Australian citizen."
cdll
The petitioner now claims that his naturalization
in Australia made him at worst only a dual
national and did not divest him of his Philippine
citizenship. Such a specious argument cannot
stand against the clear provisions of CA No. 63,
which enumerates the modes by which Philippine
citizenship may be lost. Among these are: (1)
naturalization in a foreign country; (2) express
renunciation of citizenship; and (3) subscribing to
an oath of allegiance to support the Constitution
or laws of a foreign country, all of which are
applicable to the petitioner. It is also worth
mentioning in this connection that under Article
IV, Section 5, of the present Constitution, "Dual
allegiance of citizens is inimical to the national
interest and shall be dealt with by law."
Even if it be assumed that, as the petitioner
asserts, his naturalization in Australia was
annulled after it was found that his marriage to
the Australian citizen was bigamous, that
circumstance alone did not automatically restore
his Philippine citizenship. His divestiture of
Australian citizenship does not concern us here.
That is a matter between him and his adopted
country. What we must consider is the fact that
he voluntarily and freely rejected Philippine
citizenship and willingly and knowingly embraced
the citizenship of a foreign country. The possibility
that he may have been subsequently rejected by
Australia, as he claims, does not mean that he
has been automatically reinstated as a citizen of
the Philippines.
Under CA No. 63 as amended by PD No. 725,
Philippine citizenship may be reacquired by direct
act of Congress, by naturalization, or by
repatriation. It does not appear in the record, nor
does the petitioner claim, that he has reacquired
Philippine citizenship by any of these methods.
He does not point to any judicial decree of
naturalization as to any statute directly conferring
Philippine citizenship upon him. Neither has he
shown that he has complied with PD No. 725,
providing that:
. . . (2) natural-born Filipinos who have lost their
Philippine citizenship may reacquire Philippine
citizenship through repatriation by applying with
the Special Committee on Naturalization created
by Letter of Instruction No. 270, and, if their
applications are approved, taking the necessary
oath of allegiance to the Republic of the
Philippines, after which they shall be deemed to
have reacquired Philippine citizenship. The
Commission on Immigration and Deportation shall
thereupon cancel their certificate of registration.
(Emphasis supplied.)
That is why the Commission on Immigration and
Deportation rejected his application for the
32 | C O N S T I 2 _ A r t i c l e I V _ C I T I Z E N S H I P

cancellation of his alien certificate of registration.


And that is also the reason we must deny his
present claim for recognition as a citizen of the
Philippines.
The petitioner is not now, nor was he on the day
of the local elections on January 18, 1988, a
citizen of the Philippines. In fact, he was not even
a qualified voter under the Constitution itself
because of his alienage. 21 He was therefore
ineligible as a candidate for mayor of Baguio City
under Section 42 of the Local Government Code
providing in material part as follows:
Sec. 42. Qualifications. (1) An elective local
official must be a citizen of the Philippines, at
least twenty-three years of age on election day, a
qualified voter registered as such in the
barangay, municipality, city or province where he
proposes to be elected, a resident therein for at
least one year at the time of the filing of his
certificate of candidacy, and able to read and
write English, Pilipino, or any other local language
or dialect.
The petitioner argues that his alleged lack of
citizenship is a "futile technicality" that should
not frustrate the will of the electorate of Baguio
City who elected him by a "resonant and
thunderous majority." To be accurate, it was not
as loud as all that, for his lead over the secondplacer was only about 2,100 votes. In any event,
the people of that locality could not have, even
unanimously, changed the requirements of the
Local Government Code and the Constitution. The
electorate had no power to permit a foreigner
owing his total allegiance to the Queen of
Australia, or at least a stateless individual owing
no allegiance to the Republic of the Philippines, to
preside over them as mayor of their city. Only
citizens of the Philippines have that privilege over
their countrymen.
The probability that many of those who voted for
the petitioner may have done so in the belief that
he was qualified only strengthens the conclusion
that the results of the election cannot nullify the
qualifications for the office now held by him.
These qualifications are continuing requirements;
once any of them is lost during incumbency, title
to the office itself is deemed forfeited. In the case
at bar, the citizenship and voting requirements
were not subsequently lost but were not
possessed at all in the first place on the day of
the election. The petitioner was disqualified from
running as mayor and, although elected, is not
now qualified to serve as such. LLpr
Finally, there is the question of whether or not the
private respondent, who filed the quo warranto
petition, can replace the petitioner as mayor. He
cannot. The simple reason is that as he obtained
only the second highest number of votes in the
election, he was obviously not the choice of the
people of Baguio City.
The latest ruling of the Court on this issue is
Santos v. Commission on Elections, 22 decided in
1985. In that case, the candidate who placed
second was proclaimed elected after the votes for
his winning rival, who was disqualified as a
turncoat and considered a non-candidate, were
all disregarded as stray. In effect, the second
placer won by default. That decision was

supported by eight members of the Court then,


23 with three dissenting 24 and another two
reserving their vote.25 One was on official leave.
26
Re-examining that decision, the Court finds, and
so holds, that it should be reversed in favor of the
earlier case of Geronimo v. Ramos, 27 which
represents the more logical and democratic rule.
That case, which reiterated the doctrine first
announced in 1912 in Topacio vs. Paredes, 28 was
supported by ten members of the Court, 29
without any dissent, although one reserved his
vote, 30 another took no part, 31 and two others
were an leave. 32 There the Court held:
". . . it would be extremely repugnant to the basic
concept of the constitutionally guaranteed right
to suffrage if a candidate who has not acquired
the majority or plurality of votes is proclaimed a
winner and imposed as the representative of a
constituency, the majority of which have
positively declared through their ballots that they
do not choose him.
Sound policy dictates that public elective offices
are filled by those who have received the highest
number of votes cast in the election for that
office, and it is a fundamental idea in all
republican forms of government that no one can
be declared elected and no measure can be
declared carried unless he or it receives a
majority or plurality of the legal votes cast in the
election. (20 Corpus Juris 2nd, S 243, p. 676.)
The fact that the candidate who obtained the
highest number of votes is later declared to be
disqualified or not eligible for the office to which
he was elected does not necessarily entitle the
candidate who obtained the second highest
number of votes to be declared the winner of the
elective office. The votes cast for a dead,
disqualified, or non-eligible person may not be
valid to vote the winner into office or maintain
him there. However, in the absence of a statute
which clearly asserts a contrary political and
legislative policy on the matter, if the votes were
cast in the sincere belief that the candidate was
alive, qualified, or eligible, they should not be
treated as stray, void or meaningless.
It remains to stress that the citizen of the
Philippines must take pride in his status as such
and cherish this priceless gift that, out of more
than a hundred other nationalities, God has seen
fit to grant him. Having been so endowed, he
must not lightly yield this precious advantage,
rejecting it for another land that may offer him
material and other attractions that he may not
find in his own country. To be sure, he has the
right to renounce the Philippines if he sees fit and
transfer his allegiance to a state with more
allurements for him. 33 But having done so, he
cannot expect to be welcomed back with open
arms once his taste for his adopted country turns
sour or he is himself disowned by it as an
undesirable alien.
Philippine citizenship is not a cheap commodity
that can be easily recovered after its
renunciation. It may be restored only after the
returning renegade makes a formal act of re33 | C O N S T I 2 _ A r t i c l e I V _ C I T I Z E N S H I P

dedication to the country he has abjured and he


solemnly affirms once again his total and
exclusive loyalty to the Republic of the
Philippines. This may not be accomplished by
election to public office.
WHEREFORE, petitioner Ramon J. Labo, Jr. is
hereby declared NOT a citizen of the Philippines
and therefore DISQUALIFIED from continuing to
serve as Mayor of Baguio City. He is ordered to
VACATE his office and surrender the same to the
Vice-Mayor of Baguio City once this decision
becomes final and executory. The temporary
restraining order dated January 31, 1989, is
LIFTED.
Fernan, C . J ., Narvasa, Melencio-Herrera, Paras,
Feliciano, Gancayco, Padilla, Bidin, Sarmiento,
Cortes, Grio-Aquino, Medialdea and Regalado,
JJ ., concur.
(Aznar v. COMELEC, G.R. No. 83820, May 25,
1990)
EN BANC
[G.R. No. 83820. May 25, 1990.]
JOSE B. AZNAR (as Provincial Chairman of PDP
Laban in Cebu), petitioner, vs. COMMISSION ON
ELECTIONS and EMILIO MARIO RENNER OSMEA,
respondents.
Rufino B. Requina for petitioner.
Angara, Abello, Concepcion, Regala & Cruz for
private respondent.
DECISION
PARAS, J p:
Before Us is a petition for certiorari assailing the
Resolution of the Commission on Elections
(COMELEC) dated June 11, 1988, which dismissed
the petition for the disqualification of private
respondent Emilio "Lito" Osmea as candidate for
Provincial Governor of Cebu Province. cdll
The facts of the case are briefly as follows:
On November 19, 1987, private respondent
Emilio "Lito" Osmea filed his certificate of
candidacy with the COMELEC for the position of
Provincial Governor of Cebu Province in the
January 18, 1988 local elections.
On January 22, 1988, the Cebu PDP-Laban
Provincial Council (Cebu-PDP Laban, for short), as
represented by petitioner Jose B. Aznar in his
capacity as its incumbent Provincial Chairman,
filed with the COMELEC a petition for the
disqualification of private respondent on the
ground that he is allegedly not a Filipino citizen,
being a citizen of the United States of America.
On January 27, 1988, petitioner filed a Formal
Manifestation submitting a Certificate issued by
the
then
Immigration
and
Deportation
Commissioner
Miriam
Defensor
Santiago
certifying that private respondent is an American

and is a holder of Alien Certificate of Registration


(ACR) No. B-21448 and Immigrant Certificate of
Residence (ICR) No. 133911, issued at Manila on
March 27 and 28, 1958, respectively. (Annex "B1").
The petitioner also filed a Supplemental Urgent
Ex-Parte Motion for the Issuance of a Temporary
Restraining Order to temporarily enjoin the Cebu
Provincial
Board
of
Canvassers
from
tabulating/canvassing the votes cast in favor of
private respondent and proclaiming him until the
final resolution of the main petition.
Thus, on January 28, 1988, the COMELEC en banc
resolved to order the Board to continue
canvassing but to suspend the proclamation.
At the hearing before the COMELEC (First
Division), the petitioner presented the following
exhibits tending to show that private respondent
is an American citizen: Application for Alien
Registration Form No. 1 of the Bureau of
Immigration signed by private respondent dated
November 21, 1979 (Exh. "B"); Alien Certificate of
Registration No. 015356 in the name of private
respondent dated November 21, 1979 (Exh. "C");
Permit to Re-enter the Philippines dated
November 21, 1979 (Exh. "D"); Immigration
Certificate of Clearance dated January 3, 1980
(Exh. "E"). (pp. 117-118, Rollo).
Private respondent, on the other hand,
maintained that he is a Filipino citizen, alleging:
that he is the legitimate child of Dr. Emilio D.
Osmea, a Filipino and son of the late President
Sergio Osmea, Sr.; that he is a holder of a valid
and subsisting Philippine Passport No. 0855103
issued on March 25, 1987; that he has been
continuously residing in the Philippines since birth
and has not gone out of the country for more
than six months; and that he has been a
registered voter in the Philippines since 1965.
(pp. 107-108, Rollo)
On March 3, 1988, COMELEC (First Division)
directed the Board of Canvassers to proclaim the
winning candidates. Having obtained the highest
number of votes, private respondent was
proclaimed the Provincial Governor of Cebu.
Thereafter, on June 11, 1988, COMELEC (First
Division)
dismissed
the
petition
for
disqualification for not having been timely filed
and for lack of sufficient proof that private
respondent is not a Filipino citizen. cdrep
Hence, the present petition.
The petition is not meritorious.
There are two instances where a petition
questioning the qualifications of a registered
candidate to run for the office for which his
certificate of candidacy was filed can be raised
under the Omnibus Election Code (B.P. Blg. 881),
to wit:

34 | C O N S T I 2 _ A r t i c l e I V _ C I T I Z E N S H I P

"(1) Before election, pursuant to Section 78


thereof which provides that:
'Section 78. Petition to deny due course or to
cancel a certificate of candidacy. A verified
petition seeking to deny due course or to cancel a
certificate of candidacy may be filed by any
person exclusively on the ground that any
material representation contained therein as
required under Section 74 hereof is false. The
petition may be filed at any time not later than
twenty five days from the time of the filing of the
certificate of candidacy and shall be decided,
after the notice and hearing, not later than fifteen
days before the election.
and
"(2) After election, pursuant to Section 253
thereof, viz:
'Sec. 253. Petition for quo warranto. Any voter
contesting the election of any Member of the
Batasang Pambansa, regional, provincial, or city
officer on the ground of inelligibility or of
disloyalty to the Republic of the Philippines shall
file a sworn petition for quo warranto with the
Commission
within
ten
days
after
the
proclamation of the results of the election."
The records show that private respondent filed
his certificate of candidacy on November 19,
1987 and that the petitioner filed its petition for
disqualification of said private respondent on
January 22, 1988. Since the petition for
disqualification was filed beyond the twenty fiveday period required in Section 78 of the Omnibus
Election Code, it is clear that said petition was
filed out of time.
The petition for the disqualification of private
respondent cannot also be treated as a petition
for quo warranto under Section 253 of the same
Code as it is unquestionably premature,
considering
that
private
respondent
was
proclaimed Provincial Governor of Cebu only on
March 3, 1988.
However, We deem it is a matter of public
interest to ascertain the respondent's citizenship
and qualification to hold the public office to which
he has been proclaimed elected. There is enough
basis for us to rule directly on the merits of the
case, as the COMELEC did below.
Petitioner's contention that private respondent is
not a Filipino citizen and, therefore, disqualified
from running for and being elected to the office of
Provincial Governor of Cebu, is not supported by
substantial and convincing evidence.
In the proceedings before the COMELEC, the
petitioner failed to present direct proof that
private respondent had lost his Filipino citizenship
by any of the modes provided for under C.A. No.
63. Among others, these are: (1) by naturalization
in a foreign country; (2) by express renunciation
of citizenship; and (3) by subscribing to an oath
of allegiance to support the Constitution or laws

of a foreign country. From the evidence, it is clear


that private respondent Osmea did not lose his
Philippine citizenship by any of the three
mentioned hereinabove or by any other mode of
losing Philippine citizenship.
In concluding that private respondent had been
naturalized as a citizen of the United States of
America, the petitioner merely relied on the fact
that private respondent was issued alien
certificate of registration and was given clearance
and permit to re-enter the Philippines by the
Commission on Immigration and Deportation.
Petitioner assumed that because of the foregoing,
the respondent is an American and "being an
American", private respondent "must have taken
and sworn to the Oath of Allegiance required by
the U.S. Naturalization Laws." (p. 81, Rollo)
Philippine courts are only allowed to determine
who are Filipino citizens and who are not.
Whether or not a person is considered an
American under the laws of the United States
does not concern Us here.
By virtue of his being the son of a Filipino father,
the presumption that private respondent is a
Filipino remains. It was incumbent upon the
petitioner to prove that private respondent had
lost his Philippine citizenship. As earlier stated,
however, the petitioner failed to positively
establish this fact.
The cases of Juan Gallanosa Frivaldo v. COMELEC
et al., (G.R. No. 87193, June 21, 1989) and Ramon
L. Labo v. COMELEC et al. (G.R. No. 86564, August
1, 1989) are not applicable to the case at bar.
In the Frivaldo case, evidence shows that he was
naturalized as a citizen of the United States in
1983 per certification from the United States
District Court, Northern District of California, as
duly authenticated by Vice Consul Amado P.
Cortez of the Philippine Consulate General in San
Francisco, California, U.S.A.
Frivaldo expressly admitted in his answer that he
was naturalized in the United States but claimed
that he was forced to embrace American
citizenship
to
protect
himself
from
the
persecution of the Marcos government. The
Court, however, found this suggestion of
involuntariness unacceptable, pointing out that
there were many other Filipinos in the United
States similarly situated as Frivaldo who did not
find it necessary to abandon their status as
Filipinos.
Likewise, in the case of Labo, records show that
Labo was married to an Australian citizen and
that he was naturalized as an Australian citizen in
1976, per certification from the Australian
Government through its Consul in the Philippines.
This was later affirmed by the Department of
Foreign Affairs.
The authenticity of the above evidence was not
disputed by Labo. In fact, in a number of sworn
35 | C O N S T I 2 _ A r t i c l e I V _ C I T I Z E N S H I P

statements, Labo categorically declared that he


was a citizen of Australia. LexLib
In declaring both Frivaldo and Labo not citizens of
the Philippines, therefore, disqualified from
serving as Governor of the Province of Sorsogon
and Mayor of Baguio City, respectively, the Court
considered the fact that by their own admissions,
they are indubitably aliens, no longer owing any
allegiance to the Republic of the Philippines since
they have sworn their total allegiance to a foreign
state.
In the instant case, private respondent
vehemently denies having taken the oath of
allegiance of the United States (p. 81, Rollo). He
is a holder of a valid and subsisting Philippine
passport and has continuously participated in the
electoral process in this country since 1963 up to
the present, both as a voter and as a candidate
(pp. 107-108, Rollo). Thus, private respondent
remains a Filipino and the loss of his Philippine
citizenship cannot be presumed.

In the learned dissent of Mr. Justice Teodoro


Padilla, he stresses the fact that because Osmea
obtained Certificates of Alien Registration as an
American citizen, the first in 1958 when he was
24 years old and the second in 1979, he, Osmea
should be regarded as having expressly
renounced Philippine citizenship. To Our mind,
this is a case of non sequitur (It does not follow).
Considering the fact that admittedly Osmea was
both a Filipino and an American, the mere fact
that he has a Certificate stating he is an
American does not mean that he is not still a
Filipino. Thus, by way of analogy, if a person who
has two brothers named Jose and Mario states or
certifies that he has a brother named Jose, this
does not mean that he does not have a brother
named Mario; or if a person is enrolled as student
simultaneously in two universities, namely
University X and University Y, presents a
Certification that he is a student of University X,
this does not necessarily mean that he is not still
a student of University Y. In the case of Osmea,
the Certification that he is an American does not
mean that he is not still a Filipino, possessed as
he is, of both nationalities or citizenships. Indeed,
there is no express renunciation here of Philippine
citizenship; truth to tell, there is even no implied
renunciation of said citizenship. When We
consider that the renunciation needed to lose
Philippine citizenship must be "express", it stands
to reason that there can be no such loss of
Philippine
citizenship
when
there
is
no
renunciation, either "express" or "implied ".
Parenthetically, the statement in the 1987
Constitution that "dual allegiance of citizens is
inimical to the national interest and shall be dealt
with by law" (Art. IV, Sec. 5) has no retroactive
effect. And while it is true that even before the
1987 Constitution, Our country had already
frowned upon the concept of dual citizenship or
allegiance, the fact is it actually existed. Be it
noted further that under the aforecited proviso,

the effect of such dual citizenship or allegiance


shall be dealt with by a future law. Said law has
not yet been enacted.
WHEREFORE, the petition for certiorari is hereby
DISMISSED and the Resolution of the COMELEC is
hereby AFFIRMED.
SO ORDERED.
Narvasa,
Bidin,
Grio-Aquino,
Regalado and Feliciano, JJ., concur.

Medialdea,

Fernan, C.J., took no part


Gutierrez, Jr., J., My stand in the cases of Willie Yu
v. Miriam Defensor Santiago, et al. (G.R. No.
83882, January 24, 1989) and Ramon Labo, Jr. v.
Commission on Elections (G.R. 86564, August 2,
1989) is clear. I regret, however, that I cannot
participate in this case because one of the
principal counsels is my relative by affinity within
the fourth civil degree.
Gancayco, J., is on official leave.
Cortes, J., concurs in the result.
(Mercado v. Manzano, G.R. No. 135083, May
26, 1999)
EN BANC
[G.R. No. 135083. May 26, 1999.]
ERNESTO S. MERCADO, petitioner, vs. EDUARDO
BARRIOS MANZANO and the COMMISSION ON
ELECTIONS, respondents.
Balase, Tamase, Alampay Law Office for
petitioner.
Siguion Reyna, Montecillo & Ongsiako for private
respondent.
SYNOPSIS
Petitioner Mercado and private respondent
Manzano were candidates for vice mayor of the
City of Makati in the May 11, 1998 elections. The
proclamation
of
private
respondent
was
suspended in view of a pending petition for
disqualification filed by a certain Ernesto Mamaril
who alleged that private respondent was not a
citizen of the Philippines but of the United States.
The Second Division of the COMELEC granted the
petition of Mamaril and ordered the cancellation
of the certificate of candidacy of private
respondent on the ground that he is a dual citizen
and under Sec. 40 of the Local Government Code,
persons with dual citizenship are disqualified from
running for any elective position. Private
respondent filed a motion for reconsideration. The
motion remained pending until after the election.
The board of canvassers tabulated the votes but
suspended the proclamation of the winner.
Petitioner sought to intervene in the case for
disqualification. COMELEC en banc reversed the
decision and declared private respondent
qualified to run for the position. Pursuant to the
ruling of the COMELEC en banc, the board of
36 | C O N S T I 2 _ A r t i c l e I V _ C I T I Z E N S H I P

canvassers proclaimed private respondent as vice


mayor. This petition sought the reversal of the
resolution of the COMELEC en banc and to
declare the private respondent disqualified to
hold the office of the vice mayor of Makati. cdasia
On the issue of whether the petitioner has
personality to bring this suit considering that he
was not the original party in the disqualification
case, the Supreme Court ruled that under Sec. 6
of R.A. No. 6646, otherwise known as the
Electoral Reforms Law of 1987, intervention may
be allowed in proceedings for disqualification
even after election if there has yet been no final
judgment rendered. As regards the issue of
citizenship, the Court ruled that by filing a
certificate of candidacy when he ran for his
present
post,
private
respondent
elected
Philippine citizenship and in effect renounced his
American citizenship.
SYLLABUS
1. POLITICAL LAW; ELECTORAL REFORMS LAW OF
1987 (R.A. No. 6646); INTERVENTION, ALLOWED
IN PROCEEDINGS FOR DISQUALIFICATION EVEN
AFTER ELECTION IF THERE HAS BEEN NO FINAL
JUDGMENT RENDERED; CASE AT BAR. Private
respondent argues that petitioner has neither
legal interest in the matter in litigation nor an
interest to protect because he is "a defeated
candidate for the vice-mayoralty post of Makati
City [who] cannot be proclaimed as the ViceMayor of Makati City even if the private
respondent be ultimately disqualified by final and
executory judgment." The flaw in this argument is
it assumes that, at the time petitioner sought to
intervene in the proceedings before the
COMELEC, there had already been a proclamation
of the results of the election for the vice
mayoralty contest for Makati City, on the basis of
which petitioner came out only second to private
respondent. The fact, however, is that there had
been no proclamation at that time. Certainly,
petitioner had, and still has, an interest in ousting
private respondent from the race at the time he
sought to intervene. The rule in Labo vs.
COMELEC, reiterated in several cases, only
applies to cases in which the election of the
respondent is contested, and the question is
whether one who placed second to the
disqualified candidate may be declared the
winner. In the present case, at the time petitioner
filed a "Motion for leave to File Intervention" on
May 20, 1998, there had been no proclamation of
the winner, and petitioner's purpose was
precisely to have private respondent disqualified
"from running for [an] elective local position"
under Section 40(d) of R.A. No. 7160. If Ernesto
Mamaril
(who
originally
instituted
the
disqualification proceedings), a registered voter
of Makati City, was competent to bring the action,
so was petitioner since the latter was a rival
candidate for vice mayor of Makati City. Nor is
petitioner's interest in the matter in litigation any
less because he filed a motion for intervention
only on May 20, 1998, after private respondent
had been shown to have garnered the highest
number of votes among the candidates for vice

mayor. That petitioner had a right to intervene at


that stage of the proceedings for the
disqualification against private respondent is
clear from Section 6 of R.A. No. 6646, otherwise
known as the Electoral Reforms Law of 1987,
which provides: Any candidate who has been
declared by final judgment to be disqualified shall
not be voted for, and the votes cast for him shall
not be counted. If for any reason a candidate is
not declared by final judgment before an election
to be disqualified and he is voted for and receives
the winning number of votes in such election, the
Court or Commission shall continue with the trial
and hearing of the action, inquiry, or protest and,
upon motion of the complainant or any
intervenor, may during the pendency thereof
order the suspension of the proclamation of such
candidate whenever the evidence of guilt is
strong. Under this provision, intervention may be
allowed in proceedings for disqualification even
after election if there has yet been no final
judgment rendered.
2.
ID.;
CITIZENSHIP;
DUAL
CITIZENSHIP;
DISTINGUISHED FROM DUAL ALLEGIANCE. Dual
citizenship is different from dual allegiance. The
former arises when, as a result of the concurrent
application of the different laws of two or more
states, a person is simultaneously considered a
national by the said states. For instance, such a
situation may arise when a person whose parents
are citizens of a state which adheres to the
principle of jus sanguinis is born in a state which
follows the doctrine of jus soli. Such a person,
ipso facto and without any voluntary act on his
part, is concurrently considered a citizen of both
states. Considering the citizenship clause (Art. IV)
of our Constitution, it is possible for the following
classes of citizens of the Philippines to posses
dual citizenship: (1) Those born of Filipino fathers
and/or mothers in foreign countries which follow
the principle of jus soli; (2) Those born in the
Philippines of Filipino mothers and alien fathers if
by the laws of their fathers' country such children
are citizens of that country; (3) Those who marry
aliens if by the laws of the latter's country the
former are considered citizens, unless by their act
or omission they are deemed to have renounced
Philippine citizenship. There may be other
situations in which a citizen of the Philippines
may, without performing any act, be also a
citizen of another state; but the above cases are
possible given the constitutional provisions on
citizenship. Dual allegiance, on the other hand,
refers to the situation in which a person
simultaneously owes, by some positive act,
loyalty to two or more states. While dual
citizenship is involuntary, dual allegiance is the
result of an individual's volition. With respect to
dual allegiance, Article IV, Section 5 of the
Constitution provides: "Dual allegiance of citizens
is inimical to the national interest and shall be
dealt with by law."
3. ID.; ID.; ID.; ID.; RATIONALE. In including
Section 5 in Article IV on citizenship, the concern
of the Constitutional Commission was not with
dual citizens per se but with naturalized citizens
who maintain their allegiance to their countries of
37 | C O N S T I 2 _ A r t i c l e I V _ C I T I Z E N S H I P

origin even after their naturalization. Hence, the


phrase "dual citizenship" in R.A. No. 7160, Section
40(d) and in R.A. No. 7854, Section 20 must be
understood as referring to "dual allegiance."
Consequently, persons with mere dual citizenship
do not fall under this disqualification. Unlike those
with dual allegiance, who must, therefore, be
subject to strict process with respect to the
termination of their status, for candidates with
dual citizenship, it should suffice if, upon the filing
of their certificates of candidacy, they elect
Philippine citizenship to terminate their status as
persons with dual citizenship considering that
their condition is the unavoidable consequence of
conflicting laws of different states. As Joaquin G.
Bernas, one of the most perceptive members of
the Constitutional Commission, pointed out:
"[D]ual citizenship is just a reality imposed on us
because we have no control of the laws on
citizenship of other countries. We recognize a
child of a Filipino mother. But whether or not she
is considered a citizen of another country is
something completely beyond our control." By
electing Philippine citizenship, such candidates at
the same time forswear allegiance to the other
country of which they are also citizens and
thereby terminate their status as dual citizens. It
may be that, from the point of view of the foreign
state and of its laws, such an individual has not
effectively renounced his foreign citizenship.
4. ID.; ID.; FILING OF THE CERTIFICATE OF
CANDIDACY SUFFICED TO RENOUNCE AMERICAN
CITIZENSHIP; CASE AT BAR. By filing a
certificate of candidacy when he ran for his
present
post,
private
respondent
elected
Philippine citizenship and in effect renounced his
American citizenship. The filing of such certificate
of candidacy sufficed to renounce his American
citizenship,
effectively
removing
any
disqualification he might have as a dual citizen.
Thus, in Frivaldo vs. COMELEC it was held: It is
not disputed that on January 20, 1983 Frivaldo
became an American. Would the retroactivity of
his repatriation not effectively give him dual
citizenship, which under Sec. 40 of the Local
Government Code would disqualify him "from
running for any elective local position?" We
answer this question in the negative, as there is
cogent reason to hold that Frivaldo was really
STATELESS at the time he took said oath of
allegiance and even before that, when he ran for
governor in 1988. In his Comment, Frivaldo wrote
that he "had long renounced and had long
abandoned his American citizenship long
before May 8, 1995. At best, Frivaldo was
stateless in the interim when he abandoned
and renounced his US citizenship but before he
was repatriated to his Filipino citizenship." On this
point, we quote from the assailed Resolution
dated December 19, 1995: "By the laws of the
United States, petitioner Frivaldo lost his
American citizenship when he took his oath of
allegiance to the Philippine Government when he
ran for Governor in 1988, in 1992, and in 1995.
Every certificate of candidacy contains an oath of
allegiance to the Philippine Government." These
factual findings that Frivaldo has lost his foreign
nationality long before the elections of 1995 have

not
been
effectively
rebutted
by
Lee.
Furthermore, it is basic that such findings of the
Commission are conclusive upon this Court,
absent any showing of capriciousness or
arbitrariness or abuse. Until the filing of his
certificate of candidacy on March 21, 1998,
private respondent had dual citizenship. The acts
attributed to him can be considered simply as the
assertion of his American nationality before the
termination of his American citizenship. What this
Court said in Aznar vs. COMELEC applies mutatis
mutandis to private respondent in the case at
bar: . . . Considering the fact that admittedly
Osmea was both a Filipino and an American, the
mere fact that he has a Certificate stating he is
an American does not mean that he is not still a
Filipino. . . [T]he Certification that he is an
American does not mean that he is not still a
Filipino, possessed as he is, of both nationalities
or citizenships. Indeed, there is no express
renunciation here of Philippine citizenship; truth
to tell, there is even no implied renunciation of
said citizenship. When We consider that the
renunciation needed to lose Philippine citizenship
must be "express," it stands to reason that there
can be no such loss of Philippine citizenship when
there is no renunciation, either "express" or
"implied." To recapitulate, by declaring in his
certificate of candidacy that he is a Filipino
citizen; that he is not a permanent resident or
immigrant of another country; that he will defend
and support the Constitution of the Philippines
and bear true faith and allegiance thereto and
that he does so without mental reservation,
private respondent has, as far as the laws of this
country are concerned, effectively repudiated his
American citizenship and anything which he may
have said before as a dual citizen. On the other
hand, private respondent's oath of allegiance to
the Philippines, when considered with the fact
that he has spent his youth and adulthood,
received his education, practiced his profession
as an artist, and taken part in past elections in
this country, leaves no doubt of his election of
Philippine citizenship. acCITS

DECISION
MENDOZA, J p:
Petitioner Ernesto S. Mercado and private
respondent Eduardo B. Manzano were candidates
for vice mayor of the City of Makati in the May 11,
1998 elections. The other one was Gabriel V.
Daza III. The results of the election were as
follows:
Eduardo B. Manzano 103,853
Ernesto S. Mercado 100,894
Gabriel V. Daza III 54,275 1
The proclamation of private respondent was
suspended in view of a pending petition for
disqualification filed by a certain Ernesto Mamaril
who alleged that private respondent was not a
citizen of the Philippines but of the United States.
38 | C O N S T I 2 _ A r t i c l e I V _ C I T I Z E N S H I P

In its resolution, dated May 7, 1998, 2 the Second


Division of the COMELEC granted the petition of
Mamaril and ordered the cancellation of the
certificate of candidacy of private respondent on
the ground that he is a dual citizen and, under
40(d) of the Local Government Code, persons
with dual citizenship are disqualified from running
for any elective position. The COMELEC's Second
Division said:
What is presented before the Commission is a
petition for disqualification of Eduardo Barrios
Manzano as candidate for the office of Vice-Mayor
of Makati City in the May 11, 1998 elections. The
petition is based on the ground that the
respondent is an American citizen based on the
record of the Bureau of Immigration and
misrepresented himself as a natural-born Filipino
citizen.
In his answer to the petition filed on April 27,
1998, the respondent admitted that he is
registered as a foreigner with the Bureau of
Immigration
under
Alien
Certificate
of
Registration No. B-31632 and alleged that he is a
Filipino citizen because he was born in 1955 of a
Filipino father and a Filipino mother. He was born
in the United States, San Francisco, California, on
September 14, 1955, and is considered an
American
citizen
under
US
Laws.
But
notwithstanding his registration as an American
citizen, he did not lose his Filipino citizenship.
Judging from the foregoing facts, it would appear
that respondent Manzano is both a Filipino and a
US citizen. In other words, he holds dual
citizenship.
The question presented is whether under our
laws, he is disqualified from the position for which
he filed his certificate of candidacy. Is he eligible
for the office he seeks to be elected?
Under Section 40(d) of the Local Government
Code, those holding dual citizenship are
disqualified from running for any elective local
position.
WHEREFORE, the Commission hereby declares
the respondent Eduardo Barrios Manzano
DISQUALIFIED as candidate for Vice-Mayor of
Makati City.
On May 8, 1998, private respondent filed a
motion for reconsideration. 3 The motion
remained pending even until after the election
held on May 11, 1998.
Accordingly, pursuant to Omnibus Resolution No.
3044, dated May 10, 1998, of the COMELEC, the
board of canvassers tabulated the votes cast for
vice mayor of Makati City but suspended the
proclamation of the winner.
On May 19, 1998, petitioner sought to intervene
in the case for disqualification. 4 Petitioner's
motion was opposed by private respondent.
The motion was not resolved. Instead, on August
31, 1998, the COMELEC en banc rendered its

resolution. Voting 4 to 1, with one commissioner


abstaining, the COMELEC en banc reversed the
ruling of its Second Division and declared private
respondent qualified to run for vice mayor of the
City of Makati in the May 11, 1998 elections. 5
The pertinent portions of the resolution of the
COMELEC en banc read:
As aforesaid, respondent Eduardo Barrios
Manzano was born in San Francisco, California,
U.S.A. He acquired US citizenship by operation of
the United States Constitution and laws under the
principle of jus soli.
He was also a natural born Filipino citizen by
operation of the 1935 Philippine Constitution, as
his father and mother were Filipinos at the time of
his birth. At the age of six (6), his parents brought
him to the Philippines using an American passport
as travel document. His parents also registered
him as an alien with the Philippine Bureau of
Immigration. He was issued an alien certificate of
registration. This, however, did not result in the
loss of his Philippine citizenship, as he did not
renounce Philippine citizenship and did not take
an oath of allegiance to the United States.
It is an undisputed fact that when respondent
attained the age of majority, he registered
himself as a voter, and voted in the elections of
1992, 1995 and 1998, which effectively
renounced his citizenship under American law.
Under Philippine law, he no longer had U.S.
citizenship.
At the time of the May 11, 1998 elections, the
resolution of the Second Division, adopted on May
7, 1998, was not yet final. Respondent Manzano
obtained the highest number of votes among the
candidates for vice-mayor of Makati City,
garnering one hundred three thousand eight
hundred fifty-three (103,853) votes over his
closest rival, Ernesto S. Mercado, who obtained
one hundred thousand eight hundred ninety-four
(100,894) votes, or a margin of two thousand
nine hundred fifty-nine (2,959) votes. Gabriel
Daza III obtained third place with fifty four
thousand two hundred seventy-five (54,275)
votes. In applying election laws, it would be far
better to err in favor of the popular choice than
be embroiled in complex legal issues involving
private international law which may well be
settled before the highest court (Cf. Frivaldo vs.
Commission on Elections, 257 SCRA 727).
WHEREFORE, the Commission en banc hereby
REVERSES the resolution of the Second Division,
adopted on May 7, 1998, ordering the
cancellation of the respondent's certificate of
candidacy.
We declare respondent Eduardo Luis Barrios
Manzano to be QUALIFIED as a candidate for the
position of vice-mayor of Makati City in the May
11, 1998, elections.
ACCORDINGLY, the Commission directs the Makati
City Board of Canvassers, upon proper notice to
the parties, to reconvene and proclaim the
39 | C O N S T I 2 _ A r t i c l e I V _ C I T I Z E N S H I P

respondent Eduardo Luis Barrios Manzano as the


winning candidate for vice-mayor of Makati City.
Pursuant to the resolution of the COMELEC en
banc, the board of canvassers, on the evening of
August 31, 1998, proclaimed private respondent
as vice mayor of the City of Makati. cdasia
This is a petition for certiorari seeking to set aside
the aforesaid resolution of the COMELEC en banc
and to declare private respondent disqualified to
hold the office of vice mayor of Makati City.
Petitioner contends that
[T]he COMELEC en banc ERRED in holding that:
A. Under Philippine law, Manzano was no longer a
U.S. citizen when he:
1. He renounced his U.S. citizenship when he
attained the age of majority when he was already
37 years old; and,
2. He renounced his U.S. citizenship when he
(merely) registered himself as a voter and voted
in the elections of 1992, 1995 and 1998.
B. Manzano is qualified to run for and or hold the
elective office of Vice-Mayor of the City of Makati;
C. At the time of the May 11, 1998 elections, the
resolution of the Second Division adopted on 7
May 1998 was not yet final so that, effectively,
petitioner may not be declared the winner even
assuming that Manzano is disqualified to run for
and hold the elective office of Vice-Mayor of the
City of Makati.
We first consider the threshold procedural issue
raised by private respondent Manzano whether
petitioner Mercado has personality to bring this
suit considering that he was not an original party
in the case for disqualification filed by Ernesto
Mamaril nor was petitioner's motion for leave to
intervene granted.
I. PETITIONER'S RIGHT TO BRING THIS SUIT
Private respondent cites the following provisions
of Rule 8 of the Rules of Procedure of the
COMELEC in support of his claim that petitioner
has no right to intervene and, therefore, cannot
bring this suit to set aside the ruling denying his
motion for intervention:
SECTION 1. When proper and when may be
permitted to intervene. Any person allowed to
initiate an action or proceeding may, before or
during the trial of an action or proceeding, be
permitted by the Commission, in its discretion to
intervene in such action or proceeding, if he has
legal interest in the matter in litigation, or in the
success of either of the parties, or an interest
against both, or when he is so situated as to be
adversely affected by such action or proceeding.
xxx xxx xxx
SECTION 3.
Discretion of Commission. In
allowing or disallowing a motion for intervention,
the Commission or the Division, in the exercise of
its discretion, shall consider whether or not the
intervention will unduly delay or prejudice the
adjudication of the rights of the original parties

and whether or not the intervenor's rights may be


fully protected in a separate action or proceeding.
Private respondent argues that petitioner has
neither legal interest in the matter in litigation
nor an interest to protect because he is "a
defeated candidate for the vice-mayoralty post of
Makati City [who] cannot be proclaimed as the
Vice-Mayor of Makati City even if the private
respondent be ultimately disqualified by final and
executory judgment."
The flaw in this argument is it assumes that, at
the time petitioner sought to intervene in the
proceedings before the COMELEC, there had
already been a proclamation of the results of the
election for the vice mayoralty contest for Makati
City, on the basis of which petitioner came out
only second to private respondent. The fact,
however, is that there had been no proclamation
at that time. Certainly, petitioner had, and still
has, an interest in ousting private respondent
from the race at the time he sought to intervene.
The rule in Labo v. COMELEC, 6 reiterated in
several cases, 7 only applies to cases in which
the election of the respondent is contested, and
the question is whether one who placed second
to the disqualified candidate may be declared the
winner. In the present case, at the time petitioner
filed a "Motion for Leave to File Intervention" on
May 20, 1998, there had been no proclamation of
the winner, and petitioner's purpose was
precisely to have private respondent disqualified
"from running for [an] elective local position"
under 40(d) of R.A. No. 7160. If Ernesto Mamaril
(who originally instituted the disqualification
proceedings), a registered voter of Makati City,
was competent to bring the action, so was
petitioner since the latter was a rival candidate
for vice mayor of Makati City.
Nor is petitioner's interest in the matter in
litigation any less because he filed a motion for
intervention only on May 20, 1998, after private
respondent had been shown to have garnered the
highest number of votes among the candidates
for vice mayor. That petitioner had a right to
intervene at that stage of the proceedings for the
disqualification against private respondent is
clear from 6 of R.A. No. 6646, otherwise known
as the Electoral Reforms Law of 1987, which
provides:
Any candidate who has been declared by final
judgment to be disqualified shall not be voted for,
and the votes cast for him shall not be counted. If
for any reason a candidate is not declared by final
judgment before an election to be disqualified
and he is voted for and receives the winning
number of votes in such election, the Court or
Commission shall continue with the trial and
hearing of the action, inquiry, or protest and,
upon motion of the complainant or any
intervenor, may during the pendency thereof
order the suspension of the proclamation of such
candidate whenever the evidence of guilt is
strong.
Under this provision, intervention may be allowed
in proceedings for disqualification even after
40 | C O N S T I 2 _ A r t i c l e I V _ C I T I Z E N S H I P

election if there has yet been no final judgment


rendered.
The failure of the COMELEC en banc to resolve
petitioner's
motion
for
intervention
was
tantamount to a denial of the motion, justifying
petitioner in filing the instant petition for
certiorari. As the COMELEC en banc instead
decided the merits of the case, the present
petition properly deals not only with the denial of
petitioner's motion for intervention but also with
the substantive issues respecting private
respondent's alleged disqualification on the
ground of dual citizenship.
This brings us to the next question, namely,
whether private respondent Manzano possesses
dual citizenship and, if so, whether he is
disqualified from being a candidate for vice
mayor of Makati City.
II. DUAL CITIZENSHIP AS A GROUND FOR
DISQUALIFICATION
The disqualification of private respondent
Manzano is being sought under 40 of The Local
Government Code of 1991 (R.A. No. 7160), which
declares as "disqualified from running for any
elective local position: . . . (d) Those with dual
citizenship." This provision is incorporated in the
Charter of the City of Makati. 8
Invoking the maxim dura lex sed lex, petitioner,
as well as the Solicitor General, who sides with
him in this case, contends that through 40(d) of
the Local Government Code, Congress has
"command[ed] in explicit terms the ineligibility of
persons possessing dual allegiance to hold local
elective office."
To begin with, dual citizenship is different from
dual allegiance. The former arises when, as a
result of the concurrent application of the
different laws of two or more states, a person is
simultaneously considered a national by the said
states. 9 For instance, such a situation may arise
when a person whose parents are citizens of a
state which adheres to the principle of jus
sanguinis is born in a state which follows the
doctrine of jus soli. Such a person, ipso facto and
without any voluntary act on his part, is
concurrently considered a citizen of both states.
Considering the citizenship clause (Art. IV) of our
Constitution, it is possible for the following
classes of citizens of the Philippines to possess
dual citizenship:
(1) Those born of Filipino fathers and/or mothers
in foreign countries which follow the principle of
jus soli;
(2) Those born in the Philippines of Filipino
mothers and alien fathers if by the laws of their
fathers' country such children are citizens of that
country;
(3) Those who marry aliens if by the laws of the
latter's country the former are considered
citizens, unless by their act or omission they are
deemed to have renounced Philippine citizenship.
There may be other situations in which a citizen
of the Philippines may, without performing any
act, be also a citizen of another state; but the
above cases are clearly possible given the
constitutional provisions on citizenship.

Dual allegiance, on the other hand, refers to the


situation in which a person simultaneously owes,
by some positive act, loyalty to two or more
states. While dual citizenship is involuntary, dual
allegiance is the result of an individual's volition.
With respect to dual allegiance, Article IV, 5 of
the Constitution provides: "Dual allegiance of
citizens is inimical to the national interest and
shall be dealt with by law." This provision was
included in the 1987 Constitution at the instance
of Commissioner Blas F. Ople who explained its
necessity as follows: 10
. . . I want to draw attention to the fact that dual
allegiance is not dual citizenship. I have
circulated a memorandum to the Bernas
Committee according to which a dual allegiance
and I reiterate a dual allegiance is larger and
more threatening than that of mere double
citizenship which is seldom intentional and,
perhaps, never insidious. That is often a function
of the accident of mixed marriages or of birth on
foreign soil. And so, I do not question double
citizenship at all.
What we would like the Committee to consider is
to take constitutional cognizance of the problem
of dual allegiance. For example, we all know what
happens in the triennial elections of the
Federation of Filipino-Chinese Chambers of
Commerce which consists of about 600 chapters
all over the country. There is a Peking ticket, as
well as a Taipei ticket. Not widely known is the
fact that the Filipino-Chinese community is
represented in the Legislative Yuan of the
Republic of China in Taiwan. And until recently,
the sponsor might recall, in Mainland China in the
People's Republic of China, they have the
Associated Legislative Council for overseas
Chinese wherein all of Southeast Asia including
some European and Latin countries were
represented, which was dissolved after several
years because of diplomatic friction. At that time,
the Filipino-Chinese were also represented in that
Overseas Council.
When I speak of double allegiance, therefore, I
speak of this unsettled kind of allegiance of
Filipinos, of citizens who are already Filipinos but
who, by their acts, may be said to be bound by a
second allegiance, either to Peking or Taiwan. I
also took close note of the concern expressed by
some
Commissioners
yesterday,
including
Commissioner Villacorta, who were concerned
about the lack of guarantees of thorough
assimilation,
and
especially
Commissioner
Concepcion who has always been worried about
minority claims on our natural resources.
Dual allegiance can actually siphon scarce
national capital to Taiwan, Singapore, China or
Malaysia, and this is already happening. Some of
the great commercial places in downtown Taipei
are Filipino-owned, owned by Filipino-Chinese it
is of common knowledge in Manila. It can mean a
tragic capital outflow when we have to endure a
capital famine which also means economic
stagnation, worsening unemployment and social
unrest.
41 | C O N S T I 2 _ A r t i c l e I V _ C I T I Z E N S H I P

And so, this is exactly what we ask that the


Committee kindly consider incorporating a new
section, probably Section 5, in the article on
Citizenship which will read as follows: DUAL
ALLEGIANCE IS INIMICAL TO CITIZENSHIP AND
SHALL BE DEALT WITH ACCORDING TO LAW.
In another session of the Commission, Ople spoke
on the problem of these citizens with dual
allegiance, thus: 11
. . . A significant number of Commissioners
expressed their concern about dual citizenship in
the sense that it implies a double allegiance
under a double sovereignty which some of us who
spoke then in a freewheeling debate thought
would be repugnant to the sovereignty which
pervades the Constitution and to citizenship itself
which implies a uniqueness and which elsewhere
in the Constitution is defined in terms of rights
and obligations exclusive to that citizenship
including, of course, the obligation to rise to the
defense of the State when it is threatened, and
back of this, Commissioner Bernas, is, of course,
the concern for national security. In the course of
those debates, I think some noted the fact that as
a result of the wave of naturalizations since the
decision to establish diplomatic relations with the
People's Republic of China was made in 1975, a
good number of these naturalized Filipinos still
routinely go to Taipei every October 10; and it is
asserted that some of them do renew their oath
of allegiance to a foreign government maybe just
to enter into the spirit of the occasion when the
anniversary of the Sun Yat-Sen Republic is
commemorated. And so, I have detected a
genuine and deep concern about double
citizenship, with its attendant risk of double
allegiance which is repugnant to our sovereignty
and national security. I appreciate what the
Committee said that this could be left to the
determination of a future legislature. But
considering the scale of the problem, the real
impact on the security of this country, arising
from, let us say, potentially great numbers of
double citizens professing double allegiance, will
the Committee entertain a proposed amendment
at the proper time that will prohibit, in effect, or
regulate double citizenship?
Clearly, in including 5 in Article IV on citizenship,
the concern of the Constitutional Commission was
not with dual citizens per se but with naturalized
citizens who maintain their allegiance to their
countries of origin even after their naturalization.
Hence, the phrase "dual citizenship" in R.A. No.
7160, 40(d) and in R.A. No. 7854, 20 must be
understood as referring to "dual allegiance."
Consequently, persons with mere dual citizenship
do not fall under this disqualification. Unlike those
with dual allegiance, who must, therefore, be
subject to strict process with respect to the
termination of their status, for candidates with
dual citizenship, it should suffice if, upon the filing
of their certificates of candidacy, they elect
Philippine citizenship to terminate their status as
persons with dual citizenship considering that
their condition is the unavoidable consequence of
conflicting laws of different states. As Joaquin G.
Bernas, one of the most perceptive members of

the Constitutional Commission, pointed out:


"[D]ual citizenship is just a reality imposed on us
because we have no control of the laws on
citizenship of other countries. We recognize a
child of a Filipino mother. But whether or not she
is considered a citizen of another country is
something completely beyond our control." 12
By
electing
Philippine
citizenship,
such
candidates at the same time forswear allegiance
to the other country of which they are also
citizens and thereby terminate their status as
dual citizens. It may be that, from the point of
view of the foreign state and of its laws, such an
individual has not effectively renounced his
foreign citizenship. That is of no moment as the
following discussion on 40(d) between Senators
Enrile and Pimentel clearly shows: 13
SENATOR ENRILE. Mr. President, I would like to
ask clarification of line 41, page 17: "Any person
with dual citizenship" is disqualified to run for any
elective local position. Under the present
Constitution, Mr. President, someone whose
mother is a citizen of the Philippines but his
father is a foreigner is a natural-born citizen of
the Republic. There is no requirement that such a
natural born citizen, upon reaching the age of
majority, must elect or give up Philippine
citizenship.
On the assumption that this person would carry
two passports, one belonging to the country of
his or her father and one belonging to the
Republic of the Philippines, may such a situation
disqualify the person to run for a local
government position?
SENATOR PIMENTEL. To my mind, Mr. President, it
only means that at the moment when he would
want to run for public office, he has to repudiate
one of his citizenships.
SENATOR ENRILE. Suppose he carries only a
Philippine passport but the country of origin or
the country of the father claims that person,
nevertheless, as a citizen? No one can renounce.
There are such countries in the world.
SENATOR PIMENTEL. Well, the very fact that he is
running for public office would, in effect, be an
election for him of his desire to be considered as
a Filipino citizen.
SENATOR ENRILE. But, precisely, Mr. President,
the Constitution does not require an election.
Under the Constitution, a person whose mother is
a citizen of the Philippines is, at birth, a citizen
without any overt act to claim the citizenship.
SENATOR PIMENTEL. Yes. What we are saying, Mr.
President, is: Under the Gentleman's example, if
he does not renounce his other citizenship, then
he is opening himself to question. So, if he is
really interested to run, the first thing he should
do is to say in the Certificate of Candidacy that: "I
am a Filipino citizen, and I have only one
citizenship."

42 | C O N S T I 2 _ A r t i c l e I V _ C I T I Z E N S H I P

SENATOR ENRILE. But we are talking from


viewpoint of Philippine law, Mr. President. He
always have one citizenship, and that is
citizenship invested upon him or her in
Constitution of the Republic.

the
will
the
the

SENATOR PIMENTEL. That is true, Mr. President.


But if he exercises acts that will prove that he
also acknowledges other citizenships, then he will
probably fall under this disqualification.
This is similar to the requirement that an
applicant for naturalization must renounce "all
allegiance and fidelity to any foreign prince,
potentate, state, or sovereignty" 14 of which at
the time he is a subject or citizen before he can
be issued a certificate of naturalization as a
citizen of the Philippines. In Parado v. Republic, 15
it was held:
[W]hen a person applying for citizenship by
naturalization takes an oath that he renounces
his loyalty to any other country or government
and solemnly declares that he owes his
allegiance to the Republic of the Philippines, the
condition imposed by law is satisfied and
complied with. The determination whether such
renunciation is valid or fully complies with the
provisions of our Naturalization Law lies within
the province and is an exclusive prerogative of
our courts. The latter should apply the law duly
enacted by the legislative department of the
Republic. No foreign law may or should interfere
with its operation and application. If the
requirement of the Chinese Law of Nationality
were to be read into our Naturalization Law, we
would be applying not what our legislative
department has deemed it wise to require, but
what a foreign government has thought or
intended to exact. That, of course, is absurd. It
must be resisted by all means and at all cost. It
would be a brazen encroachment upon the
sovereign will and power of the people of this
Republic.
III. PETITIONER'S ELECTION OF PHILIPPINE
CITIZENSHIP
The record shows that private respondent was
born in San Francisco, California on September 4,
1955, of Filipino parents. Since the Philippines
adheres to the principle of jus sanguinis, while
the United States follows the doctrine of jus soli,
the parties agree that, at birth at least, he was a
national both of the Philippines and of the United
States. However, the COMELEC en banc held that,
by participating in Philippine elections in 1992,
1995, and 1998, private respondent "effectively
renounced his U.S. citizenship under American
law," so that now he is solely a Philippine
national.
Petitioner challenges this ruling. He argues that
merely taking part in Philippine elections is not
sufficient evidence of renunciation and that, in
any event, as the alleged renunciation was made
when private respondent was already 37 years
old, it was ineffective as it should have been
made when he reached the age of majority.
In holding that by voting in Philippine elections
private respondent renounced his American
citizenship, the COMELEC must have in mind

349 of the Immigration and Nationality Act of the


United States, which provided that "A person who
is a national of the United States, whether by
birth or naturalization, shall lose his nationality
by: . . . (e) Voting in a political election in a
foreign state or participating in an election or
plebiscite to determine the sovereignty over
foreign territory." To be sure this provision was
declared unconstitutional by the U.S. Supreme
Court in Afroyim v. Rusk 16 as beyond the power
given to the U.S. Congress to regulate foreign
relations. However, by filing a certificate of
candidacy when he ran for his present post,
private respondent elected Philippine citizenship
and in effect renounced his American citizenship.
Private respondent's certificate of candidacy, filed
on March 27, 1998, contained the following
statements made under oath:
6. I AM A FILIPINO CITIZEN (STATE IF "NATURALBORN" OR "NATURALIZED") NATURAL-BORN
xxx xxx xxx
10. I AM A REGISTERED VOTER OF PRECINCT NO.
747-A,
BARANGAY
SAN
LORENZO,
CITY/MUNICIPALITY OF MAKATI, PROVINCE OF
NCR.
11. I AM NOT A PERMANENT RESIDENT OF, OR
IMMIGRANT TO, A FOREIGN COUNTRY.
12. I AM ELIGIBLE FOR THE OFFICE I SEEK TO BE
ELECTED. I WILL SUPPORT AND DEFEND THE
CONSTITUTION OF THE PHILIPPINES AND WILL
MAINTAIN
TRUE
FAITH
AND
ALLEGIANCE
THERETO; THAT I WILL OBEY THE LAWS, LEGAL
ORDERS AND DECREES PROMULGATED BY THE
DULY CONSTITUTED AUTHORITIES OF THE
REPUBLIC OF THE PHILIPPINES; AND THAT I
IMPOSE
THIS
OBLIGATION
UPON
MYSELF
VOLUNTARILY, WITHOUT MENTAL RESERVATION
OR PURPOSE OF EVASION. I HEREBY CERTIFY
THAT THE FACTS STATED HEREIN ARE TRUE AND
CORRECT OF MY OWN PERSONAL KNOWLEDGE.
The filing of such certificate of candidacy sufficed
to renounce his American citizenship, effectively
removing any disqualification he might have as a
dual citizen. Thus, in Frivaldo v. COMELEC it was
held: 17
It is not disputed that on January 20, 1983
Frivaldo became an American. Would the
retroactivity of his repatriation not effectively
give him dual citizenship, which under Sec. 40 of
the Local Government Code would disqualify him
"from running for any elective local position?" We
answer this question in the negative, as there is
cogent reason to hold that Frivaldo was really
STATELESS at the time he took said oath of
allegiance and even before that, when he ran for
governor in 1988. In his Comment, Frivaldo wrote
that he "had long renounced and had long
abandoned his American citizenship-long before
May 8, 1995. At best, Frivaldo was stateless in the
interim-when he abandoned and renounced his
US citizenship but before he was repatriated to
his Filipino citizenship."
43 | C O N S T I 2 _ A r t i c l e I V _ C I T I Z E N S H I P

On this point, we quote from the assailed


Resolution dated December 19, 1995:
"By the laws of the United States, petitioner
Frivaldo lost his American citizenship when he
took his oath of allegiance to the Philippine
Government when he ran for Governor in 1988, in
1992, and in 1995. Every certificate of candidacy
contains an oath of allegiance to the Philippine
Government."
These factual findings that Frivaldo has lost his
foreign nationality long before the elections of
1995 have not been effectively rebutted by Lee.
Furthermore, it is basic that such findings of the
Commission are conclusive upon this Court,
absent any showing of capriciousness or
arbitrariness or abuse.
There is, therefore, no merit in petitioner's
contention that the oath of allegiance contained
in private respondent's certificate of candidacy is
insufficient to constitute renunciation of his
American citizenship. Equally without merit is
petitioner's contention that, to be effective, such
renunciation should have been made upon
private respondent reaching the age of majority
since no law requires the election of Philippine
citizenship to be made upon majority age.
Finally, much is made of the fact that private
respondent admitted that he is registered as an
American citizen in the Bureau of Immigration
and Deportation and that he holds an American
passport which he used in his last travel to the
United States on April 22, 1997. There is no merit
in this. Until the filing of his certificate of
candidacy on March 21, 1998, he had dual
citizenship. The acts attributed to him can be
considered simply as the assertion of his
American nationality before the termination of his
American citizenship. What this Court said in
Aznar vs. COMELEC 18 applies mutatis mutandis
to private respondent in the case at bar:
. . . Considering the fact that admittedly Osmea
was both a Filipino and an American, the mere
fact that he has a Certificate stating he is an
American does not mean that he is not still a
Filipino. . . . [T]he Certification that he is an
American does not mean that he is not still a
Filipino, possessed as he is, of both nationalities
or citizenships. Indeed, there is no express
renunciation here of Philippine citizenship; truth
to tell, there is even no implied renunciation of
said citizenship. When We consider that the
renunciation needed to lose Philippine citizenship
must be "express," it stands to reason that there
can be no such loss of Philippine citizenship when
there is no renunciation, either "express" or
"implied."

To recapitulate, by declaring in his certificate of


candidacy that he is a Filipino citizen; that he is
not a permanent resident or immigrant of another
country; that he will defend and support the
Constitution of the Philippines and bear true faith
and allegiance thereto and that he does so
without mental reservation, private respondent

has, as far as the laws of this country are


concerned, effectively repudiated his American
citizenship and anything which he may have said
before as a dual citizen.
On the other hand, private respondent's oath of
allegiance to the Philippines, when considered
with the fact that he has spent his youth and
adulthood, received his education, practiced his
profession as an artist, and taken part in past
elections in this country, leaves no doubt of his
election of Philippine citizenship.
His declarations will be taken upon the faith that
he will fulfill his undertaking made under oath.
Should he betray that trust, there are enough
sanctions for declaring the loss of his Philippine
citizenship through expatriation in appropriate
proceedings. In Yu v. Defensor-Santiago, 19 we
sustained the denial of entry into the country of
petitioner on the ground that, after taking his
oath as a naturalized citizen, he applied for the
renewal of his Portuguese passport and declared
in commercial documents executed abroad that
he was a Portuguese national. A similar sanction
can be taken against any one who, in electing
Philippine citizenship, renounces his foreign
nationality, but subsequently does some act
constituting renunciation of his Philippine
citizenship. cdasia
WHEREFORE, the petition for certiorari is
DISMISSED for lack of merit.
SO ORDERED.
Davide, Jr., C.J., Romero, Bellosillo, Melo, Puno,
Vitug, Kapunan, Quisumbing, Buena, GonzagaReyes, and Ynares-Santiago, JJ., concur.
Panganiban and Purisima, JJ., are on leave.
Pardo, J., took no part.
(Re: Vicente D. Ching, B.M. No. 914
(Resolution), October 01, 1999)
EN BANC
[B.M. No. 914 . October 1, 1999.]
RE: APPLICATION FOR ADMISSION TO THE
PHILIPPINE BAR
VICENTE D. CHING, applicant.
The Solicitor General for applicant.
SYNOPSIS
Vicente D. Ching is the legitimate son of spouses
Tat Ching, a Chinese citizen and Prescila A. Dulay,
a Filipino, Ching was born in Francia West, Tubao,
La Union on 11 April 1964. Since birth, he resided
in the Philippines. He is also a Certified Public
Accountant and a registered voter of Tubao, La
Union. In fact, he was elected as member of the
Sangguniang Bayan of Tubao, La Union during the
12 May 1992 synchronized elections. On 17 July
1998, having completed a Bachelor of Laws
course at the St. Louis University, Baguio City, he
filed an application to take the 1998 Bar
Examinations. He was conditionally admitted to
take the Bar Examinations, subject to the
condition that he must submit to the Court proof
of his Philippine citizenship. On 5 April 1999, the
1998 Bar Examinations were released and Ching
was one of the successful examinees. However,
because of the questionable status of his
citizenship, he was not allowed to take his oath
and instead, he was required to submit further
44 | C O N S T I 2 _ A r t i c l e I V _ C I T I Z E N S H I P

proof of his citizenship. In compliance therewith,


on 27 July 1999, Ching filed a Manifestation with
attached Affidavit of Election of Philippine
Citizenship and Oath of Allegiance dated 15 July
1999. DSAEIT
The Court held that Ching failed to validly elect
Philippine citizenship. The span of fourteen (14)
years that lapsed from the time he reached the
age of majority until he finally expressed his
intention to elect Philippine citizenship was
clearly way beyond the contemplation of the
requirement of electing "upon reaching the age of
majority." Moreover, Ching had offered no reason
why he delayed his election of Philippine
citizenship. The prescribed procedure in electing
Philippine citizenship is certainly not a tedious
and painstaking process. All that is required of
the elector is to execute an affidavit of election of
Philippine citizenship and, thereafter, file the
same with the nearest civil registry. Ching's
unreasonable and unexplained delay in making
his election cannot be simply glossed over.
SYLLABUS
1. POLITICAL LAW; CONSTITUTIONAL LAW;
CITIZENSHIP; CHILDREN WITH ALIEN FATHER AND
FILIPINO MOTHER BORN BEFORE JANUARY 17,
1973 MUST ELECT THEIR CITIZENSHIP PURSUANT
TO 1935 Constitution. When Ching was born in
1964, the governing charter was the 1935
Constitution. Under Article IV, Section 1(3) of the
1935 Constitution, the citizenship of a legitimate
child born of a Filipino mother and an alien father
followed the citizenship of the father, unless,
upon reaching the age of majority, the child
elected Philippine citizenship. This right to elect
Philippine citizenship was recognized in the 1973
Constitution when it provided that "(t)hose who
elect Philippine citizenship pursuant to the
provisions of the Constitution of nineteen
hundred and thirty-five" are citizens of the
Philippines. Likewise, this recognition by the 1973
Constitution was carried over to the 1987
Constitution which states that "(t)hose born
before January 17, 1973 of Filipino mothers, who
elect Philippine citizenship upon reaching the age
of majority" are Philippine citizens.
2. ID.; ID.; ID.; 1973 AND 1987 ConstitutionAL
PROVISIONS ON ELECTION OF PHILIPPINE
CITIZENSHIP HAVE NO CURATIVE EFFECT. It
should be noted, however, that the 1973 and
1987 Constitutional provisions on the election of
Philippine citizenship should not be understood as
having a curative effect on any irregularity in the
acquisition of citizenship for those covered by the
1935 Constitution. If the citizenship of a person
was subject to challenge under the old charter, it
remains subject to challenge under the new
charter even if the judicial challenge had not
been commenced before the effectivity of the
new Constitution.
3. ID.; ID.; ID.; COMMONWEALTH ACT NO. 625;
PRESCRIBES PROCEDURE FOR ELECTION OF
CITIZENSHIP. C.A. No. 625 which was enacted
pursuant to Section 1(3), Article IV of the 1935

Constitution, prescribes the procedure that


should be followed in order to make a valid
election of Philippine citizenship. Under Section 1
thereof, legitimate children born of Filipino
mothers may elect Philippine citizenship by
expressing such intention "in a statement to be
signed and sworn to by the party concerned
before any officer authorized to administer oaths,
and shall be filed with the nearest civil registry.
The said party shall accompany the aforesaid
statement with the oath of allegiance to the
Constitution and the Government of the
Philippines."
4. ID.; ID.; ID.; ID.; ID.; ELECTION SHOULD BE
MADE
WITHIN
REASONABLE
TIME
AFTER
ATTAINING AGE OF MAJORITY. However, the
1935 Constitution and C.A. No. 625 did not
prescribe a time period within which the election
of Philippine citizenship should be made. The
1935 Charter only provides that the election
should be made "upon reaching the age of
majority." The age of majority then commenced
upon reaching twenty-one (21) years. In the
opinions of the Secretary of Justice on cases
involving the validity of election of Philippine
citizenship, this dilemma was resolved by basing
the time period on the decisions of this Court
prior to the effectivity of the 1935 Constitution. In
these decisions, the proper period for electing
Philippine citizenship was, in turn, based on the
pronouncements of the Department of State of
the United States Government to the effect that
the election should be made within a "reasonable
time" after attaining the age of majority.
5. ID.; ID.; ID.; ID.; ID.; ID.; "REASONABLE TIME";
CONSTRUED. The phrase "reasonable time"
has been interpreted to mean that the election
should be made within three (3) years from
reaching the age of majority. However, we held in
Cuenco vs. Secretary of Justice, that the three (3)
year period is not an inflexible rule. We said: It is
true that this clause has been construed to mean
a reasonable period after reaching the age of
majority, and that the Secretary of Justice has
ruled that three (3) years is the reasonable time
to elect Philippine citizenship under the
constitutional provision adverted to above, which
period may be extended under certain
circumstances, as when the person concerned
has always considered himself a Filipino.
However, we cautioned in Cuenco that the
extension of the option to elect Philippine
citizenship is not indefinite: Regardless of the
foregoing, petitioner was born on February 16,
1923. He became of age on February 16, 1944.
His election of citizenship was made on May 15,
1951, when he was over twenty-eight (28) years
of age, or over seven (7) years after he had
reached the age of majority. It is clear that said
election has not been made "upon reaching the
age of majority."
6. ID.; ID.; ID.; ID.; ID.; ID.; NOT COMPLIED IN
CASE AT BAR. In the present case, Ching,
having been born 11 April 1964, was already
thirty-five (35) years old when he complied with
the requirements of C.A. No. 625 on 15 June
45 | C O N S T I 2 _ A r t i c l e I V _ C I T I Z E N S H I P

1999, or over fourteen (14) years after he had


reached the age of majority. Based on the
interpretation of the phrase "upon reaching the
age of majority," Ching's election was clearly
beyond, by any reasonable yardstick, the
allowable period within which to exercise the
privilege. It should be stated, in this connection,
that the special circumstances invoked by Ching,
i.e., his continuous and uninterrupted stay in the
Philippines and his being a certified public
accountant, a registered voter and a former
elected public official, cannot vest in him
Philippine citizenship as the law specifically lays
down the requirements for acquisition of
Philippine citizenship by election.
7. ID.; ID.; ID.; ID.; APPLICANT FAILED TO VALIDLY
ELECT PHILIPPINE CITIZENSHIP; CASE AT BAR.
Consequently, we hold that Ching failed to validly
elect Philippine citizenship. The span of fourteen
(14) years that lapsed from the time he reached
the age of majority until he finally expressed his
intention to elect Philippine citizenship is clearly
way
beyond
the
contemplation
of
the
requirement of electing "upon reaching the age of
majority." Moreover, Ching has offered no reason
why he delayed his election of Philippine
citizenship. The prescribed procedure in electing
Philippine citizenship is certainly not a tedious
and painstaking process. All that is required of
the elector is to execute an affidavit of election of
Philippine citizenship and, thereafter, file the
same with the nearest civil registry. Ching's
unreasonable and unexplained delay in making
his election cannot be simply glossed over.
HaAISC
8. ID.; ID.; ID.; PERSON PRIVILEGED TO ELECT
PHILIPPINE CITIZENSHIP HAS ONLY AN INCHOATE
RIGHT TO SUCH CITIZENSHIP. Philippine
citizenship can never be treated like a commodity
that can be claimed when needed and
suppressed when convenient. One who is
privileged to elect Philippine citizenship has only
an inchoate right to such citizenship. As such, he
should avail of the right with fervor, enthusiasm
and promptitude.
RESOLUTION
KAPUNAN, J p:
Can a legitimate child born under the 1935
Constitution of a Filipino mother and an alien
father validly elect Philippine citizenship fourteen
(14) years after he has reached the age of
majority? This is the question sought to be
resolved in the present case involving the
application for admission to the Philippine Bar of
Vicente D. Ching. cdlex
The facts of this case are as follows:
Vicente D. Ching, the legitimate son of the
spouses Tat Ching, a Chinese citizen, and Prescila
A. Dulay, a Filipino, was born in Francia West,
Tubao, La Union on 11 April 1964. Since his birth,
Ching has resided in the Philippines.

On 17 July 1998, Ching, after having completed a


Bachelor of Laws course at the St. Louis
University in Baguio City, filed an application to
take the 1998 Bar Examinations. In a Resolution
of this Court, dated 1 September 1998, he was
allowed to take the Bar Examinations, subject to
the condition that he must submit to the Court
proof of his Philippine citizenship.

In compliance with the above resolution, Ching


submitted on 18 November 1998, the following
documents:
1. Certification, dated 9 June 1986, issued by the
Board of Accountancy of the Professional
Regulations Commission showing that Ching is a
certified public accountant;
2. Voter Certification, dated 14 June 1997, issued
by Elizabeth B. Cerezo, Election Officer of the
Commission on Elections (COMELEC) in Tubao, La
Union showing that Ching is a registered voter of
the said place; and
3. Certification, dated 12 October 1998, also
issued by Elizabeth B. Cerezo, showing that Ching
was elected as a member of the Sangguniang
Bayan of Tubao, La Union during the 12 May 1992
synchronized elections. cdphil
On 5 April 1999, the results of the 1998 Bar
Examinations were released and Ching was one
of the successful Bar examinees. The oath-taking
of the successful Bar examinees was scheduled
on 5 May 1999. However, because of the
questionable status of Ching's citizenship, he was
not allowed to take his oath. Pursuant to the
resolution of this Court, dated 20 April 1999, he
was required to submit further proof of his
citizenship. In the same resolution, the Office of
the Solicitor General (OSG) was required to file a
comment on Ching's petition for admission to the
bar and on the documents evidencing his
Philippine citizenship.
The OSG filed its comment on 8 July 1999, stating
that Ching, being the "legitimate child of a
Chinese father and a Filipino mother born under
the 1935 Constitution was a Chinese citizen and
continued to be so, unless upon reaching the age
of majority he elected Philippine citizenship" 1 in
strict compliance with the provisions of
Commonwealth Act No. 625 entitled "An Act
Providing for the Manner in which the Option to
Elect Philippine Citizenship shall be Declared by a
Person Whose Mother is a Filipino Citizen." The
OSG adds that "(w)hat he acquired at best was
only an inchoate Philippine citizenship which he
could perfect by election upon reaching the age
of majority." 2 In this regard, the OSG clarifies
that "two (2) conditions must concur in order that
the election of Philippine citizenship may be
effective, namely: (a) the mother of the person
making the election must be a citizen of the
Philippines; and (b) said election must be made
'upon reaching the age of majority.'" 3 The OSG
46 | C O N S T I 2 _ A r t i c l e I V _ C I T I Z E N S H I P

then explains the meaning of the phrase "upon


reaching the age of majority:"
The clause "upon reaching the age of majority"
has been construed to mean a reasonable time
after reaching the age of majority which had been
interpreted by the Secretary of Justice to be three
(3) years (VELAYO, supra at p. 51 citing Op., Sec.
of Justice No. 70, s. 1940, Feb. 27, 1940). Said
period may be extended under certain
circumstances, as when a (sic) person concerned
has always considered himself a Filipino (ibid.,
citing Op. Nos. 355 and 422, s. 1955; 3, 12, 46,
86 and 97, s. 1953). But in Cuenco, it was held
that an election done after over seven (7) years
was not made within a reasonable time.
In conclusion, the OSG points out that Ching has
not formally elected Philippine citizenship and, if
ever he does, it would already be beyond the
"reasonable
time"
allowed
by
present
jurisprudence. However, due to the peculiar
circumstances surrounding Ching's case, the OSG
recommends the relaxation of the standing rule
on the construction of the phrase "reasonable
period" and the allowance of Ching to elect
Philippine citizenship in accordance with C.A. No.
625 prior to taking his oath as a member of the
Philippine Bar. llcd
On 27 July 1999, Ching filed a Manifestation,
attaching therewith his Affidavit of Election of
Philippine Citizenship and his Oath of Allegiance,
both dated 15 July 1999. In his Manifestation,
Ching states:
1. I have always considered myself as a Filipino;
2. I was registered as a Filipino and consistently
declared myself as one in my school records and
other official documents;
3. I am practicing a profession (Certified Public
Accountant) reserved for Filipino citizens;
4. I participated in electoral process[es] since the
time I was eligible to vote;
5. I had served the people of Tubao, La Union as a
member of the Sangguniang Bayan from 1992 to
1995;
6. I elected Philippine citizenship on July 15, 1999
in accordance with Commonwealth Act No. 625;
7. My election was expressed in a statement
signed and sworn to by me before a notary
public;
8. I accompanied my election of Philippine
citizenship with the oath of allegiance to the
Constitution and the Government of the
Philippines; LexLib
9. I filed my election of Philippine citizenship and
my oath of allegiance to (sic) the Civil Registrar of
Tubao La Union, and

10. I paid the amount of TEN PESOS (Ps. 10.00) as


filing fees.
Since Ching has already elected Philippine
citizenship on 15 July 1999, the question raised is
whether he has elected Philippine citizenship
within a "reasonable time." In the affirmative,
whether his citizenship by election retroacted to
the time he took the bar examination.
When Ching was born in 1964, the governing
charter was the 1935 Constitution. Under Article
IV, Section 1(3) of the 1935 Constitution, the
citizenship of a legitimate child born of a Filipino
mother and an alien father followed the
citizenship of the father, unless, upon reaching
the age of majority, the child elected Philippine
citizenship. 4 This right to elect Philippine
citizenship was recognized in the 1973
Constitution when it provided that "(t)hose who
elect Philippine citizenship pursuant to the
provisions of the Constitution of nineteen
hundred and thirty-five" are citizens of the
Philippines. 5 Likewise, this recognition by the
1973 Constitution was carried over to the 1987
Constitution which states that "(t)hose born
before January 17, 1973 of Filipino mothers, who
elect Philippine citizenship upon reaching the age
of majority" are Philippine citizens. 6 It should be
noted, however, that the 1973 and 1987
Constitutional provisions on the election of
Philippine citizenship should not be understood as
having a curative effect on any irregularity in the
acquisition of citizenship for those covered by the
1935 Constitution. 7 If the citizenship of a person
was subject to challenge under the old charter, it
remains subject to challenge under the new
charter even if the judicial challenge had not
been commenced before the effectivity of the
new Constitution. 8
C.A. No. 625 which was enacted pursuant to
Section 1(3), Article IV of the 1935 Constitution,
prescribes the procedure that should be followed
in order to make a valid election of Philippine
citizenship. Under Section 1 thereof, legitimate
children born of Filipino mothers may elect
Philippine
citizenship by expressing
such
intention "in a statement to be signed and sworn
to by the party concerned before any officer
authorized to administer oaths, and shall be filed
with the nearest civil registry. The said party shall
accompany the aforesaid statement with the oath
of allegiance to the Constitution and the
Government of the Philippines." cdasia
However, the 1935 Constitution and C.A. No. 625
did not prescribe a time period within which the
election of Philippine citizenship should be made.
The 1935 Charter only provides that the election
should be made "upon reaching the age of
majority." The age of majority then commenced
upon reaching twenty-one (21) years. 9 In the
opinions of the Secretary of Justice on cases
involving the validity of election of Philippine
citizenship, this dilemma was resolved by basing
the time period on the decisions of this Court
prior to the effectivity of the 1935 Constitution. In
these decisions, the proper period for electing
47 | C O N S T I 2 _ A r t i c l e I V _ C I T I Z E N S H I P

Philippine citizenship was, in turn, based on the


pronouncements of the Department of State of
the United States Government to the effect that
the election should be made within a "reasonable
time" after attaining the age of majority. 10 The
phrase "reasonable time" has been interpreted to
mean that the election should be made within
three (3) years from reaching the age of majority.
11 However, we held in Cuenco vs. Secretary of
Justice, 12 that the three (3) year period is not an
inflexible rule. We said:
It is true that this clause has been construed to
mean a reasonable period after reaching the age
of majority, and that the Secretary of Justice has
ruled that three (3) years is the reasonable time
to elect Philippine citizenship under the
constitutional provision adverted to above, which
period may be extended under certain
circumstances, as when the person concerned
has always considered himself a Filipino. 13
However, we cautioned in Cuenco that the
extension of the option to elect Philippine
citizenship is not indefinite:
Regardless of the foregoing, petitioner was born
on February 16, 1923. He became of age on
February 16, 1944. His election of citizenship was
made on May 15, 1951, when he was over
twenty-eight (28) years of age, or over seven (7)
years after he had reached the age of majority. It
is clear that said election has not been made
"upon reaching the age of majority." 14
In the present case, Ching, having been born on
11 April 1964, was already thirty-five (35) years
old when he complied with the requirements of
C.A. No. 625 on 15 June 1999, or over fourteen
(14) years after he had reached the age of
majority. Based on the interpretation of the
phrase "upon reaching the age of majority,"
Ching's election was clearly beyond, by any
reasonable yardstick, the allowable period within
which to exercise the privilege. It should be
stated, in this connection, that the special
circumstances invoked by Ching, i.e., his
continuous and uninterrupted stay in the
Philippines and his being a certified public
accountant, a registered voter and a former
elected public official, cannot vest in him
Philippine citizenship as the law specifically lays
down the requirements for acquisition of
Philippine citizenship by election. cda

Definitely, the so-called special circumstances


cannot constitute what Ching erroneously labels
as informal election of citizenship. Ching cannot
find a refuge in the case of In re: Florencio
Mallare, 15 the pertinent portion of which reads:
And even assuming arguendo that Ana Mallare
were (sic) legally married to an alien, Esteban's
exercise of the right of suffrage when he came of
age, constitutes a positive act of election of
Philippine citizenship. It has been established that
Esteban Mallare was a registered voter as of April

14, 1928, and that as early as 1925 (when he was


about 22 years old), Esteban was already
participating in the elections and campaigning for
certain candidate[s]. These acts are sufficient to
show his preference for Philippine citizenship. 16
Ching's reliance on Mallare is misplaced. The
facts and circumstances obtaining therein are
very different from those in the present case,
thus, negating its applicability. First, Esteban
Mallare was born before the effectivity of the
1935 Constitution and the enactment of C.A. No.
625. Hence, the requirements and procedures
prescribed under the 1935 Constitution and C.A.
No. 625 for electing Philippine citizenship would
not be applicable to him. Second, the ruling in
Mallare was an obiter since, as correctly pointed
out by the OSG, it was not necessary for Esteban
Mallare to elect Philippine citizenship because he
was already a Filipino, he being a natural child of
a Filipino mother. In this regard, the Court stated:
Esteban Mallare, natural child of Ana Mallare, a
Filipina, is therefore himself a Filipino, and no
other act would be necessary to confer on him all
the rights and privileges attached to Philippine
citizenship (U.S. vs. Ong Tianse, 29 Phil. 332;
Santos Co vs. Government of the Philippine
Islands, 42 Phil. 543, Serra vs. Republic, L-4223,
May 12, 1952, Sy Quimsuan vs. Republic, L-4693,
Feb. 16, 1953; Pitallano vs. Republic, L-5111, June
28, 1954). Neither could any act be taken on the
erroneous belief that he is a non-Filipino divest
him of the citizenship privileges to which he is
rightfully entitled. 17
The ruling in Mallare was reiterated and further
elaborated in Co vs. Electoral Tribunal of the
House of Representatives, 18 where we held:
We have jurisprudence that defines 'election' as
both a formal and an informal process.
In the case of In re: Florencio Mallare (59 SCRA 45
[1974]), the Court held that the exercise of the
right of suffrage and the participation in election
exercises constitute a positive act of election of
Philippine
citizenship.
In
the
exact
pronouncement of the Court, we held:
"Esteban's exercise of the right of suffrage when
he came of age, constitutes a positive act of
Philippine
citizenship".
(p.
52;
emphasis
supplied)"
The private respondent did more than merely
exercise his right of suffrage. He has established
his life here in the Philippines.
For those in the peculiar situation of the
respondent who cannot be expected to have
elected Philippine citizenship as they were
already citizens, we apply the In Re Mallare rule.
xxx xxx xxx
The filing of sworn statement or formal
declaration is a requirement for those who still
have to elect citizenship. For those already
48 | C O N S T I 2 _ A r t i c l e I V _ C I T I Z E N S H I P

Filipinos when the time to elect came up, there


are acts of deliberate choice which cannot be less
binding. Entering a profession open only to
Filipinos, serving in public office where citizenship
is a qualification, voting during election time,
running for public office, and other categorical
acts of similar nature are themselves formal
manifestations for these persons.
An election of Philippine citizenship presupposes
that the person electing is an alien. Or his status
is doubtful because he is a national of two
countries. There is no doubt in this case about Mr.
Ong's being a Filipino when he turned twenty-one
(21). cdpr
We repeat that any election of Philippine
citizenship on the part of the private respondent
would not only have been superfluous but it
would also have resulted in an absurdity. How can
a Filipino citizen elect Philippine citizenship? 19
The Court, like the OSG, is sympathetic with the
plight of Ching. However, even if we consider the
special circumstances in the life of Ching like his
having lived in the Philippines all his life and his
consistent belief that he is a Filipino, controlling
statutes and jurisprudence constrain us to
disagree with the recommendation of the OSG.
Consequently, we hold that Ching failed to validly
elect Philippine citizenship. The span of fourteen
(14) years that lapsed from the time he reached
the age of majority until he finally expressed his
intention to elect Philippine citizenship is clearly
way
beyond
the
contemplation
of
the
requirement of electing "upon reaching the age of
majority." Moreover, Ching has offered no reason
why he delayed his election of Philippine
citizenship. The prescribed procedure in electing
Philippine citizenship is certainly not a tedious
and painstaking process. All that is required of
the elector is to execute an affidavit of election of
Philippine citizenship and, thereafter, file the
same with the nearest civil registry. Ching's
unreasonable and unexplained delay in making
his election cannot be simply glossed over.
Philippine citizenship can never be treated like a
commodity that can be claimed when needed and
suppressed when convenient. 20 One who is
privileged to elect Philippine citizenship has only
an inchoate right to such citizenship. As such, he
should avail of the right with fervor, enthusiasm
and promptitude. Sadly, in this case, Ching slept
on his opportunity to elect Philippine citizenship
and, as a result, this golden privilege slipped
away from his grasp. LLjur
IN VIEW OF THE FOREGOING, the Court Resolves
to DENY Vicente D. Ching's application for
admission to the Philippine Bar.
SO ORDERED.
Davide, Jr., C.J., Bellosillo, Melo, Puno, Vitug,
Mendoza, Panganiban, Quisumbing, Purisima,
Pardo, Buena, Gonzaga-Reyes and YnaresSantiago, JJ., concur.

(Bengson III v. HRET, G.R. No. 142840, May


07, 2001)
EN BANC
[G.R. No. 142840. May 7, 2001.]
ANTONIO BENGSON III, petitioner, vs. HOUSE OF
REPRESENTATIVES ELECTORAL TRIBUNAL and
TEODORO C. CRUZ, respondents.
DECISION
KAPUNAN, J p:
The citizenship of respondent Teodoro C. Cruz is
at issue in this case, in view of the constitutional
requirement that "no person shall be a Member of
the House of Representatives unless he is a
natural-born citizen." 1
Respondent Cruz was a natural-born citizen of the
Philippines. He was born in San Clemente, Tarlac,
on April 27, 1960, of Filipino parents. The
fundamental law then applicable was the 1935
Constitution. 2
On November 5, 1985, however, respondent Cruz
enlisted in the United States Marine Corps and,
without the consent of the Republic of the
Philippines, took an oath of allegiance to the
United States. As a consequence, he lost his
Filipino citizenship for under COMMONWEALTH
ACT NO. 63, Section 1(4), a Filipino citizen may
lose his citizenship by, among others, "rendering
service to or accepting commission in the armed
forces of a foreign country." Said provision of law
reads:
SECTION 1. How citizenship may be lost. A
Filipino citizen may lose his citizenship in any of
the following ways and/or events:
xxx xxx xxx
(4) By rendering services to, or accepting
commission in, the armed forces of a foreign
country: Provided, That the rendering of service
to, or the acceptance of such commission in, the
armed forces of a foreign country, and the taking
of an oath of allegiance incident thereto, with the
consent of the Republic of the Philippines, shall
not divest a Filipino of his Philippine citizenship if
either of the following circumstances is present:
(a) The Republic of the Philippines has a
defensive and/or offensive pact of alliance with
said foreign country; or
(b) The said foreign country maintains armed
forces on Philippine territory with the consent of
the Republic of the Philippines: Provided, That the
Filipino citizen concerned, at the time of
rendering said service, or acceptance of said
commission, and taking the oath of allegiance
incident thereto, states that he does so only in
connection with his service to said foreign
country; And provided, finally, That any Filipino
citizen who is rendering service to, or is
49 | C O N S T I 2 _ A r t i c l e I V _ C I T I Z E N S H I P

commissioned in, the armed forces of a foreign


country under any of the circumstances
mentioned in paragraph (a) or (b), shall not be
permitted to participate nor vote in any election
of the Republic of the Philippines during the
period of his service to, or commission in, the
armed forces of said country. Upon his discharge
from the service of the said foreign country, he
shall be automatically entitled to the full
enjoyment of his civil and political rights as a
Filipino citizen . . . .
Whatever doubt that remained regarding his loss
of Philippine citizenship was erased by his
naturalization as a U.S. citizen on June 5, 1990, in
connection with his service in the U.S. Marine
Corps.
On March 17, 1994, respondent Cruz reacquired
his Philippine citizenship through repatriation
under Republic Act No. 2630. 3 He ran for and
was elected as the Representative of the Second
District of Pangasinan in the May 11, 1998
elections. He won by a convincing margin of
26,671 votes over petitioner Antonio Bengson III,
who was then running for reelection.
Subsequently, petitioner filed a case for Quo
Warranto Ad Cautelam with respondent House of
Representatives
Electoral
Tribunal
(HRET)
claiming that respondent Cruz was not qualified
to become a member of the House of
Representatives since he is not a natural-born
citizen as required under Article VI, Section 6 of
the Constitution. 4
On March 2, 2000, the HRET rendered its decision
5 dismissing the petition, for quo warranto and
declaring respondent Cruz the duly elected
Representative of the Second District of
Pangasinan in the May 1998 elections. The HRET
likewise
denied
petitioner's
motion
for
reconsideration of the decision in its resolution
dated April 27, 2000. 6
Petitioner thus filed the present petition for
certiorari assailing the HRET's decision on the
following grounds:
1. The HRET committed serious errors and grave
abuse of discretion, amounting to excess of
jurisdiction, when it ruled that private respondent
is a natural-born citizen of the Philippines despite
the fact that he had ceased being such in view of
the loss and renunciation of such citizenship on
his part. SDEHCc
2. The HRET committed serious errors and grave
abuse of discretion, amounting to excess of
jurisdiction,
when
it
considered
private
respondent as a citizen of the Philippines despite
the fact that he did not validly acquire his
Philippine citizenship.
3. Assuming that private respondent's acquisition
of Philippine citizenship was invalid, the HRET
committed serious errors and grave abuse of
discretion, amounting to excess of jurisdiction,
when it dismissed the petition despite the fact

that such reacquisition could not legally and


constitutionally restore his natural-born status. 7
The issue now before us is whether respondent
Cruz, a natural-born Filipino who became an
American citizen, can still be considered a
natural-born Filipino upon his reacquisition of
Philippine citizenship.
Petitioner asserts that respondent Cruz may no
longer be considered a natural-born Filipino since
he lost his Philippine citizenship when he swore
allegiance to the United States in 1995, and had
to reacquire the same by repatriation. He insists
that Article IV, Section 2 of the Constitution
expressly states that natural-born citizens are
those who are citizens from birth without having
to perform any act to acquire or perfect such
citizenship.
Respondent on the other hand contends that he
reacquired his status as a natural-born citizen
when he was repatriated since the phrase "from
birth" in Article IV, Section 2 refers to the innate,
inherent and inborn characteristic of being a
natural-born citizen.
The petition is without merit.
The 1987 Constitution enumerates
Filipino citizens as follows:

who are

(1) Those who are citizens of the Philippines at


the time of the adoption of this Constitution;
(2) Those whose fathers or mothers are citizens
of the Philippines;
(3) Those born before January 17, 1973 of Filipino
mothers, who elect Philippine citizenship upon
reaching the age of majority, and
(4) Those who are naturalized in accordance with
law. 8
There are two ways of acquiring citizenship: (1)
by birth, and (2) by naturalization. These ways of
acquiring citizenship correspond to the two kinds
of citizens: the natural-born citizen, and the
naturalized citizen. A person who at the time of
his birth is a citizen of a particular country, is a
natural-born citizen thereof. 9
As defined in the same Constitution, natural-born
citizens "are those citizens of the Philippines from
birth without having to perform any act to acquire
or perfect his Philippine citizenship." 10
On the other hand, naturalized citizens are those
who have become Filipino citizens through
naturalization, generally under Commonwealth
Act No. 473, otherwise known as the Revised
Naturalization Law, which repealed the former
Naturalization Law (Act No. 2927), and by
Republic Act No. 530. 11 To be naturalized, an
applicant has to prove that he possesses all the
qualifications 12 and none of the disqualifications
13 provided by law to become a Filipino citizen.
The decision granting Philippine citizenship
50 | C O N S T I 2 _ A r t i c l e I V _ C I T I Z E N S H I P

becomes executory only after two (2) years from


its promulgation when the court is satisfied that
during the intervening period, the applicant has
(1) not left the Philippines; (2) has dedicated
himself to a lawful calling or profession; (3) has
not been convicted of any offense or violation of
Government promulgated rules; or (4) committed
any act prejudicial to the interest of the nation or
contrary to any Government announced policies.
14
Filipino citizens who have lost their citizenship
may however reacquire the same in the manner
provided by law. COMMONWEALTH ACT NO. 63
(CA No. 63), enumerates the three modes by
which Philippine citizenship may be reacquired by
a former citizen: (1) by naturalization, (2) by
repatriation, and (3) by direct act of Congress. 15
Naturalization is a mode for both acquisition and
reacquisition of Philippine citizenship. As a mode
of initially acquiring Philippine citizenship,
naturalization is governed by Commonwealth Act
No. 473, as amended. On the other hand,
naturalization as a mode for reacquiring
Philippine
citizenship
is
governed
by
COMMONWEALTH ACT NO. 63. 16 Under this law,
a former Filipino citizen who wishes to reacquire
Philippine citizenship must possess certain
qualifications 17 and none of the disqualifications
mentioned in Section 4 of C.A. 473. 18
Repatriation, on the other hand, may be had
under various statutes by those who lost their
citizenship due to: (1) desertion of the armed
forces; 19 (2) service in the armed forces of the
allied forces in World War II; 20 (3) service in the
Armed Forces of the United States at any other
time; 21 (4) marriage of a Filipino woman to an
alien; 22 and (5) political and economic necessity.
23
As distinguished from the lengthy process of
naturalization, repatriation simply consists of the
taking of an oath of allegiance to the Republic of
the Philippines and registering said oath in the
Local Civil Registry of the place where the person
concerned resides or last resided.
In Angara v. Republic, 24 we held:
. . . . Parenthetically, under these statutes
[referring to RA Nos. 965 and 2630], the person
desiring to reacquire Philippine citizenship would
not even be required to file a petition in court,
and all that he had to do was to take an oath of
allegiance to the Republic of the Philippines and
to register that fact with the civil registry in the
place of his residence or where he had last
resided in the Philippines. [Emphasis in the
original.] 25

Moreover, repatriation results in the recovery of


the original nationality. 26 This means that a
naturalized Filipino who lost his citizenship will be
restored to his prior status as a naturalized
Filipino citizen. On the other hand, if he was

originally a natural-born citizen before he lost his


Philippine citizenship, he will be restored to his
former status as a natural-born Filipino.
In respondent Cruz's case, he lost his Filipino
citizenship when he rendered service in the
Armed Forces of the United States. However, he
subsequently reacquired Philippine citizenship
under R.A. No. 2630, which provides:
SECTION 1.
Any person who had lost his
Philippine citizenship by rendering service to, or
accepting commission in, the Armed Forces of the
United States, or after separation from the Armed
Forces of the United States, acquired United
States citizenship, may reacquire Philippine
citizenship by taking an oath of allegiance to the
Republic of the Philippines and registering the
same with Local Civil Registry in the place where
he resides or last resided in the Philippines. The
said oath of allegiance shall contain a
renunciation of any other citizenship.
Having thus taken the required oath of allegiance
to the Republic and having registered the same in
the Civil Registry of Magantarem, Pangasinan in
accordance with the aforecited provision,
respondent Cruz is deemed to have recovered his
original status as a natural-born citizen, a status
which he acquired at birth as the son of a Filipino
father. 27 It bears stressing that the act of
repatriation allows him to recover, or return to,
his original status before he lost his Philippine
citizenship.
Petitioner's contention that respondent Cruz is no
longer a natural-born citizen since he had to
perform an act to regain his citizenship is
untenable. As correctly explained by the HRET in
its decision, the term "natural-born citizen" was
first defined in Article III, Section 4 of the 1973
Constitution as follows:
SECTION 4. A natural-born citizen is one who is a
citizen of the Philippines from birth without
having to perform any act to acquire or perfect
his Philippine citizenship.
Two requisites must concur for a person to be
considered as such: (1) a person must be a
Filipino citizen from birth and (2) he does not
have to perform any act to obtain or perfect his
Philippine citizenship.
Under the 1973 Constitution definition, there
were two categories of Filipino citizens which
were not considered natural-born: (1) those who
were naturalized and (2) those born before
January 17, 1973, 28 of Filipino mothers who,
upon reaching the age of majority, elected
Philippine citizenship. Those "naturalized citizens"
were not considered natural-born obviously
because they were not Filipinos at birth and had
to perform an act to acquire Philippine
citizenship. Those born of Filipino mothers before
the effectivity of the 1973 Constitution were
likewise not considered natural-born because
they also had to perform an act to perfect their
Philippine citizenship.
51 | C O N S T I 2 _ A r t i c l e I V _ C I T I Z E N S H I P

The present Constitution, however, now considers


those born of Filipino mothers before the
effectivity of the 1973 Constitution and who
elected Philippine citizenship upon reaching the
majority age as natural-born. After defining who
are natural-born citizens, Section 2 of Article IV
adds a sentence: "Those who elect Philippine
citizenship in accordance with paragraph (3),
Section 1 hereof shall be deemed natural-born
citizens." Consequently, only naturalized Filipinos
are considered not natural-born citizens. It is
apparent from the enumeration of who are
citizens under the present Constitution that there
are only two classes of citizens: (1) those who are
natural-born and (2) those who are naturalized in
accordance with law. A citizen who is not a
naturalized Filipino, i.e., did not have to undergo
the process of naturalization to obtain Philippine
citizenship, necessarily is a natural-born Filipino.
Noteworthy is the absence in said enumeration of
a separate category for persons who, after losing
Philippine citizenship, subsequently reacquire it.
The reason therefor is clear: as to such persons,
they would either be natural-born or naturalized
depending on the reasons for the loss of their
citizenship and the mode prescribed by the
applicable law for the reacquisition thereof. As
respondent Cruz was not required by law to go
through naturalization proceedings in order to
reacquire his citizenship, he is perforce a naturalborn Filipino. As such, he possessed all the
necessary qualifications to be elected as member
of the House of Representatives.
A final point. The HRET has been empowered by
the Constitution to be the "sole judge" of all
contests relating to the election, returns, and
qualifications of the members of the House. 29
The Court's jurisdiction over the HRET is merely
to check "whether or not there has been a grave
abuse of discretion amounting to lack or excess
of jurisdiction" on the part of the latter. 30 In the
absence thereof, there is no occasion for the
Court to exercise its corrective power and annul
the decision of the HRET nor to substitute the
Court's judgment for that of the latter for the
simple reason that it is not the office of a petition
for certiorari to inquire into the correctness of the
assailed decision. 31 There is no such showing of
grave abuse of discretion in this case.
WHEREFORE, the petition is hereby DISMISSED.
Davide, Jr., C.J., Bellosillo and Puno, JJ., concur.
Melo, J., took no part. Chairman of the HRET
which rendered the decision under review.
Vitug, J.,took no part. A member of the HRET
which rendered the appealed judgment.
Mendoza, J., took no part, being ponente of
decision under review.
Panganiban, J., please see concurring opinion.
Quisumbing, Buena and De Leon, Jr., JJ., are on
leave.
Pardo, J., concurs on this and the concurring
opinion of J. Panganiban.
Gonzaga-Reyes, J., also joins concurring opinion
of J. Panganiban.

Ynares-Santiago, J., hereby certifies that J.


Santiago joins with the majority opinion of J.
Kapunan.
Gutierrez, J., Please see dissenting opinion.
(Moy Ya Lim Yao v. Commissioner of
Immigration, G.R. No. L-21289, October 04,
1971)
EN BANC
[G.R. No. L-21289. October 4, 1971.]
MOY YA LIM YAO alias EDILBERTO AGUINALDO LIM
and LAU YUEN YEUNG, petitioners-appellants, vs.
THE COMMISSIONER OF IMMIGRATION,
respondent-appellee.
Aruego, Mamaril & Associates for petitionersappellants.
Solicitor General Arturo A. Alafriz, Asst. Sol. Gen.
Frine C . Zaballero and Solicitor Sumilang V .
Bernardo for respondent-appellee.
SYLLABUS
1. POLITICAL LAW; CITIZENSHIP; IMMIGRATION
ACT; SECTION 9 (G) THEREOF, NOT APPLICABLE
TO ALIEN WHO LEGITIMATELY BECOMES FILIPINO.
Section 9 (g) of the Immigration Act does not
apply to aliens who after coming into the
Philippines as temporary visitors, legitimately
become Filipino citizens or acquire Filipino
citizenship. Such change of nationality naturally
bestows upon them the right to stay in the
Philippines permanently or not, as they may
choose, and if they elect to reside here, the
immigration authorities may neither deport them
nor confiscate their bonds.
2. ID.; ID.; NATURALIZATION; EFFECTS. The
naturalization of an alien visitor as a Philippine
citizen logically produces the effect of conferring
upon him ipso facto all the rights of citizenship
including that of being entitled to permanently
stay in the Philippines outside the orbit of
authority of the Commissioner of Immigration visavis aliens, if only because by its very nature and
express provisions, the Immigration Law is a law
only for aliens and is inapplicable to citizens of
the Philippines.
3.
STATUTORY
CONSTRUCTION;
WHERE
LANGUAGE OF STATUTE IS SUSCEPTIBLE OF TWO
CONSTRUCTIONS, THAT WHICH CARRIES OUT
OBJECT PREVAILS. A statute is to be construed
with reference to its manifest object, and if the
language is susceptible of two constructions, one
which will carry out and the other defeat such
manifest object, it should receive the former
construction.
A
construction
will
cause
objectionable results should be avoided and the
court will, if possible, place on the statute a
construction which will not result in injustice, and
in accordance with the decisions construing
statutes, a construction will not result in
oppression, hardship, or inconveniences will also
be avoided, as will a construction which will
52 | C O N S T I 2 _ A r t i c l e I V _ C I T I Z E N S H I P

prejudice public interest, or construction resulting


in unreasonableness, as well as a construction
which will result in absurd consequences.
4. ID.; CONSTRUCTION AVOIDED IF INCONSISTENT
WITH LEGISLATIVE INTENT. So a construction
should, if possible, be avoided if the result would
be an apparent inconsistency in legislative intent,
as has been determined by the judicial decisions,
or which would result in futility, redundancy, or a
conclusion not contemplated by the legislature;
and the court should adopt that construction
which will be the least likely to produce mischief.
Unless plainly shown to have been the intention
of the legislature an interpretation which would
render the requirements of the statute uncertain
and vague is to be avoided, and the court will not
ascribe to the legislature an intent to confer an
illusory right.
5.
POLITICAL
LAW;
CITIZENSHIP;
NATURALIZATION;
POLICY
OF
SELECTIVE
ADMISSION, EXPLAINED. The avowed policy of
"selective admission" more particularly refers to a
case where a citizenship is sought to be acquired
in a judicial proceeding for naturalization. In such
a case, the courts should no doubt apply the
national policy of selecting only those who are
worthy to be come citizens. There is here a choice
between accepting or rejecting the application for
citizenship. But this policy finds no application is
cases where citizenship is conferred by operation
of law. In such cases, the courts have no choice to
accept or reject. If the individual claiming
citizenship by operations of law proves in legal
proceedings that he satisfies the statutory
requirements, the cannot do otherwise than to
declare that he is a citizens of the Philippines.
6. ID.; ID.; ID.; ALIEN WOMAN MARRYING FILIPINO
IPSO FACTO BECOME CITIZEN PROVIDED NOT
DISQUALIFIED BY LAW. We now hold, all
previous decisions of this Court indicating
otherwise notwithstanding, that under Section 15
of Commonwealth Act 473, an alien woman
marrying a Filipino, native-born or naturalized,
becomes ipso facto a Filipina provided she is not
disqualified to be a citizen of the Philippines
under Section 4 of the same law. Likewise, an
alien woman married an alien who is
subsequently naturalized here follows the
Philippine citizenship of her husband the moment
he takes his oath as Filipino citizens, provided
that she does not suffer from any of the
disqualifications under said Section 4.
7.
ID.;
ID.;
ID.;
ID.;
NATURALIZATION
PROCEEDING, NOT REQUIRED. Section 16 is a
parallel provision to Section 15. If the widow of an
applicant for naturalization a Filipino, who dies
during the proceedings, is not required to go
through a naturalization proceeding, in order to
be considered as a Filipino citizen hereof, it
should not follow that the wife of a living Filipino
cannot be denied that same privilege. This is
plain common sense and there is absolutely no
evidence that the Legislature intended to treat
them differently.

8. ID.; ID.; ID.; MODES OF. The Constitution


itself recognizes as Philippine citizens "Those who
are naturalized in accordance with law" (Section 1
[5], Article IV, Philippine Constitution). Citizens by
naturalization, under this provision, include not
only those who are naturalized in accordance
with legal proceedings for the acquisition of
citizenship, but also those who acquire citizenship
by "derivative naturalization" or by operation of
law, as, for example, the "naturalization" of an
alien wife through the naturalization of her
husband, or by marriage of an alien woman to a
citizen.
9. ID.; ID.; ID.; SECTION 15 OF REVISED
NATURALIZATION LAW; PURPOSE. The leading
idea or purpose of Section 15 was to confer
Philippine citizenship by operation of law upon
certain classes of aliens as a legal consequence
of their relationship, by blood or by affinity, to
persons who are already citizens of the
Philippines. Whenever the fact of relationship of
the persons enumerated in the provisions concurs
with the fact of citizenship of the person to who
they are related, the effect is for said person to
become ipso facto citizens of the Philippines.
"Ipso facto" as here used does not mean that all
alien wives and all minor children of the
Philippine citizens, from the mere fact of
relationship, necessarily become such citizens
also. Those who do not meet the statutory
requirements do not ipso facto become citizens;
they must apply for naturalization in order to
acquire such status. What it does mean, however,
is that in respect of those persons enumerated in
Section 15, the relationship to a citizen of the
Philippines is the operative fact which establishes
the acquisition of Philippine citizenship by them.
Necessarily, it also determines the point of time
at which such citizenship commences.
10. ID.; ID.; ID.; ID.; ALIEN WIFE DEEMED A
CITIZEN IF SHE MIGHT HERSELF BE NATURALIZED.
The legislature could not have intended that an
alien wife should not be deemed a Philippine
citizen unless and until she proves that she might
herself be lawfully naturalized. Far from it, the law
states in plain terms that she shall be deemed a
citizen of the Philippines if she is one "who might
herself be lawfully naturalized." The proviso that
she must be one "who might herself be lawfully
naturalized" is not a condition precedent to the
vesting or acquisition of citizenship; it is only a
condition or a state of fact necessary to establish
her citizenship as a factum probandum, i.e., as a
fact established and proved in evidence. The
word "might," as used in that phrase, precisely
implies that at the time of her marriage to
Philippine citizen, the alien woman "had (the)
power" to become such a citizen herself under
the laws then in force.
11. ID.; ID.; RES JUDICATA NOT APPLICABLE TO
RULINGS THEREON. Everytime the citizenship
of a person is material or indispensable in a
judicial or administrative case, whatever the
corresponding court or administrative authority
decides therein as to such citizenship is generally
not considered as res adjudicata, hence it has to
53 | C O N S T I 2 _ A r t i c l e I V _ C I T I Z E N S H I P

be threshed out again and again as the occasion


may demand.
12. ID.; ID.; NATURALIZATION; PROCEDURES FOR
ALIEN WIFE TO ACQUIRE PHILIPPINE CITIZENSHIP.
Regarding the steps that should be taken by an
alien woman married to a Filipino citizen in order
to acquire Philippine citizenship, the procedure
followed in the Bureau of Immigration is as
follows: The alien woman must file a petition for
the cancellation of her alien certificate of
registration alleging, among other things, that
she is married to a Filipino citizen and that she is
not disqualified from acquiring her husband's
citizenship
pursuant
to
Section
4
of
Commonwealth Act No. 473, as amended. Upon
the filing of the said petition, which should be
accompanied or supported by the joint affidavit of
the petitioner and her Filipino husband to the
effect that the petitioner does not belong to any
of the groups disqualified by the cited Section
from becoming naturalized Filipino citizen, the
Bureau of Immigration conducts an investigation
and thereafter promulgates its order or decision
granting or denying the petition.
REYES, J.B.L., J., dissenting:
POLITICAL LAW; CITIZENSHIP; NATURALIZATION;
ALIEN WOMAN MARRIED TO FILIPINO MUST
PROVE QUALIFICATIONS UNDER SECTION 3.
Our naturalization law separates qualifications
from disqualifications; the positive qualifications
under Section 3 thereof express a policy of
restriction as to candidates for naturalization as
much as the disqualifications under Section 4.
And it has been shown in our decision in the
second Ly Giok Ha case (Ly Giok Ha vs. Galang, L21332 March 18, 1966, 16 SCRA 416) that those
not disqualified under Section 4 would not
necessarily qualify under Section 3, even if the
residence qualification were disregarded. In other
words, by giving to Section 15 of our
Naturalization Law the effect of excluding only
those women suffering from disqualification
under Section 3 could result in admitting to
citizenship woman that Section 2 intends to
exclude. In these circumstances, I do not see why
American interpretation of the words who might
herself be lawfully naturalized should be
considered hinding in this jurisdiction.

DECISION
BARREDO, J p:
Appeal from the following decision of the Court of
First Instance of Manila in its Civil Case No. 49705
entitled Moy Ya Lim Yao, etc., et al. vs. The
Commissioner of Immigration which, brief as it is,
sufficiently depicts the factual setting of and the
fundamental issues involved in this case thus:
"In the instant case, petitioners seek the issuance
of a writ of injunction against the Commissioner
of Immigration, 'restraining the latter and/or his
authorized representative from ordering plaintiff

Lau Yuen Yeung to leave the Philippines and


causing her arrest and deportation and the
confiscation of her bond, upon her failure to do
so.'
"The prayer for preliminary injunction embodied
in the complaint, having been denied, the case
was heard on the merits and the parties
submitted their respective evidence.
"The facts of the case, as substantially and
correctly stated by the Solicitor General are
these:
'On February 8, 1961, Lau Yuen Yeung applied for
a passport visa to enter the Philippines as a nonimmigrant. In the interrogation made in
connection with her application for a temporary
visitor's visa to enter the Philippines, she stated
that she was a Chinese residing at Kowloon,
Hongkong, and that she desired to take a
pleasure trip to the Philippines to visit her great
(grand) uncle Lau Ching Ping for a period of one
month (Exhibits '1,' '1-a,' and '2'). She was
permitted to come into the Philippines on March
13, 1961, and was permitted to stay for a period
of one month which would expire on April 13,
1961. On the date of her arrival, Asher Y, Cheng
filed a bond in the amount of P1,000.00 to
undertake, among others, that said Lau Yuen
Yeung would actually depart from the Philippines
on or before the expiration of her authorized
period of stay in this country or within the period
as in his discretion the Commissioner of
Immigration or his authorized representative
might properly allow. After repeated extensions,
petitioner Lau Yuen Yeung was allowed to stay in
the Philippines up to February 13, 1962 (Exhibit
'4'). On January 25, 1962, she contracted
marriage with Moy Ya Lim Yao alias Edilberto
Aguinaldo Lim an alleged Filipino citizen. Because
of the contemplated action of respondent to
confiscate her bond and order her arrest and
immediate deportation, after the expiration of her
authorized stay, she brought this action for
injunction with preliminary injunction. At the
hearing which took place one and a half years
after her arrival, it was admitted that petitioner
Lau Yuen Yeung could not write either English or
Tagalog. Except for a few words, she could not
speak either English or Tagalog. She could not
name any Filipino neighbor, with a Filipino name
except one, Rosa. She did not know the names of
her brothers-in-law, or sisters-in-law.'
"Under the facts unfolded above, the Court is of
the considered opinion, and so holds, that the
instant petition for injunction cannot be sustained
for the same reasons set forth in the Order of this
Court, dated March 19, 1962, the pertinent
portions of which read:
'First, Section 15 of the Revised Naturalization
Law provides:
"'Effect of the naturalization on wife and children.
Any woman who is now or may hereafter be
married to a citizen of the Philippines, and who
54 | C O N S T I 2 _ A r t i c l e I V _ C I T I Z E N S H I P

might herself be lawfully naturalized shall be


deemed a citizen of the Philippines."
The above-quoted provision is clear and its
import unequivocal and hence it should be held
to mean what it plainly and explicitly expresses in
unmistakable terms. The clause 'who might
herself be lawfully naturalized' incontestably
implies that an alien woman may be deemed a
citizen of the Philippines by virtue of her marriage
to a Filipino citizen only if she possesses all the
qualifications and none of the disqualifications
specified in the law, because these are the
explicit requisites provided by law for an alien to
be naturalized. (Lee Suan Ay, Alberto Tan and Lee
Chiao vs. Emilio Galang, etc., G. R. No. L-11855).
However, from the allegation of paragraph 3 of
the complaint, to wit:
"'3. That plaintiff Lau Yuen Yeung, Chinese by
birth, who might herself be lawfully naturalized as
a Filipino citizen (not being disqualified to become
such by naturalization), is a Filipino citizen by
virtue of her marriage on January 25, 1962 to
plaintiff MOY YA LIM YAO alias EDILBERTO
AGUINALDO LIM, under the Naturalization Laws of
the Philippines."
it can be deduced beyond debate that petitioner
Lau Yuen Yeung while claiming not to be
disqualified, does not and cannot allege that she
possesses all the qualifications to be naturalized,
naturally because, having been admitted as a
temporary visitor only on March 13, 1961, it is
obvious at once that she lacks at least, the
requisite length of residence in the Philippines
(Revised Naturalization Law, Sec. 2, Case No. 2,
Sec. 3, Case No. 3).
'Were if the intention of the law that the alien
woman, to be deemed a citizen of the Philippines
by virtue of marriage to a Filipino citizen, need
only be not disqualified under the Naturalization
Law, it would have been worded "and who herself
is not disqualified to become a citizen of the
Philippines."
'Second, Lau Yuen Yeung, a temporary Chinese
woman visitor, whose authorized stay in the
Philippines, after repeated extensions thereof,
was to expire last February 28, 1962, having
married her co-plaintiff only on January 25, 1962,
or just a little over one month before the expiry
date of her stay, it is evident that said marriage
was effected merely for convenience to defeat or
avoid her then impending compulsory departure,
not to say deportation. This cannot be permitted.
'Third, as the Solicitor General has well stated:
"'5. That petitioner Lau Yuen Yeung, having been
admitted as a temporary alien visitor on the
strength
of
a
deliberate
and
voluntary
representation that she will enter and stay only
for a period of one month and thereby secured a
visa, cannot go back on her representation to
stay permanently without first departing from the
Philippines as she had promised." (Chung Tiao
Bing, et al. vs. Commissioner of Immigration, G.

R. No. L-9966, September 29, 1956; Ong Se Lun


vs. Board of Commissioners, G. R. No. L-6017,
September 16, 1954; Sec. 9, last par., Phil.
Immigration Law).
The aforequoted argument of the Solicitor
General is well buttressed, not only by the
decided cases of the Supreme Court on the point
mentioned above, but also on the very provisions
of Section 9, sub-paragraph (g) of the Philippine
Immigration Act of 1940 which reads:
" 'An alien who is admitted as a non-immigrant
cannot remain in the Philippines permanently. To
obtain permanent admission, a non-immigrant
alien must depart voluntarily to some foreign
country and procure from the appropriate
Philippine Consul the proper visa and thereafter
undergo examination by the Officers of the
Bureau of Immigration at a Philippine port of
entry for determination of his admissibility in
accordance with the requirements of this Act.
(This paragraph is added by Republic Act 503).'"
(Sec. 9, subparagraph (g) of the Philippine
Immigration Act of 1940).
'And fourth, respondent Commissioner of
Immigration is charged with the administration of
all laws relating to immigration (Sec. 3, Com. Act
No. 613) and in the performance of his duties in
relation to alien immigrants, the law gives the
Commissioner of Immigration a wide discretion, a
quasi-judicial function in determining cases
presented to him (Pedro Uy So vs. Commissioner
of Immigration CA-G. R. No. 23336-R, Dec 15,
1960), so that his decision thereon may not be
disturbed unless he acted with abuse of
discretion or in excess of his jurisdiction.'
"It may also be not amiss to state that wife Lau
Yuen Yeung, while she barely and insufficiently
talk in broken Tagalog and English, she admitted
that she cannot write either language."
The only matter of fact not clearly passed upon
by His Honor which could have some bearing in
the resolution of this appeal is the allegation in
the brief of petitioners-appellants, not denied in
the government's brief, that "in the hearing . . . ,
it was shown thru the testimony of the plaintiff
Lau Yuen Yeung that she does not possess any of
the disqualifications for naturalization." Of course,
as an additional somehow relevant factual
matter, it is also emphasized by said appellants
that during the hearing in the lower court, held
almost ten months after the alleged marriage of
petitioners, "Lau Yuen Yeung was already carrying
in her womb for seven months a child by her
husband."
Appellants have assigned six errors allegedly
committed by the court a quo, thus:
I
THE LOWER COURT ERRED IN HOLDING THAT THE
CLAUSE 'WHO MIGHT HERSELF BE LAWFULLY
NATURALIZED' (OF SECTION 15, REVISED
NATURALIZATION LAW) INCONTESTABLY IMPLIES
THAT AN ALIEN WOMAN MAY BE DEEMED A
55 | C O N S T I 2 _ A r t i c l e I V _ C I T I Z E N S H I P

CITIZEN OF THE PHILIPPINES BY VIRTUE OF HER


MARRIAGE TO A FILIPINO CITIZEN, ONLY IF SHE
POSSESSES ALL THE QUALIFICATIONS AND NONE
OF THE DISQUALIFICATIONS SPECIFIED IN THE
LAW.
II
THE LOWER COURT ERRED IN HOLDING THAT A
WOMAN FOREIGNER WHO DOES NOT POSSESS
ANY
OF
THE
DISQUALIFICATIONS
FOR
CITIZENSHIP AND WHO MARRIED A FILIPINO
CITIZEN IS STILL CONSIDERED AN ALIEN EVEN
AFTER SUCH MARRIAGE AS TO FALL WITHIN THE
REQUIREMENT OF SECTION 9, SUB-PARAGRAPH
(9) OF THE PHILIPPINE IMMIGRATION ACT OF
1940.
III
THE COURT ERRED IN CONCLUDING THAT LAU
YUEN YEUNG'S MARRIAGE TO A FILIPINO CITIZEN
WAS ONLY FOR CONVENIENCE, MERELY BECAUSE
THE SAME WAS CELEBRATED JUST OVER A
MONTH BEFORE THE EXPIRY DATE OF HER
AUTHORIZED STAY.
IV
THE LOWER COURT ERRED IN FAILING TO FIND
THAT THE COMMISSIONER OF IMMIGRATION
ACTED WITH ABUSE OF DISCRETION OR IN
EXCESS OF HIS JURISDICTION WHEN SAID
OFFICER THREATENED TO SEND OUT OF THE
COUNTRY PLAINTIFF LAU YUEN YEUNG WITH
WARNING THAT HER FAILURE TO DO SO WOULD
MEAN CONFISCATION OF HER BOND, ARREST
AND IMMEDIATE DEPORTATION, IN SPITE OF THE
FACT THAT LAU YUEN YEUNG IS NOW A FILIPINO
CITIZEN.
V
THE LOWER COURT ERRED IN DISMISSING
PLAINTIFFS-APPELLANTS' COMPLAINT AND IN
REFUSING TO PERMANENTLY ENJOIN THE
COMMISSIONER FROM ORDERING PLAINTIFF LAU
YUEN YEUNG TO LEAVE THE PHILIPPINES AS A
TEMPORARY VISITOR WHICH SHE IS NOT.

VI
THE LOWER COURT ERRED IN REFUSING TO
GRANT PLAINTIFFS-APPELLANTS' MOTION FOR
PRELIMINARY INJUNCTION EMBODIED IN THEIR
COMPLAINT, IN AN ORDER DATED MARCH 19,
1962. (PAGES 36-41, RECORD ON APPEAL).
We need not discuss these assigned errors
separately. In effect, the above decision upheld
the two main grounds of objection of the Solicitor
General to the petition in the court below, viz:
"That petitioner Lau Yuen Yeung, having been
admitted as a temporary alien visitor on the
strength
of
a
deliberate
and
voluntary
representation that she will enter and stay only

for a period of one month and thereby secured a


visa, cannot go back on her representation to
stay permanently without first departing from the
Philippines as she had promised (Chung Tiao
Bing, et al. vs. Commissioner of Immigration, G.R.
No. L-9966, September 29, 1956; Ong Se Lun vs.
Board of Commissioners, G.R. No. L-6017, Sept.
16, 1954, Sec. 9, last par. Phil. Immigration Law);
"That the mere marriage of a Filipino citizen to an
alien does not automatically confer on the latter
Philippine citizenship. The alien wife must
possess all the qualifications required by law to
become a Filipino citizen by naturalization and
none of the disqualifications. (Lee Suan Ay,
Alberto Tan and Lee Chiao vs. Galang, etc., G. R.
No. L-11855, Dec. 25, 1959)"
It is obvious from the nature of these objections
that their proper resolution would necessarily
cover all the points raised in appellants'
assignments of error, hence, We will base our
discussions, more or less, on said objections.
I.
The first objection of the Solicitor General which
covers the matters dealt with in appellants'
second and fourth assignments of error does not
require any lengthy discussion. As a matter of
fact, it seems evident that the Solicitor General's
pose that an alien who has been admitted into
the Philippines as a non-immigrant cannot remain
here permanently unless he voluntarily leaves the
country first and goes to a foreign country to
secure thereat from the appropriate Philippine
consul the proper visa and thereafter undergo
examination by officers of the Bureau of
Immigration at a Philippine port of entry for
determination of his admissibility in accordance
with the requirements of the Philippine
Immigration Act of 1940, as amended by Republic
Act 503, is premised on the assumption that
petitioner Lau Yuen Yeung is not a Filipino citizen.
We note the same line of reasoning in the
appealed decision of the court a quo. Accordingly,
it is but safe to assume that were the Solicitor
General and His Honor of the view that said
petitioner had become ipso facto a Filipina by
virtue of her marriage to her Filipino husband,
they would have held her as entitled to assume
the status of a permanent resident without
having to depart as required of aliens by Section
9(g) of the law.
In any event, to set this point at rest, We hereby
hold that portion of Section 9(g) of the
Immigration Act providing:
"An alien who is admitted as a non-immigrant
cannot remain in the Philippines permanently. To
obtain permanent admission, a non-immigrant
alien must depart voluntarily to some foreign
country and procure from the appropriate
Philippine consul the proper visa and thereafter
undergo examination by the officers of the
Bureau of Immigration at a Philippine port of
entry for determination of his admissibility in
accordance with the requirements of this Act."

56 | C O N S T I 2 _ A r t i c l e I V _ C I T I Z E N S H I P

does not apply to aliens who after coming into


the Philippines as temporary visitors, legitimately
become Filipino citizens or acquire Filipino
citizenship. Such change of nationality naturally
bestows upon them the right to stay in the
Philippines permanently or not, as they may
choose, and if they elect to reside here, the
immigration authorities may neither deport them
nor confiscate their bonds. True it is that this
Court has vehement]y expressed disapproval of
convenient ruses employed by aliens to convert
their status from temporary visitors to permanent
residents in circumvention of the procedure
prescribed by the legal provision already
mentioned, such as in Chiong Tiao Bing vs.
Commissioner of Immigration, 99 Phil. 1020,
wherein, thru Mr. Justice J.B.L. Reyes, the Court,
reiterating the ruling in Ong Se Lun vs. Board of
Immigration Commissioners, 95 Phil. 785, said:
". . . It is clear that if an alien gains admission to
the Islands on the strength of a deliberate and
voluntary representation that he will enter only
for a limited time, and thereby secures the
benefit of a temporary visa, the law will not allow
him
subsequently
to
go
back
on
his
representation and stay permanently, without
first departing from the Philippines as he had
promised. No officer can relieve him of the
departure requirements of section 9 of the
Immigration Act, under the guise of 'change' or
'correction', for the law makes no distinctions,
and no officer is above the law. Any other ruling
would, as stated in our previous decision,
encourage aliens to enter the Islands on false
pretences; every alien so permitted to enter for a
limited time, might then claim a right to
permanent admission, however flimsy such claim
should be, and thereby compel our government
to spend time, money and effort to examining
and verifying whether or not every such alien
really has a right to take up permanent residence
here. In the meanwhile, the alien would be able
to prolong his stay and evade his return to the
port whence he came, contrary to what he
promised to do when he entered. The damages
inherent in such ruling are self-evident."
On the other hand, however, We cannot see any
reason why an alien who has been here as a
temporary visitor but who has in the meanwhile
become a Filipino should be required to still leave
the Philippines for a foreign country, only to apply
thereat for a re-entry here and undergo the
process of showing that he is entitled to come
back, when after all, such right has become
incontestible as a necessary concomitant of his
assumption of our nationality by whatever legal
means this hag been conferred upon him.
Consider, for example, precisely the case of the
minor children of an alien who is naturalized. It is
indubitable that they become ipso facto citizens
of the Philippines. Could it be the law that before
they can be allowed permanent residence, they
still have to be taken abroad so that they may be
processed to determine whether or not they have
a right to have permanent residence here? The
difficulties and hardships which such a
requirement
entails
and
its
seeming
unreasonableness argue against such a rather

absurd construction. Indeed, as early as 1957, in


Ly Giok Ha vs. Galang, 101 Phil. 459, Mr. Justice
Concepcion, our present Chief Justice, already
ruled thus:
". . . (P)etitioners allege that, upon her marriage
to a Filipino, Ly Giok Ha became also a citizen of
the Philippines. Indeed, if this conclusion were
correct, it would follow that, in consequence of
her marriage, she had been naturalized as such
citizen, and, hence the decision appealed from
would have to be affirmed, for section 40(c) of
Commonwealth Act 613 provides that 'in the
event of the naturalization as a Philippine citizen .
. . of the alien on whose behalf the bond deposit
is given, the bond shall be cancelled or the be
deposited shall be returned to the depositor or his
legal representative.'" (At. pp. 462-463) In other
words, the applicable statute itself more than
implies that the naturalization of an alien visitor
as a Philippine citizen logically produces the
effect of conferring upon him ipso facto all the
rights of citizenship including that of being
entitled to permanently stay in the Philippines
outside the orbit of authority of the Commissioner
of Immigration vis-a-vis aliens, if only because by
its very nature and express provisions, the
Immigration Law is a law only for aliens and is
inapplicable to citizens of the Philippines. In the
sense thus discussed, therefore, appellants'
second and fourth assignments of error are well
taken.
II.
Precisely, the second objection of the Solicitor
General sustained by the trial judge is that
appellant Lau Yuen Yeung's marriage to appellant
Moya Lim Yao alias Edilberto Aguinaldo whose
Filipino citizenship is not denied did not have the
effect of making her a Filipino, since it has not
been shown that she "might herself be lawfully
naturalized," it appearing clearly in the record
that she does not possess all the qualifications
required of applicants for naturalization by the
Revised Naturalization Law, Commonwealth Act
473, even if she has proven that she does not
suffer from any of the disqualifications
thereunder. In other words, the Solicitor General
implicitly concedes that had it been established
in the proceedings below that appellant Lau Yuen
Yeung possesses all the qualifications required by
the law of applicants for naturalization, she would
have been recognized by the respondent as a
Filipino citizen in the instant case, without
requiring her to submit to the usual proceedings
for naturalization.
To be sure, this position of the Solicitor General is
in accord with what used to be the view of this
Court since Lee Suan Ay, et al. v. Emilio Galang,
etc., et al., G.R. No. L-11855, promulgated
December 23, 1959, 106 Phil., 706, 713, 1 for it
was only in Zita Ngo Burca vs. Republic, G.R. No.
L-24252 which was promulgated on January 30,
1967 (19 SCRA 186), that over the pen of Mr.
Justice Conrado Sanchez, this Court held that for
an alien woman who marries a Filipino to be
deemed a Filipina, she has to apply for
naturalization in accordance with the procedure
57 | C O N S T I 2 _ A r t i c l e I V _ C I T I Z E N S H I P

prescribed by the Revised Naturalization Law and


prove in said naturalization proceeding not only
that she has all the qualifications and none of the
disqualifications provided in the law but also that
she has complied with all the formalities required
thereby
like
any
other
applicant
for
naturalization, 2 albeit said decision is not yet
part of our jurisprudence inasmuch as the motion
for its reconsideration is still pending resolution.
Appellants are in effect urging Us, however, in
their first and second assignments of error, not
only to reconsider Burca but to even reexamine
Lee Suan Ay which, as a matter of fact, is the
prevailing rule, having been reiterated in all
subsequent decisions up to Go Im Ty. 3

Actually, the first case in which Section 15 of the


Naturalization Law, Commonwealth Act 473,
underwent judicial construction was in the first Ly
Giok Ha case, 4 one almost identical to the one at
bar. Ly Giok Ha, a woman of Chinese nationality,
was a temporary visitor here whose authority to
stay was to expire on March 14, 1956. She filed a
bond to guaranty her timely departure. On March
8, 1956, eight days before the expiration of her
authority to stay, she married a Filipino by the
name of Restituto Lacasta. On March 9, 1956, her
husband
notified
the
Commissioner
of
Immigration of said marriage and, contending
that his wife had become a Filipina by reason of
said marriage, demanded for the cancellation of
her bond, but instead of acceding to such
request, the Commissioner required her to leave,
and upon her failure to do so, on March 16, 1956,
the Commissioner confiscated her bond; a suit
was filed for the recovery of the bond; the lower
court sustained her contention that she had no
obligation to leave because she had become
Filipina by marriage, hence her bond should be
returned. The Commissioner appealed to this
Court. In the said appeal, Mr. Justice Roberto
Concepcion, our present Chief Justice, spoke for
the Court, thus:
"The next and most important question for
determination is whether her marriage to a
Filipino justified or, at least, excused the aforesaid
failure of Ly Giok Ha to depart from the
Philippines on or before March 14, 1956. In
maintaining the affirmative view, petitioners
alleged that, upon her marriage to a Filipino, Ly
Giok Ha became, also, a citizen of the Philippines.
Indeed, if this conclusion were correct, it would
follow that, in consequence of her marriage, she
had been naturalized as such citizen, and, hence,
the decision appealed from would have to be
affirmed, for section 40(c) of Commonwealth Act
No. 613 provides that 'in the event of the
naturalization as a Philippine citizen . . . of the
alien on whose behalf the bond deposit is given,
the bond shall be cancelled or the sum deposited
shall be returned to the depositor or his legal
representative." Thus the issue boils down to
whether an alien female who marries a male
citizen of the Philippines follows ipso facto his
political status.

"The
pertinent
part
of
section
15
of
Commonwealth Act No. 473, upon which
petitioners rely, reads:
'Any woman who is now or may hereafter be
married to a citizen of the Philippines, and who
might herself be lawfully naturalized shall be
deemed a citizen of the Philippines.'
"Pursuant thereto, marriage to a male Filipino
does not vest Philippine citizenship to his foreign
wife, unless she 'herself may be lawfully
naturalized.' As correctly held in an opinion of the
Secretary of Justice (O.p. No. 52, series of 1950),
* this limitation of section 15 excludes, from the
benefits of naturalization by marriage, those
disqualified from being naturalized as citizens of
the Philippines under section 4 of said
Commonwealth Act No. 473, namely:
'(a) Persons opposed to organized government or
affiliated with any association or group of persons
who uphold and teach doctrines opposing all
organized governments;
'(b) Persons defending or teaching the necessity
or propriety of violence, personal assault, or
assassination for the success and predominance
of their ideas;
'(c) Polygamists or believers in the practice of
polygamy;
'(d) Persons convicted of crimes involving moral
turpitude;
'(e) Persons suffering from mental alienation or
incurable contagious diseases;
'(f) Persons who, during the period of their
residence in the Philippines, have not mingled
socially with the Filipinos, or who have not
evinced a sincere desire to learn and embrace the
customs, traditions, and ideals of the Filipinos;
'(g) Citizens or subjects of nations with whom
the . . . Philippines are at war, during the period
of such war;
'(h) Citizens or subjects of a foreign country other
than the United States, whose laws does not
grant Filipinos the right to become naturalized
citizens or subjects thereof.'
"In the case at bar, there is neither proof nor
allegation in the pleadings that Ly Giok Ha does
not fall under any of the classes disqualified by
law. Moreover, as the parties who claim that,
despite her failure to depart from the Philippines
within the period specified in the bond in
question, there has been no breach thereof,
petitioners have the burden of proving her
alleged change of political status, from alien to
citizen. Strictly speaking, petitioners have not
made out, therefore a case against the
respondents-appellants.
"Considering, however, that neither in the
administrative proceedings, nor in the lower
58 | C O N S T I 2 _ A r t i c l e I V _ C I T I Z E N S H I P

court, had the parties seemingly felt that there


was an issue on whether Ly Giok Ha may 'be
lawfully naturalized,' and this being a case of first
impression in our courts, we are of the opinion
that, in the interest of equity and justice, the
parties herein should be given an opportunity to
introduce evidence, if they have any, on said
issue." (At pp. 462-464.).
As may be seen, although not specifically in so
many words, no doubt was left in the above
decision as regards the following propositions:
1. That under Section 15 of Commonwealth Act
473, the Revised Naturalization Law, the marriage
of an alien woman to a Filipino makes her a
Filipina, if she "herself might be lawfully
naturalized";
2. That this Court declared as correct the opinion
of the Secretary of Justice that the limitation of
Section 15 of the Naturalization Law excludes
from the benefits of naturalization by marriage,
only those disqualified from being naturalized
under Section 4 of the law quoted in the decision;
3. That evidence to the effect that she is not
disqualified may be presented in the action to
recover
her
bond
confiscated
by
the
Commissioner of Immigration;
4. That upon proof of such fact, she may be
recognized as Filipina; and
5. That in referring to the disqualifications
enumerated in the law, the Court somehow left
the impression that no inquiry need be made as
to qualifications, 5 specially considering that the
decision cited and footnoted several opinions of
the Secretary of Justice, the immediate superior
of the Commissioner of Immigration, the most
important of which are the following:
"Paragraph (a), section 13 of Act No. 2927, as
amended, (now section 15, Commonwealth Act
No. 473), provided that 'any woman who is now
or may hereafter be married to a citizen of the
Philippines, and who might herself be lawfully
naturalized shall be deemed a citizen of the
Philippines.'
A
similar
provision
in
the
naturalization law of the United States has been
construed as not requiring the woman to have
the qualifications of residence, good character,
etc., as in the case of naturalization by judicial
proceedings, but merely that she is of the race of
persons who may be naturalized. (Kelly v. Owen
[Dist. Col. 1868] 7 Wall 496, 5F, 11, 12; ex parte
Tryason [D. C. Wash. 1914] 215 F. 449, 27 Op.
Atty. Gen. 507). (Op. No. 168, s. 1940 of Justice
Sec. Jose Abad Santos.)
"In a previous opinion rendered for your Office, I
stated that the clause 'who might herself be
lawfully naturalized', should be construed as not
requiring the woman to have the qualifications of
residence, good character, etc., as in cases of
naturalization by judicial proceedings, but merely
that she is of the race of persons who may be
naturalized. (Op. No. 79, s. 1940)

"Inasmuch as the race qualification has been


removed by the Revised Naturalization Law, it
results that any woman who married a citizen of
the Philippines prior to or after June 17, 1939, and
the marriage not having been dissolved, and on
the assumption that she possesses none of the
disqualifications mentioned in Section 4 of
Commonwealth Act No. 473, follows the
citizenship of her husband." (Op. No. 176, v. 1940
of Justice Sec. Jose Abad Santos.)
"From the foregoing narration of facts, it would
seem that the only material point of inquiry is as
to the citizenship of Arce Machura. If he shall be
found to be a citizen of the Philippines, his wife,
Mrs. Lily James Machura, shall likewise be
deemed a citizen of the Philippines pursuant to
the provision of Section 15, Commonwealth Act
No. 473, which reads in part as follows:
'Any woman who is now or may hereafter be
married to a citizen of the Philippines, and who
might herself be lawfully naturalized shall be
deemed a citizen of the Philippines.'
"The phrase 'who might herself be lawfully
naturalized', as contained in the above provision,
means that the woman who is married to a
Filipino citizen must not belong to any of the
disqualified classes enumerated in Section 4 of
the Naturalization Law (Ops., Sec. of Jus., No. 28,
s. 1950; No. 43, s. 1948, No. 95, s. 1941: Nos. 79
and 168, s. 1940). Under the facts stated in the
within papers, Mrs. Machura does not appear to
be among the disqualified classes mentioned in
the law.
"It having been shown that Arce Machura or
Arsenio Guevara was born as an illegitimate of a
Filipino mother, he should be considered as a
citizen of the Philippines in consonance with the
well-settled rule that an illegitimate child follows
the citizenship of his only legally recognized
parent, the mother (Op., Sec. of Jus., Nos. 58, 98
& 281, s. 1948; No. 96, s. 1949). Her husband
being a Filipino, Mrs. Machura must necessarily
be deemed as a citizen of the Philippines by
marriage (Sec. 15, Com. Act No. 473.) (Op. No.
52, s. 1950 of Justice Sec. Ricardo Nepomuceno.)
The logic and authority of these opinions,
compelling as they are, must have so appealed to
this Court that five days later, on May 22, 1957,
in Ricardo Cua v. The Board of Commissioners,
101 Phil. 521, Mr. Justice J.B.L. Reyes, reiterated
the same ruling on the basis of the following
facts:
Tjioe Wu Suan, an Indonesian, arrived in Manila
on November 1, 1952, but it turned out that her
passport was forged. On December 10, 1953, a
warrant was issued for her arrest for purposes of
deportation. Later, on December 20, 1953, she
married Ricardo Cua, a Filipino, and because of
said marriage, the Board of Special Inquiry
considered her a Filipina. Upon a review of the
case, however, the Board of Immigration
Commissioners insisted on continuing with the
59 | C O N S T I 2 _ A r t i c l e I V _ C I T I Z E N S H I P

deportation proceedings and so, the husband


filed prohibition and mandamus proceedings. The
lower court denied the petition. Although this
Court affirmed said decision, it held, on the other
hand, that:

"Granting the validity of marriage, this Court has


ruled in the recent case of Ly Giok Ha v. Galang,
supra, p. 459, that the bare fact of a valid
marriage to a citizen does not suffice to confer his
citizenship upon the wife. Section 15 of the
Naturalization Law requires that the alien woman
who marries a Filipino must show, in addition,
that she 'might herself be lawfully naturalized' as
a Filipino citizen. As construed in the decision
cited, this last condition requires proof that the
woman who married a Filipino is herself not
disqualified under section 4 of the Naturalization
Law.
"No such evidence appearing on record, the claim
of assumption of Filipino citizenship by Tjioe Wu
Suan, upon her marriage to petitioner, is
untenable. The lower court, therefore, committed
no error in refusing to interfere with the
deportation proceedings, where she can anyway
establish the requisites indispensable for her
acquisition of Filipino citizenship, as well as the
alleged validity of her Indonesian passport."
(Ricardo Cua v. The Board of Immigration
Commissioners, G. R. No. L-9997, May 22, 1957,
101 Phil. 521, 523.) [Emphasis supplied]
For emphasis, it is reiterated that in the above
two cases, this Court expressly gave the parties
concerned opportunity to prove the fact that they
were
not
suffering
from
any
of
the
disqualifications of the law without the need of
undergoing any judicial naturalization proceeding.
It may be stated, therefore, that according to the
above decisions, the law in this country, on the
matter of the effect of marriage of an alien
woman to a Filipino is that she thereby becomes
a Filipina, if it can be proven that at the time of
such marriage, she does not possess any of the
disqualifications enumerated in Section 4 of the
Naturalization Law, without the need of
submitting to any naturalization proceedings
under said law.
It is to be admitted that both of the above
decisions made no reference to qualifications,
that is, as to whether or not they need also to be
proved, but, in any event, it is a fact that the
Secretary of Justice understood them to mean
that such qualifications need not be possessed
nor proven. Then Secretary of Justice Jesus
Barrera, who later became a distinguished
member of this Court, 6 so ruled in opinions
rendered by him subsequent to Ly Giok Ha, the
most illustrative of which held:
"At the outset it is important to note that an alien
woman married to a Filipino citizen needs only to
show that she 'might herself be lawfully
naturalized' in order to acquire Philippine
citizenship. Compliance with other conditions of

the statute, such as those relating to the


qualifications of an applicant for naturalization
through judicial proceedings, is not necessary
(See: Leonard v. Grant, 5 Fed. 11; 27 Ops. Atty.
Gen [U.S.] 507; Ops Sec. of Justice, No. 776, s.
1940, and No. 111, s. 1953.
"This view finds support in the case of Ly Giok Ha
et al. v. Galang et al., G.R. No. L-10760,
promulgated May 17, 1957, where the Supreme
Court, construing the abovequoted section of the
Naturalization Law, held that 'marriage to a male
Filipino does not vest Philippine citizenship to his
foreign wife, unless she 'herself may he lawfully
naturalized,' and that 'this limitation of Section 15
excludes, from the benefits of naturalization by
marriage,
those
disqualified
from
being
naturalized as citizens of the Philippines under
Section 4 of said Commonwealth Act No. 473.' In
other words, disqualification for any of the causes
enumerated in Section 4 of the Act is the decisive
factor that defeats the right of the foreign wife of
a Philippine citizen to acquire Philippine
citizenship.
xxx xxx xxx
"Does petitioner, Lim King Bian, belong to any of
these groups ? The Commissioner of Immigration
does not say so but merely predicates his
negative action on the ground that a warrant of
deportation for 'overstaying' is pending against
the petitioner.
"We do not believe the position is well taken.
Since the grounds for disqualification for
naturalization are expressly enumerated in the
law, a warrant of deportation not based on a
finding of unfitness to become naturalized for any
of those specified causes may not be invoked to
negate acquisition of Philippine citizenship by a
foreign wife of a Philippine citizen under Section
15 of the Naturalization Law. (Inclusio unius est
exclusio alterius)" (Op. No. 12, s. 1958 of Justice
Undersec. Jesus G. Barrera.)
"Regarding the steps that should be taken by an
alien woman married to a Filipino citizen in order
to acquire Philippine citizenship, the procedure
followed in the Bureau of Immigration is as
follows: The alien woman must file a petition for
the cancellation of her alien certificate of
registration alleging, among other things, that
she is married to a Filipino citizen and that she is
not disqualified from acquiring her husband's
citizenship
pursuant
to
section
4
of
Commonwealth Act No. 473, as amended. Upon
the filing of said petition, which should be
accompanied or supported by the joint affidavit of
the petitioner and her Filipino husband to the
effect that the petitioner does not belong to any
of the groups disqualified by the cited section
from becoming naturalized Filipino citizen (please
see attached CEB Form 1), the Bureau of
Immigration conducts an investigation and
thereafter promulgates its order or decision
granting or denying the petition." (Op. No. 38, B.
1958 of Justice Sec. Jesus G. Barrera.)

60 | C O N S T I 2 _ A r t i c l e I V _ C I T I Z E N S H I P

"This view finds support in the case of Ly Giok Ha


et al., v. Galang et al. (G.R. No. L-10760,
promulgated May 17, 1957), where the Supreme
Court, construing the above-quoted section in the
Revised Naturalization Law, held that 'marriage to
a male Filipino does not vest Philippine citizenship
to his foreign wife, unless she 'herself may be
lawfully naturalized,' and that 'this limitation of
Section 15 excludes from the benefits of
naturalization by marriage those disqualified from
being naturalized as citizens of the Philippines
under Section 4 of said Commonwealth Act No.
473.' In other words, disqualification for any of
the causes enumerated in section 4 of the Act is
the decisive factor that defeats the right of an
alien woman married to a Filipino citizen to
acquire Philippine citizenship." (Op. 57, s. 1958 of
Justice Sec. Jesus G. Barrera.)
"The contention is untenable. The doctrine
enunciated in the Ly Giok Ha case is not a new
one. In that case, the Supreme Court held that
under
paragraph
1
of
Section
15
of
Commonwealth Act No. 473, 'marriage to a male
Filipino does not vest Philippine citizenship to his
foreign wife unless she "herself may be lawfully
naturalized"', and, quoting several earlier
opinions of the Secretary of Justice, namely: No.
52, s. 1950; No. 168, s. 1940; No. 95, s. 1941; No.
63, s. 1948; No. 28, s. 1950, 'this limitation of
section 15 excludes from the benefits of
naturalization by marriage, those disqualified
from being naturalized as citizens of the
Philippines
under
section
4
of
said
Commonwealth Act No. 473." (Op. 134, B. 1962
of Justice Undersec. Magno S. Gatmaitan.)
It was not until more than two years later that, in
one respect, the above construction of the law
was importantly modified by this Court in Lee
Suan Ay, supra, in which the facts were as
follows:
"Upon expiration of the appellant Lee Suan Ay's
authorized period of temporary stay in the
Philippines (25 March 1955), on 26 March 1955
the Commissioner of Immigration asked the
bondsman to present her to the Bureau of
Immigration within 24 hours from receipt of
notice, otherwise the bond will be confiscated
(Annex 1). For failure of the bondsman to comply
with the foregoing order, on 1 April 1955 the
Commissioner of Immigration ordered the cash
bond confiscated (Annex E). Therefore, there was
an order issued by the Commissioner of
Immigration confiscating or forfeiting the cash
bond. Unlike in forfeiture of bail bonds in criminal
proceedings, where the Court must enter an
order forfeiting the bail bond and the bondsman
must be given an opportunity to present his
principal or give a satisfactory reason for his
inability to do so, before final judgment may be
entered against the bondsman, (section 15, Rule
110; U.S. v. Bonoan, 22 Phil. 1.) in forfeiture of
bonds posted for the temporary stay of an alien
in the Philippines, no court proceeding is
necessary. Once a breach of the terms and
conditions of the undertaking in the bond is
committed, the Commissioner of Immigration

may, under the terms and conditions thereof,


declare it forfeited in favor of the Government."
(In the meanwhile, on April 1, 1955, Lee Suan Ay
and Alberto Tan, a Filipino, were joined in
marriage by the Justice of the Peace of Las Pias,
Rizal.)
Mr. Justice Sabino Padilla speaking for a
unanimous
court
which
included
Justices
Concepcion and Reyes who had penned Ly Giok
Ha and Ricardo Cua, ruled thus:
"The fact that Lee Suan Ay (a Chinese) was
married to a Filipino citizen does not relieve the
bondsman from his liability on the bond. The
marriage took place on 1 April 1955, and the
violation of the terms and conditions of; the
undertaking in the bond failure to depart from
the Philippines upon expiration of her authorized
period of temporary stay in the Philippines (25
March 1955) and failure to report to the
Commissioner of Immigration within 24 hours
from receipt of notice were committed before
the marriage. Moreover, the marriage of a Filipino
citizen to an alien does not automatically confer
Philippine citizenship upon the latter. She must
possesses the qualifications required by law to
become a Filipino citizen by naturalization. **
There is no showing that the appellant Lee Suan
Ay possesses all the qualifications and none of
the disqualifications provided for by law to
become a Filipino citizen by naturalization."
Pertinently to be noted at once in this ruling,
which, to be sure, is the one relied upon in the
appealed decision now before Us, is the fact that
the footnote of the statement therein that the
alien wife "must possess the qualifications
required by law to become a Filipino citizen by
naturalization" makes reference to Section 15,
Commonwealth Act 473 and precisely, also to Ly
Giok Ha v. Galang, supra. As will be recalled, on
the other hand, in the opinions of the Secretary of
Justice explicitly adopted by the Court in Ly Giok
Ha, among them, Opinion No. 176, Series of
1940, above-quoted, it was clearly held that "(I)n
a previous opinion rendered for your Office, I
stated that the clause 'who might herself be
lawfully naturalized', should be construed as not
requiring the woman to have the qualifications of
residence, good character, etc., as in cases of
naturalization by judicial proceedings, but merely
that she is of the race by persons who may be
naturalized." (Op. Na. 79, s. 1940)

Since Justice Padilla gave no reason at all for the


obviously
significant
modification
of
the
construction of the law, it could be said that there
was need for clarification of the seemingly new
posture of the Court. The occasion for such
clarification should have been in Kua Suy, etc., et
al. vs. The Commissioner of Immigration, G.R. No.
L-13790, October 31, 1963, penned by Mr. Justice
J.B.L. Reyes, who had rendered the opinion in
Ricardo Cua, supra, which followed that in Ly Giok
Ha, supra, but apparently seeing no immediate
relevancy in the case on hand then of the
particular point in issue now, since it was not
61 | C O N S T I 2 _ A r t i c l e I V _ C I T I Z E N S H I P

squarely raised therein similarly as in Lee Suan


Ay, hence, anything said on the said matter would
at best be no more than obiter dictum, Justice
Reyes limited himself to holding that "Under
Section 15 of the Naturalization Act, the wife is
deemed a citizen of the Philippines only if she
'might herself be lawfully naturalized,' so that the
fact of marriage to a citizen, by itself alone, does
not suffice to confer citizenship, as this Court has
previously ruled in Ly Giok Ha v. Galang, 54 O.G.
356, and in Cua v. Board of Immigration
Commissioners, 53 O.G. 8567; and there is here
no evidence of record as to the qualifications or
absence of disqualifications of appellee Kua Suy",
without explaining the apparent departure
already pointed out from Ly Giok Ha and Ricardo
Cua. Even Justice Makalintal, who wrote a
separate concurring and dissenting opinion
merely lumped together Ly Giok Ha, Ricardo Cua
and Lee Suan Ay and opined that both
qualifications and non-disqualifications have to be
shown without elucidating on what seemed to be
departure from the said first two decisions.
It was only on November 30, 1963 that to Mr.
Justice Roberto Regala fell the task of
rationalizing the Court's position. In La San Tuang
v. Galang, G.R. No. L-18775, November 30, 1963,
9 SCRA 638, the facts were simply these: 10 San
Tuang, a Chinese woman, arrived in the
Philippines on July 1, 1960 as a temporary visitor
with authority to stay up to June 30, 1961. She
married a Filipino on January 7, 1961, almost six
months before the expiry date at her permit, and
when she was refused to leave after her authority
to stay had expired, she refused to do so,
claiming she had become a Filipina by marriage,
and to bolster her position, she submitted an
affidavit stating explicitly that she does not
possess any of the disqualifications enumerated
in the Naturalization Law, Commonwealth Act
473. When the case reached the court, the trial
judge held for the government that in addition to
not having any of the disqualifications referred to,
there was need that Lo San Tuang should have
also possessed all the qualifications of residence,
moral character, knowledge of a native principal
dialect, etc., provided by the law. Recognizing
that the issue squarely to be passed upon was
whether or not the possession of all the
qualifications were indeed needed to be shown
apart from non-disqualification, Justice Regala
held affirmatively for the Court, reasoning out
thus:
"It is to be noted that the petitioner has anchored
her claim for citizenship on the basis of the
decision laid down in the case of Leonard v.
Grant, 5 Swy. 603, 5 F 11, where the Circuit Court
of Oregon held that it was only necessary that the
woman 'should be a person of the class or race
permitted to be naturalized by existing laws, and
that in respect of the qualifications arising out of
her conduct or opinions, being the wife of a
citizen, she is to be regarded as qualified for
citizenship, and therefore considered a citizen.'
(In explanation of its conclusion, the Court said:
'If, whenever during the life of the woman or
afterwards, the question of her citizenship arises

in a legal proceeding, the party asserting her


citizenship by reason of her marriage with a
citizen must not only prove such marriage, but
also that the woman then possessed all the
further qualifications necessary to her becoming
naturalized under existing laws, the statute will
be practically nugatory, if not a delusion and a
snare. The proof of the facts may have existed at
the time of the marriage, but years after, when a
controversy arises upon the subject, it may be
lost or difficult to find.')
"In other words, all that she was required to prove
was that she was a free white woman or a woman
of African descent or nativity, in order to be
deemed an American citizen, because, with
respect to the rest of the qualifications on
residence, moral character, etc., she was
presumed to be qualified.
"Like the law in the United States, our former
Naturalization Law (Act No. 2927, as amended by
Act No. 3448) specified the classes of persons
who alone might become citizens of the
Philippines, even as it provided who were
disqualified. Thus, the pertinent provisions of that
law provided:
'Section 1. Who may become Philippine citizens.
Philippine citizenship may be acquired by (a)
natives of the Philippines who are not citizens
thereof under the Jones Law; (b) natives of the
Insular possessions of the United States; (c)
citizens of the United States, or foreigners who
under the laws of the United States may become
citizens of said country if residing therein.
'Section 2. Who are disqualified. The following
cannot be naturalized as Philippine citizens: (a)
Persons opposed to organized government or
affiliated with any association or group of persons
who uphold and teach doctrines opposing all
organized government; (b) persons defending or
teaching the necessity or propriety of violence,
personal assault or assassination for the success
and predominance of their ideas; (c) polygamists
or believers in the practice of polygamy; (d)
persons convicted of crimes involving moral
turpitude; (e) persons suffering from mental
alienation or incurable contagious diseases; (f)
citizens or subjects of nations with whom the
United States and the Philippines are at war,
during the period of such war.
'Section 3. Qualifications. The persons
comprised in subsection (a) of section one of this
Act, in order to be able to acquire Philippine
citizenship, must be not less than twenty-one
years of age on the day of the hearing of their
petition.
'The persons comprised in subsections (b) and (c)
of said section one shall, in addition to being not
less than twenty-one years of age on the day of
the hearing of the petition, have all and each of
the following qualifications:

62 | C O N S T I 2 _ A r t i c l e I V _ C I T I Z E N S H I P

'First. Residence in the Philippine Islands for a


continuous period of not less than five years,
except as provided in the next following section;
'Second. To have conducted themselves in a
proper and irreproachable manner during the
entire Period of their residence in the Philippine
Islands, in their relation with the constituted
government as well as with the community in
which they are living;
'Third. To hold in the Philippine Islands real estate
worth not less than one thousand pesos,
Philippine currency, or have some known trade or
profession; and
'Fourth. To speak and write English, Spanish, or
some native tongue.
'In case the petitioner is a foreign subject, he
shall, besides, declare in writing and under oath
his intention of renouncing absolutely and
perpetually all faith and allegiance to the foreign
authority, state or sovereignty of which he was a
native, citizen or subject.'
"Applying the interpretation given by Leonard v.
Grant, supra, to our law as it then stood, alien
women married to citizens of the Philippines
must, in order to be deemed citizens of the
Philippines, be either (1) natives of the Philippines
who were not citizens thereof under the Jones
Law, or (2) natives of other Insular possessions of
the United States, or (3) citizens of the United
States or foreigners who under the laws of the
United States might become citizens of that
country if residing therein. With respect to the
qualifications set forth in Section 3 of the former
law, they were deemed to have the same for all
intents and purposes.
"But, with the approval of the Revised
Naturalization Law (Commonwealth Act No. 473)
on June 17, 1939, Congress has since discarded
class
or
racial
consideration
from
the
qualifications of applicants for naturalization
(according to its proponent, the purpose in
eliminating this consideration was, first, to
remove the features of the existing naturalization
act which discriminated in favor of the
Caucasian} and against Asiatics who are our
neighbors, and are related to us by racial affinity
and, second, to foster amity with all nations
[Sinco, Phil. Political Law 502 11 ed.]), even as
it retained in Section 15 the phrase in question.
The result is that the phrase 'who might herself
be lawfully naturalized' must be understood in
the context in which it is now found, in a setting
so different from that in which it was found by the
Court in Leonard v. Grant.
"The only logical deduction from the elimination
of class or racial consideration is that, as the
Solicitor General points out, the phrase 'who
might herself be lawfully naturalized' must now
be understood as referring to those who under
Section 2 of the law are qualified to become
citizens of the Philippines.

"There is simply no support for the view that the


phrase 'who might herself be lawfully naturalized'
must now be understood as requiring merely that
the alien woman must not belong to the class of
disqualified persons under Section 4 of the
Revised Naturalization Law. Such a proposition
misreads the ruling laid down in Leonard v. Grant.
A person who is not disqualified is not necessarily
qualified to become a citizen of the Philippines,
because the law treats 'qualifications' and
'disqualifications' in separate sections. And then
it must not be lost sight of that even under the
interpretation given to the former law, it was to
be understood that the alien woman was not
disqualified under Section 2 of that law. Leonard
v. Grant did not rule that it was enough if the
alien woman does not belong to the class of
disqualified persons in order that she may be
deemed to follow the citizenship of her husband:
What that case held was that the phrase 'who
might herself be lawfully naturalized, merely
means that she belongs to the class or race of
persons qualified to become citizens by
naturalization the assumption being always
that she is not otherwise disqualified.

"We therefore hold that under the first paragraph


of Section 15 of the Naturalization Law, an alien
woman, who is married to a citizen of the
Philippines, acquires the citizenship of her
husband only if she has all the qualifications and
none of the disqualifications provided by law.
Since there is no proof in this case that petitioner
has all the qualifications and is not in any way
disqualified, her marriage to a Filipino citizen
does not automatically make her a Filipino citizen.
Her affidavit to the effect that she is not in any
way disqualified to become a citizen of this
country was correctly disregarded by the trial
court, the same being self-serving."
Naturally, almost a month later in Sun Peck Yong
V. Commissioner of Immigration, G.R. No L-20784,
December 27, 1963, 9 SCRA 875, wherein the
Secretary of Foreign Affairs reversed a previous
resolution of the preceding administration to
allow Sun Peck Yong and her minor son to await
the taking of the oath of Filipino citizenship of her
husband two years after the decision granting
him nationalization and required her to leave and
this order was contested in court, Justice Barrera
held:
"In the case of Lo San Tuang v. Commissioner of
Immigration (G.R. No. L-18775, promulgated
November 30, 1963; Kua Suy vs. Commissioner of
Immigration, L-13790, promulgated October 31,
1963), we held that the fact that the husband
became
a
naturalized
citizen
does
not
automatically make the wife a citizen of the
Philippines. It must also be shown that she herself
possesses all the qualifications, and none of the
disqualifications, to become a citizen. In this
case, there is no allegation, much less showing,
that petitioner-wife is qualified to become a
Filipino citizen herself. Furthermore, the fact that
a decision was favorably made on the
63 | C O N S T I 2 _ A r t i c l e I V _ C I T I Z E N S H I P

naturalization petition of her husband is no


assurance that he (the husband) would become a
citizen, as to make a basis for the extension of
her temporary stay."
On the same day, in Tong Siok Sy v. Vivo, G.R. No.
L-21136, December 27, 1963, 9 SCRA 876, Justice
Barrera reiterated the same ruling and citing
particularly Lo San Tuang and Kua Suy, held that
the marriage of Tong Siok Sy to a Filipino on
November 12, 1960 at Taichung, Taiwan and her
taking oath of Filipino citizenship before the
Philippine Vice Consul at Taipeh, Taiwan on
January 6, 1961 did not make her a Filipino
citizen, since she came here only in 1961 and
obviously, she had not had the necessary tenyear residence in the Philippines required by the
law.
Such then was the status of the jurisprudential
law on the matter under discussion when Justice
Makalintal sought a reexamination thereof in
Choy King Tee v. Galang, G.R. No. L-18351, March
26, 1965, 13 SCRA 402. Choy King Tee's husband
was granted Philippine citizenship on January 13,
1959 and took the oath on January 31 of the
same year, Choy King Tee first came to the
Philippines in 1955 and kept commuting between
Manila and Hongkong since then, her last visa
before the case being due to expire on February
14, 1961. On January 27, 1961, her husband
asked the Commissioner of Immigration to cancel
her alien certificate of registration, as well as
their child's, for the reason that they were
Filipinos, and when the request was denied as to
the wife, a mandamus was sought, which the trial
court granted. Discussing anew the issue of the
need for qualifications, Justice Makalintal not on]y
reiterated the arguments of Justice Regala in Lo
San Tuang but added further that the ruling is
believed to be in line with the national policy of
selective admission to Philippine citizenship. 7
No wonder, upon this authority, in Austria v.
Conchu, G.R. No. L-20716, June 22, 1965, 14
SCRA 336, Justice J.P. Bengzon readily reversed
the decision of the lower court granting the writs
of mandamus and prohibition against the
Commissioner of Immigration, considering that
Austria's wife, while admitting she did not
possess all the qualifications for naturalization,
had submitted only an affidavit that she had none
of the disqualifications therefor. So also did
Justice Dizon similarly hold eight days later in
Brito v. Commissioner, G.R. No. L-16829, June 30,
1965, 14 SCRA 539.
Then came the second Ly Giok Ha case 8 wherein
Justice J. B. L. Reyes took occasion to expand on
the reasoning of Choy King Tee by illustrating with
examples "the danger of relying exclusively on
the absence of disqualifications, without taking
into account the other affirmative requirements of
the law." 9
Lastly, in Go Im Ty v. Republic, G.R. No. L-17919,
decided on July 30, 1966, 10 Justice Zaldivar held
for the Court that an alien woman who is
widowed
during
the
pendency
of
the
naturalization proceedings of her husband, in

order that she may be allowed to take the oath as


Filipino, must, aside from proving compliance with
the requirements of Republic Act 530, show that
she possesses all the qualifications and does not
suffer from any of the disqualifications under the
Naturalization Law, citing in the process the
decision to such effect discussed above, 1 1 even
as he impliedly reversed pro tanto the ruling in
Tan Lin v. Republic, G.R. No. L-13786, May 31,
1961, 2 SCRA 383.
Accordingly, in Burca, Justice Sanchez premised
his opinion on the assumption that the point now
under discussion is settled law.
In the case now at bar, the Court is again called
upon to rule on the same issue. Under Section 15
of the Naturalization Law, Commonwealth Act
473, providing that:
"SEC. 15. Effect of the naturalization on wife and
children. Any woman, who is now or may
hereafter be married to a citizen of the
Philippines, and who might herself be lawfully
naturalized shall be deemed a citizen of the
Philippines.
"Minor children of persons naturalized under this
law who have been born in the Philippines shall
be considered citizens thereof.
"A foreign-born minor child, if dwelling in the
Philippines at the time of the naturalization of the
parent, shall automatically become a Philippine
citizen, and a foreign-born child, who is not in the
Philippines at the time the parent is naturalized,
shall be deemed a Philippine citizen only during
his minority, unless he begins to reside
permanently in the Philippines when still a minor,
in which case, he will continue to be a Philippine
citizen even after becoming of age.
"A child born outside of the Philippines after the
naturalization of his parent, shall be considered a
Philippine citizen, unless within one year after
reaching the age of majority he fails to register
himself as a Philippine citizen at the American
Consulate of the country where he resides, and to
take the necessary oath of allegiance.
is it necessary, in order that an alien woman who
marries a Filipino or who is married to a man who
subsequently becomes a Filipino, may become a
Filipino citizen herself, that, aside from not
suffering from any of the disqualifications
enumerated in the law, she must also possess all
the qualifications required by said law? If nothing
but the unbroken line from Lee Suan Ay to Go Im
Ty, as recounted above, were to be considered, it
is obvious that an affirmative answer to the
question would be inevitable, specially, if it is
noted that the present case was actually
submitted for decision on January 21, 1964 yet,
shortly after Lo San Tuang, Tong Siok Sy and Sun
Peck Yong, all supra, and even before Choy King
Tee, supra, were decided. There are other
circumstances, however, which make it desirable,
if not necessary, that the Court take up the
matter anew. There has been a substantial
64 | C O N S T I 2 _ A r t i c l e I V _ C I T I Z E N S H I P

change in the membership of the Court since Go


Im Ty, and of those who were in the Court already
when Burca was decided, two members, Justice
Makalintal and Castro concurred only in the
result, precisely, according to them, because they
wanted to leave the point now under discussion
open in so far as they are concerned. 12 Truth to
tell, the views and arguments discussed at length
with copious relevant authorities, in the motion
for
reconsideration
as
well
as
in
the
memorandum of the amici curiae 13 in the Burca
case cannot just be taken lightly and summarily
ignored, since they project in the most forceful
manner, not only the legal and logical angles of
the issue, but also the imperative practical
aspects thereof in the light of the actual situation
of the thousands of alien wives of Filipinos who
have so long, even decades, considered
themselves as Filipinas and have always lived and
acted as such, officially or otherwise, relying on
the long standing continuous recognition of their
status as such by the administrative authorities in
charge of the matter, as well as by the courts.
Under these circumstances, and if only to afford
the Court an opportunity to consider the views of
the five justices who took no part in Ga Im Ty
(including the writer of this opinion), the Court
decided to further reexamine the matter. After all,
the ruling first laid in Lee Suan Ay, and later in Lo
San Tuang, Choy King Tee and the second (1966)
Ly Giok Ha, did not categorically repudiate the
opinions of the Secretary of Justice relied upon by
the first (1959) Ly Giok Ha. Besides, some points
brought to light during the deliberations in this
case would seem to indicate that the premises of
the
later
cases
can
still
bear
further
consideration.
Whether We like it or not, it is undeniably factual
that the legal provision We are construing,
Section 15, aforequoted, of the Naturalization
Law has been taken directly, copied and adopted
from its American counterpart. To be more
accurate, said provision is nothing less than a
reenactment of the American provision. A brief
review of its history proves this beyond per
adventure of doubt.
The first Naturalization Law of the Philippines
approved by the Philippine Legislature under
American sovereignty was that of March 26,
1920, Act No. 2927. Before then, as a
consequence of the Treaty of Paris, our citizenship
laws were found only in the Organic Laws, the
Philippine Bill of 1902, the Act of the United
States Congress of March 23, 1912 and later the
Jones Law of 1916. In fact, Act No. 2927 was
enacted pursuant to express authority granted by
the Jones Law. For obvious reasons, the
Philippines gained autonomy on the subjects of
citizenship and immigration only after the
effectivity of the Philippine Independence Act.
This made it practically impossible for our laws on
said subject to have any perspective or
orientation of our own; everything was American.

The Philippine Bill of 1902 provided pertinently:

"SECTION 4. That all inhabitants of the Philippine


Islands continuing to reside therein who were
Spanish subjects on the eleventh day of April,
eighteen-hundred and ninety-nine, and then
resided in said Islands, and their children born
subsequent thereto, shall be deemed and held to
be citizens of the Philippine Islands and as such
entitled to the protection of the United States,
except such as shall have elected to preserve
their allegiance to the Crown of Spain in
accordance with the provisions of the treaty of
peace between the United States and Spain
signed at Paris December tenth, eighteen
hundred and ninety-eight."
This Section 4 of the Philippine Bill of 1902 was
amended by Act of Congress of March 23, 1912,
by adding a provision as follows:
"Provided, That the Philippine Legislature is
hereby authorized to provide by law for the
acquisition of Philippine citizenship by those
natives of the Philippine Islands who do not come
within the foregoing provisions, the natives of
other insular possessions of the United States,
and such other persons residing in the Philippine
Islands who would become citizens of the United
States, under the laws of the United States, if
residing therein."
The Jones Law
substantially:

reenacted

these

provisions

"SECTION 2. That all inhabitants of the Philippine


Islands who were Spanish subjects on the
eleventh day of April, eighteen hundred and
ninety-nine, and then resided in said islands, and
their children born subsequent thereto, shall be
deemed and held to be citizens of the Philippine
Islands, except such as shall have elected to
preserve their allegiance to the Crown of Spain in
accordance with the provisions of the treaty of
peace between the United States and Spain,
signed at Paris December tenth, eighteen
hundred and ninety-eight and except such others
as have since become citizens of some other
country: Provided, That the Philippine Legislature,
herein provided for, is hereby authorized to
provide by law for the acquisition of Philippine
citizenship by those natives of the Philippine
Islands who do not come within the foregoing
provisions, the natives of the insular possessions
of the United States, and such other persons
residing in the Philippine Islands who are citizens
of the United States under the laws of the United
States if residing therein."
For aught that appears, there was nothing in any
of the said organic laws regarding the effect of
marriage to a Filipino upon the nationality of an
alien woman, albeit under the Spanish Civil Code
provisions on citizenship, Articles 17 to 27, which
were, however, abrogated upon the change of
sovereignty, it was unquestionable that the
citizenship of the wife always followed that of the
husband. Not even Act 2927 contained any
provision regarding the effect of naturalization of
an alien upon the citizenship of his alien wife, nor
of the marriage of such alien woman with a
native born Filipino or one who had become a
65 | C O N S T I 2 _ A r t i c l e I V _ C I T I Z E N S H I P

Filipino before the marriage, although Section 13


thereof provided thus:
"SEC. 13. Right of widow and children of
petitioners who have died. In case a petitioner
should die before the final decision has been
rendered, his widow and minor children may
continue the proceedings. The decision rendered
in the case shall, so far as the widow and minor
children are concerned, produce the same legal
effect as if it had been rendered during the life of
the petitioner."
It was not until November 30, 1928, upon the
approval of Act 3448, amending Act 2977, that
the following provisions were added to the above
Section 13:
"SECTION 1. The following new sections are
hereby inserted between sections thirteen and
fourteen of Act Numbered Twenty-nine hundred
and Twenty-seven:
'SEC. 13 (a). Any woman who is now or may
hereafter be married to a citizen of the Philippine
Islands and who might herself be lawfully
naturalized, shall be deemed a citizen of the
Philippine Islands.
'SEC. 13 (b). Children of persons who have been
duly naturalized under this law, being under the
age of twenty-one years at the time of the
naturalization of their parents, shall, if dwelling in
the Philippine Islands, be considered citizens
thereof.
'SEC. 13 (c). Children of persons naturalized
under this law who have been born in the
Philippine Islands after the naturalization of their
parents shall be considered citizens thereof.' "
When Commonwealth Act 473, the current
naturalization law, was enacted on June 17, 1939,
the above Section 13 became its Section 15
which has already been quoted earlier in this
decision. As can be seen, Section 13(a)
abovequoted was re-enacted practically word for
word in the first paragraph of this Section 15
except for the change of Philippine Islands to
Philippines. And it could not have been on any
other basis than this legislative history of our
naturalization law that each and everyone of the
decisions of this Court from the first Ly Giok Ha to
Go Im Ty, discussed above, were rendered.
As stated earlier, in the opinion of Chief Justice
Concepcion in the first Ly Giok Ha, it was quite
clear that for an alien woman who marries a
Filipino to become herself a Filipino citizen, there
is no need for any naturalization proceeding
because she becomes a Filipina ipso facto from
the time of such marriage, provided she does not
suffer any of the disqualifications enumerated in
Section 4 of Commonwealth Act 473, with no
mention being made of whether or not the
qualifications enumerated in Section 2 thereof
need be shown. It was only in Lee Suan Ay in
1959 that the possession of qualifications were
specifically required, but it was not until 1963, in

Lo San Tuang, that Justice Regala reasoned out


why the possession of the qualifications provided
by the law should also be shown to be possessed
by the alien wife of a Filipino, for her to become a
Filipina by marriage.
As may be recalled, the basic argument advanced
by Justice Regala was briefly as follows: That "like
the law in the United States, our Naturalization
Law specified the classes of persons who alone
might become citizens, even as it provided who
were
disqualified,"
and
inasmuch
as
Commonwealth Act 473, our Naturalization Law
since 1939 did not reenact the section providing
who might become citizens, allegedly in order to
remove racial discrimination in favor of
Caucasians and against Asiatics, "the only logical
deduction . . . is that the phrase 'who might
herself be lawfully naturalized' must now be
understood as referring to those who under
Section 2 of the law are qualified to become
citizens of the Philippines" and "there is simply no
support for the view that the phrase 'who might
herself be lawfully naturalized' must now be
understood as requiring merely that the alien
woman must not belong to the class of
disqualified persons under Section 4 of the
Revised Naturalization Law." 14
A similar line of reasoning was followed in Choy
King Tee, which for ready reference may be
quoted:
"The question has been settled by the uniform
ruling of this Court in a number of cases. The
alien wife of a Filipino citizen must first prove that
she has all the qualifications required by Section
2 and none of the disqualifications enumerated in
Section 4 of the Naturalization Law before she
may he deemed a Philippine citizen (Lao Chay v.
Galang, L-19977, Oct. 30, 1964, citing Lo San
Tuang v. Galang, L-18775, Nov. 30, 1963; Sun
Peck Yong v. Commissioner of Immigration, L20784, December 27, 1963; Tong Siok Sy v. Vivo,
L-21136, December 27, 1963). The writer of this
opinion has submitted the question anew to the
court for a possible reexamination of the said
ruling in the light of the interpretation of a similar
law in the United States after which Section 15 of
our Naturalization Law was patterned. That law
was section 2 of the Act of February 10, 1855
(Section 1994 of the Revised Statutes of the
U.S.). The local law, Act No. 3448, was passed on
November 30, 1928 as an amendment to the
former Philippine Naturalization Law, Act No.
2927, which was approved on March 26, 1920.
Under this Naturalization Law, acquisition of
Philippine citizenship was limited to three classes
of persons, (a) Natives of the Philippines who
were not citizens thereof; (b) natives of the other
insular possessions of the United States; and (c)
citizens of the United States, or foreigners who,
under the laws of the United States, may become
citizens of the latter country if residing therein.
The reference in subdivision (c) to foreigners who
may become American Citizens is restrictive in
character, for only persons of certain specified
races were qualified thereunder. In other words,
in so far as racial restrictions were concerned
66 | C O N S T I 2 _ A r t i c l e I V _ C I T I Z E N S H I P

there was at the time a similarity between the


naturalization laws of the two countries, and
hence there was reason to accord here
persuasive force to the interpretation given in the
United States to the statutory provision
concerning the citizenship of alien women
marrying American citizens.
"This Court, however, believes that such reason
has ceased to exist since the enactment of the
Revised Naturalization Law (Commonwealth Act
No. 473) on June 17, 1939. The racial restrictions
have been eliminated in this Act, but the
provision found in Act No. 3448 has been
maintained. It is logical to presume that when
Congress chose to retain the said provision
that to be deemed a Philippine citizen upon
marriage the alien wife must be one 'who might
herself be lawfully naturalized,' the reference is
no longer to the class or race to which the woman
belongs, for class or race has become immaterial,
but to the qualifications and disqualifications for
naturalization as enumerated in Sections 2 and 4
of the statute. Otherwise the requirement that
the woman 'might herself be lawfully naturalized'
would be meaningless surplusage, contrary to
settled norms of statutory construction.
"The rule laid down by this Court in this and in
other cases heretofore decided is believed to be
in line with the national policy of selective
admission to Philippine citizenship, which after all
is a privilege granted only to those who are found
worthy thereof, and not indiscriminately to
anybody at all on the basis alone of marriage to a
man who is a citizen of the Philippines,
irrespective of moral character, ideological
beliefs, and identification with Filipino ideals,
customs and traditions.

"Appellee here having failed to prove that she has


all the qualifications for naturalization, even,
indeed, that she has none of the disqualifications,
she is not entitled to recognition as a Philippine
citizen."
In the second Ly Giok Ha, the Court further
fortified the arguments in favor of the same
conclusion thus:
"On cross-examination, she (Ly Giok Ha) failed to
establish that: (1) she has been residing in the
Philippines for a continuous period of at least (10)
years (p. 27, t.s.n., id.); (2) she has a lucrative
trade, profession, or lawful occupation (p. 13.
t.s.n., id.); and (3) she can speak and write
English, or any of the principal Philippine
languages (pp. 12, 13, t.s.n., id.)
"While the appellant Immigration Commissioner
contends that the words emphasized indicate
that the present Naturalization Law requires that
an alien woman who marries a Filipino husband
must possess the qualifications prescribed by
section 2 in addition to not being disqualified
under any of the eight ('a' to 'h') subheadings of
section 4 of Commonwealth Act No. 473, in order

to claim our citizenship by marriage, both the


appellee and the court below (in its second
decision) sustain the view that all that the law
demands is that the woman be not disqualified
under section 4.
"At the time the present case was remanded to
the court of origin (1960) the question at issue
could be regarded as not conclusively settled,
there being only the concise pronouncement in
Lee Suan Ay, et al. v. Galang, G. R. No. L-11855,
Dec. 23, 1959, to the effect that:
'The marriage of a Filipino citizen to an alien does
not automatically confer Philippine citizenship
upon the latter. She must possess the
qualifications required by law to become a Filipino
citizen by naturalization.'
"Since that time, however, a long line of decisions
of this Court has firmly established the rule that
the requirement of section 15 of Commonwealth
Act 473 (the Naturalization Act), that an alien
woman married to a citizen should be one who
'might herself be lawfully naturalized," means not
only woman free from the disqualifications
enumerated in section 4 of the Act but also one
who possesses the qualifications prescribed by
section 2 of Commonwealth Act 473 (San Tuan v.
Galang, L-18775, Nov. 30, 1963; Sun Peck Yong v.
Com. of Immigration, L-20784, Dec. 27, 1963;
Tong Siok Sy v. Vivo, L-21136, Dec. 27, 1963;
Austria v. Conchu, L-20716, June 22, 1965; Choy
King Tee v. Galang, L-18351, March 26, 1965;
Brito v. Com. of Immigration, L-16829, June 30,
1965).
"Reflection will reveal why this must be so. The
qualifications prescribed under section 2 of the
Naturalization Act, and the disqualifications
enumerated in its section 4 are not mutually
exclusive; and if all that were to be required is
that the wife of a Filipino be not disqualified
under section 4, the result might well be that
citizenship would be conferred upon persons in
violation of the policy of the statute. For example,
section 4 disqualifies only
'(c) Polygamists or believers in the practice of
polygamy; and
'(d) Persons convicted of crimes involving moral
turpitude,'
so that a blackmailer, or a maintainer of gambling
or bawdy houses, not previously convicted by a
competent court would not be thereby
disqualified; still, it is certain that the law did not
intend such person to be admitted as a citizen in
view of the requirement of section 2 that an
applicant for citizenship 'must be of good moral
character.'
"Similarly, the citizen's wife might be a convinced
believer in racial supremacy, in government by
certain selected classes, in the right to vote
exclusively by certain 'herrenvolk', and thus
disbelieve in the principles underlying the
Philippine Constitution; yet she would not be
67 | C O N S T I 2 _ A r t i c l e I V _ C I T I Z E N S H I P

disqualified under section 4, as long as she is not


'opposed to organized government,' nor affiliated
to groups 'upholding or teaching doctrines
opposing all organized governments', nor
'defending or teaching the necessity or propriety
of violence, personal assault or assassination for
the success or predominance of their ideas.' Et
sic de caeteris.
"The foregoing instances should suffice to
illustrate the danger of relying exclusively on the
absence of disqualifications, without taking into
account the other affirmative requirements of the
law, which, in the case at bar, the appellee Ly
Giok Ha admittedly does not possess.
"As to the argument that the phrase 'might
herself be lawfully naturalized' was derived from
the U.S. Revised Statutes (section 1994) and
should be given the same territorial and racial
significance given to it by American courts, this
Court has rejected the same in Lon San Tuang v.
Galang, L-18775, November 30, 1963; and in
Choy King Tee v. Galang, L-18351, March 26,
1965."
It is difficult to minimize the persuasive force of
the foregoing rationalizations, but a closer study
thereof cannot but reveal certain relevant
considerations which adversely affect the
premises on which they are predicated, thus
rendering the conclusions arrived thereby not
entirely unassailable.
1. The main proposition, for instance, that in
eliminating Section 1 of Act 2927 providing who
are eligible for Philippine citizenship, the purpose
of Commonwealth Act 473, the Revised
Naturalization Law, was to remove the racial
requirements for naturalization, thereby opening
the door of Filipino nationality to Asiatics instead
of allowing the admission thereto of Caucasians
only, suffers from lack of exact accuracy. It is
important to note, to start with, that
Commonwealth Act 473 did away with the whole
Section 1 of Act 2927 which reads thus:
"SECTION 1. Who may become Philippines
citizens. Philippine citizenship may be acquired
by: (a) natives of the Philippines who are not
citizens thereof under the Jones Law; (b) natives
of the other Insular possessions of the United
States; (c) citizens of the United States, or
foreigners who under the laws of the United
States may become citizens of said country if
residing therein."
and not only subdivision (c) thereof. Nowhere in
this whole provision was there any mention of
race or color of the persons who were then
eligible for Philippine citizenship. What is more
evident from said provision is that it reflected the
inevitable subordination of our legislation during
the pre-Commonwealth American regime to the
understandable limitations flowing from our
status as a territory of the United States by virtue
of the Treaty of Paris. In fact, Section 1 of Act
2927 was precisely approved pursuant to express
authority, without which it could not have been

done, granted by an amendment to Section 4 of


the Philippine Bill of 1902 introduced by the Act
of the United States Congress of March 23, 1912
and which was reenacted as part of the Jones Law
of 1916, the pertinent provisions of which have
already been quoted earlier. In truth, therefore, it
was because of the establishment of the
Philippine Commonwealth and in the exercise of
our legislative autonomy on citizenship matters
under the Philippine Independence Act that
Section 1 of Act 2927 was eliminated, 15 and not
purposely to eliminate any racial discrimination
contained in our Naturalization Law. The
Philippine Legislature naturally wished to free our
Naturalization Law from the impositions of
American legislation. In other words, the fact that
such discrimination was removed was one of the
effects rather than the intended purpose of the
amendment.
2. Again, the statement in Choy King Tee to the
effect that "the reference in subdivision (c) (of
Section 1 of Act 2927) to foreigners who may
become American citizens is restrictive in
character, for only persons of certain specified
races were qualified thereunder" fails to consider
the exact import of the said subdivision.
Explicitly, the thrust of the said subdivision was
to confine the grant under it of Philippine
citizenship only to the three classes of persons
therein mentioned, the third of which were
citizens of the United States and, corollarily,
persons who could be American citizens under
her laws. The words used in the provision do not
convey any idea of favoring aliens of any
particular race or color and of excluding others,
but more accurately, they refer to all the
disqualifications of foreigners for American
citizenship under the laws of the United States.
The fact is that even as of 1906, or long before
1920, when our Act 2927 became a law, the
naturalization laws of the United States already
provided for the following disqualifications in the
Act of the Congress of June 29, 1906:
"SEC. 7. That no person who disbelieves in or who
is opposed to organized government, or who is a
member of or affiliated with any organization
entertaining and teaching such disbelief in or
opposition to organized government, or who
advocates or teaches the duty, necessity, or
propriety of the unlawful assaulting or killing of
any officer or officers, either of specific
individuals or of officers generally, of the
Government of the United States, or of any other
organized government, because of his or their
official character, or who is a polygamist, shall be
naturalized or be made a citizen of the United
States."
and all these disqualified persons were, therefore,
ineligible for Philippine citizenship under Section
1 of Act 2927 even if they happened to be
Caucasians. More importantly, as a matter of fact,
said American law, which was the first "Act to
Establish
a
Bureau
of
Immigration
and
Naturalization and to Provide for a Uniform Rule
for Naturalization of Aliens throughout the United
States" contained no racial disqualification
requirement, except as to Chinese, the Act of May
68 | C O N S T I 2 _ A r t i c l e I V _ C I T I Z E N S H I P

6, 1882 not being among those expressly


repealed by this law, hence it is clear that when
Act 2927 was enacted, subdivision (c) of its
Section 1 could not have had any connotation of
racial exclusion necessarily, even if it were traced
back to its origin in the Act of the United States
Congress of 1912 already mentioned above. 16
Thus, it would seem that the nationalization in
the quoted decisions predicated on the theory
that the elimination of Section 1 of Act 2927 by
Commonwealth Act 473 was purposely for no
other end than the abolition of racial
discrimination in our naturalization law has no
clear factual basis. 17
3. In view of these considerations, there appears
to be no cogent reason, why the construction
adopted in the opinions of the Secretary of Justice
referred to in the first Ly Giok Ha decision of the
Chief Justice should not prevail. It is beyond
dispute that the first paragraph of Section 15 of
Commonwealth Act 473 is a reenactment of
Section 13(a) of Act 2927, as amended by Act
3448, and that the latter is nothing but an exact
copy, deliberately made, of Section 1994 of the
Revised Statutes of the United States as it stood
before it repeal in 1922. 18 Before such repeal,
the phrase "who might herself be lawfully
naturalized" found in said Section 15 had a
definite unmistakable construction uniformly
followed in all courts of the United States that
had occasion to apply the same and which,
therefore, must be considered as if it were written
in the statute itself. It is almost trite to say that
when our legislators enacted said section, they
knew of its unvarying construction in the United
States and that, therefore, in adopting verbatim
the American statute, they have in effect
incorporated into the provision, as thus enacted,
the construction given to it by the American
courts as well as the Attorney General of the
United States and all administrative authorities
charged with the implementation of the
naturalization and immigration laws of that
country. (Lo Cham v. Ocampo, 77 Phil., 635
[1946]; Laxamana v. Baltazar, 92 Phil., 32 [1952];
Hartley v. Commissioner, 295 U.S. 216, 79 L. ed.
1399, 55 S Ct. 756 [1935]; Helvering v. Windmill,
305 U.S. 79, 83 L ed. 52, 59 S Ct. 45 [1938];
Helvering v. R. J. Reynolds Tobacco Co., 306 U.S.
110, 83 L ed. 536, 59 S Ct. 423 [1939]. [p. 32,
Memo of Amicus Curiae]).
A fairly comprehensive summary of the said
construction by the American courts and
administrative authorities is contained in United
Stats of America ex rel. Dora Sejnensky v. Robert
E. Tod, Commissioner of Immigration, Appt., 285
Fed. 523, decided November 14, 1922, 26 A. L. R.
1316 as follows:
"Section 1994 of the Revised Statutes (Comp.
Stat. 3948, 2 Fed. Sta. Anno. 2d ed. p. 117)
provides as follows: 'Any woman who is now or
may hereafter be married to a citizen of the
United States, and who might herself be lawfully
naturalized, shall be deemed a citizen.'

"Section 1944 of the Revised Stat. is said to


originate in the Act of Congress of February 10,
1855 (10 Stat. at L. 604, chap. 71), which in its
second section provided 'that any woman, who
might lawfully be naturalized under the existing
laws, married, or who shall be married to a citizen
of the United States, shall be deemed and taken
to be a citizen.'

construed as in effect declaring that an alien


woman, who is of the class or race that may be
lawfully naturalized under the existing laws, and
who marries a citizen of the United States, is such
a citizen also, and it was not necessary that it
should appear affirmatively that she possessed
the other qualifications at the time of her
marriage to entitle her to naturalization.

"And the American Statute of 1855 is


substantially a copy of the earlier British Statute
7 & 8 Vict. chap. 66, . . . 16, 1844, which provided
that 'any woman married, or who shall be
married, to a natural-born subject or person
naturalized, shall be deemed and taken to be
herself naturalized, and have all the rights and
privileges of a natural born subject.'

"In 1882, the Act of 1855 came before Mr. Justice


Harlan, sitting in the circuit court, in United States
v. Kellar, 13 Fed. 82. An alien woman, a subject of
Prussia came to the United States and married
here a naturalized citizen. Mr. Justice Harlan, with
the concurrence of Judge Treat, held that upon
her marriage she became ipso facto a citizen of
the United States as fully as if she had complied
with all of the provisions of the statutes upon the
subject of naturalization. He added: 'There can be
no doubt of this, in view of the decision of the
Supreme Court of the United States in Kelly v.
Owen, 7 Wall. 496, 19 L. ed. 283.' The alien
'belonged to the class of persons' who might be
lawfully naturalized.

"The Act of Congress of September 22, 1922 (42


Stat. at L. 1021, chap. 411, Comp. Stat. 4358b,
Fed. Stat. Anno. Supp. 1922, p. 255), being 'An
Act Relative to the Naturalization and Citizenship
of Married Women,' in 2, provides 'that any
woman who marries a citizen of the United States
after the passage of this Act, . . . shall not
become a citizen of the United States by reason
of such marriage . . .'
"Section 6 of the act also provides 'that . . . 1994
of the Revised Statutes . . . are repealed.'
"Section 6 also provides that 'such repeal shall
not terminate citizenship acquired or retained
under either of such sections, . . .' meaning 2
and 6. So that this Act of September 22, 1922,
has no application to the facts of the present
case, as the marriage of the relator took place
prior to its passage. This case, therefore, depends
upon the meaning to be attached to 1994 of the
Revised Statutes.
"In 1868 the Supreme Court, in Kelly v. Owen, 7
Wall. 496, 498, 19 L. ed. 283, 284, construed this
provision as found in the Act of 1855 as follows:
'The term, "who might lawfully be naturalized
under the existing laws," only limits the
application of the law to free white women. The
previous Naturalization Act, existing at the time,
only required that the person applying for its
benefits should be "a free white person," and not
an alien enemy.'
"This construction limited the effect of the statute
to those aliens who belonged to the class or race
which might be lawfully naturalized, and did not
refer to any of the other provisions of the
naturalization laws as to residence or moral
character, or to any of the provisions of the
immigration laws relating to the exclusion or
deportation of aliens.
"In 1880, in Leonard v. Grant (C. C.) 5 Fed. 11,
District Judge Deady also construed the Act of
1855, declaring that 'any woman who is now or
may hereafter be married to a citizen of the
United States, and might herself be lawfully
naturalized, shall be deemed a citizen.' He held
that 'upon the authorities, and the reason, if not
the necessity, of the case,' the statute must be
69 | C O N S T I 2 _ A r t i c l e I V _ C I T I Z E N S H I P

"In 1904, in Hopkins v. Fachant, 65 C. C. A. 1, 130


Fed. 839, an alien woman came to the United
States from France and entered the country
contrary
to
the
immigration
laws.
The
immigration authorities took her into custody at
the port of New York, with the view of deporting
her. She applied for her release under a writ of
habeas corpus, and pending the disposition of the
matter she married a naturalized American
citizen. The circuit court of appeals for the ninth
circuit held, affirming the court below, that she
was entitled to be discharged from custody. The
court declared: 'The rule is well settled that her
marriage to a naturalized citizen of the United
States entitled her to be discharged. The status of
the wife follows that of her husband, . . . and by
virtue of her marriage her husband's domicil
became her domicil.'
"In 1908, the circuit court for the district of Rhode
Island in Re Rustigian, 165 Fed. 980, had before it
the application of a husband for his final decree
of naturalization. It appeared that at that time his
wife was held by the immigration authorities at
New York on the ground that she was afflicted
with a dangerous and contagious disease.
Counsel on both sides agreed that the effect of
the husband's naturalization would be to confer
citizenship upon the wife. In view of that
contingency District Judge Brown declined to pass
upon the husband's application for naturalization,
and thought it best to wait until it was
determined whether the wife's disease was
curable. He placed his failure to act on the
express ground that the effect of naturalizing the
husband might naturalize her. At the same time
he expressed his opinion that the husband's
naturalization would not effect her naturalization,
as she was not one who could become lawfully
naturalized. 'Her own capacity (to become
naturalized),' the court stated, 'is a prerequisite
to her attaining citizenship. If herself lacking in
that capacity, the married status cannot confer it
upon her.' Nothing, however, was actually

decided in that case, and the views expressed


therein are really nothing more than mere dicta.
But, if they can be regarded as something more
than that, we find ourselves, with all due respect
for the learned judge, unable to accept them.
"In 1909, in United States ex rel. Nicola v.
Williams, 173 Fed, 626, District Judge Learned
Hand held that an alien woman, a subject of the
Turkish Empire, who married an American citizen
while visiting Turkey, and then came to the United
States, could not be excluded, although she had,
at the time of her entry, a disease which under
the immigration laws would have been sufficient
ground for her exclusion, if she had not had the
status of a citizen. The case was brought into this
court on appeal, and in 1911 was affirmed, in 106
C. C. A. 464, 184 Fed. 322. In that case, however
at the time the relators married, they might have
been lawfully naturalized, and we said: 'Even if
we assume the contention of the district attorney
to be correct that marriage will not make a citizen
of a woman who would be excluded under our
immigration laws, it does not affect these
relators.'
"We held that, being citizens, they could not be
excluded as aliens; and it was also said to be
inconsistent with the policy of our law that the
husband should be a citizen and the wife an alien.
The distinction between that case and the one
now before the court is that, in the former case,
the marriage took place before any order of
exclusion had been made, while in this the
marriage was celebrated after such an order was
made. But such an order is a mere administrative
provision, and has not the force of a judgment of
a
court,
and
works
no
estoppel.
The
administrative
order
is
based
on
the
circumstances that existed at the time the order
of exclusion was made. If the circumstances
change prior to the order being carried into
effect, it cannot be executed. For example, if an
order of exclusion should be based on the ground
that the alien was at the time afflicted with a
contagious disease, and it should be made
satisfactorily
to appear,
prior
to
actual
deportation, that the alien had entirely recovered
from the disease, we think it plain that the order
could not be carried into effect. So, in this case,
if, after the making of the order of exclusion and
while she is permitted temporarily to remain, she
in good faith marries an American citizen, we
cannot doubt the validity of her marriage, and
that she thereby acquired, under international
law and under 1994 of the Revised Statutes,
American citizenship, and ceased to be an alien.
There upon, the immigration authorities lost their
jurisdiction over her, as that jurisdiction applies
only to aliens, and not to citizens.

"In 1910, District Judge Dodge, in Ex parte


Kaprielian, 188 Fed. 694, sustained the right of
the officials to deport a woman under the
following circumstances: She entered this country
in July, 1910, being an alien and having been
born in Turkey. She was taken into custody by the
70 | C O N S T I 2 _ A r t i c l e I V _ C I T I Z E N S H I P

immigration
authorities
in
the
following
September, and in October a warrant for her
deportation was issued. Pending hearings as to
the validity of that order, she was paroled in the
custody of her counsel. The ground alleged for
her deportation was that she was afflicted with a
dangerous and contagious disease at the time of
her entry. One of the reasons assigned to defeat
deportation was that the woman had married a
citizen of the United States pending the
proceedings for her deportation. Judge Dodge
declared himself unable to believe that a
marriage under such circumstances 'is capable of
having the effect claimed, in view of the facts
shown.' He held that it was no part of the
intended policy of 1994 to annul or override the
immigration laws, so as to authorize the
admission into the country of the wife of a
naturalized alien not otherwise entitled to enter,
and that an alien woman, who is of a class of
persons excluded by law from admission to the
United States does not come within the provisions
of that section. The court relied wholly upon the
dicta contained in the Rustigian Case. No other
authorities were cited.
"In 1914, District Judge Neterer, in Ex parte
Grayson, 215 Fed. 449, construed 1994 and
held that where, pending proceedings to deport
an alien native of France as an alien prostitute,
she was married to a citizen of the United States,
she thereby became a citizen, and was not
subject to deportation until her citizenship was
revoked by due process of law. It was his opinion
that if, as was contended, her marriage was
conceived in fraud, and was entered into for the
purpose of evading the immigration laws and
preventing her deportation, such fact should be
established in a court of competent jurisdiction in
an action commenced for the purpose. The case
was appealed and the appeal was dismissed. 134
C. C. A. 666, 219 Fed. 1022.
"It is interesting also to observe the construction
placed upon the language of the statute by the
Department of Justice. In 1874, Attorney General
Williams, 14 Ops. Atty. Gen. 402, passing upon
the Act of February 10, 1855, held that residence
within the United States for the period required
by the naturalization laws was not necessary in
order to constitute an alien woman a citizen, she
having married a citizen of the United States
abroad, although she never resided in the United
States, she and her husband having continued to
reside abroad after the marriage.
"In 1909, a similar construction was given to the
Immigration Act of May 5, 1907, in an opinion
rendered by Attorney General Wickersham. It
appeared an unmarried woman, twenty-eight
years of age and a native of Belgium, arrived in
New York and went at once to a town in
Nebraska, where she continued to reside. About
fifteen months after her arrival she was taken
before a United States commissioner by way of
instituting proceedings under the Immigration Act
(34 Stat. at L. 898, chap. 1134, Comp. Stat.
4242, 3 Fed. Stat. Anno. 2d ed. p. 637) for her
deportation, on the ground that she had entered

this country for the purpose of prostitution, and


had been found an inmate of a house of
prostitution and practicing the same within three
years after landing. It appeared, however, that
after she was taken before the United States
commissioner, but prior to her arrest under a
warrant by the Department of Justice, she was
lawfully married to a native-born citizen of the
United States. The woman professed at the time
of her marriage an intention to abandon her
previous mode of life and to remove with her
husband to his home in Pennsylvania. He knew
what her mode of life had been, but professed to
believe in her good intentions. The question was
raised as to the right to deport her, the claim
being advance that by her marriage she had
become an American citizen and therefore could
not be deported. The Attorney General ruled
against the right to deport her as she had
become an American citizen. He held that the
words, 'who might herself be lawfully naturalized,'
refer to a class or race who might be lawfully
naturalized, and that compliance with the other
conditions of the naturalization laws was not
required. 27 Ops. Atty. Gen. 507.
"Before concluding this opinion, we may add that
it has not escaped our observation that Congress,
in enacting the Immigration Act of 1917, co as to
provide, in 19, 'that the marriage to an
American citizen of a female of the sexually
immoral classes . . . shall not invest such female
with United States citizenship if the marriage of
such alien female shall be solemnized after her
arrest or after the commission of acts which make
her liable to deportation under this act.'
"Two conclusions seem irresistibly to follow from
the above change in the law:
"(1) Congress deemed legislation essential to
prevent women of the immoral class avoiding
deportation through the device of marrying an
American citizen.
"(2) If Congress intended that the marriage of an
American citizen with an alien woman of any
other of the excluded classes, either before or
after her detention should not confer upon her
American citizenship, thereby entitling her to
enter the country, its intention would have been
expressed, and 19 would not have been
confined solely to women of the immoral class."
Indeed, We have examined all the leading
American decisions on the, subject and We have
found no warrant for the proposition that the
phrase
"who
might
herself
be
lawfully
naturalized" in Section 1994 of the Revised Status
was meant solely as a racial bar, even if loose
statements in some decisions and other treaties
and other writings on the subject would seem to
give such impression. The case of Kelly v. Owen,
supra, which appears to be the most cited among
the first of these decisions 19 simply held:
"As we construe this Act, it confers the privileges
of citizenship upon women married to citizens of
the United States, if they are of the class of
71 | C O N S T I 2 _ A r t i c l e I V _ C I T I Z E N S H I P

persons for whose naturalization the previous


Acts of Congress provide. The terms 'married' or
'who shall be married,' do not refer, in our
judgment, to the time when the ceremony of
marriage is celebrated, but to a state of marriage.
They mean that, whenever a woman, who under
previous Acts might be naturalized, is in a state of
marriage to a citizen, whether his citizenship
existed at the passage of the Act or
subsequently, or before or after the marriage, she
becomes, by that fact, a citizen also. His
citizenship, whenever it exists, confers, under the
Act, citizenship upon her. The construction which
would restrict the Act to women whose husbands,
at the time of marriage, are citizens, would
exclude far the greater number, for whose
benefit, as we think, the Act was intended. Its
object, in our opinion, was to allow her citizenship
to follow that of her husband, without the
necessity of any application for naturalization on
her part; and, if this was the object, there is no
reason for the restriction suggested.
"The terms, 'who might lawfully be naturalized
under the existing laws,' only limit the application
of the law to free white women. The previous
Naturalization Act, existing at the time only
required that the person applying for its benefits
should be 'a free white person,' and not an alien
enemy. Act of April 14th, 1802, 2 Stat. at L. 153.
"A similar construction was given to the Act by
the Court of Appeals of New York, in Burton v.
Burton, 40 N. Y. 373; and is the one which gives
the widest extension to its provisions"
Note that while the court did say that "the terms,
'who might lawfully be naturalized under existing
laws' only limit the application to free white
women" 20 it hastened to add that "the previous
Naturalization Act, existing at the time, . . .
required that the person applying for its benefits
should be (not only) a 'free white person' (but
also) . . . not an alien enemy." This is simply
because under the Naturalization Law of the
United States at the time the case was decided,
the disqualification of enemy aliens had already
been removed by the Act of July 30, 1813, as may
be seen in the corresponding footnote hereof
anon. In other words, if in the case of Kelly v.
Owen only the race requirement was mentioned,
the reason was that there was no other non-racial
requirement
or
no
more
alien-enemy
disqualification at the time; and this is
demonstrated by the fact that the court took care
to make it clear that under the previous
naturalization law, there was also such
requirement in addition to race. This is important,
since as stated in re Rustigian, 165 Fed. Rep. 980,
"The expression used by Mr. Justice Field, (in Kelly
v. Owen) the terms 'who might lawfully be
naturalized under existing laws' only limit the
application of the law to free white women, must
be interpreted in the application to the special
facts and to the incapacities under the then
existing laws," (at p. 982) meaning that whether
or not an alien wife marrying a citizen would be a
citizen was dependent, not only on her race and
nothing more necessarily, but on whether or not

there were other disqualifications under the law


in force at the time of her marriage or the
naturalization of her husband.
4. As already stated, in Lo San Tuang, Choy King
Tee and the second Ly Giok Ha, the Court drew
the inference that because Section 1 of Act 2927
was eliminated by Commonwealth Act 473, it.
follows that in place of the said eliminated
section, particularly its subdivision (c), being the
criterion of whether or not an alien wife "may be
lawfully naturalized," what should be required is
not only that she must not be disqualified under
Section 4 but that she must also possess the
qualifications enumerated in Section 2, such as
those of age, residence, good moral character,
adherence to the underlying principles of the
Philippine Constitution, irreproachable conduct,
lucrative employment or ownership of real estate,
capacity to speak and write English or Spanish
and one of the principal local languages,
education of children in certain schools, etc.,
thereby implying that, in effect, said Section 2
has been purposely intended to take the place of
Section 1 of Act 2927. Upon further consideration
of the proper premises, We have come to the
conclusion that such inference is not sufficiently
justified.

To begin with, nothing extant in the legislative


history, which We have already examined above
of the mentioned provisions has been shown or
can be shown to indicate that such was the clear
intent of the legislature. Rather, what is definite is
that Section 15 an exact copy of Section 1994 of
the Revised Statutes of the United States, which,
at the time of the approval of Commonwealth Act
473 had already a settled construction by
American courts and administrative authorities.
Secondly, as may be gleaned from the summary
of pertinent American decisions quoted above,
there can be no doubt that in the construction of
the identically worded provision in the Revised
Statutes of the United States, (Section 1994,
which was taken from the Act of February 10,
1855) all authorities in the United States are
unanimously agreed that the qualifications of
residence, good moral character, adherence to
the Constitution, etc. are not supposed to be
considered, and that the only eligibility to be
taken into account is that of the race or class to
which the subject belongs, the conceptual scope
of which, We have just discussed. 21 In the very
case of Leonard v. Grant, supra, discussed by
Justice Regala in Lo San Tuang, the explanation
for such posture of the American authorities was
made thus:
"The phrase, 'shall be deemed a citizen,' in
section 1994 Rev. St., or as it was in the Act of
1855, supra, 'shall be deemed and taken to be a
citizen,' while it may imply that the person to
whom it relates has not actually become a citizen
by ordinary means or in the usual way, as by the
judgment of a competent court, upon a proper
application and proof, yet it does not follow that
72 | C O N S T I 2 _ A r t i c l e I V _ C I T I Z E N S H I P

such person is on that account practically any the


less a citizen. The word 'deemed' is the
equivalent of 'considered' or 'judged'; and,
therefore, whatever an act of Congress requires
to be 'deemed' or 'taken' as true of any person or
thing, must, in law, he considered as having been
duly adjudged or established concerning such
person or thing, and have force and effect
accordingly. When, therefore, Congress declares
that an alien woman shall, under certain
circumstances, be 'deemed' an American citizen,
the effect when the contingency occurs, is
equivalent to her being naturalized directly by an
act of Congress, or in the usual mode thereby
prescribed."
Unless We disregard now the long settled familiar
rule of statutory construction that in a situation
like this wherein our legislature has copied an
American statute word for word, it is understood
that the construction already given to such
statute before its being copied constitute part of
our own law, there seems to be no reason how
We can give a different connotation or meaning
to the provision in question. At least, We have
already seen that the views sustaining the
contrary conclusion appear to be based on
inaccurate factual premises related to the real
legislative background of the framing of our
naturalization law in its present form.
Thirdly, the idea of equating the qualifications
enumerated in Section 2 of Commonwealth Act
473 with the eligibility requirements of Section 1
of Act 2927 cannot bear close scrutiny from any
point of view. There is no question that Section 2
of Commonwealth Act 473 is more or less
substantially the same as Section 3 of Act 2927.
In other words, Section 1 of Act 2927 co-existed
already with practically the same provision as
Section 2 of Commonwealth Act 473. If it were
true that the phrase "who may be lawfully
naturalized" in Section 13(a) of Act 2927, as
amended by Act 3448, referred to the so called
racial requirement in Section 1 of the same Act,
without regard to the provisions of Section 3
thereof, how could the elimination of Section 1
have the effect of shifting the reference to
Section 3, when precisely, according to the
American jurisprudence, which was prevailing at
the time Commonwealth Act 473 was approved,
such qualifications as were embodied in said
Section 3, which had their counterpart in the
corresponding American statutes, are not
supposed to be taken into account and that what
should be considered only are the requirements
similar to those provided for in said Section 1
together with the disqualifications enumerated in
Section 4?
Fourthly, it is difficult to conceive that the phrase
"who might be lawfully naturalized" in Section 15
could have been intended to convey a meaning
different than that given to it by the American
courts and administrative authorities. As already
stated, Act 3448 which contained said phrase and
from which it was taken by Commonwealth Act
473, was enacted in 1928. By that time, Section
1994 of the Revised Statutes of the United States
was no longer in force because it had been

repealed expressly the Act of September 22,


1922 which did away with the automatic
naturalization of alien wives of American citizens
and required, instead, that they submit to regular
naturalization proceedings, albeit under more
liberal terms than those of other applicants. In
other words, when our legislature adopted the
phrase
in
question,
which,
as
already
demonstrated, had a definite construction in
American law, the Americans had already
abandoned said phraseology in favor of a
categorical compulsion for alien wives to be
naturalized judicially. Simple logic would seem to
dictate that, since our lawmakers, at the time of
the approval of Act 3448, had two choices, one to
adopt the phraseology of Section 1994 with its
settled construction and the other to follow the
new posture of the Americans of requiring judicial
naturalization, and it appears that they have
opted for the first, We have no alternative but to
conclude that our law still follows the old or
previous American law on the subject. Indeed,
when Commonwealth Act 473 was approved in
1939,
the
Philippine
Legislature,
already
autonomous then from the American Congress,
had a clearer chance to disregard the old
American law and make one of our own, or, at
least, follow the trend of the Act of the U.S.
Congress of 1922, but still, our legislators chose
to maintain the language of the old law. What
then is significantly important is not that the
legislature maintained said phraseology after
Section 1 of Act 2927 was eliminated, but that it
continued insisting on using it even after the
Americans had amended their law in order to
provide for what is now contended to be the
construction that should be given to the phrase in
question. Stated differently, had our legislature
adopted a phrase from an American statute
before the American courts had given it a
construction which was acquiesced to by those
given upon to apply the same, it would be
possible for Us to adopt a construction here
different from that of the Americans, but as
things stand, the fact is that our legislature
borrowed the phrase when there was already a
settled construction thereof, and what is more, it
appears that our legislators even ignored the
modification of the American law and persisted in
maintaining the old phraseology. Under these
circumstances, it would be in defiance of reason
and the principles of Statutory construction to say
that Section 15 has a nationalistic and selective
orientation and that it should be construed
independently of the previous American posture
because of the difference of circumstances here
and in the United States. It is always safe to say
that in the construction of a statute, We cannot
fall on possible judicial fiat or perspective when
the demonstrated legislative point of view seems
to indicate otherwise.
5. Viewing the matter from another angle, there
is need to emphasize that in reality and in effect,
the so-called racial requirements, whether under
the American laws or the Philippine laws, have
hardly been considered as qualifications in the
same sense as those enumerated in Section 3 of
Act 2927 and later in Section 2 of Commonwealth
73 | C O N S T I 2 _ A r t i c l e I V _ C I T I Z E N S H I P

Act 473. More accurately, they have always been


considered as disqualifications, in the sense that
those who did not possess them were the ones
who could not "be lawfully naturalized," just as if
they
were
suffering
from
any
of
the
disqualifications under Section 2 of Act 2927 and
later those under Section 4 of Commonwealth Act
473, which, incidentally, are practically identical
to those in the former law, except those in
paragraphs (f) and (h) of the latter. 22 Indeed,
such is the clear impression anyone will surely
get after going over all the American decisions
and opinions quoted and/or cited in the latest
USCA (1970), Title 8, section 1430, pp. 598-602,
and the first decisions of this Court on the matter,
Ly Giok Ha (1959) and Ricardo Cua, citing with
approval the opinions of the Secretary of Justice.
23 Such being the case, that is, that the so-called
racial requirements were always treated as
disqualifications in the same light as the other
disqualifications under the law, why should their
elimination not be viewed or understood as a
subtraction from or a lessening of the
disqualifications? Why should such elimination
have instead the meaning that what were
previously considered as irrelevant qualifications
have become disqualifications, as seems to be
the import of the holding in Choy King Tee to the
effect that the retention in Section 15 of
Commonwealth Act 473 of the same language of
what used to be Section 13 (a) of Act 2927 (as
amended by Act 3448), notwithstanding the
elimination of Section 1 of the latter, necessarily
indicates that the legislature had in mind making
the phrase in question "who may be lawfully
naturalized" refer no longer to any racial
disqualification but to the qualification under
Section 2 of Commonwealth Act 473? Otherwise
stated, under Act 2927, there were two groups of
persons that could not be naturalized, namely,
those falling under Section 1 and those falling
under Section 2, and surely, the elimination of
one group, i.e. those belonging to Section 1,
could not have had, by any process of reasoning,
the effect of increasing, rather than decreasing
the disqualifications that used to be before such
elimination. We cannot see by what alchemy of
logic such elimination could have converted
qualifications into disqualifications, specially in
the light of the fact that, after all, these are
disqualifications clearly set out as such in the law
distinctly and separately from qualifications and,
as
already
demonstrated,
in
American
jurisprudence, qualifications had never been
considered to be of any relevance in determining
"who might be lawfully naturalized," as such
phrase is used in the statute governing the status
of alien wives of American citizens, and our law
on the matter was merely copied verbatim from
the American statutes.

6. In addition to these arguments based on the


applicable legal provisions and judicial opinions,
whether here or in the United States, there are
practical considerations that militate towards the
same conclusions. As aptly stated in the motion
for reconsideration of counsel for petitioner-

appellee dated February 23, 1967, filed in the


case of Zita Ngo Burca v. Republic, supra:

can she hope to acquire a lucrative income of her


own to qualify her for citizenship?

"Unreasonableness of requiring alien wife to


prove 'qualifications

"3. Under Section 2 of the law, the applicant for


naturalization 'must have enrolled his minor
children of school age, in any of the public
schools or private schools recognized by the
Office of the Private Education of the Philippines,
where Philippine history, government and civics
are taught or prescribed as part of the school
curriculum during the entire period of residence
in the Philippines required of him prior to the
hearing of his petition for naturalization as
Philippine citizen.' If an alien woman has minor
children by a previous marriage to another alien
before she marries a Filipino, and such minor
children had not been enrolled in Philippine
schools during her period of residence in the
country, she cannot qualify for naturalization
under the interpretation of this Court. The reason
behind the requirement that children should be
enrolled in recognized educational institutions is
that they follow the citizenship of their father.
(Chan Ho Lay v. Republic, L-5666, March 30,
1954; Tan Hi v. Republic, 88 Phil. 117 [1951]; Hao
Lian Chu v. Republic, 87 Phil. 668 [950]; Yap Chin
v. Republic, L-4177, May 29, 1953; Lim Lian Hong
v. Republic, L-3575, Dec. 26, 1950). Considering
that said minor children by her first husband
generally follow the citizenship of their alien
father, the basis for such requirement as applied
to her does not exist. Cessante ratione legis
cessat ipsa lex.

"There is one practical consideration that strongly


militates against a construction that Section 15 of
the law requires that an alien wife of a Filipino
must affirmatively prove that she possesses the
qualifications prescribed under Section 2, before
she may be deemed a citizen. Such condition, if
imposed
upon
an
alien
wife,
becomes
unreasonably onerous and compliance therewith
manifestly difficult. The unreasonableness of such
requirement is shown by the following:
"1. One of the qualifications required of an
applicant for naturalization under Section 2 of the
law is that the applicant 'must have resided in the
Philippines for a continuous period of not less
than ten years.' If this requirement is applied to
an alien wife married to a Filipino citizen, this
means that for a period of ten years at least, she
cannot hope to acquire the citizenship of her
husband. If the wife happens to be a citizen of a
country whose law declares that upon her
marriage to a foreigner she automatically loses
her citizenship and acquires the citizenship of her
husband, this could mean that for a period of ten
years at least, she would be stateless. And even
after having acquired continuous residence in the
Philippines for ten years, there is no guarantee
that her petition for naturalization will be granted,
in which case she would remain stateless for an
indefinite period of time.
"2. Section 2 of the law likewise requires of the
applicant for naturalization that he 'must own real
estate in the Philippines worth not less than five
thousand pesos, Philippine currency, or must
have some known lucrative trade, profession, or
lawful occupation.' Considering the constitutional
prohibition against acquisition by an alien of real
estate except in cases of hereditary succession
(Art. XIII, Sec. 5, Constitution), an alien wife
desiring to acquire the citizenship of her husband
must have to prove that she has a lucrative
income derived from a lawful trade, profession or
occupation. The income requirement has been
interpreted to mean that the petitioner herself
must be the one to possess the said income. (Uy
v. Republic, L-19578, Oct. 27, 1964; Tanpa Ong
vs. Republic, L-20605, June 30, 1965; Li Tong Pek
v. Republic, L-20912, November 29, 1965). In
other words, the wife must prove that she has a
lucrative income derived from sources other than
her husband's trade, profession or calling. It is of
common knowledge, and judicial notice may be
taken of the fact that most wives in the
Philippines do not have gainful occupations of
their own. Indeed, Philippine law, recognizing the
dependence of the wife upon the husband,
imposes upon the latter the duty of supporting
the former. (Art. 291, Civil Code). It should be
borne in mind that universally, it is an accepted
concept that when a woman marries, her primary
duty is to be a wife, mother and housekeeper. If
an alien wife is not to be remiss in this duty, how
74 | C O N S T I 2 _ A r t i c l e I V _ C I T I Z E N S H I P

"4. Under Section 3 of the law, the 10-year


continuous residence prescribed by Section 2
'shall be understood as reduced to five years for
any petitioner (who is) married to a Filipino
woman.' It is absurd that an alien male married to
a Filipino wife should be required to reside only
for five years in the Philippines to qualify for
citizenship, whereas an alien woman married to a
Filipino husband must reside for ten years.
"Thus under the interpretation given by this
Court, it is more difficult for an alien wife related
by marriage to a Filipino citizen to become such
citizen, than for a foreigner who is not so related.
And yet, it seems more than clear that the
general purpose of the first paragraph of Section
15 was obviously to accord to an alien woman, by
reason of her marriage to a Filipino, a privilege
not similarly granted to other aliens. It will be
recalled that prior to the enactment of Act No.
3448 in 1928, amending Act No. 2927 (the old
Naturalization Law), there was no law granting
any special privilege to alien wives of Filipinos.
They were treated as any other foreigner. It was
precisely to remedy this situation that the
Philippine legislature enacted Act No. 3448. On
this point, the observation made by the Secretary
of Justice in 1941 is enlightening:
'It is true that under Article 22 of the (Spanish)
Civil Code, the wife follows the nationality of the
husband; but the Department of State of the
United States on October 31, 1921, ruled that the
alien wife of a Filipino citizen is not a Filipino
citizen, pointing out that our Supreme Court in

the leading case of Roa v. Collector of Customs


(23 Phil. 315) held that Articles 17 to 27 of the
Civil Code being political have been abrogated
upon the cession of the Philippine Islands to the
United States. Accordingly, the stand taken by
the Attorney-General prior to the enactment of
Act No. 3448, was that marriage of alien women
to Philippine citizens did not make the former
citizens of this country.' (Op. Atty. Gen., March 16,
1928).
'To remedy this anomalous condition, Act No.
5448 was enacted in 1928 adding section 13(a)
to Act No. 2997 which provides that "any woman
who is now or may hereafter be married to a
citizen of the Philippine Islands, and who might
herself be lawfully naturalized, shall be deemed a
citizen of the Philippine Islands.' (Op. No. 22, s.
1941; emphasis ours)
"If Section 15 of the Revised Naturalization Law
were to be interpreted, as this Court did, in such
a way as to require that the alien wife must prove
the qualifications prescribed in Section 2, the
privilege granted to alien wives would become
illusory. It is submitted that such a construction,
being contrary to the manifested object of the
statute, must be rejected.
'A statute is to be construed with reference to its
manifest object, and if the language is
susceptible of two constructions, one which will
carry out and the other defeat such manifest
object, it should receive the former construction.'
(In re National Guard, 71 Vt. 493, 45 A. 1051;
Singer v. United States, 323 U.S. 333, 89 L. ed.
285. See also, U.S. v. Navarro, 19 Phil. 134
[1911]; U. S. v. Toribio, 15 Phil. 85 [1910]).
'. . . A construction which will cause objectionable
results should be avoided and the court will, if
possible, place on the statute a construction
which will not result in injustice, and in
accordance with the decisions construing
statutes, a construction which will result in
oppression, hardship, or inconveniences will also
be avoided, as will a construction which will
prejudice public interest, or construction resulting
in unreasonableness, as well as a construction
which will result in absurd consequences.'
'So a construction should, if possible, be avoided
if the result would be an apparent inconsistency
in legislative intent, as has been determined by
the judicial decisions, or which would result in
futility, redundancy, or a conclusion not
contemplated by the legislature; and the court
should adopt that construction which will be the
least likely to produce mischief. Unless plainly
shown to have been the intention of the
legislature, an interpretation which would render
the requirements of the statute uncertain and
vague is to be avoided, and the court will not
ascribe to the legislature an intent to confer an
illusory right . . .' (82 C.J.S., Statutes, sec. 326,
pp. 623-632)."
7. In Choy King Tee and the second Ly Giok Ha,
emphasis was laid on the need for aligning the
75 | C O N S T I 2 _ A r t i c l e I V _ C I T I Z E N S H I P

construction of Section 15 with "the national


policy of selective admission to Philippine
citizenship." But the question may be asked, is it
reasonable to suppose that in the pursuit of such
policy, the legislature contemplated to make it
more difficult, if not practically impossible in
some instances, for an alien woman marrying a
Filipino to become a Filipina than any ordinary
applicant for naturalization, as has just been
demonstrated above? It seems but natural and
logical to assume that Section 15 was intended to
extend special treatment to alien women who by
marrying
a
Filipino
irrevocably
deliver
themselves, their possessions, their fate and
fortunes and all that marriage implies to a citizen
of this country, "for better or for worse." Perhaps
there can and will be cases wherein the personal
conveniences and benefits arising from Philippine
citizenship may motivate such marriage, but
must the minority, as such cases are bound to be,
serve as the criterion for the construction of law?
Moreover, it is not farfetched to believe that in
joining a Filipino family, the alien woman is
somehow disposed to assimilate the customs,
beliefs and ideals of Filipinos among whom, after
all, she has to live and associate, but surely, no
one should expect her to do so even before
marriage. Besides, it may be considered that in
reality the extension of citizenship to her is made
by the law not so much for her sake as for the
husband. Indeed, We find the following
observations
anent
the
national
policy
rationalization in Choy King Tee and Ly Giok Ha
(the second) to be quite persuasive:

"We respectfully suggest that this articulation of


the national policy begs the question. The
avowed policy of 'selective admission' more
particularly refers to a case where citizenship is
sought to be acquired in a judicial proceeding for
naturalization. In such a case, the courts should
no doubt apply the national policy of selecting
only those who are worthy to become citizens.
There is here a choice between accepting or
rejecting the application for citizenship. But this
policy finds no application in cases where
citizenship is conferred by operation of law. In
such cases, the courts have no choice to accept
or reject. If the individual claiming citizenship by
operation of law proves in legal proceedings that
he satisfies the statutory requirements, the
courts cannot do otherwise than to declare that
he is a citizen of the Philippines. Thus, an
individual who is able to prove that his father is a
Philippine citizen, is a citizen of the Philippines,
'irrespective of his moral character, ideological
beliefs, and identification with Filipino ideals,
customs, and traditions.' A minor child of a
person naturalized under the law, who is able to
prove the fact of his birth in the Philippines, is
likewise a citizen, regardless of whether he has
lucrative income, or he adheres to the principles
of the Constitution. So it is with an alien wife of a
Philippine citizen. She is required to prove only
that she may herself be lawfully naturalized, i.e.,
that she is not one of the disqualified persons

enumerated in Section 4 of the law, in order to


establish her citizenship status as a fact.
"A paramount policy consideration of graver
import should not be overlooked in this regard,
for it explains and justifies the obviously
deliberate choice of words. It is universally
accepted that a State, in extending the privilege
of citizenship to an alien wife of one of its citizens
could have had no other objective than to
maintain a unity of allegiance among the
members of the family. (Nelson v. Nelson, 113
Neb. 453, 203 N. W. 640 [1925]; see also
'Convention on the Nationality of Married Women:
Historical Background and Commentary.' UNITED
NATIONS, Department of Economic and Social
Affairs E/CN, 6/399, pp. 8 et seq.). Such objective
can only be satisfactorily achieved by allowing
the wife to acquire citizenship derivatively
through the husband. This is particularly true in
the Philippines where tradition and law has
placed the husband as head of the family, whose
personal status and decisions govern the life of
the family group. Corollary to this, our laws look
with favor on the unity and solidarity of the family
(Art. 220, Civil Code), in whose preservation of
State as a vital and enduring interest. (See Art.
216, Civil Code). Thus, it has been said that by
tradition in our country, there is a theoretic
identity of person and interest between husband
and wife, and from the nature of the relation, the
home of one is that of the other. (See De la Via
v. Villareal, 41 Phil. 13). It should likewise be said
that because of the theoretic identity of husband
and wife, and the primacy of the husband, the
nationality of husband should be the nationality
of the wife, and the laws upon one should be the
law upon the other. For as the court, in Hopkins v.
Fachant (9th Cir., 1904) 65 C.C.A., 1, 130 Fed.
839, held: 'The status of the wife follows that of
the husband, . . . and by virtue of her marriage
her husband's domicile became her domicile.'
And the presumption under Philippine law being
that the property relations of husband and wife
are under the regime of conjugal partnership (Art.
119, Civil Code), the income of one is also that of
the other.
"It is, therefore, not congruent with our cherished
traditions of family unity and identity that a
husband should be a citizen and the wife an alien,
and that the national treatment of one should be
different from that of the other. Thus, it cannot be
that the husband's interests in property and
business activities reserved by law to citizens
should not form part of the conjugal partnership
and be denied to the wife, nor that she herself
cannot, through her own efforts but for the
benefit of the partnership, acquire such interests.
Only in rare instances should the identity of
husband and wife be refused recognition, and we
submit that in respect of our citizenship laws, it
should only be in the instances where the wife
suffers from the disqualifications stated in Section
4 of the Revised Naturalization Law." (Motion for
Reconsideration, Burca vs. Republic, supra.)
With all these considerations in mind, We are
persuaded that it is in the best interest of all
76 | C O N S T I 2 _ A r t i c l e I V _ C I T I Z E N S H I P

concerned that Section 15 of the Naturalization


Law be given effect in the same way as it was
understood and construed when the phrase "who
may be lawfully naturalized," found in the
American statute from which it was borrowed and
copied verbatim, was applied by the American
courts and administrative authorities. There is
merit, of course, in the view that Philippine
statutes should be construed in the light of
Philippine circumstances, and with particular
reference to our naturalization laws. We should
realize the disparity in the circumstances
between the United States, as the so-called
"melting pot" of peoples from all over the world,
and the Philippines as a developing country
whose Constitution is nationalistic almost in the
extreme. Certainly, the writer of this opinion
cannot be the last in rather passionately insisting
that our jurisprudence should speak our own
concepts and resort to American authorities, to
be sure, entitled to admiration and respect,
should not be regarded as source of pride and
indisputable authority. Still, We cannot close our
eyes to the undeniable fact that the provision of
law now under scrutiny has no local origin and
orientation; it is purely American, factually taken
bodily from American law when the Philippines
was under the dominating influence of statutes of
the United States Congress. It is indeed a sad
commentary on the work of our own legislature of
the late 1920's and 1930's that given the
opportunity to break away from the old American
pattern, it took no step in that direction. Indeed,
even after America made it patently clear in the
Act of Congress of September 22, 1922 that alien
women marrying Americans cannot be citizens of
the
United
States
without
undergoing
naturalization proceedings, our legislators still
chose to adopt the previous American law of
August 10, 1855 as embodied later in Section
1994 of the Revised Statutes of 1874, which, it is
worth reiterating, was consistently and uniformly
understood as conferring American citizenship to
alien women marrying Americans ipso facto,
without having to submit to any naturalization
proceeding and without having to prove that they
possess the special qualifications of residence,
moral character, adherence to American ideals
and American constitution, provided they show
they did not suffer from any of the
disqualifications enumerated in the American
Naturalization Law. Accordingly, We now hold, all
previous decisions of this Court indicating
otherwise notwithstanding, that under Section 15
of Commonwealth Act 473, an alien woman
marrying a Filipino, native born or naturalized,
becomes ipso facto a Filipina provided she is not
disqualified to be a citizen of the Philippines
under Section 4 of the same law. Likewise, an
alien woman married to an alien who is
subsequently naturalized here follows the
Philippine citizenship of her husband the moment
he takes his oath as Filipino citizen, provided that
she does not suffer from any of the
disqualifications under said Section 4.
As under any other law rich in benefits for those
coming under it, doubtless there will be instances
where unscrupulous persons will attempt to take

advantage of this provision of law by entering


into fake and fictitious marriages or mala fide
matrimonies. We cannot as a matter of law hold
that just because of these possibilities, the
construction of the provision should be otherwise
than as dictated inexorably by more ponderous
relevant considerations, legal, juridical and
practical. There can always be means of
discovering such undesirable practices and every
case can be dealt with accordingly as it arises.
III.
The third aspect of this case requires necessarily
a re-examination of the ruling of this Court in
Burca, supra, regarding the need of judicial
naturalization proceedings before the alien wife
of a Filipino may herself be considered or deemed
a Filipino. If this case which, as already noted,
was submitted for decision in 1964 yet, had only
been decided earlier, before Go Im Ty, the
foregoing discussions would have been sufficient
to dispose of it. The Court could have held that
despite her apparent lack of qualifications, her
marriage to her co-petitioner made her a Filipina,
without her undergoing any naturalization
proceedings, provided she could sustain her claim
that she is not disqualified under Section 4 of the
law. But as things stand now, with the Burca
ruling, the question We have still to decide is,
may she be deemed a Filipina without submitting
to a naturalization proceeding?
Naturally, if Burca is to be followed, it is clear that
the answer to this question must necessarily be
in the affirmative. As already stated, however,
the decision in Burca has not yet become final
because there is still pending with Us a motion for
its reconsideration which vigorously submits
grounds worthy of serious consideration by this
Court. On this account, and for the reasons
expounded earlier in this opinion, this case is as
good an occasion as any other to re-examine the
issue.
In the said decision, Justice Sanchez held for the
Court:
"We accordingly rule that: (1) An alien woman
married to a Filipino who desires to be a citizen of
this country must apply therefore by filing a
petition for citizenship reciting that she possesses
all the qualifications set forth in Section 2 and
none of the disqualifications under Section 4,
both of the Revised Naturalization Law; (2) Said
petition must be filed in the Court of First
Instance where petitioner has resided at least one
year immediately preceding the filing of the
petition; and (3) Any action by any other office,
agency, board or official, administrative or
otherwise other than the judgment of a
competent court of justice certifying or
declaring that an alien wife of the Filipino citizen
is also a Filipino citizen, is hereby declared null
and void.

"3. We treat the present petition as one for


naturalization. Or, in the words of law, a 'petition
77 | C O N S T I 2 _ A r t i c l e I V _ C I T I Z E N S H I P

for citizenship'. This is as it should be. Because a


reading of the petition will reveal at once that
efforts were made to act forth therein. and to
prove afterwards, compliance with Sections 2 and
4 of the Revised Naturalization law. The trial court
itself apparently considered the petition as one
for naturalization, and, in fact, declared petitioner
'a citizen of the Philippines.'"
In other words, under this holding, in order for an
alien woman marrying a Filipino to be vested with
Filipino citizenship, it is not enough that she
possesses the qualifications prescribed by
Section 2 of the law and none of the
disqualifications enumerated in its Section 4.
Over and above all these, she has to pass thru
the whole process of judicial naturalization,
apparently from declaration of intention to oathtaking, before she can become a Filipina. In plain
words, her marriage to a Filipino is absolutely of
no consequence to her nationality vis-a-vis that of
her Filipino husband; she remains to be the
national of the country to which she owed
allegiance before her marriage, and if she desires
to be of one nationality with her husband, she has
to wait for the same time that any other applicant
for naturalization needs to complete, the required
period of ten year residence, gain the knowledge
of English or Spanish and one of the principal
local languages, make her children study in
Filipino schools, acquire real property or engage
in some lawful occupation of her own
independently of her husband, file her declaration
of intention and after one year her application for
naturalization, with the affidavits of two credible
witnesses of her good moral character and other
qualifications, etc., etc., until a decision is
rendered in her favor, after which, she has to
undergo the two years of probation, and only
then, but not before she takes her oath as citizen,
will she begin to be considered and deemed to be
a citizen of the Philippines. Briefly; she can
become a Filipino citizen only by judicial
declaration.
Such being the import of, the Court's ruling, and
it being quite obvious, on the other hand, upon a
cursory reading of the provision, in question, that
the law intends by it to spell out what is the
"effect of naturalization on (the) wife and
children" of an alien, as plainly indicated by its
title, and inasmuch as the language of the
provision itself clearly conveys the thought that
some effect beneficial to the wife is intended by
it, rather than that she is not in any manner to be
benefited thereby, it behooves Us to take a
second hard look at the ruling, if only to see
whether or not the Court might have overlooked
any
relevant
consideration
warranting
a
conclusion different from that contained therein.
It is undeniable that the issue before Us is of
grave importance, considering its consequences
upon tens of thousands of persons affected by
the ruling therein made by the Court, and surely,
it is for Us to avoid, whenever possible, that Our
decision in any case should produce any adverse
effect upon them not contemplated either by the
law or by the national policy it seeks to enforce.

AMICI CURIAE in the Burca case, respectable and


impressive by their number and standing in the
Bar and well known for their reputation for
intellectual integrity, legal acumen and incisive
and comprehensive resourcefulness in research,
truly evident in the quality of the memorandum
they have submitted in said case, invite Our
attention to the impact of the decision therein
thus:
"The doctrine announced by this Honorable Court
for the first time in the present case that an
alien woman who marries a Philippine citizen not
only does not ipso facto herself become a citizen
but can acquire such citizenship only through
ordinary naturalization proceedings under the
Revised Naturalization Law, and that all
administrative actions 'certifying or declaring'
such woman to be a Philippine citizen are 'null
and void' has consequences that reach far
beyond the confines of the present case.
Considerably more people are affected, and
affected deeply, than simply Mrs. Zita N. Burca.
The newspapers report that as many as 15
thousand women married to Philippine citizens
are affected by this decision of the Court. These
are women of many and diverse nationalities,
including Chinese, Spanish, British, American,
Columbian, Finnish, Japanese, Chilean, and so on.
These members of the community, some of
whom have been married to citizens for two or
three decades, have all exercised rights and
privileges reserved by law to Philippine citizens.
They will have acquired, separately or in conjugal
partnership with their citizen husbands, real
property, and they will have sold and transferred
such property. Many of these women may be in
professions membership in which is limited to
citizens. Others are doubtless stockholders or
officers or employees in companies engaged in
business activities for which a certain percentage
of Filipino equity content is prescribed by law. All
these married women are now faced with
possible divestment of personal status and of
rights acquired and privileges exercised in
reliance, in complete good faith, upon a reading
of the law that has been accepted as correct for
more than two decades by the very agencies of
government charged with the administration of
that law. We must respectfully suggest that
judicial doctrines which would visit such
comprehensive and far-reaching injury upon the
wives and mothers of Philippine citizens deserve
intensive scrutiny and re-examination."
To be sure, this appeal can be no less than what
this Court attended to in Gan Tsitung vs. Republic,
G.R. No L-20819, Feb. 21, 1967, 19 SCRA 401
when Chief Justice Concepcion observed:
"The Court realizes, however, that the rulings in
the Barretto and Delgado cases although
referring to situations the equities of which are
not identical to those obtaining in the case at bar
may have contributed materially to the
irregularities committed therein and in other
analogous cases, and induced the parties
concerned to believe, although erroneously, that
the procedure followed was valid under the law.
78 | C O N S T I 2 _ A r t i c l e I V _ C I T I Z E N S H I P

"Accordingly, and in view of the implications of


the issue under consideration, the Solicitor
General was required, not only, to comment
thereon, but, also, to state 'how many cases
there are, like the one at bar, in which certificates
of naturalization have been issued after notice of
the filing of the petition for naturalization had
been published in the Official Gazette only once,
within the periods (a) from January 28, 1950'
(when the decision in Delgado v. Republic was
promulgated) 'to May 29, 1957' (when the Ong
Son Cui was decided) 'and (b) from May 29, 1957
to November 29, 1965' (when the decision in the
present case was rendered).
"After mature deliberation, and in the light of the
reasons adduced in appellant's motion for
reconsideration and in the reply thereto of the
Government, as well as of the data contained in
the latter, the Court holds that the doctrine laid
down in the Ong Son Cui case shall apply and
affect the validity of certificates of naturalization
issued after, not on or before May 29, 1957."
Here We are met again by the same problem. In
Gan Tsitung, the Court had to expressly enjoin the
prospective application of its construction of the
law made in a previous decision 24 which had
already become final, to serve the ends of justice
and equity. In the case at bar, We do not have to
go that far. As already observed, the decision in
Burca is still under reconsideration, while the
ruling in Lee Suan Ay, Lo San Tuang, Choy King
Tee and other that followed them have at the
most become the law of the case only for the
parties thereto. If there are good grounds
therefor, all We have to do now is to re-examine
the said rulings and clarify them.
For ready reference, We requote Section 15:
"Sec. 15. Effect of the naturalization on wife and
children . Any woman who is now or may
hereafter be married to a citizen of the
Philippines, and who might herself be lawfully
naturalized shall be deemed a citizen of the
Philippines.
"Minor children of persons naturalized under this
law who have been born in the Philippines shall
be consider citizens thereof.
"A foreign-born minor child, if dwelling in the
Philippines at the time of naturalization of the
parents, shall automatically become a Philippine
citizen, and a foreign-born minor child, who is not
in the Philippines at the time the parent is
naturalized, shall be deemed a Philippines citizen
only during his minority, unless he begins to
reside permanently in the Philippines when still a
minor, in which case, he will continue to be a
Philippine citizen even after becoming of age.
"A child born outside of the Philippines after the
naturalization of his parent, shall be considered a
Philippine citizen, unless within one year after
reaching the age of minority, he fails to register
himself as a Philippine citizen at the American

Consulate of the country where he resides, and to


take the necessary oath of allegiance."
It is obvious that the main subject-matter and
purpose of the statute, the Revised Naturalization
Law or Commonwealth Act 473, as a whole is to
establish a complete procedure for the judicial
conferment of the of the status of citizenship
upon qualified aliens. After having out such a
procedure, remarkable for its elaborate and
careful inclusion of all safeguards against the
possibility of any undesirable persons becoming a
part of our citizenry, it carefully but categorically
states the consequence of the naturalization of
an alien undergoing such procedure it prescribes
upon members of his immediate family, his wife
and children, 25 and, to that end, in no uncertain
terms it ordains that: (a) all his minor children
who have been born in the Philippines shall be
"considered citizens" also; (b) all such minor
children, if born outside the Philippines but
dwelling here at the time of such naturalization
"shall automatically become" Filipinos also, but
those not born in the Philippines and not in the
Philippines at the time of such naturalization, are
also "deemed citizens" of this country provided
that they shall lose said status if they transfer
their permanent residence to a foreign country
before becoming of age; (c) all such minor
children, if born outside of the Philippines after
such naturalization, shall also be "considered"
Filipino
citizens,
unless
they
expatriate
themselves by failing to register as Filipinos at
the Philippine (American) Consulate of the
country where they reside and take the necessary
oath of allegiance; and (d) as to the wife, she
"shall be deemed a citizen of the Philippines" if
she is one "who might herself be lawfully
naturalized". 26

No doubt whatever is entertained, so Burca holds


very correctly, as to the point that the minor
children, failing within the conditions of place and
time of birth and residence prescribed in the
provision, are vested with Philippines citizenship
directly by legislative fiat or by force of the law
itself and without the need for any judicial
proceeding or declaration. (At p. 192 SCRA).
Indeed, the language of the provision is not
susceptible of any other interpretation. But it is
claimed that the same expression "shall be
deemed a citizen of the Philippines" in reference
to the wife, does not necessarily connote the
vesting of citizenship status upon her by
legislative fiat because the antecedent phrase
requiring that she must be one "who might
herself be lawfully naturalized" implies that such
status is intended to attach only after she has
undergone the whole process of judicial
naturalization required of any person desiring to
become a Filipino. Stated otherwise, the ruling in
Burca is that while Section 15 envisages and
intends legislative naturalization as to the minor
children, the same section deliberately treats the
wife differently and leaves her out for ordinary
judicial naturalization.
79 | C O N S T I 2 _ A r t i c l e I V _ C I T I Z E N S H I P

Of course, it goes without saying that it is


perfectly within the constitutional authority of the
Congress of the Philippines to confer or vest
citizenship status by legislative fiat. (U.S. v. Wong
Kim Ark, 169 U.S. 649, 42 L ed. 890 [1898]; See,
1 Taada & Carreon, Political Law of the
Philippines 152 [1961 ed.] ) In fact, it has done so
for particular individuals, like two foreign religious
prelates, 27 hence there is no reason it cannot do
it for classes or groups of persons under general
conditions applicable to all of the members of
such class or group, like women who marry
Filipinos, whether native-born or naturalized. The
issue before Us in this case is whether or not the
legislature has done so in the disputed provisions
of Section 15 of the Naturalization Law. And Dr.
Vicente G. Sinco, one of the most respected
authorities on political law in the Philippines 28
observes in this connection thus: " A special form
of naturalization is often observed by some states
with respect to women. Thus in the Philippines a
foreign woman married to a Filipino citizen
becomes ipso facto naturalized, if she belongs to
any of the classes who may apply for
naturalization under the Philippine Laws" (Sinco,
Phil. Political Law 498-499 [10th ed. 1954];
emphasis ours; this comment is substantially
reiterated in the 1962 edition, citing Ly Giok Ha
and Ricardo Cua , supra.)
More importantly, it may be stated at this
juncture, that in construing the provision of the
United States statutes from which our law has
been copies, 28a the American citizenship by
choice but by operation of law. "In the Revised
Statutes the words 'and taken' are omitted. The
effect of this statute is that every alien woman
who marries a citizen of the United States
becomes perforce a citizen herself, without the
formality of naturalization, and regardless of her
wish in that respect." (USCA 8, p. 601 [1970 ed.],
citing Mackenzie v. Hare, 1913, 134 P. 713, 165
Cal. 766, affirmed 36 S. Ct. 106, 239 U.S. 299, 60
L ed. 297.)
We need not recount here again how this
provision in question was first enacted as
paragraph (a) of Section 13, by way of an
insertion into Act 2927 by Act 3448 of November
30, 1928, and that , in turn, and paragraph was
copied verbatim from Section 1994 of the Revised
Statutes of the United States, which by that time
already had a long accepted construction among
the courts and administrative authorities in that
country holding that under such provision an
alien woman who married a citizen became, upon
such marriage, likewise a citizen by force of law
and as a consequence of the marriage itself
without having to undergo any naturalization
proceedings, provided that it could be shown that
at the time of such marriage, she was not
disqualified to be naturalized under the laws then
in force. To repeat the discussion We already
made
of
these
undeniable
facts
would
unnecessarily
make
this
decision
doubly
extensive. The only point which might be
reiterated for emphasis at this juncture is that
whereas in the United States, the American
Congress, recognizing the uniform construction of

Section 1994 of the Revised Statutes to be as


stated above, and finding it desirable to avoid the
effects of such construction, approved the Act of
September 22, 1922 explicitly requiring all such
alien wives to submit to judicial naturalization,
albeit under more liberal terms than those for
other applicants for citizenship, on the other
hand, the Philippines Legislature, instead of
following suit and adopting a requirement,
enacted Act 3448 on November 30, 1928 which
copied verbatim the aforementioned Section
1994 of the Revised Statutes, thereby indicating
its preferences to adopts the latter law and its
settled constitution rather than the reform
introduced by the Act of 1992.
Obviously, these considerations leave Us no
choice. Much as this Court may feel that as the
United States herself has evidently found it to be
an improvement of her national policy vis-a-vis
the alien wives of her citizens to discontinue their
automatic incorporation into the body of her
citizenry without passing through the judicial
scrutiny of a naturalization proceeding, as it used
to be before 1922, it seems but proper, without
evidencing any bit of colonial mentality, that as a
developing country, the Philippines adopt a
similar policy, unfortunately, the manner in which
our own legislature has enacted our laws on the
subject, as recounted above, provides no basis
for Us to construe said law along the line of the
1922 modification of the American Law. For Us to
do so would be to indulge in judicial legislation
which it is not constitutionally permissible for this
Court to do. Worse, this Court would be going
precisely against the grain of the implicit
Legislative intent.
There is at least one decision of this Court before
Burca wherein it seems it is quite clearly implied
that this Court is of the view that under Section
16 of the Naturalization Law, the widow and
children of an applicant for naturalization who
dies during the proceeding do not have to submit
themselves to another naturalization proceeding
in order to avail of the benefits of the proceedings
involving the husband. Section 16 provides:
"SEC. 16. Right of widow and children of
petitioners who have died. In case a petitioner
should die before the final decision has been
rendered, his widow and minor children may
continue the proceedings. The decision rendered
in the case shall, so far same legal effect as if it
had been rendered during the life of the
petitioner."
In Tan Lin v. Republic, G.R. No. L-13706, May 31,
1961, 2 SCRA 383 this Court held:
"Invoking the above provisions in their favor,
petitioners-appellants argue (1) that under said
Sec. 16, the widow and minor children are
allowed to continue the same proceedings and
are not substituted for the original petitioner; (2)
that the qualifications of the original petitioner
remain to be in issue and not those of the widow
and minor children, and (3) that said Section 16
applies whether the petitioner dies before or after
80 | C O N S T I 2 _ A r t i c l e I V _ C I T I Z E N S H I P

final decision is rendered,


judgment becomes executory.

but

before

the

"There is force in the first and second arguments.


Even the second sentence of said Section 16
contemplates the fact that the qualifications of
the original petitioner remains the subject of
inquiry, for the simple reason that it states that
"The decision rendered in the case shall, so far as
the widow and minor children are concerned,
produce the same legal effect as if it had been
rendered during the life of the petitioner.' This
phraseology emphasizes the intent of the law to
continue the proceedings with the deceased as
the theoretical petitioner, for if it were otherwise,
it would have been unnecessary to consider the
decision rendered, as far as it effected the widow
and the minor children.
xxx xxx xxx
"The Chua Chian case (supra), cited by the
appellee, declared that a dead person can not be
bound to do things stipulated in the oath of
allegiance, because an oath is a personal matter.
Therein, the widow prayed that she be allowed to
take the oath of allegiance for the deceased. IN
the case at bar, petitioner Tan Lin merely asked
that she be allowed to take the oath of allegiance
and the proper certificate of naturalization, once
the naturalization proceedings of her deceased
husband, shall have been completed, not on
behalf of the deceased, but on her own behalf
and of her children, as recipients of the benefits
of his naturalization. In other words, the herein
petitioner proposed to take the oath of allegiance,
as a citizen of the Philippines, by virtue of the
legal provision that 'any woman who is now or
may hereafter be married to a citizen of the
Philippines and who might be lawfully naturalized
shall be deemed a citizen of the Philippines. Minor
children of persons naturalized under this law
who have been born in the Philippines shall be
considered
citizens
thereof.'
(Section
15,
Commonwealth Act No. 473). The decision
granting citizenship to Lee Pa and the record of
the case at bar, do not show that the petitioning
widow could not have been lawfully naturalized,
at the time Lee Pa filed his petition, apart from
the fact that his 9 minor children were all born in
the Philippines. (Decision, In the Matter of the
P)etition of Lee Pa to be admitted a citizen of the
Philippines, Civil Case No. 16287, CFI, Manila,
Annex A; Record on Appeal, pp. 8-11). The
reference for Chua case is, therefore, premature."
Section 16, as may be seen, is a parallel provision
to Section 15. If the widow of an applicant for
naturalization as Filipino, who dies during the
proceedings, is not required to go through a
naturalization proceedings, in order to be
considered as a Filipino citizen hereof, it should
follow that the wife of a living Filipino cannot be
denied the same privilege. This is plain common
sense and there is absolutely no evidence that
the Legislature intended to treat them differently.

Additionally, We have carefully considered the


arguments advanced in the motion for
reconsideration in Burca, and We see no reason
to disagree with the following views of counsel:
"It is obvious that the provision itself is a
legislative declaration of who may be considered
citizens of the Philippines. It is a proposition too
plain to be disputed that Congress has the power
not only to prescribe the mode or manner under
which foreigners may acquire citizenship, but also
the very power of conferring citizenship by
legislative fiat. (U.S. v. Wong Kim Ark, 169 U.S.
649, 42 L. Ed. 890 [1898]; see 1 Taada and
Carreon, Political Law of the Philippine citizens
ed.]). The constitutional itself recognizes as
Philippines citizens 'Those who are naturalized in
accordance with law' (Section 1[5], Article IV,
Philippine
Constitution).
Citizens
by
naturalization, under this provision, include not
only those who are naturalized in accordance
with legal proceedings for the acquisition of
citizenship, but also those who acquire citizenship
by 'derivative naturalization' or by operation of
law, as. for example, the 'naturalization' of an
alien wife through the naturalization of her
husband, or by marriage of an alien woman to a
citizen. (See Taada & Carreon, op. cit supra, at
152 172; Velayo, Philippine Citizenship and
Naturalization 2 [1965 ed.]: 1 Paras, Civil code
186 [1967 ed.]; see also 3 Hackworth, Digest of
International Law 3).
"The phrase 'shall be deemed a citizen of the
Philippines found in Section 14 of the Revised
Naturalization Law clearly manifests an intent to
confer citizenship. Construing a similar phrase
found in the old U.S. naturalization law (Revised
Statutes, 1994) , American courts have uniformly
taken it to mean that upon her marriage, the
alien woman becomes by operation of law a
citizen of the United States as fully as if she had
complied with all the provisions of the statutes
upon the subject of naturalization. (U.S. v. Keller,
13 F. 82; U.S. Opinions of the US Attorney General
dated June 4, 1874 [14 Op. 402], July 20, 1909
[27 Op. 507], December 1, 1910 [28 Op. 508],
Jan. 15, 1920 [32 Op. 209] and Jan. 12, 1923 [23
398] ).
'The phrase "shall be deemed a citizen, " in
Section 1994 Revised Statute (U.S. Comp. Stat.
1091 1268) or as it was in the Act of 1855 910
Stat. at L. 604, Chapt. 71, Sec. 2), "shall be
deemed and taken to be a citizen", while it may
imply that the person to whom it relates has not
actually become a citizen by the ordinary means
or in the usual way, as by the judgment of a
competent court, upon a proper application and
proof, yet it does not follow that such person is on
that account practically any the less a citizen.
The word "deemed" is the equivalent of
"considered" or "judged," and therefore, whatever
an Act of Congress requires to be "deemed" or
"taken" as true of any person or thing must, in
law, be considered as having been duly adjudged
or established concerning such person or thing,
and have force and effect accordingly. When,
therefore, Congress declares that an alien woman
81 | C O N S T I 2 _ A r t i c l e I V _ C I T I Z E N S H I P

shall, under certain circumstances, be "deemed"


an American to her being naturalized directly by
an Act of Congress or in the usual mode thereby
prescribed.' (Van Dyne, Citizenship of the United
States 239, cited in Velayo, Philippine Citizenship
and Naturalization 146-147 [1965 ed.] ; italics
ours).
"That this was likewise the intent of the Philippine
legislature when it enacted the first paragraph of
Section 15 of the Revised Naturalization
provision. In its entirely, Section 15 reads:
(See supra)
The phrases 'shall be deemed,' shall be
considered,' and 'shall automatically become,' as
used in the above provision , are undoubtedly
synonymous. The leading idea or purpose of the
provision was to confer Philippine citizenship by
operation of law upon certain classes of aliens as
a legal consequence of their relationship, by
blood affinity, to persons who are already citizens
of the Philippines. Whenever the fact of
relationship of the persons enumerated in the
provision concurs related, the effect is for said
persons to become ipso facto citizens of the
Philippines. 'Ipso facto' as here used does not
mean that all alien wives and all minor children of
Philippine citizens, from the mere fact of
relationship, necessarily become such citizens
also. Those who do not meet the statutory
requirements do not ipso facto become citizens;
they must apply for naturalization in order to
acquire such status. What it does mean, however,
is that in respect of those persons numerated in
Section 15, the relationship to a citizen of the
Philippines is the operative fact which establishes
the acquisition of Philippine citizenship by them.
Necessarily, it also determines the points of time
at which such citizenship commences. Thus,
under the second paragraph of Section 15, a
minor child of a Filipino naturalized under the law,
who was born in the Philippines, becomes ipso
facto a citizen of the Philippines from the time the
fact of relationship concurs with the fact of a
citizenship of his parent, and the time when child
became a citizen does not depend upon the time
that he is able to prove that he was born in the
Philippines. The child may prove some 25 years
after the naturalization of his father that he was
born in the Philippines and should, therefore, be
'considered' a citizen thereof. It does not mean
that he became a Philippine citizen only at that
later time. Similarly, an alien woman who married
a Philippine citizen may be able to prove only
some 25 years after her marriage (perhaps,
because it was only 25 years after the marriage
that her citizenship status became in question),
that she is one who 'might herself be lawfully
naturalized.' It is not reasonable to conclude that
she acquired Philippine citizenship only after she
had proven that she 'might herself be lawfully
naturalized.'
"The point that bears emphasis in this regard is
that in adopting the very phraseology of the law ,
the legislature could not have intended that an
alien wife should not be deemed a Philippine

citizen unless and until she proves that she might


herself be lawfully naturalized' is not a condition
precedent to the vesting or acquisition of
citizenship; it is only a condition or a state of fact
necessary to establish her citizenship as a factum
probandum i.e., as a fact established and proved
in evidence. The word 'might,' as used in that
phrase, precisely implies that at the time of her
marriage to a Philippine citizen, the alien woman
'had (the) power' to become such a citizen herself
under the laws then in force. (Owen v. Kelly, 6 DC
191 [1867], aff'd Kelly v. Owen, power long after
her marriage does not alter the fact that at her
marriage, she became a citizen.
"(This Court has held) that 'an alien wife of a
Filipino citizen may not acquire the status of a
citizen of the Philippines unless there is proof that
she herself may be lawfully naturalized'
(Decision, pp. 3-4). Under this view, the
acquisition' of citizenship by the alien wife
depends on her having proven her qualifications
for citizenship, that is, she is not a lawfully
naturalized. It is clear from the words of the law
that the proviso does not mean that she must
first prove that deemed (by Congress, not by the
courts) a citizen. Even the 'uniform' decisions
cited by this Court (at fn. 2) to support its holding
did not rule that the alien wife becomes a citizen
only after she has proven her qualifications for
citizenship. What those decisions ruled was that
the alien wives in those cases failed to prove their
qualifications and therefore they failed to
establish their claim to citizenship. Thus in Ly
Giok Ha v. Galang, 101 Phil. 459 [1957], the case
was remanded to the lower court for
determination of whether petitioner, whose claim
to citizenship by marriage to a Filipino was
disputed by the Government, 'might herself be
lawfully naturalized,' for the purpose of 'proving
her alleged change of political status from alien
to citizen' (at 464). In Cua v. Board, 101 Phil. 521
[1957], the alien wife who was being deported,
claimed she was a Philippine citizen by marriage
to a Filipino. This Court finding that there was no
proof that she was not disqualified under Section
4 of the Revised Naturalization Law, ruled that:
'No such evidence appearing on record, the claim
of assumption of Philippine citizenship by Tijoe
Wu Suan, upon her marriage to petitioner, is
untenable.' (at 523) It will be observed that in
these decisions cited by this Court, the lack of
proof that the alien wives 'might (themselves) be
lawfully naturalized' did not necessarily imply
that they did not become, in truth and in fact,
citizens upon their marriage to Filipinos. What the
decisions merely held was that these wives failed
to establish their claim to that status as a proven
fact.
"In all instances where citizenship is conferred by
operation of law, the time when citizenship is
conferred should not be confused with the time
when citizenship status is established as a proven
fact. Thus, even a natural-born citizen of the
Philippines, whose citizenship status is put in
issue in any proceeding would be required to
prove, for instance, that his father is a citizen of
the Philippines in order to factually establish his
82 | C O N S T I 2 _ A r t i c l e I V _ C I T I Z E N S H I P

claim to citizenship. *** His citizenship status


commences from the time of birth, although his
claim thereto is established as a fact only at a
subsequent time. Likewise, an alien woman who
might herself be a lawfully naturalized becomes a
Philippine citizen at the time of her marriage to a
Filipino husband, not at the time she is able to
establish that status as a proven fact by showing
that she might herself be lawfully naturalized.
Indeed, there is no difference between a statutory
declaration that a person is deemed a citizen of
the Philippines provided his father is such citizen
from a declaration that an alien woman married
to a Filipino citizen of the Philippines provided his
father is such citizen from a declaration that an
alien woman married to a Filipino citizen of the
Philippines provided she might herself be lawfully
naturalized. Both become citizens by operation of
law; the former becomes a citizen ipso facto upon
birth; the later ipso facto upon marriage.

"It is true that unless and until the alien wife


proves that she might herself be lawfully
naturalized, it cannot be said that she has
established her status as a proven fact. But
neither can it be said that on account, she did not
become a citizen of the Philippines. If her
citizenship status is not questioned in any legal
proceeding, she obviously has no obligation to
establish her status as a fact. In such a case, the
presumption of law should be what she claims to
be. (U.S. v. Roxas, 5 Phil. 375 [1905] : Hilado v.
Assad, 51 O.G. 4527 [1955] ). There is a
presumption that a representation shown to have
been made is true. (Aetna Indemnity Co. v.
George A. Fuller, Co., 73 A. 738 A. 369, 111 ME.
321)."
The question that keeps bouncing back as a
consequence of the foregoing views is, what
substitute is there for naturalization proceeding
to enable the alien wife of a Philippine citizen to
have the matter of her own citizenship settled
and established so that she may not have to be
called upon to prove it everytime she has to
perform an act or enter into a transaction or
business or exercise a right reserved only to
Filipinos? The ready answer to such question is
that as the laws of our country, both substantive
and procedural, stand today, there is no such
procedure, but such is no proof that the
citizenship under discussion is not vested as of
the date of marriage or the husband's acquisition
of citizenship, as the case may be, for the truth is
that the situation obtains even as to native-born
Filipinos. Everytime the citizenship of a person is
material or indispensible in a judicial or
administrative case, Whatever the corresponding
court or administrative authority decides therein
as to such citizenship is generally not considered
as res adjudicata, hence it has to be threshed out
again and again as the occasion may demand.
This, as We view it, is the sense in which Justice
Dizon referred to "appropriate proceeding" in
Brito v. Commissioner, supra. Indeed, only the
good sense and judgment of those subsequently
inquiring into the matter may make the effort

easier or simpler for the persons concerned by


relying somehow on the antecedent official
findings, even if these are not really binding.

Fernando, J ., concurs except as the interpretation


accorded some American decisions as to which
he is not fully persuaded.

It may not be amiss to suggest, however, that in


order to have a good starting point and so that
the most immediate relevant public records may
be kept in order the following observations in
Opinion No. 38, series of 1958, of then Acting
Secretary of Justice Jesus G. Barrera, may be
considered as the most appropriate initial step by
the interested parties.

APPENDIX
The following review of all naturalization statutes
of the United States from 1790 to 1970 ravel: (1)
that
aside
from
race,
various
other
disqualifications have also been provided for in
the said statutes from time to time, although it
was only in 1906 that the familiar and usual
grounds of disqualification, like not being
anarchists, polygamists, etc. were incorporated
therein, and (2) that qualifications of applicants
for naturalization also varied from time to time.

"Regarding the steps that should be taken by an


alien woman married to a Filipino citizen in order
to acquire Philippine citizenship, the procedure
followed in the Bureau of Immigration is as
follows: The alien woman must file a petition for
the cancellation of her alien certificate of
registration alleging, among other things, that
she is married to a Filipino citizen and that she is
not disqualified from acquiring her husband's
citizenship
pursuant
to
section
4
of
Commonwealth Act No. 473, as amended. Upon
the filing of said petition, which should be
accompanied or supported by the joint affidavit of
the petitioner and her Filipino husband to the
effect that the petitioner does not belong to any
of the groups disqualified by the cited section
from becoming naturalized Filipino citizen (please
see attached CEB Form 1), the Bureau of
Immigration conducts an investigation and
thereafter promulgates its order or decision
granting or denying the petition."
Once the Commissioner of Immigration cancels
the subject's registration as an alien, there will
probably be less difficulty in establishing her
Filipino citizenship in any other proceeding,
depending naturally on the substance and vigor
of the opposition.
Before closing, it is perhaps best to clarify that
this third issue We have passed upon was not
touched by the trial court, but as the point is
decisive in this case, the Court prefers that the
matter be settled once and for all now.
IN VIEW OF ALL THE FOREGOING, the judgment of
the Court a quo dismissing appellants' petition for
injunction
is
hereby
reversed
and
the
Commissioner
of
Immigration
and/or
his
authorized
representative
is
permanently
enjoined from causing the arrest and deportation
and the confiscation of the bond of appellant Lau
Yuen Yeung, who is hereby declared to have
become a Filipino citizen from and by virtue of
her marriage to her co-appellant Moy Ya Lim Yao
al as Edilberto Aguinaldo Lim, a Filipino citizen of
January 25, 1962. No costs.
Dizon, Castro, Teehankee and Villamor, JJ .,
concur.
Makalintal J ., reserves his separate concurring
opinion.

83 | C O N S T I 2 _ A r t i c l e I V _ C I T I Z E N S H I P

A DISQUALIFICATIONS
1. In the first naturalization statute of March 26,
1790, only a "free white person" could be
naturalized, provided he was not "proscribed" by
any state, unless it be with the consent of such
state. (Chap. V. 1 Stat. 103)
2. In the Act of January 29, 1795, to the same
provisions was added the disqualification of those
"legally convicted of having joined the army of
Great Britain, during the late war." (Chap. XX, 1
Stat. 414).
3. In the Act of June 18, 1798, Section 1 thereto
provided:
"SECTION 1. Be it enacted by the Senate and
House of Representatives of the United States of
America in Congress assembled, That no alien
shall be admitted to become a citizen of the
United States, or of any state, unless in the
manner prescribed by the act, entitled 'An Act to
establish an uniform rule of naturalization; and to
repeal the act heretofore passed on that subject,
'he shall have declared his intention to become a
citizen of the United States, five years, at least,
before his admission, and shall, at the time of his
application to be admitted, declare and prove, to
the satisfaction of the court having jurisdiction in
the case, that he has resided within the United
States fourteen years, at least, and within the
state or territory where, or for which such court is
at the time held five years, at least, besides
conforming
to
the
other
declarations,
renunciations and proofs, by the said act
required, any thing therein to the contrary hereof
notwithstanding: Provided, that any alien, who
was residing within the limelights, and under the
jurisdiction of the United States, before the
twenty-ninth day of January, one thousand seven
hundred and ninety-five, may, within one year
after the passing of this actand any alien who
shall have made the declaration of his intention
to become a citizen of the United States, in
conformity to the provisions of the act, entitled
'An act to establish an uniform rule of
naturalization, and to repeal the act heretofore
passed on that subject,' may, within four years
after having made the declaration aforesaid, be
admitted to become a citizen, in the manner
prescribed by the said act, upon his making proof
that he has resided five years, at least, within the
limits, and under the jurisdiction of the United

States: And provided also, that no alien, who shall


be a native, citizen, denizen or subject of any
nation or state with whom the United States shall
be at war, at the time of his application, shall be
then admitted to become a citizen of the United
States."
There is here no mention of "white persons."
(Chap. LIV, 1 Stat. 566).
4. In the Act of April 14, 1802, mentioned in Kelly
v. Owen, supra, reference was made again to
"free white persons," and the same enemy alien
and "state-proscribed" disqualifications in the
former statutes were carried over. (Chap. XXVIII,
2 Stat. 153.)
5. The Act of March 26, 1804 provided in its
Section 1 thus:
"Be it enacted by the Senate and House of
Representatives of the United States of America
in Congress assembled, That any alien, being a
free white person, who was residing within the
limits and under the jurisdiction of the United
States, at any time between the eighteenth day
of June, one thousand seven hundred and ninetyeight, and the fourteenth day of April one
thousand eight hundred and two, and who has
continued to reside within the same, may be
admitted to become a citizen of the United
States, without a compliance with the first
condition specified in the first section of the act,
entitled 'An act to establish an uniform rule of
naturalization, and to repeal the acts heretofore
passed on that subject.' "
In its Section 2, this Act already provided that:
"SEC. 2. And be it further enacted, That when any
alien who shall have complied with the first
condition specified in ,the first section of the said
original act, and who shall have pursued the
directions prescribed in the second section of the
said act, may die, before he is actually
naturalized, the widow and the children of such
alien shall be considered as citizens of the United
States, and shall be entitled to rights and
privileges as such, upon taking the oaths
prescribed by law." (CHAP. XLVII, 2 Stat. 292)
6. In the Act of July 30, 1813, the disqualification
of enemy aliens was removed as follows:
"CHAP. XXXVI. An Act supplementary to the
acts heretofore passed on the subject of an
uniform rule of naturalization. (a)
"Be it enacted by the Senate and House of
Representatives of the United States of America
in Congress assembled, That persons resident
within the United States, or the territories thereof,
on the eighteenth day of June, in the year one
thousand eight hundred and twelve, who had
before that day made declaration according to
law, of their intention to become citizens of the
United States, or who by the existing laws of the
United States, were on that day entitled to
become
citizens,
without
making
such
declaration, may be admitted to become citizens
thereof, notwithstanding they shall be alien
84 | C O N S T I 2 _ A r t i c l e I V _ C I T I Z E N S H I P

enemies at the times and in the manner


prescribed by the laws heretofore passed on that
subject: Provided, That nothing herein contained
shall be taken or construed to interfere with or
prevent the apprehension and removal, agreeably
to law, of any alien enemy at any time previous
to the actual naturalization of such alien." (Chap.
XXXVI, 3 Stat. 53)

7. Neither the Act of March 22, 1816 nor those of


May 26, 1824 and May 24, 1828 made any
change in the above requirements. (Chap. XXXII,
3 Stat. 258; Chap. CLXX-XVI, 4 Stat. 69; and
Chap. CXVI, 4 Stat. 310).
8. Then the Act of February 10, 1855, important
because it gave alien wives of citizens ,the status
of citizens, was enacted providing:
"CHAP. LXXI. An Act to secure the Right of
Citizenship to Children of Citizens of the United
States born out of the Limits thereof.
"Be it enacted by the Senate and House of
Representatives of the United States of America
in Congress assembled, That persons heretofore
born, or hereafter to be born, out of the limits and
jurisdiction of the United States, whose fathers
were or shall be at the time of their birth citizens
of the United States, shall be deemed and
considered and are hereby declared to be citizens
of the United States: Provided, however, That the
rights of citizenship shall not descend to persons
whose fathers never resided in the United States.
"SEC. 2. And be it further enacted, That any
woman who might lawfully be naturalized under
the existing laws, married, or who shall be
married to a citizen of the United States, shall
loyal be deemed and taken to be a citizen."
(Chap. LXXI, 10 Stat. 604.)
9. The Act of July 14, 1870 mainly provided only
for penalties for certain acts related to
naturalization, as punished thereby, but added in
its Section 7 "that the naturalization laws are
hereby extended to aliens of African nativity and
to African descent." (Chap. CCLIV, 16 Stat. 254.)
10. The Act of February 1, 1876 contained no
relevant amendment. (Chap. 5, 19 Stat. 2.)
11. When the statutes of the United States were
revised on June 22, 1874, the naturalization law
of the country was embodied in Sections 21652174 of saddle Revised Statutes. This contained
no racial disqualification. In fact, it reenacted
;Section 2 of the Act of February 10, 1855 as its
Section 1994 thereof, thus:
"SEC. 1994. Any person who is now or may
hereafter be married to a citizen of the United
States, and who might herself be lawfully
naturalized, shall be deemed a citizen." (18 Stat.
351.)

12. The Act of May 6, 1882 provided expressly


that no State court or court of the United State
shall admit Chinese to citizenship. (Chap. 126,
Sec. 14, 22 Stat. , 61.)
13. The Act of August 9, 1888 extended the
benefits of American citizenship to Indian woman
married to Americans thus:
"CHAP. 818. An Act in relation to marriage
between white men and Indian women.
"Be it enacted, That no white man, not otherwise
a member of any tribe of Indians, who may
hereafter marry, an Indian woman, member of
any Indian tribe in the United States, or any of its
Territories except the five civilized tribes in the
Indian Territory, shall by such marriage hereafter
acquire any right to any tribal property, privilege,
or interest whatever to which any member of
such tribe is entitled.
"SEC. 2. That every Indian woman, member of
any such tribe of Indians, who may hereafter be
married to any citizen of the United States, is
hereby declared to become by such marriage a
citizen of the United States, with all the right,
privileges, and immunities of any such citizen,
being a married woman:
"Provided, That nothing in this act contained shall
impair or in any way affect the right or title of
such married woman to any tribal property or any
interest therein.
"SEC. 2. That whenever the marriage of any white
man with any Indian woman, a member of any
such tribe of Indians, is required or offered to be
proved in any judicial ,proceeding, evidence of
the admission of such fact by the party against
whom the proceeding is had, or evidence of
general repute, or of cohabitation as married
persons, or any other circumstantial or
presumptive evidence from which the fact may
be inferred, shall be competent. (Aug. 9, 1888) "
[25 Stat. 392, Suppl. 1.]
14. The Act of April 19, 1900 extended American
citizenship to all citizens of the Republic of Hawaii
on August 12, 1898 as well as the laws of the
United States to said Republic, including, of
course, those on naturalization. (Chap. 339, Sec.
4, 31 Stat. 141.)
15. On June 29, 1906. "An Act to establish a
Bureau of Immigration and Naturalization, and to
provide a uniform rule for the naturalization of
aliens throughout the United States" was
approved. No reference was made therein to "free
white persons''; it merely provided in its Section 7
that:
"SEC. 7. That no person who disbelieve in or who
is opposed to organized government, or who is a
member of or affiliated with any organization
entertaining and teaching such disbelief in or
opposition to organized government, or who
advocates or teaches the duty, necessity, or
propriety of the unlawful assaulting or killing of
85 | C O N S T I 2 _ A r t i c l e I V _ C I T I Z E N S H I P

any officer or officers, either of specific


individuals or of officers generally of the
Government of the United States, or of any other
organized government, because of his or their
official character, or who is a polygamist, shall be
naturalized or be made a citizen of the United
States." (36 Stat. 598)
Incidentally, the 6th paragraph of its Section 4
provided:
"Sixth. When any alien who has declared his
intention to become a citizen of the United States
dies before he is actually naturalized the widow
and minor children of such alien may, by
complying with the other provisions of this Act, be
naturalized without making any declaration of
intention." (36 Stat. 598)
16. By the Act of March 2, 1907, alien women
who acquired American citizenship by marriage
retained said citizenship, if she continued to
reside in the United States and did not renounce
it, or, if she resided outside of the United States
by registering with the U.S. Consul of her place of
residence. (CHAP. 2534, Sec. 4, 34 Stat. 1229.)
17. Since United States legislation treats
naturalization and citizenship per se separately,
Section 1994 of the Revised Statutes remained
untouched. In the Act of February 24, 1911 it was
provided:
"Be it enacted by the Senate and House of
Representatives of the United States of America
in Congress assembled, That when any alien, who
has declared his intention to become a citizen of
the United States, becomes insane before he is
actually naturalized, and his wife shall thereafter
make a homestead entry under the land laws of
the United States, she and their minor children
may, by complying with the other provisions of
the naturalization laws be naturalized without
making any declaration of intention." (36 Stat.
929.)
18. The Act of August 11, 1916 merely validated
entries filed in certain countries. (CHAP. 316, 39
Stat. 926.)
19. In the Act of May 9, 1918, the U.S. Congress
amended the naturalization laws to make
possible
the
admission
of
Filipino
navy
servicemen, and understandably, because of the
war then, it provided:
"Seventh. Any native-born Filipino of the age of
twenty-one years and upward who has declared
his intention to become a citizen of the United
States and who has enlisted or may hereafter
enlist in the United States Navy or Marine Corps
or the Naval Auxiliary Service, and who, after
service of not less than three years, may be
honorably discharged therefrom, or who may
receive
an
ordinary
discharge
with
recommendation for reenlistment; or any alien, or
any Porto Rican not a citizen of the United States,
. . . ." (40 Stat. 542.)

20. On September 22, 1922, "An Act Relative to


the Naturalization and citizenship of married
women" was appareled repeating Section 1994 of
the Revised Statutes and otherwise adopting a
different attitude as regards the citizenship and
naturalization of married women thus:

status she; shall have resided within the United


States. After her naturalization she shall have the
same citizenship status as if her marriage had
taken place after the passage of this Act.

"Be it enacted by the Senate cleaned House of


Representatives of the United States of America
in Congress assembled, That the right of any
woman to become a naturalized citizen of the
United States shall not be denied or abridged
because of her sex or because she is a married
woman.

"Sec. 5. That no woman whose husband is not


eligible to citizenship shall be naturalized during
the continuance of the marital status.

"Sec. 2. That any woman who marries a citizen of


the United States after the passage of this Act, or
any woman whose husband is naturalized after
the passage of this Act, shall not become a
citizen of the United States by reason of such
marriage or naturalization; but, if eligible to
citizenship, she may be naturalized upon full and
complete compliance with all the requirements of
the naturalization laws, with the following
exceptions:

"Sec. 6. That section 1994 of the Revised Statutes


and section 4 of the Expatriation Act of 1907 are
repealed. Such repeal shall not terminate
citizenship acquired or retained under either of
such sections nor restore citizenship lost under
section 4 of the Expatriation Act of 1907.

(a) No declaration of intention shall be required;

"Sec. 7. That section 3 of the Expatriation Act of


1901 is repealed. Such repeal shall not restore
citizenship lost under such section nor terminate
citizenship resumed under such section. A woman
who has resumed under such section citizenship
lost by marriage shall, upon the passage of this
Act, have for all purposes the same citizenship
status as immediately preceding her marriage."
(Chap. 411, 42 Stat. 10211022.)

(b) In lieu of the five-year period of residence


within the United States and the one-year period
of residence within the State or Territory where
the naturalization court is held, she shall have
resided continuously in the United States Hawaii,
Alaska, or Porto Rico for at least one year
immediately preceding the filing of the petition.

21. When "The Code of the Laws of to United


States of America of a General and Permanent
Character in Force on December 7, 1925" was
approved, the provisions, corresponding to the
disqualifications for naturalization and the
citizenship
and
naturalization
of
women
embodied therein were:

"Sec. 3. That a woman citizen of the United


States shall not cease to be a citizen of the
United States by reason of her marriage after the
passage of this Act, unless she makes a formal
renunciation of her citizenship before a court
having jurisdiction over naturalization of aliens;
Provided, That any woman citizen who marries an
alien ineligible to citizenship shall cease to be a
citizen of the United States. If at the termination
of the marital status she is a citizen of the United
States she shall retain her citizenship regardless
of her residence. If during the continuance of the
marital status she resides continuously for two
years in a foreign State of which her husband is a
citizen or subject, or for five years continuously
outside the United States, she shall thereafter be
subject to the same presumption as is a
naturalized citizen of the United States under the
second paragraph of section 2 of the Act entitled
"An Act in reference to the expatriation of citizens
and their protection abroad," approved March 2
1907. Nothing herein shall be construed to repeal
or amend the provisions of Revised Statutes 1999
or of section 2 of the Expatriation Act of 1907
with reference to expatriation.

"367. Naturalization of woman; sex or marriage


not a bar. The right of any woman to become a
naturalized citizen of the United States shall not
be denied or abridged because of her sex or
because she is a married woman. ( Sept. 22,
1922, c.411, 1, 42 Stat. 1021.)

"Sec. 4. That a woman who, before the passage


of this Act, has lost her United States citizenship
by reason of her marriage to an alien eligible for
citizenship, may be naturalized as provided by
section 2 of this Act: Provided, That no certificate
of arrival shall be required to be filed with her
petition if during the continuance of the marital
86 | C O N S T I 2 _ A r t i c l e I V _ C I T I Z E N S H I P

"368. Same; women marrying citizens or persons


becoming naturalized; procedure. Any woman
who marries a citizen of the United States after
September 22, 1922, or any woman whose
husband is naturalized after that date, shall not
become a citizen of the United States by reason
of such marriage or naturalization; but, if eligible
to citizenship, she may be naturalized upon full
and complete compliance with the following
exceptions:
(a) No declaration of intention shall be required;
(b) In lieu of the five-year period of residence
within the United States and the one-year period
of residence within the State or Territory where
the naturalization court is held, she shall have
resided continuously in the United States, Hawaii,
Alaska, or Porto Rico for at least one year
immediately preceding the filing of the petition.
(Sept. 22, 1922, c. 411, 2, 42 Stat. 1022.)
"369. Same; women who have lost citizenship by
Marrying aliens eligible to citizenship; procedure.
A woman, who, before September 22, 1922,
has lost her United States citizenship by reason

for her marriage to an alien eligible for


citizenship, may be naturalized as provided in the
preceding section. No certificate of arrival shall
be required to be filed with her petition if during
the continuance of the marital status she shall
have resided within the United Stators. After her
naturalization she shall have the same citizenship
status as if her marriage had taken place after
September 22, 1922. (Swept. 22, 1922, c. 411,
4, 42 Stat. 1022.)
"370. Same; Women married to persons ineligible
to citizenship. No woman whose husband is not
eligible to citizenship shall be naturalized during
the continuance of the marital status. (Swept. 22,
1922, c. 411, 5, 42 Stat. 1022.)
"371. Same, wife of alien declaring becoming
insane before naturalization; minor children.
When any alien, who has declared his intention to
become a citizen of the United States, becomes
insane before he is actually naturalized, and his
wife shall thereafter make a homestead entry
under the land laws of the United States, she and
their minor children may, by complying with the
other provisions of the naturalization laws be
naturalized without making any declaration of
intention. (Feb. 24, 1911, c. 151, 36 Stat. 929.)"
(Chap. 9, 44 Stat. 156, 158.)
which, of course, must be read together with the
provisions
on
inadmissibility
of
Chinese,
anarchists, polygamists, non-English speaking
persons, etc. in Sections 363-365 of the same
Code.
22. The Act of May 26, 1926 extended
naturalization privileges to alien veterans of
World War I, thus:
"Be it enacted by the Senate anal House of
Representatives of the United States of America
in Congress assembled, That (a) as used in this
Act, the term "alien veteran" means an individual,
a member of the military or naval forces of the
United States at any time after April 5, 1917, and
before November 12, 1918, who is now an alien
not ineligible to citizenship; but does not include
(1) any individual at anytime during such period
or thereafter separated from such forces under
other than honorable conditions, (2) any
conscientious objector who performed no military
duty whatever or refused to wear the uniform, or
(3) any alien at any time during such period or
thereafter discharged from the military or naval
forces on account of his alienage.
(b) Terms defined in the Immigration Act of 1924
shall, when used in this Act, have the meaning
assigned to such terms in that Act.
"Sec. 2. An alien veteran shall for the purposes of
the Immigration Act of 1924 be considered as a
non-quota immigrant, but shall be subject to all
the other provisions of that Act and of the
immigration laws, except that
(a) He shall not be subject to the head tax
imposed by section 2 of the Immigration Act of
1917;
87 | C O N S T I 2 _ A r t i c l e I V _ C I T I Z E N S H I P

(b) He shall not be required to pay any fee under


section 2 or section 7 of the Immigration Act of
1924;
(c) If otherwise admissible, he shall not be
excluded under section 3 of the Immigration Act
of 1917, unless excluded under the provisions of
that section relating to
(1) Persons afflicted with a loathsome or
dangerous
contagious
disease,
except
tuberculosis in any form;
(2) Polygamy;
(3) Prostitutes, procurers, or other like immoral
persons;
(4) Contract laborers;
(5) Persons previously deported;
(6) Persons convicted of crime.
"Sec. 3. The unmarried child under eighteen
years of age, the wife, or the husband, of an alien
veteran shall, for the purposes of the Immigration
Act of 1924, be considered as a non quota
immigrant when accompanying or following
within six months to join him, but shall be subject
to all other provisions of that Act and of the
immigration laws.
'Sec. 4. The foregoing provisions of this Act shall
not apply to any alien unless the immigration visa
is issued to him before the expiration of one year
after the enactment of this Act." (Chap. 398, 44
Stat. 654-655.)
23. The Act of June 21, 1930 authorized
repatriation of certain veterans of World War I.
(Chap. 559, 46 Stat. 791.)
24. On March 3, 1931, the Act of September 22,
1922 as amended as follows:
"Sec. 4.(a) Section 3 of the Act entitled "An Act
relative to the naturalization and citizenship of
married women," approved September 22, 1922,
as amended, is amended to read as follows:
'Sec. 3.(a) A woman citizen of the United States
shall not cease to be a citizen of the United
States by reason of her marriage after this
section, as amended, takes effect, unless she
makes a formal renunciation of her citizenship
before
a
court
having
jurisdiction
over
naturalization of aliens.
'(b) Any woman who before this section, as
amended takes effect, has lost her United States
citizenship by residence abroad after marriage to
an alien or by marriage to an alien ineligible to
citizenship may, if she has not acquired any other
nationality by affirmative act, be naturalized in
the manner prescribed in section 4 of this Act, as
amended. Any woman who was a citizen of the
United States at birth shall not be denied

naturalization under section 4 on account of her


race.
'(c) No woman shall be entitled to naturalization
under section 4 of this Act, as amended, if her
United States citizenship originated solely by a
reason of her marriage to a citizen of the United
States or by reason of the acquisition of United
States citizenship by her husband.'
"(b) Section 5 of such Act of September 22, 1922,
is repealed." (Chap. 442, 46 Stat. 1511-1512.)
25. The Act of May 25, 1932 contained the
following somewhat pertinent provisions:
"Be it enacted by the Senate and House of
Representatives of the United States of America
in Congress assembled, That
(a) an alien veteran, as defined in section 1 of the
Act of May 26, 1926 (ch. 398, 44 Stat. 654; title 8,
sec. 241, U.S. C. Supp. 1), if residing in the United
States, be entitled at any time within two years
after the enactment of this Act to naturalization
upon the same terms, conditions, and exemptions
which would have been accorded to such alien if
he had petitioned before the armistice of the
World War, except that (1) such alien shall be
required to prove that immediately preceding the
date of this petition he has resided continuously
within the United States for at least two years, in
pursuance of a legal admission for permanent
residence, and that during all such period he has
behaved as a person of good moral character; (2)
if such admission was subsequent to March 3,
1924, such alien shall file with his petition a
certificate of arrival issued by the Commissioner
of Naturalization; (3) final action shall not be had
upon the petition until at least ninety days have
elapsed after filing of such petition; and (4) such
alien shall be required to appear and file his
petition in person, and to take the prescribed
oath of allegiance in open court. Such residence
and good moral character shall be proved either
by the affidavits of two credible witnesses who
are citizens of the United States, or by
depositions by two such witnesses made before a
naturalization examiner, for each place of
residence.
"(b) All petitions for citizenship made outside the
United States in accordance with the seventh
subdivision of section 4 of the Naturalization Act
of June 29, 1906, as amended, upon which
naturalization has not been heretofore granted,
are hereby declared to be invalid for all purposes.
"Sec. 2. (a) The seventh subdivision of section 4
of the Naturalization Act of June 29, 1906, as
amended, is amended by striking out 'the
National Guard or Naval Militia of any State,
Territory, or the District of Columbia, or the State
Militia in Federal Service.'

88 | C O N S T I 2 _ A r t i c l e I V _ C I T I Z E N S H I P

"(b) This section shall not be applied in the case


of any individual whose petition for naturalization
has been filed before the enactment of this Act.
"Sec. 3. The last proviso in the first paragraph of
the seventh subdivision of section 4 of such Act of
June 29, 1906, as amended, is amended by
striking out the period at the end thereof and
inserting in lieu thereof a semicolon and the
following: 'except that this proviso shall not apply
in the case of service on American-owned vessels
by an alien who has been lawfully admitted to the
United States for permanent residence.'
"Sec. 4. Section 32 of such Act of June 29, 1906,
as amended, is amended by adding at the end
thereof the following new subdivisions:
'(c) If the name of any naturalized citizen has,
subsequent to naturalization, been changed by
order of a court of competent jurisdiction, or by
marriage, the citizen may, upon the payment to
the commissioner of a fee of $10, make
application (accompanied by two photographs of
the applicant) for a new certificate of citizenship
in the new name of such citizen. If the
commissioner finds the name of the applicant to
have been changed as claimed he shall issue to
the applicant a new certificate with one of such
photographs of the applicant affixed thereto.
'(d) The Commissioner of Naturalization is
authorized to make and issue, without fee,
certifications of any part of the naturalization
records of any court, or of any certificate of
citizenship, for use in complying with any statute,
State or Federal, or in any judicial proceeding.
Any such certification shall be admitted in
evidence equally with the original from which
such certification was made in any case in which
the original thereof might be admissible as
evidence. No such certification shall be made by
any clerk of court except upon order of the court.'
"Sec. 5. So much of subdivision (a) of section 33
of such Act of June 29, 1906, as amended, as
read 'Upon obtaining a certificate from the
Secretary of Labor showing the date, place, and
manner of arrival in the United States,' is hereby
repealed.
"Sec. 6. Section 4 of the Act entitled 'An Act to
supplement the naturalization laws, and for other
purposes,' approved March 2, 1929, is amended
by striking out the period at the end thereof and
inserting in lieu thereof a semicolon and the
following: 'except that no such certificate shall be
required if the entry was on or before June 29,
1906.'
"Sec. 7. Despite the provisions of subdivision (a)
of section 1 of the Act entitled 'An Act making it a
felony with penalty for certain aliens to enter the
United States of America under certain conditions
in violation of law,' approved March 4, 1929, as
amended, an alien, if otherwise admissible, shall
not be excluded from admission to the United
States under the provisions of such subdivision
after the expiration of one year after the date of

deportation if, prior to his reembarkation at a


place outside of the United States, or prior to his
application in foreign contiguous territory for
admission to the United States, the Secretary of
Labor, in his discretion, shall have granted such
alien permission to reapply for admission.
"Sec. 8. The compilation of the statistics to show
races nationalities, and other information,
authorized and directed to be prepared by the
Commissioner
of
Naturalization,
shall
be
completed and published at the same time, as
near as practicable, as the Publication of the
statistics of the 1930 census except that reports
covering the census of 1910 shall be completed
and submitted not later than January 31, 1933,
and reports covering the census of 1920 not later
than December 31, 1938. Such statistics shall
show the records of registry made under the
provisions of the Act entitled 'An Act to
supplement the naturalization laws, and for other
purposes,' approved March 2, 1929. Payment for
the
equipment
used
in
preparing
such
compilation shall be made from appropriations for
miscellaneous expenses of the Bureau of
Naturalization.
"Sec. 9. The Secretary of the Treasury, upon the
recommendation of the Secretary of Labor, is
authorized to provide quarters without payment
of rent, in the building occupied by the
Naturalization Service in New York City, for a
photographic
studio
operated
by
welfare
organizations without profit and solely for the
benefit of aliens seeking naturalization. Such
studio shall be under the supervision of the
Commissioner of Naturalization.
"Sec. 10. The tenth subdivision of section 4 of the
Act of June 29, 1906 (ch. 3592, 34 Stat. 598), as
amended by the Act of May 9, 1918 (ch. 69, 40,
40 Stat. 545; U.S.C., title 8 sec. 377), is hereby
amended to read as follows:
'Tenth. That any person not an alien enemy, who
resided uninterruptedly within the United States
during the period of five years next preceding July
1, 1920, and was on that date otherwise qualified
to become a citizen of the United States, except
that he had not made a declaration of intention
required by law and who during or prior to that
time, because of misinformation regarding his
citizenship status erroneously exercised the rights
and performed the duties of a citizen of the
United States in good faith, may file the petition
for naturalization prescribed by law without
making the preliminary declaration of intention
required of other aliens, and upon satisfactory
proof to the court that he has so acted may be
admitted as a citizen of the United States upon
complying in all respects with the other
requirements of the naturalization law.' (Chap.
203, 47 Stat. 165-167.)
26. By June 27, 1952, the right of a person to be
naturalized could no longer be denied by reason
of race or sex or because such person was
married, although various disqualifications were
still maintained, such as lack understanding,
89 | C O N S T I 2 _ A r t i c l e I V _ C I T I Z E N S H I P

capacity to read and write English, or of the


principles of the constitution and form of
government of the United States, being opposed
to organized government of law, favoring
totalitarian forms of government, deserters from
the armed forces, etc. (Secs. 1422 to 1426, USCA
8-9, 1953; See also Secs. 1421 et seq., USCA 8,
1970.)
B QUALIFICATIONS
Apart from the above disqualifications, the
statutes
referred
tea
contained
express
requirements as to qualifications as follows:
(1) The Act of 1790 required residence, good
moral character and adherence to the principles
of the United States Constitution.
(2) That of 1795 required a declaration of
intention. residence, adherence to the U.S.
Constitution, good moral character and no title of
nobility.
(3) That of 1798 referred only declaration of
intention and residence.
(4) That of 1802 required residence, renunciation
of allegiance to former government, adherence to
U.S. Constitution, good moral character and
declaration of intention.
(5) That of 1804 was practically I the same as
that of 1802.
(6) So also were those of 1813, 1816 and 1824.
(7) That of 1828 mentioned only residence and
declaration of intention.
(8) Those of 1855, 1870 and 1888 amended the
law in other respects.
(9) That of
provisions:

1906

contained

the

following

"SEC. 4. That an alien may be admitted to


become a citizen of the United States in the
following manner and not otherwise:
"First. He A hall declare on oath before the clerk
of any court authorized by this Act to naturalize
aliens, or his authorized deputy, in the district in
which such alien resides, two years at least prior
to his admission, and after he has reached the
age of eighteen years, that it is bona fide his
intention to become a citizen of the united States,
and to renounce forever all allegiance and fidelity
to any foreign prince, potentate, state, or
sovereignty, and particularly, by name, to the
prince, potentate, state, or sovereignty of which
the alien may be at the time a citizen or subject.
And such declaration shall set forth, the name,
age, occupation, personal description, place of
birth, last foreign residence and allegiance, the
date of arrival, the, name of the vessel, if any, in
which he came to the United states, and the
present place of residence in the United States of
said alien: Provided, however, That no alien who,
in conformity with the law in force at the date of

his declaration, has declared his intention to


become a citizen of the United States shall be
required to renew such declaration.
"Second. Not less than two years nor more than
seven years after he has made such declaration
of intention he shall make and file, in duplicate, a
petition in writing, signed by the applicant in his
own handwriting and duly verified, in which
petition such applicant shall state his full name,
his place of residence (by street and number, if
possible), his occupation, and, if possible, the
date and place of his birth; the place from which
he emigrated, and the date and place of his
arrival in the United States, and, if he entered
through a port, the name of the vessel on which
he arrived; the time when and the place and
name of the court where he declared his intention
to become a citizen of the United States; if he is
married he shall state the name of his wife and, if
possible, the country of her nativity and her place
of residence at the time of filing his petition; and
if he has children, the name, date, and place of
birth and place of residence of each child living at
the time of his petition: Provided, That if he has
filed his declaration before the passage of this Act
he shall not be required to sign the petition in his
own handwriting.
"The petition shall set forth that he is not a
disbeliever
in
or
opposed
to
organized
government, or a member of or affiliated with any
organization or body of persons teaching disbelief
in or opposed to organized government, a
polygamist or believer in the practice of
polygamy, and that it is his intention to become a
citizen of the United States and to renounce
absolutely and forever all allegiance and fidelity
to any foreign prince, potentate, state, or
sovereignty, and particularly by name to the
prince, potentate, state, or sovereignty of which
he at the time of filing of his petition may be a
citizen or subject, and that it is his intention to
reside permanently within the United States, and
whether or not he has been denied admission a, a
citizen of the United States, and, if denied, the
ground or grounds of such denial, the court or
courts in which such decision was rendered, and
that the cause for such denial has since been
cured or removed, and every fact material to his
naturalization ailed required to be proved upon
the final hearing of his application.

"The petition shall also be verified by the


affidavits of at least two credible witnesses, who
are citizens of the United States, and who shall
state in their affidavits that they have personally
known the applicant to be a resident of the
United States for a period of at least five years
continuously, and of the State, Territory, or
district in which the application is made for a
period of at least one year immediately preceding
the date of the filing of his petition, and that they
each have personal knowledge that the petitioner
is a person of good moral character, and that he
is in every way qualified, in their opinion, to be
admitted as a citizen of the United States.
90 | C O N S T I 2 _ A r t i c l e I V _ C I T I Z E N S H I P

"At the time of filing of his petition there shall be


filed with the clerk of the court a certificate from
the Department of Commerce and Labor, if the
petitioner arrives in the United States after the
passage of this Act, stating the date, place and
manner of his arrival in the United States, and the
declaration of intention of such petitioner, which
certificate and declaration shall be attached to
and made a part of said petition.
"Third. He shall, before he is admitted to
citizenship, declare on oath in open court that he
will support the Constitution of the United States,
and he absolutely and entirely renounces and
abjures all allegiance and fidelity to any foreign
prince potentate, state, or sovereignty, and
particularly by name to the prince, potentate,
state, or sovereignty of which he was before a
citizen or subject; that he will support and defend
the Constitution and laws of the United States
against all enemies, foreign and domestic, and
bear true faith and allegiance to the same.
"Fourth. It shall be made to appear to the
satisfaction of the court admitting any alien to
citizenship that immediately preceding the date
of his application he has resided continuously
within the United States five years at least, and
within the State or Territory where such court is at
the time held one year at least, and that during
that time he has behaved as a man of good moral
character, attached to the principles of the
Constitution of the United States, and well
disposed to the good order and happiness of the
same. In addition to the oath of the applicant, the
testimony of at least two witnesses, citizens of
the United States, as to the facts of residence,
moral character, and attachment to the principles
of the Constitution shall be required, and the
name, ,place of residence, and occupation of
each witness shall be set forth in the record.
"Fifth. In case the alien applying to be admitted to
citizenship has borne any hereditary title, or has
been of any of the orders of nobility in the
kingdom or state from which he came, he shall, in
addition to the above requisites make an express
renunciation of his title or order of nobility in the
court to which his application is made, and his
renunciation shall be recorded in the court.
"Sixth. When any alien who has declared his
intention to become a citizen of the United States
dies before he is actually naturalized the widow
and minor children of such alien may, by
complying with the other provisions of this Act, be
naturalized without making any declaration of
intention." (34 stat. 596-98.)
10. Those of 1911 and 1916 contained
amendments as to other matters.
11. That of 1918 provided for different
qualifications for Filipinos, Porto Ricans, etc. for
naturalization in addition to service in the U.S.
Navy or Philippine Constabulary.
12. Those of years after 1922 when Section 1994
was repealed would have no material bearing in
this case.
Amen.

(Tecson v. COMELEC, G.R. No. 161434,


161634, 161824, March 03, 2004)
EN BANC
[G.R. No. 161434. March 3, 2004.]
MARIA JEANETTE C. TECSON and FELIX B.
DESIDERIO, JR., petitioners, vs. The COMMISSION
ON ELECTIONS, RONALD ALLAN KELLY POE (a.k.a.
FERNANDO POE, JR.) and VICTORINO X. FORNIER,
respondents.
[G.R. No. 161634. March 3, 2004.]
ZOILO ANTONIO VELEZ, petitioner, vs. RONALD
ALLAN KELLEY POE, a.k.a. FERNANDO POE, JR.,
respondent.
[G.R. No. 161824. March 3, 2004.]
VICTORINO X. FORNIER, petitioner, vs. HON.
COMMISSION ON ELECTIONS and RONALD ALLAN
KELLEY POE, ALSO KNOWN AS FERNANDO POE
JR., respondents.
DECISION
VITUG, J p:
Citizenship is a treasured right conferred on those
whom the state believes are deserving of the
privilege. It is a "precious heritage, as well as an
inestimable acquisition," 1 that cannot be taken
lightly by anyone either by those who enjoy it
or by those who dispute it.
Before the Court are three consolidated cases, all
of which raise a single question of profound
importance to the nation. The issue of citizenship
is brought up to challenge the qualifications of a
presidential candidate to hold the highest office
of the land. Our people are waiting for the
judgment of the Court with bated breath. Is
Fernando Poe, Jr., the hero of silver screen, and
now one of the main contenders for the
presidency, a natural-born Filipino or is he not?
The moment of introspection takes us face to
face with Spanish and American colonial roots
and reminds us of the rich heritage of civil law
and common law traditions, the fusion resulting
in a hybrid of laws and jurisprudence that could
be no less than distinctly Filipino.
Antecedent Case Settings
On 31 December 2003, respondent Ronald Allan
Kelly Poe, also known as Fernando Poe, Jr.
(hereinafter "FPJ"), filed his certificate of
candidacy for the position of President of the
Republic of the Philippines under the Koalisyon ng
Nagkakaisang Pilipino (KNP) Party, in the
forthcoming national elections. In his certificate
of candidacy, FPJ, representing himself to be a
natural-born citizen of the Philippines, stated his
name to be "Fernando Jr.," or "Ronald Allan" Poe,
his date of birth to be 20 August 1939 and his
place of birth to be Manila.
Victorino X. Fornier, petitioner in G.R. No. 161824,
entitled "Victorino X. Fornier, Petitioner, versus
Hon. Commission on Elections and Ronald Allan
Kelley Poe, also known as Fernando Poe, Jr.,
Respondents," initiated, on 09 January 2004, a
petition docketed SPA No. 04-003 before the
91 | C O N S T I 2 _ A r t i c l e I V _ C I T I Z E N S H I P

Commission on Elections ("COMELEC") to


disqualify FPJ and to deny due course or to cancel
his certificate of candidacy upon the thesis that
FPJ made a material misrepresentation in his
certificate of candidacy by claiming to be a
natural- born Filipino citizen when in truth,
according to Fornier, his parents were foreigners;
his mother, Bessie Kelley Poe, was an American,
and his father, Allan Poe, was a Spanish national,
being the son of Lorenzo Pou, a Spanish subject.
Granting, petitioner asseverated, that Allan F. Poe
was a Filipino citizen, he could not have
transmitted his Filipino citizenship to FPJ, the
latter being an illegitimate child of an alien
mother. Petitioner based the allegation of the
illegitimate birth of respondent on two assertions
first, Allan F. Poe contracted a prior marriage to
a certain Paulita Gomez before his marriage to
Bessie Kelley and, second, even if no such prior
marriage had existed, Allan F. Poe, married Bessie
Kelly only a year after the birth of respondent.
In the hearing before the Third Division of the
COMELEC on 19 January 2004, petitioner, in
support of his claim, presented several
documentary exhibits 1) a copy of the
certificate of birth of FPJ, 2) a certified photocopy
of an affidavit executed in Spanish by Paulita Poe
y Gomez attesting to her having filed a case for
bigamy and concubinage against the father of
respondent, Allan F. Poe, after discovering his
bigamous relationship with Bessie Kelley, 3) an
English translation of the affidavit aforesaid, 4) a
certified photocopy of the certificate of birth of
Allan F. Poe, 5) a certification issued by the
Director of the Records Management and
Archives Office, attesting to the fact that there
was no record in the National Archives that a
Lorenzo Poe or Lorenzo Pou resided or entered
the Philippines before 1907, and 6) a certification
from the Officer-In-Charge of the Archives
Division of the National Archives to the effect that
no available information could be found in the
files of the National Archives regarding the birth
of Allan F. Poe.
On his part, respondent, presented twenty-two
documentary pieces of evidence, the more
significant ones being a) a certification issued
by Estrella M. Domingo of the Archives Division of
the National Archives that there appeared to be
no available information regarding the birth of
Allan F. Poe in the registry of births for San Carlos,
Pangasinan, b) a certification issued by the
Officer-In-Charge of the Archives Division of the
National Archives that no available information
about the marriage of Allan F. Poe and Paulita
Gomez could be found, c) a certificate of birth of
Ronald Allan Poe, d) Original Certificate of Title
No. P-2247 of the Registry of Deeds for the
Province of Pangasinan, in the name of Lorenzo
Pou, e) copies of Tax Declaration No. 20844, No.
20643, No. 23477 and No. 23478 in the name of
Lorenzo Pou, f) a copy of the certificate of death
of Lorenzo Pou, g) a copy of the purported
marriage contract between Fernando Pou and
Bessie Kelley, and h) a certification issued by the
City Civil Registrar of San Carlos City, Pangasinan,
stating that the records of birth in the said office

during the period of from 1900 until May 1946


were totally destroyed during World War II.
On 23 January 2004, the COMELEC dismissed SPA
No. 04-003 for lack of merit. Three days later, or
on 26 January 2004, Fornier filed his motion for
reconsideration. The motion was denied on 06
February 2004 by the COMELEC en banc. On 10
February 2004, petitioner assailed the decision of
the COMELEC before this Court conformably with
Rule 64, in relation to Rule 65, of the Revised
Rules of Civil Procedure. The petition, docketed
G.R. No. 161824, likewise prayed for a temporary
restraining order, a writ of preliminary injunction
or any other resolution that would stay the
finality and/or execution of the COMELEC
resolutions.
The other petitions, later consolidated with G.R.
No. 161824, would include G.R. No. 161434,
entitled "Maria Jeanette C. Tecson, and Felix B.
Desiderio, Jr., vs. The Commission on Elections,
Ronald Allan Kelley Poe (a.k.a. 'Fernando Poe, Jr.'),
and Victorino X. Fornier," and the other, docketed
G.R. No. 161634, entitled "Zoilo Antonio G. Velez,
vs. Ronald Allan Kelley Poe, a.k.a. Fernando Poe,
Jr.," both challenging the jurisdiction of the
COMELEC and asserting that, under Article VII,
Section 4, paragraph 7, of the 1987 Constitution,
only the Supreme Court had original and
exclusive jurisdiction to resolve the basic issue on
the case.
Jurisdiction of the Court
In G.R. No. 161824
In seeking the disqualification of the candidacy of
FPJ and to have the COMELEC deny due course or
to cancel FPJ's certificate of candidacy for alleged
misrepresentation of a material fact (i.e., that FPJ
was a natural-born citizen) before the COMELEC,
petitioner Fornier invoked Section 78 of the
Omnibus Election Code
"Section 78.Petition to deny due course or to
cancel a certificate of candidacy. A verified
petition seeking to deny due course or to cancel a
certificate of candidacy may be filed by any
person exclusively on the ground that any
material representation contained therein as
required under Section 74 hereof is false"
in consonance with the general powers of
COMELEC expressed in Section 52 of the Omnibus
Election Code
"Section 52.Powers and functions of the
Commission on Elections. In addition to the
powers and functions conferred upon it by the
Constitution, the Commission shall have exclusive
charge of the enforcement and administration of
all laws relative to the conduct of elections for the
purpose of ensuring free, orderly and honest
elections"
and in relation to Article 69 of the Omnibus
Election Code which would authorize "any
interested party" to file a verified petition to deny
or cancel the certificate of candidacy of any
nuisance candidate.
92 | C O N S T I 2 _ A r t i c l e I V _ C I T I Z E N S H I P

Decisions of the COMELEC on disqualification


cases may be reviewed by the Supreme Court per
Rule 64 2 in an action for certiorari under Rule 65
3 of the Revised Rules of Civil Procedure. Section
7, Article IX, of the 1987 Constitution also reads

"Each Commission shall decide by a majority vote


of all its Members any case or matter brought
before it within sixty days from the date of its
submission for decision or resolution. A case or
matter is deemed submitted for decision or
resolution upon the filing of the last pleading,
brief, or memorandum, required by the rules of
the Commission or by the Commission itself.
Unless otherwise provided by this Constitution or
by law, any decision, order, or ruling of each
Commission may be brought to the Supreme
Court on certiorari by the aggrieved party within
thirty days from receipt of a copy thereof."
Additionally, Section 1, Article VIII, of the same
Constitution provides that judicial power is vested
in one Supreme Court and in such lower courts as
may be established by law which power "includes
the duty of the courts of justice to settle actual
controversies involving rights which are legally
demandable and enforceable, and to determine
whether or not there has been a grave abuse of
discretion amounting to lack or excess of
jurisdiction on the part of any branch or
instrumentality of the Government."
It is sufficiently clear that the petition brought up
in G.R. No. 161824 was aptly elevated to, and
could well be taken cognizance of, by this Court.
A contrary view could be a gross denial to our
people of their fundamental right to be fully
informed, and to make a proper choice, on who
could or should be elected to occupy the highest
government post in the land.

In G.R. No. 161434 and G.R. No. 161634


Petitioners Tecson, et al., in G.R. No. 161434, and
Velez, in G.R. No. 161634, invoke the provisions
of Article VII, Section 4, paragraph 7, of the 1987
Constitution in assailing the jurisdiction of the
COMELEC when it took cognizance of SPA No. 04003 and in urging the Supreme Court to instead
take on the petitions they directly instituted
before it. The Constitutional provision cited reads:
"The Supreme Court, sitting en banc, shall be the
sole judge of all contests relating to the election,
returns, and qualifications of the President or
Vice-President, and may promulgate its rules for
the purpose."
The provision is an innovation of the 1987
Constitution. The omission in the 1935 and the
1973 Constitution to designate any tribunal to be
the sole judge of presidential and vicepresidential contests, has constrained this Court
to declare, in Lopez vs. Roxas, 4 as "not (being)
justiciable" controversies or disputes involving

contests
on
the
elections,
returns
and
qualifications of the President or Vice-President.
The constitutional lapse prompted Congress, on
21 June 1957, to enact Republic Act No. 1793, "An
Act Constituting an Independent Presidential
Electoral Tribunal to Try, Hear and Decide Protests
Contesting the Election of the President-Elect and
the Vice-President-Elect of the Philippines and
Providing for the Manner of Hearing the Same."
Republic Act 1793 designated the Chief Justice
and the Associate Justices of the Supreme Court
to be the members of the tribunal. Although the
subsequent adoption of the parliamentary form of
government under the 1973 Constitution might
have implicitly affected Republic Act No. 1793,
the statutory set-up, nonetheless, would now be
deemed revived under the present Section 4,
paragraph 7, of the 1987 Constitution.

It is fair to conclude that the jurisdiction of the


Supreme Court, defined by Section 4, paragraph
7, of the 1987 Constitution, would not include
cases directly brought before it, questioning the
qualifications of a candidate for the presidency or
vice-presidency before the elections are held.

Ordinary usage would characterize a "contest" in


reference to a post-election scenario. Election
contests consist of either an election protest or a
quo warranto which, although two distinct
remedies, would have one objective in view, i.e.,
to dislodge the winning candidate from office. A
perusal of the phraseology in Rule 12, Rule 13,
and Rule 14 of the "Rules of the Presidential
Electoral Tribunal," promulgated by the Supreme
Court en banc on 18 April 1992, would support
this premise

Perhaps, the earliest understanding of citizenship


was that given by Aristotle, who, sometime in 384
to 322 B.C., described the "citizen" to refer to a
man who shared in the administration of justice
and in the holding of an office. 6 Aristotle saw its
significance if only to determine the constituency
of the "State," which he described as being
composed of such persons who would be
adequate in number to achieve a self-sufficient
existence. 7 The concept grew to include one who
would both govern and be governed, for which
qualifications like autonomy, judgment and
loyalty could be expected. Citizenship was seen
to deal with rights and entitlements, on the one
hand, and with concomitant obligations, on the
other. 8 In its ideal setting, a citizen was active in
public life and fundamentally willing to submit his
private interests to the general interest of society.

"Rule 12.Jurisdiction. The Tribunal shall be the


sole judge of all contests relating to the election,
returns, and qualifications of the President or
Vice-President of the Philippines.
"Rule 13.How Initiated. An election contest is
initiated by the filing of an election protest or a
petition for quo warranto against the President or
Vice-President. An election protest shall not
include a petition for quo warranto. A petition for
quo warranto shall not include an election
protest.
"Rule 14.Election Protest. Only the registered
candidate for President or for Vice-President of
the Philippines who received the second or third
highest number of votes may contest the election
of the President or the Vice-President, as the case
may be, by filing a verified petition with the Clerk
of the Presidential Electoral Tribunal within thirty
(30) days after the proclamation of the winner."
The rules categorically speak of the jurisdiction of
the tribunal over contests relating to the election,
returns and qualifications of the "President" or
"Vice-President", of the Philippines, and not of
"candidates" for President or Vice-President. A
quo warranto proceeding is generally defined as
being an action against a person who usurps,
intrudes into, or unlawfully holds or exercises a
public office. 5 In such context, the election
contest can only contemplate a post-election
scenario. In Rule 14, only a registered candidate
who would have received either the second or
third highest number of votes could file an
election protest. This rule again presupposes a
post-election scenario.
93 | C O N S T I 2 _ A r t i c l e I V _ C I T I Z E N S H I P

Accordingly, G.R. No. 161434, entitled "Maria


Jeanette C. Tecson, et al., vs. Commission on
Elections, et al.," and G.R. No. 161634, entitled
"Zoilo Antonio Velez vs. Ronald Allan Kelley Poe
a.k.a. Fernando Poe, Jr." would have to be
dismissed for want of jurisdiction.
The Citizenship Issue
Now, to the basic issue; it should be helpful to
first give a brief historical background on the
concept of citizenship.

The concept of citizenship had undergone


changes over the centuries. In the 18th century,
the concept was limited, by and large, to civil
citizenship,
which
established
the
rights
necessary for individual freedom, such as rights
to property, personal liberty and justice. 9 Its
meaning expanded during the 19th century to
include political citizenship, which encompassed
the right to participate in the exercise of political
power. 10 The 20th century saw the next stage of
the development of social citizenship, which laid
emphasis on the right of the citizen to economic
well-being and social security. 11 The idea of
citizenship has gained expression in the modern
welfare state as it so developed in Western
Europe. An ongoing and final stage of
development, in keeping with the rapidly
shrinking global village, might well be the
internationalization of citizenship. 12
The Local Setting from Spanish Time to the
Present
There was no such term as "Philippine citizens"
during the Spanish regime but "subjects of Spain"
or "Spanish subjects." 13 In church records, the
natives were called 'indios', denoting a low regard
for the inhabitants of the archipelago. Spanish
laws on citizenship became highly codified during
the 19th century but their sheer number made it
difficult to point to one comprehensive law. Not
all of these citizenship laws of Spain however,

were made to apply to the Philippine Islands


except for those explicitly extended by Royal
Decrees. 14
Spanish laws on citizenship were traced back to
the Novisima Recopilacion, promulgated in Spain
on 16 July 1805 but as to whether the law was
extended to the Philippines remained to be the
subject of differing views among experts; 15
however, three royal decrees were undisputably
made applicable to Spaniards in the Philippines
the Order de la Regencia of 14 August 1841, 16
the Royal Decree of 23 August 1868 specifically
defining the political status of children born in the
Philippine Islands, 17 and finally, the Ley
Extranjera de Ultramar of 04 July 1870, which was
expressly made applicable to the Philippines by
the Royal Decree of 13 July 1870. 18
The Spanish Constitution of 1876 was never
extended to the Philippine Islands because of the
express mandate of its Article 89, according to
which the provisions of the Ultramar among
which this country was included, would be
governed by special laws. 19
It was only the Civil Code of Spain, made effective
in this jurisdiction on 18 December 1889, which
came out with the first categorical enumeration
of who were Spanish citizens.
"(a)Persons born in Spanish territory,
"(b)Children of a Spanish father or mother, even if
they were born outside of Spain,
"(c)Foreigners who have obtained naturalization
papers,
"(d)Those who, without such papers, may have
become domiciled inhabitants of any town of the
Monarchy." 20
The year 1898 was another turning point in
Philippine history. Already in the state of decline
as a superpower, Spain was forced to so cede her
sole colony in the East to an upcoming world
power, the United States. An accepted principle
of international law dictated that a change in
sovereignty, while resulting in an abrogation of all
political laws then in force, would have no effect
on civil laws, which would remain virtually intact.
The Treaty of Paris was entered into on 10
December 1898 between Spain and the United
States. 21 Under Article IX of the treaty, the civil
rights and political status of the native
inhabitants of the territories ceded to the United
States would be determined by its Congress
"Spanish subjects, natives of the Peninsula,
residing in the territory over which Spain by the
present treaty relinquishes or cedes her
sovereignty may remain in such territory or may
remove therefrom, retaining in either event all
their rights of property, including the right to sell
or dispose of such property or of its proceeds;
and they shall also have the right to carry on
their industry, commerce, and professions, being
94 | C O N S T I 2 _ A r t i c l e I V _ C I T I Z E N S H I P

subject in respect thereof to such laws as are


applicable to foreigners. In case they remain in
the territory they may preserve their allegiance
to the Crown of Spain by making, before a court
of record, within a year from the date of the
exchange of ratifications of this treaty, a
declaration of their decision to preserve such
allegiance; in default of which declaration they
shall be held to have renounced it and to have
adopted the nationality of the territory in which
they reside.

Thus
"The civil rights and political status of the native
inhabitants of the territories hereby ceded to the
United States shall be determined by the
Congress." 22
Upon the ratification of the treaty, and pending
legislation by the United States Congress on the
subject, the native inhabitants of the Philippines
ceased to be Spanish subjects. Although they did
not become American citizens, they, however,
also ceased to be "aliens" under American laws
and were thus issued passports describing them
to be citizens of the Philippines entitled to the
protection of the United States. LibLex
The term "citizens of the Philippine Islands"
appeared for the first time in the Philippine Bill of
1902, also commonly referred to as the Philippine
Organic Act of 1902, the first comprehensive
legislation of the Congress of the United States
on the Philippines
". . . that all inhabitants of the Philippine Islands
continuing to reside therein, who were Spanish
subjects on the 11th day of April, 1891, and then
resided in said Islands, and their children born
subsequent thereto, shall be deemed and held to
be citizens of the Philippine Islands and as such
entitled to the protection of the United States,
except such as shall have elected to preserve
their allegiance to the Crown of Spain in
accordance with the provisions of the treaty of
peace between the United States and Spain,
signed at Paris, December tenth eighteen
hundred and ninety eight." 23
Under the organic act, a "citizen of the
Philippines" was one who was an inhabitant of the
Philippines, and a Spanish subject on the 11th
day of April 1899. The term "inhabitant" was
taken to include 1) a native-born inhabitant, 2) an
inhabitant who was a native of Peninsular Spain,
and 3) an inhabitant who obtained Spanish
papers on or before 11 April 1899. 24
Controversy arose on to the status of children
born in the Philippines from 11 April 1899 to 01
July 1902, during which period no citizenship law
was extant in the Philippines. Weight was given to
the view, articulated in jurisprudential writing at
the time, that the common law principle of jus
soli, otherwise also known as the principle of
territoriality, operative in the United States and
England, governed those born in the Philippine

Archipelago within that period. 25 More about this


later.

"Section 1, Article III, 1935 Constitution. The


following are citizens of the Philippines

In 23 March 1912, the Congress of the United


States made the following amendment to the
Philippine Bill of 1902

"(1)Those who are citizens of the Philippine


Islands at the time of the adoption of this
Constitution

"Provided, That the Philippine Legislature is


hereby authorized to provide by law for the
acquisition of Philippine citizenship by those
natives of the Philippine Islands who do not come
within the foregoing provisions, the natives of
other insular possession of the United States, and
such other persons residing in the Philippine
Islands who would become citizens of the United
States, under the laws of the United States, if
residing therein." 26

"(2)Those born in the Philippines Islands of


foreign parents who, before the adoption of this
Constitution, had been elected to public office in
the Philippine Islands.

With the adoption of the Philippine Bill of 1902,


the concept of "Philippine citizens" had for the
first time crystallized. The word "Filipino" was
used by William H. Taft, the first Civil Governor
General in the Philippines when he initially made
mention of it in his slogan, "The Philippines for
the Filipinos." In 1916, the Philippine Autonomy
Act, also known as the Jones Law restated
virtually the provisions of the Philippine Bill of
1902, as so amended by the Act of Congress in
1912
"That all inhabitants of the Philippine Islands who
were Spanish subjects on the eleventh day of
April, eighteen hundred and ninety-nine, and then
resided in said Islands, and their children born
subsequently thereto, shall be deemed and held
to be citizens of the Philippine Islands, except
such as shall have elected to preserve their
allegiance to the Crown of Spain in accordance
with the provisions of the treaty of peace
between the United States and Spain, signed at
Paris December tenth, eighteen hundred and
ninety-eight and except such others as have
since become citizens of some other country;
Provided, That the Philippine Legislature, herein
provided for, is hereby authorized to provide for
the acquisition of Philippine citizenship by those
natives of the Philippine Islands who do not come
within the foregoing provisions, the natives of the
insular possessions of the United States, and such
other persons residing in the Philippine Islands
who are citizens of the United States, or who
could become citizens of the United States under
the laws of the United States, if residing therein."
Under the Jones Law, a native-born inhabitant of
the Philippines was deemed to be a citizen of the
Philippines as of 11 April 1899 if he was 1) a
subject of Spain on 11 April 1899, 2) residing in
the Philippines on said date, and, 3) since that
date, not a citizen of some other country.
While there was, at one brief time, divergent
views on whether or not jus soli was a mode of
acquiring citizenship, the 1935 Constitution
brought to an end to any such link with common
law, by adopting, once and for all, jus sanguinis
or blood relationship as being the basis of Filipino
citizenship

95 | C O N S T I 2 _ A r t i c l e I V _ C I T I Z E N S H I P

"(3)Those whose fathers or mothers are citizens


of the Philippines.
"(4)Those whose mothers are citizens of the
Philippines and upon reaching the age of
majority, elect Philippine citizenship.
"(5)Those who are naturalized in accordance with
law."
Subsection (4), Article III, of the 1935
Constitution, taken together with existing civil law
provisions at the time, which provided that
women would automatically lose their Filipino
citizenship and acquire that of their foreign
husbands, resulted in discriminatory situations
that effectively incapacitated the women from
transmitting their Filipino citizenship to their
legitimate children and required illegitimate
children of Filipino mothers to still elect Filipino
citizenship upon reaching the age of majority.
Seeking to correct this anomaly, as well as fully
cognizant of the newly found status of Filipino
women as equals to men, the framers of the 1973
Constitution crafted the provisions of the new
Constitution on citizenship to reflect such
concerns
"Section 1, Article III, 1973 Constitution The
following are citizens of the Philippines:
"(1)Those who are citizens of the Philippines at
the time of the adoption of this Constitution.
"(2)Those whose fathers or mothers are citizens
of the Philippines.
"(3)Those who elect Philippine citizenship
pursuant to the provisions of the Constitution of
nineteen hundred and thirty-five.
"(4)Those who are naturalized in accordance with
law."
For good measure, Section 2 of the same article
also further provided that
"A female citizen of the Philippines who marries
an alien retains her Philippine citizenship, unless
by her act or omission she is deemed, under the
law to have renounced her citizenship."
The 1987 Constitution generally adopted the
provisions of the 1973 Constitution, except for
subsection (3) thereof that aimed to correct the
irregular situation generated by the questionable
proviso in the 1935 Constitution.

Section 1, Article IV, 1987 Constitution now


provides:
"The following are citizens of the Philippines:
"(1)Those who are citizens of the Philippines at
the time of the adoption of this Constitution.
"(2)Those whose fathers or mothers are citizens
of the Philippines.
"(3)Those born before January 17, 1973 of Filipino
mothers, who elect Philippine citizenship upon
reaching the age of majority; and
"(4)Those who are naturalized in accordance with
law."
The Case Of FPJ
Section 2, Article VII, of the 1987 Constitution
expresses:
"No person may be elected President unless he is
a natural-born citizen of the Philippines, a
registered voter, able to read and write, at least
forty years of age on the day of the election, and
a resident of the Philippines for at least ten years
immediately preceding such election."
The term "natural-born citizens," is defined to
include "those who are citizens of the Philippines
from birth without having to perform any act to
acquire or perfect their Philippine citizenship." 27
The date, month and year of birth of FPJ appeared
to be 20 August 1939 during the regime of the
1935 Constitution. Through its history, four
modes of acquiring citizenship naturalization,
jus soli, res judicata and jus sanguinis 28 had
been in vogue. Only two, i.e., jus soli and jus
sanguinis, could qualify a person to being a
"natural-born" citizen of the Philippines. Jus soli,
per Roa vs. Collector of Customs 29 (1912), did
not last long. With the adoption of the 1935
Constitution and the reversal of Roa in Tan Chong
vs. Secretary of Labor 30 (1947), jus sanguinis or
blood relationship would now become the primary
basis of citizenship by birth.
Documentary evidence adduced by petitioner
would tend to indicate that the earliest
established direct ascendant of FPJ was his
paternal grandfather Lorenzo Pou, married to
Marta Reyes, the father of Allan F. Poe. While the
record of birth of Lorenzo Pou had not been
presented in evidence, his death certificate,
however, identified him to be a Filipino, a resident
of San Carlos, Pangasinan, and 84 years old at
the time of his death on 11 September 1954. The
certificate of birth of the father of FPJ, Allan F.
Poe, showed that he was born on 17 May 1915 to
an Espaol father, Lorenzo Pou, and a mestiza
Espaol mother, Marta Reyes. Introduced by
petitioner was an "uncertified" copy of a
supposed certificate of the alleged marriage of
Allan F. Poe and Paulita Gomez on 05 July 1936.
The marriage certificate of Allan F. Poe and Bessie
Kelley reflected the date of their marriage to be
on 16 September 1940. In the same certificate,
96 | C O N S T I 2 _ A r t i c l e I V _ C I T I Z E N S H I P

Allan F. Poe was stated to be twenty-five years


old, unmarried, and a Filipino citizen, and Bessie
Kelley to be twenty-two years old, unmarried, and
an American citizen. The birth certificate of FPJ,
would disclose that he was born on 20 August
1939 to Allan F. Poe, a Filipino, twenty-four years
old, married to Bessie Kelly, an American citizen,
twenty-one years old and married.

Considering the reservations made by the parties


on the veracity of some of the entries on the birth
certificate of respondent and the marriage
certificate of his parents, the only conclusions
that could be drawn with some degree of
certainty from the documents would be that
1.The parents of FPJ were Allan F. Poe and Bessie
Kelley;
2.FPJ was born to them on 20 August 1939;
3.Allan F. Poe and Bessie Kelley were married to
each other on 16 September, 1940;
4.The father of Allan F. Poe was Lorenzo Poe; and
5.At the time of his death on 11 September 1954,
Lorenzo Poe was 84 years old.
Would the above facts be sufficient or insufficient
to establish the fact that FPJ is a natural-born
Filipino citizen? The marriage certificate of Allan F.
Poe and Bessie Kelley, the birth certificate of FPJ,
and the death certificate of Lorenzo Pou are
documents of public record in the custody of a
public officer. The documents have been
submitted in evidence by both contending parties
during the proceedings before the COMELEC.
The birth certificate of FPJ was marked Exhibit "A"
for petitioner and Exhibit "3" for respondent. The
marriage certificate of Allan F. Poe to Bessie
Kelley was submitted as Exhibit "21" for
respondent. The death certificate of Lorenzo Pou
was submitted by respondent as his Exhibit "5."
While the last two documents were submitted in
evidence for respondent, the admissibility
thereof, particularly in reference to the facts
which they purported to show, i.e., the marriage
certificate in relation to the date of marriage of
Allan F. Poe to Bessie Kelley and the death
certificate relative to the death of Lorenzo Pou on
11 September 1954 in San Carlos, Pangasinan,
were all admitted by petitioner, who had utilized
those material statements in his argument. All
three documents were certified true copies of the
originals.
Section 3, Rule 130, Rules of Court states that
"Original
document
must
be
produced;
exceptions. When the subject of inquiry is the
contents of a document, no evidence shall be
admissible other than the original document
itself, except in the following cases:
xxx xxx xxx

"(d)When the original is a public record in the


custody of a public office or is recorded in a
public office."
Being public documents, the death certificate of
Lorenzo Pou, the marriage certificate of Allan F.
Poe and Bessie Kelly, and the birth certificate of
FPJ, constitute prima facie proof of their contents.
Section 44, Rule 130, of the Rules of Court
provides:
"Entries in official records. Entries in official
records made in the performance of his duty by a
public officer of the Philippines, or by a person in
the performance of a duty specially enjoined by
law, are prima facie evidence of the facts therein
stated."
The trustworthiness of public documents and the
value given to the entries made therein could be
grounded on 1) the sense of official duty in the
preparation of the statement made, 2) the
penalty which is usually affixed to a breach of
that duty, 3) the routine and disinterested origin
of most such statements, and 4) the publicity of
record which makes more likely the prior
exposure of such errors as might have occurred.
31
The death certificate of Lorenzo Pou would
indicate that he died on 11 September 1954, at
the age of 84 years, in San Carlos, Pangasinan. It
could thus be assumed that Lorenzo Pou was born
sometime in the year 1870 when the Philippines
was still a colony of Spain. Petitioner would argue
that Lorenzo Pou was not in the Philippines during
the crucial period of from 1898 to 1902
considering that there was no existing record
about such fact in the Records Management and
Archives Office. Petitioner, however, likewise
failed to show that Lorenzo Pou was at any other
place during the same period. In his death
certificate, the residence of Lorenzo Pou was
stated to be San Carlos, Pangasinan. In the
absence of any evidence to the contrary, it
should be sound to conclude, or at least to
presume, that the place of residence of a person
at the time of his death was also his residence
before death. It would be extremely doubtful if
the Records Management and Archives Office
would have had complete records of all residents
of the Philippines from 1898 to 1902.
Proof of Paternity and Filiation Under Civil Law.
Petitioner submits, in any case, that in
establishing filiation (relationship or civil status of
the child to the father [or mother]) or paternity
(relationship or civil status of the father to the
child) of an illegitimate child, FPJ evidently being
an illegitimate son according to petitioner, the
mandatory rules under civil law must be used.
Under the Civil Code of Spain, which was in force
in the Philippines from 08 December 1889 up
until the day prior to 30 August 1950 when the
Civil Code of the Philippines took effect,
acknowledgment was required to establish
filiation or paternity. Acknowledgment was either
judicial (compulsory) or voluntary. Judicial or
97 | C O N S T I 2 _ A r t i c l e I V _ C I T I Z E N S H I P

compulsory acknowledgment was possible only if


done during the lifetime of the putative parent;
voluntary acknowledgment could only be had in a
record of birth, a will, or a public document. 32
Complementary to the new code was Act No.
3753 or the Civil Registry Law expressing in
Section 5 thereof, that
"In case of an illegitimate child, the birth
certificate shall be signed and sworn to jointly by
the parents of the infant or only by the mother if
the father refuses. In the latter case, it shall not
be permissible to state or reveal in the document
the name of the father who refuses to
acknowledge the child, or to give therein any
information by which such father could be
identified."
In order that the birth certificate could then be
utilized to prove voluntary acknowledgment of
filiation or paternity, the certificate was required
to be signed or sworn to by the father. The failure
of such requirement rendered the same useless
as
being
an
authoritative
document
of
recognition. 33 In Mendoza vs. Mella, 34 the
Court ruled
"Since Rodolfo was born in 1935, after the
registry law was enacted, the question here really
is whether or not his birth certificate (Exhibit 1),
which is merely a certified copy of the registry
record, may be relied upon as sufficient proof of
his having been voluntarily recognized. No such
reliance, in our judgment, may be placed upon it.
While it contains the names of both parents,
there is no showing that they signed the original,
let alone swore to its contents as required in
Section 5 of Act No. 3753. For all that might have
happened, it was not even they or either of them
who furnished the data to be entered in the civil
register. Petitioners say that in any event the
birth certificate is in the nature of a public
document wherein voluntary recognition of a
natural child may also be made, according to the
same Article 131. True enough, but in such a
case, there must be a clear statement in the
document that the parent recognizes the child as
his or her own."
In the birth certificate of respondent FPJ,
presented by both parties, nowhere in the
document was the signature of Allan F. Poe found.
There being no will apparently executed, or at
least shown to have been executed, by decedent
Allan F. Poe, the only other proof of voluntary
recognition remained to be "some other public
document." In Pareja vs. Pareja, 35 this Court
defined what could constitute such a document
as proof of voluntary acknowledgment:
"Under the Spanish Civil Code there are two
classes of public documents, those executed by
private individuals which must be authenticated
by notaries, and those issued by competent
public officials by reason of their office. The public
document pointed out in Article 131 as one of the
means by which recognition may be made
belongs to the first class."
Let us leave it at that for the moment.

The
1950
Civil
Code
categorized
the
acknowledgment or recognition of illegitimate
children into voluntary, legal or compulsory.
Voluntary recognition was required to be
expressedly made in a record of birth, a will, a
statement before a court of record or in any
authentic writing. Legal acknowledgment took
place in favor of full blood brothers and sisters of
an illegitimate child who was recognized or
judicially declared as natural. Compulsory
acknowledgment could be demanded generally in
cases when the child had in his favor any
evidence to prove filiation. Unlike an action to
claim legitimacy which would last during the
lifetime of the child, and might pass exceptionally
to the heirs of the child, an action to claim
acknowledgment, however, could only be brought
during the lifetime of the presumed parent.
Amicus Curiae Ruben F. Balane defined, during
the oral argument, "authentic writing," so as to
be an authentic writing for purposes of voluntary
recognition, simply as being a genuine or
indubitable writing of the father. The term would
include
a
public
instrument
(one
duly
acknowledged before a notary public or other
competent official) or a private writing admitted
by the father to be his.
The Family Code has further liberalized the rules;
Article 172, Article 173, and Article 175 provide:
"Art. 172.The filiation of legitimate children is
established by any of the following:
"(1)The record of birth appearing in the civil
register or a final judgment; or
"(2)An admission of legitimate filiation in a public
document or a private handwritten instrument
and signed by the parent concerned.
"In the absence of the foregoing evidence, the
legitimate filiation shall be proved by:
"(1)The open and continuous possession of the
status of a legitimate child; or
"(2)Any other means allowed by the Rules of
Court and special laws.
"Art. 173.The action to claim legitimacy may be
brought by the child during his or her lifetime and
shall be transmitted to the heirs should the child
die during minority or in a state of insanity. In
these cases, the heirs shall have a period of five
years within which to institute the action.

"The action already commenced by the child shall


survive notwithstanding the death of either or
both the parties.
"xxx xxx xxx

98 | C O N S T I 2 _ A r t i c l e I V _ C I T I Z E N S H I P

"Art. 175.Illegitimate children may establish their


illegitimate filiation in the same way and on the
same evidence as legitimate children.
"The action must be brought within the same
period specified in Article 173, except when the
action is based on the second paragraph of
Article 172, in which case the action may be
brought during the lifetime of the alleged parent."
The provisions of the Family Code are
retroactively applied; Article 256 of the code
reads:
"Art. 256.This Code shall have retroactive effect
insofar as it does not prejudice or impair vested
or acquired rights in accordance with the Civil
Code or other laws."
Thus, in Vda. De SyQuia vs. Court of Appeals, 36
the Court has ruled:
"We hold that whether Jose was a voluntarily
recognized natural child should be decided under
Article 278 of the Civil Code of the Philippines.
Article 2260 of that Code provides that 'the
voluntary recognition of a natural child shall take
place according to this Code, even if the child
was born before the effectivity of this body of
laws' or before August 30, 1950. Hence, Article
278 may be given retroactive effect."
It should be apparent that the growing trend to
liberalize the acknowledgment of recognition of
illegitimate children is an attempt to break away
from the traditional idea of keeping well apart
legitimate and non-legitimate relationships within
the family in favor of the greater interest and
welfare of the child. The provisions are intended
to merely govern the private and personal affairs
of the family. There is little, if any, to indicate that
the legitimate or illegitimate civil status of the
individual would also affect his political rights or,
in general, his relationship to the State. While,
indeed, provisions on "citizenship" could be found
in the Civil Code, such provisions must be taken
in the context or private relations, the domain of
civil law; particularly
"Civil Law is that branch of law which has for its
double purpose the organization of the family and
the regulation of property. It has thus [been]
defined as the mass of precepts which determine
and regulate the relations of assistance, authority
and obedience among member of a family, and
those which exist among members of a society
for the protection of private interests." 37
In Yaez de Barnuevo vs. Fuster, 38 the Court has
held:
"In accordance with Article 9 of the Civil Code of
Spain, . . . the laws relating to family rights and
duties, or to the status, condition and legal
capacity of persons, govern Spaniards although
they reside in a foreign country; that, in
consequence, 'all questions of a civil nature, such
as those dealing with the validity or nullity of the
matrimonial bond, the domicile of the husband
and wife, their support, as between them, the

separation of their properties, the rules governing


property, marital authority, division of conjugal
property, the classification of their property, legal
causes for divorce, the extent of the latter, the
authority to decree it, and, in general, the civil
effects of marriage and divorce upon the persons
and properties of the spouses, are questions that
are governed exclusively by the national law of
the husband and wife."
The relevance of "citizenship" or "nationality" to
Civil Law is best exemplified in Article 15 of the
Civil Code, stating that
"Laws relating to family rights and duties, or to
the status, condition and legal capacity of
persons are binding upon citizens of the
Philippines, even though living abroad"
that explains the need to incorporate in the code
a reiteration of the Constitutional provisions on
citizenship. Similarly, citizenship is significant in
civil relationships found in different parts of the
Civil Code, 39 such as on successional rights and
family relations. 40 In adoption, for instance, an
adopted child would be considered the child of his
adoptive parents and accorded the same rights
as their legitimate child but such legal fiction
extended only to define his rights under civil law
41 and not his political status.
Civil law provisions point to an obvious bias
against illegitimacy. This discriminatory attitude
may be traced to the Spanish family and property
laws, which, while defining proprietary and
successional rights of members of the family,
provided distinctions in the rights of legitimate
and illegitimate children. In the monarchial set-up
of old Spain, the distribution and inheritance of
titles and wealth were strictly according to
bloodlines and the concern to keep these
bloodlines uncontaminated by foreign blood was
paramount.
These distinctions between legitimacy and
illegitimacy were codified in the Spanish Civil
Code, and the invidious discrimination survived
when the Spanish Civil Code became the primary
source of our own Civil Code. Such distinction,
however, remains and should remain only in the
sphere of civil law and not unduly impede or
impinge on the domain of political law.
The proof of filiation or paternity for purposes of
determining his citizenship status should thus be
deemed independent from and not inextricably
tied up with that prescribed for civil law purposes.
The Civil Code or Family Code provisions on proof
of filiation or paternity, although good law, do not
have preclusive effects on matters alien to
personal and family relations. The ordinary rules
on evidence could well and should govern. For
instance, the matter about pedigree is not
necessarily precluded from being applicable by
the Civil Code or Family Code provisions.
Section 39, Rule 130, of the Rules of Court
provides

99 | C O N S T I 2 _ A r t i c l e I V _ C I T I Z E N S H I P

"Act or Declaration about pedigree. The act or


declaration of a person deceased, or unable to
testify, in respect to the pedigree of another
person related to him by birth or marriage, may
be received in evidence where it occurred before
the controversy, and the relationship between the
two persons is shown by evidence other than
such act or declaration. The word 'pedigree'
includes relationship, family genealogy, birth,
marriage, death, the dates when and the places
where these facts occurred, and the names of the
relatives. It embraces also facts of family history
intimately connected with pedigree."
For the above rule to apply, it would be necessary
that (a) the declarant is already dead or unable to
testify, (b) the pedigree of a person must be at
issue, (c) the declarant must be a relative of the
person whose pedigree is in question, (d)
declaration must be made before the controversy
has occurred, and (e) the relationship between
the declarant and the person whose pedigree is in
question must be shown by evidence other than
such act or declaration.
Thus, the duly notarized declaration made by
Ruby Kelley Mangahas, sister of Bessie Kelley Poe
submitted as Exhibit 20 before the COMELEC,
might be accepted to prove the facts of Allan F.
Poe, recognizing his own paternal relationship
with FPJ, i.e., living together with Bessie Kelly and
his children (including respondent FPJ) in one
house, and as one family
"I, Ruby Kelly Mangahas, of legal age and sound
mind, presently residing in Stockton, California,
U.S.A., after being sworn in accordance with law
do hereby declare that:
"1.I am the sister of the late Bessie Kelly Poe.
"2.Bessie Kelley Poe was the wife of Fernando
Poe, Sr.
"3.Fernando and Bessie Poe had a son by the
name of Ronald Allan Poe, more popularly known
in the Philippines as 'Fernando Poe, Jr., or FPJ'.
"4.Ronald Allan Poe 'FPJ' was born on August 20,
1939 at St. Luke's Hospital, Magdalena Street,
Manila.
"xxx xxx xxx
"7.Fernando Poe Sr., and my sister Bessie, met
and became engaged while they were students at
the University of the Philippines in 1936. I was
also introduced to Fernando Poe Sr., by my sister
that same year.
"8.Fernando Poe, Sr., and my sister Bessie had
their first child in 1938.
"9.Fernando Poe, Sr., my sister Bessie and their
first three children, Elizabeth, Ronald, Allan and
Fernando II, and myself lived together with our
mother at our family's house on Dakota St. (now
Jorge Bocobo St.), Malate until the liberation of
Manila in 1945, except for some months between
1943-1944.

"10.Fernando Poe, Sr., and my sister, Bessie, were


blessed with four (4) more children after Ronald
Allan Poe.
"xxx xxx xxx
"18.I am executing this Declaration to attest to
the fact that my nephew, Ronald Allan Poe is a
natural born Filipino, and that he is the legitimate
child of Fernando Poe, Sr.
"Done in City of Stockton, California, U.S.A., this
12th day of January 2004.
Ruby Kelly Mangahas
Declarant
DNA Testing
In case proof of filiation or paternity would be
unlikely to satisfactory establish or would be
difficult to obtain, DNA testing, which examines
genetic codes obtained from body cells of the
illegitimate child and any physical residue of the
long dead parent could be resorted to. A positive
match would clear up filiation or paternity. In
Tijing vs. Court of Appeals, 42 this Court has
acknowledged the strong weight of DNA testing

"Parentage
will
still
be
resolved
using
conventional methods unless we adopt the
modern and scientific ways available. Fortunately,
we have now the facility and expertise in using
DNA test for identification and parentage testing.
The University of the Philippines Natural Science
Research Institute (UP-NSRI) DNA Analysis
Laboratory has now the capability to conduct
DNA typing using short tandem repeat (STR)
analysis. The analysis is based on the fact that
the DNA of a child/person has two (2) copies, one
copy from the mother and the other from the
father. The DNA from the mother, the alleged
father and the child are analyzed to establish
parentage. Of course, being a novel scientific
technique, the use of DNA test as evidence is still
open to challenge. Eventually, as the appropriate
case comes, courts should not hesitate to rule on
the admissibility of DNA evidence. For it was said,
that courts should apply the results of science
when completely obtained in aid of situations
presented, since to reject said result is to deny
progress."

Petitioner's
Argument
For
Jurisprudential
Conclusiveness
Petitioner would have it that even if Allan F. Poe
were a Filipino citizen, he could not have
transmitted his citizenship to respondent FPJ, the
latter being an illegitimate child. According to
petitioner, prior to his marriage to Bessie Kelly,
Allan F. Poe, on July 5, 1936, contracted marriage
with a certain Paulita Gomez, making his
subsequent marriage to Bessie Kelly bigamous
and respondent FPJ an illegitimate child. The
veracity of the supposed certificate of marriage
between Allan F. Poe and Paulita Gomez could be
most doubtful at best. But the documentary
evidence introduced by no less than respondent
100 | C O N S T I 2 _ A r t i c l e I V _ C I T I Z E N S H I P

himself, consisting of a birth certificate of


respondent and a marriage certificate of his
parents showed that FPJ was born on 20 August
1939 to Filipino father and an American mother
who were married to each other a year later, or
on 16 September 1940. Birth to unmarried
parents would make FPJ an illegitimate child.
Petitioner contended that as an illegitimate child,
FPJ so followed the citizenship of his mother,
Bessie Kelly, an American citizen, basing his
stand on the ruling of this Court in Morano vs.
Vivo, 43 citing Chiongbian vs. de Leon 44 and
Serra vs. Republic. 45
On the above score, the disqualification made by
amicus curiae Joaquin G. Bernas, SJ, is most
convincing; he states
"We must analyze these cases and ask what the
lis mota was in each of them. If the procurement
of the Court on jus sanguinis was on the lis mota,
the pronouncement would be a decision
constituting doctrine under the rule of stare
decisis. But if the pronouncement was irrelevant
to the lis mota, the pronouncement would not be
a decision but a mere obiter dictum which did not
establish doctrine. I therefore invite the Court to
look closely into these cases.
"First, Morano vs. Vivio. The case was not about
an illegitimate child of a Filipino father. It was
about a stepson of a Filipino, a stepson who was
the child of a Chinese mother and a Chinese
father. The issue was whether the stepson
followed the naturalization of the stepfather.
Nothing about jus sanguinis there. The stepson
did not have blood of the naturalized stepfather.
"Second, Chiongbian vs. de Leon. This case was
not about the illegitimate son of a Filipino father.
It was about a legitimate son of a father who had
become Filipino by election to public office before
the 1935 Constitution pursuant to Article IV,
Section 1(2) of the 1935 Constitution. No one was
illegitimate here.
"Third, Serra vs. Republic. The case was not about
the illegitimate son of a Filipino father. Serra was
an illegitimate child of a Chinese father and a
Filipino mother. The issue was whether one who
was already a Filipino because of his mother who
still needed to be naturalized. There is nothing
there about invidious jus sanguinis.
"Finally, Paa vs. Chan. 46 This is more
complicated case. The case was about the
citizenship of Quintin Chan who was the son of
Leoncio Chan. Quintin Chan claimed that his
father, Leoncio, was the illegitimate son of a
Chinese father and a Filipino mother. Quintin
therefore argued that he got his citizenship from
Leoncio, his father. But the Supreme Court said
that there was no valid proof that Leoncio was in
fact the son of a Filipina mother. The Court
therefore concluded that Leoncio was not Filipino.
If Leoncio was not Filipino, neither was his son
Quintin. Quintin therefore was not only not a
natural-born Filipino but was not even a Filipino.

"The Court should have stopped there. But


instead it followed with an obiter dictum. The
Court said obiterthat even if Leoncio, Quintin's
father, were Filipino, Quintin would not be Filipino
because Quintin was illegitimate. This statement
about Quintin, based on a contrary to fact
assumption, was absolutely necessary for the
case. . . . It was obiter dictum, pure and simple,
simply repeating the obiter dictum in Morano vs.
Vivo.
"xxx xxx xxx
"Aside from the fact that such a pronouncement
would have no textual foundation in the
Constitution, it would also violate the equal
protection clause of the Constitution not once but
twice. First, it would make an illegitimate
distinction between a legitimate child and an
illegitimate child, and second, it would make an
illegitimate distinction between the illegitimate
child of a Filipino father and the illegitimate child
of a Filipino mother.
"The doctrine on constitutionality allowable
distinctions was established long ago by People
vs. Cayat. 47 I would grant that the distinction
between legitimate children and illegitimate
children rests on real differences. . . . But real
differences alone do not justify invidious
distinction.
Real
differences
may
justify
distinction for one purpose but not for another
purpose.
". . . What is the relevance of legitimacy to
elective public service? What possible state
interest can there be for disqualifying an
illegitimate child from becoming a public officer. It
was not the fault of the child that his parents had
illicit liaison. Why deprive the child of the fullness
of political rights for no fault of his own? To
disqualify an illegitimate child from holding an
important public office is to punish him for the
indiscretion of his parents. There is neither justice
nor rationality in that. And if there is neither
justice nor rationality in the distinction, then the
distinction transgresses the equal protection
clause and must be reprobated."

Constitution, the fundamental law prevailing on


the day, month and year of birth of respondent
FPJ, can never be more explicit than it is.
Providing neither conditions nor distinctions, the
Constitution states that among the citizens of the
Philippines are "those whose fathers are citizens
of the Philippines." There utterly is no cogent
justification to prescribe conditions or distinctions
where there are clearly none provided.
In Sum
(1)The Court, in the exercise of its power of
judicial review, possesses jurisdiction over the
petition in G.R. No. 161824, filed under Rule 64,
in relation to Rule 65, of the Revised Rules of Civil
Procedure. G.R. No. 161824 assails the resolution
of the COMELEC for alleged grave abuse of
discretion in dismissing, for lack of merit, the
petition in SPA No. 04-003 which has prayed for
the disqualification of respondent FPJ from
running for the position of President in the 10th
May 2004 national elections on the contention
that FPJ has committed material representation in
his certificate of candidacy by representing
himself to be a natural-born citizen of the
Philippines.
(2)The Court must dismiss, for lack of jurisdiction
and prematurity, the petitions in G.R. No. 161434
and No. 161634 both having been directly
elevated to this Court in the latter's capacity as
the only tribunal to resolve a presidential and
vice-presidential election contest under the
Constitution. Evidently, the primary jurisdiction of
the Court can directly be invoked only after, not
before, the elections are held.

Where jurisprudence regarded an illegitimate


child as taking after the citizenship of its mother,
it did so for the benefit of the child. It was to
ensure a Filipino nationality for the illegitimate
child of an alien father in line with the assumption
that the mother had custody, would exercise
parental authority and had the duty to support
her illegitimate child. It was to help the child, not
to prejudice or discriminate against him.

(3)In ascertaining, in G.R. No. 161824, whether


grave abuse of discretion has been committed by
the COMELEC, it is necessary to take on the
matter of whether or not respondent FPJ is a
natural-born citizen, which, in turn, depended on
whether or not the father of respondent, Allan F.
Poe, would have himself been a Filipino citizen
and, in the affirmative, whether or not the alleged
illegitimacy of respondent prevents him from
taking after the Filipino citizenship of his putative
father. Any conclusion on the Filipino citizenship
of Lorenzo Pou could only be drawn from the
presumption that having died in 1954 at 84 years
old, Lorenzo would have been born sometime in
the year 1870, when the Philippines was under
Spanish rule, and that San Carlos, Pangasinan, his
place of residence upon his death in 1954, in the
absence of any other evidence, could have well
been his place of residence before death, such
that Lorenzo Pou would have benefited from the
"en masse Filipinization" that the Philippine bill
had effected in 1902. That citizenship (of Lorenzo
Pou), if acquired, would thereby extend to his son,
Allan F. Poe, father of respondent FPJ. The 1935
Constitution, during which regime respondent FPJ
has seen first light, confers citizenship to all
persons whose fathers are Filipino citizens
regardless of whether such children are
legitimate or illegitimate.

The fact of the matter perhaps the most


significant consideration is that the 1935

(4)But while the totality of the evidence may not


establish conclusively that respondent FPJ is a

The other amici curiae, Mr. Justice Vicente


Mendoza (a former member of this Court),
Professor Ruben Balane and Dean Martin
Magallona, at bottom, have expressed similar
views. The thesis of petitioner, unfortunately
hinging solely on pure obiter dicta, should indeed
fail.

101 | C O N S T I 2 _ A r t i c l e I V _ C I T I Z E N S H I P

natural-born citizen of the Philippines, the


evidence on hand still would preponderate in his
favor enough to hold that he cannot be held
guilty
of
having
made
a
material
misrepresentation in his certificate of candidacy
in violation of Section 78, in relation to Section
74, of the Omnibus Election Code. Petitioner has
utterly failed to substantiate his case before the
Court, notwithstanding the ample opportunity
given to the parties to present their position and
evidence, and to prove whether or not there has
been material misrepresentation, which, as so
ruled in Romualdez-Marcos vs. COMELEC, 48
must not only be material, but also deliberate
and willful.

Azcuna, J., concurs in a separate opinion.


Tinga, J., dissents as per his separate opinion.

(Republic v. Lim, G.R. No. 153883, January


13, 2004)
FIRST DIVISION
[G.R. No. 153883. January 13, 2004.]

WHEREFORE, the Court RESOLVES to DISMISS


1.G.R. No. 161434, entitled "Maria Jeanette C.
Tecson and Felix B. Desiderio, Jr., Petitioners,
versus Commission on Elections, Ronald Allan
Kelley Poe (a.k.a. "Fernando Poe, Jr.,) and
Victorino X. Fornier, Respondents," and G.R. No.
161634, entitled "Zoilo Antonio Velez, Petitioner,
versus Ronald Allan Kelley Poe, a.k.a. Fernando
Poe, Jr., Respondent," for want of jurisdiction.
2.G.R. No. 161824, entitled "Victorino X. Fornier,
Petitioner, versus Hon. Commission on Elections
and Ronald Allan Kelley Poe, also known as
Fernando Poe, Jr.," for failure to show grave abuse
of discretion on the part of respondent
Commission on Elections in dismissing the
petition in SPA No. 04-003.
No Costs. ASCTac
SO ORDERED.
Davide, Jr., C.J., see separate opinion.
Puno, J., is on leave but was allowed to vote; see
separate opinion.
Panganiban, J., is on official leave; allowed to vote
but did not send his vote.
Quisumbing, J., joins the dissent of Justices Tinga
and Morales; case should have been REMANDED.
Ynares-Santiago, J., concurs and also with J.
Puno's separate opinion.
Sandoval-Gutierrez,
separate opinion.

J.,

concurs,

please

see

Carpio, J., see separate opinion.

REPUBLIC OF THE PHILIPPINES, petitioner, vs.


CHULE Y. LIM, respondent.
DECISION
YNARES-SANTIAGO, J p:
This petition for review on certiorari under Rule
45 of the Rules of Court stemmed from a petition
for correction of entries under Rule 108 of the
Rules of Court filed by respondent Chule Y. Lim
with the Regional Trial Court of Lanao del Norte,
Branch 4, docketed as Sp. Proc. No. 4933.
In her petition, respondent claimed that she was
born on October 29, 1954 in Buru-an, Iligan City.
Her birth was registered in Kauswagan, Lanao del
Norte but the Municipal Civil Registrar of
Kauswagan transferred her record of birth to
Iligan City. She alleged that both her Kauswagan
and Iligan City records of birth have four
erroneous entries, and prays that they be
corrected.
The trial court then issued an Order, 1 which
reads:
WHEREFORE, finding the petition to be sufficient
in form and substance, let the hearing of this
case be set on December 27, 1999 before this
Court, Hall of Justice, Rosario Heights, Tubod,
Iligan City at 8:30 o'clock in the afternoon at
which date, place and time any interested person
may appear and show cause why the petition
should not be granted.
Let this order be published in a newspaper of
general circulation in the City of Iligan and the
Province of Lanao del Norte once a week for three
(3) consecutive weeks at the expense of the
petitioner.

Corona, J., joins the dissenting opinion of Justice


Morales.

Furnish copies of this order the Office of the


Solicitor General at 134 Amorsolo St., Legaspi
Vill., Makati City and the Office of the Local Civil
Registrar of Iligan City at Quezon Ave., Pala-o,
Iligan City.

Carpio Morales, J., see dissenting opinion.

SO ORDERED.

Callejo, Sr., J ., see concurring opinion.

During the hearing, respondent testified thus:

Austria-Martinez, J., concurs, please see separate


opinion.

102 | C O N S T I 2 _ A r t i c l e I V _ C I T I Z E N S H I P

First, she claims that her surname "Yu" was


misspelled as "Yo". She has been using "Yu" in all
her school records and in her marriage certificate.
2 She presented a clearance from the National
Bureau of Investigation (NBI) 3 to further show
the consistency in her use of the surname "Yu".
Second, she claims that her father's name in her
birth record was written as "Yo Diu To (Co Tian)"
when it should have been "Yu Dio To (Co Tian)."
Third, her nationality was entered as Chinese
when it should have been Filipino considering that
her father and mother never got married. Only
her deceased father was Chinese, while her
mother is Filipina. She claims that her being a
registered voter attests to the fact that she is a
Filipino citizen.
Finally, it was erroneously indicated in her birth
certificate that she was a legitimate child when
she should have been described as illegitimate
considering that her parents were never married.
Placida Anto, respondent's mother, testified that
she is a Filipino citizen as her parents were both
Filipinos from Camiguin. She added that she and
her daughter's father were never married
because the latter had a prior subsisting marriage
contracted in China.
In this connection, respondent presented a
certification attested by officials of the local civil
registries of Iligan City and Kauswagan, Lanao del
Norte that there is no record of marriage between
Placida Anto and Yu Dio To from 1948 to the
present.
The Republic, through the City Prosecutor of
Iligan City, did not present any evidence although
it actively participated in the proceedings by
attending
hearings
and
cross-examining
respondent and her witnesses.
On February 22, 2000, the trial court granted
respondent's petition and rendered judgment as
follows:
WHEREFORE, the foregoing premises considered,
to set the records of the petitioner straight and in
their proper perspective, the petition is granted
and the Civil Registrar of Iligan City is directed to
make the following corrections in the birth
records of the petitioner, to wit:
1. Her family name from "YO" to "YU";
2. Her father's name from "YO DIU TO (CO TIAN)"
to "YU DIOTO (CO TIAN)";
3. Her status from "legitimate" to "illegitimate" by
changing "YES" to "NO" in answer to the question
"LEGITIMATE?"; and,
4. Her citizenship from "Chinese" to "Filipino".
SO ORDERED. 4
103 | C O N S T I 2 _ A r t i c l e I V _ C I T I Z E N S H I P

The Republic of the Philippines appealed the


decision to the Court of Appeals which affirmed
the trial court's decision. 5
Hence, this petition on the following assigned
errors:
I
THE COURT OF APPEALS ERRED IN ORDERING
THE CORRECTION OF THE CITIZENSHIP OF
RESPONDENT CHULE Y. LIM FROM "CHINESE" TO
"FILIPINO" DESPITE THE FACT THAT RESPONDENT
NEVER DEMONSTRATED ANY COMPLIANCE WITH
THE LEGAL REQUIREMENTS FOR ELECTION OF
CITIZENSHIP.
II
THE COURT OF APPEALS ERRED IN ALLOWING
RESPONDENT TO CONTINUE USING HER FATHER'S
SURNAME
DESPITE
ITS
FINDING
THAT
RESPONDENT IS AN ILLEGITIMATE CHILD. 6
To digress, it is just as well that the Republic did
not cite as error respondent's recourse to Rule
108 of the Rules of Court to effect what
indisputably are substantial corrections and
changes in entries in the civil register. To clarify,
Rule 108 of the Revised Rules of Court provides
the procedure for cancellation or correction of
entries in the civil registry. The proceedings under
said rule may either be summary or adversary in
nature. If the correction sought to be made in the
civil register is clerical, then the procedure to be
adopted is summary. If the rectification affects
the civil status, citizenship or nationality of a
party, it is deemed substantial, and the procedure
to be adopted is adversary. This is our ruling in
Republic v. Valencia 7 where we held that even
substantial errors in a civil registry may be
corrected and the true facts established under
Rule 108 provided the parties aggrieved by the
error avail themselves of the appropriate
adversary proceeding. An appropriate adversary
suit or proceeding is one where the trial court has
conducted, proceedings where all relevant facts
have been fully and properly developed, where
opposing counsel have been given opportunity to
demolish the opposite party's case, and where
the evidence has been thoroughly weighed and
considered. 8
As likewise observed by the Court of Appeals, we
take it that the Republic's failure to cite this error
amounts to a recognition that this case properly
falls under Rule 108 of the Revised Rules of Court
considering that the proceeding can be
appropriately classified as adversarial.
Instead, in its first assignment of error, the
Republic avers that respondent did not comply
with the constitutional requirement of electing
Filipino citizenship when she reached the age of
majority. It cites Article IV, Section 1(3) of the
1935 Constitution, which provides that the
citizenship of a legitimate child born of a Filipino
mother and an alien father followed the
citizenship of the father, unless, upon reaching

the age of majority, the child elected Philippine


citizenship. 9 Likewise, the Republic invokes the
provision in Section 1 of Commonwealth Act No.
625, that legitimate children born of Filipino
mothers may elect Philippine citizenship by
expressing such intention "in a statement to be
signed and sworn to by the party concerned
before any officer authorized to administer oaths,
and shall be filed with the nearest civil registry.
The said party shall accompany the aforesaid
statement with the oath of allegiance to the
Constitution and the Government of the
Philippines." 10
Plainly, the above constitutional and statutory
requirements of electing Filipino citizenship apply
only to legitimate children. These do not apply in
the case of respondent who was concededly an
illegitimate child, considering that her Chinese
father and Filipino mother were never married. As
such, she was not required to comply with said
constitutional and statutory requirements to
become a Filipino citizen. By being an illegitimate,
child
of
a
Filipino
mother,
respondent
automatically became a Filipino upon birth.
Stated differently, she is a Filipino since birth
without having to elect Filipino citizenship when
she reached the age of majority. acCITS
In Ching, Re: Application for Admission to the Bar,
11 citing In re Florencio Mallare, 12 we held:
Esteban Mallare, natural child of Ana Mallare, a
Filipina, is therefore himself a Filipino, and no
other act would be necessary to confer on him all
the rights and privileges attached to Philippine
citizenship (US. vs. Ong Tianse, 29 Phil. 332;
Santos Co vs. Government of the Philippine
Islands, 42 Phil. 543; Serra vs. Republic, L-4223,
May 12, 1952; Sy Quimsuan vs. Republic, L-4693,
Feb. 16, 1953; Pitallano vs. Republic, L-5111, June
28, 1954). Neither could any act be taken on the
erroneous belief that he is a non-Filipino divest
him of the citizenship privileges to which he is
rightfully entitled. 13
This notwithstanding, the records show that
respondent elected Filipino citizenship when she
reached the age of majority. She registered as a
voter in Misamis Oriental when she was 18 years
old. 14 The exercise of the right of suffrage and
the participation in election exercises constitute a
positive act of election of Philippine citizenship.
15
In its second assignment of error, the Republic
assails the Court of Appeals' decision in allowing
respondent to use her father's surname despite
its finding that she is illegitimate.
The Republic's submission is misleading. The
Court of Appeals did not allow respondent to use
her father's surname. What it did allow was the
correction of her father's misspelled surname
which she has been using ever since she can
remember. In this regard, respondent does not
need a court pronouncement for her to use her
father's surname.
104 | C O N S T I 2 _ A r t i c l e I V _ C I T I Z E N S H I P

We agree with the Court of Appeals when it held:


Firstly, Petitioner-appellee is now 47 years old. To
bar her at this time from using her father's
surname which she has used for four decades
without any known objection from anybody,
would only sow confusion. Concededly, one of the
reasons allowed for changing one's name or
surname is to avoid confusion.
Secondly, under Sec. 1 of Commonwealth Act No.
142, the law regulating the use of aliases, a
person is allowed to use a name "by which he has
been known since childhood."

Thirdly, the Supreme Court has already


addressed the same issue. In Pabellar v. Rep. of
the Phils., 16 we held:
Section 1 of Commonwealth Act No. 142, which
regulates the use of aliases, allows a person to
use a name "by which he has been known since
childhood" (Lim Hok Albano v. Republic, 104 Phil.
795; People v. Uy Jui Pio, 102 Phil. 679; Republic
v. Taada, infra). Even legitimate children cannot
enjoin the illegitimate children of their father from
using his surname (De Valencia v. Rodriguez, 84
Phil. 222). 17
While judicial authority is required for a change of
name or surname, 18 there is no such
requirement for the continued use of a surname
which a person has already been using since
childhood. 19
The doctrine that disallows such change of name
as would give the false impression of family
relationship remains valid but only to the extent
that the proposed change of name would in great
probability cause prejudice or future mischief to
the family whose surname it is that is involved or
to the community in general. 20 In this case, the
Republic has not shown that the Yu family in
China would probably be prejudiced or be the
object of future mischief. In respondent's case,
the change in the surname that she has been
using for 40 years would even avoid confusion to
her community in general.
WHEREFORE, in view of the foregoing, the instant
petition for review is DENIED. The decision of the
Court of Appeals in CA-G.R. CV No. 68893 dated
May 29, 2002, is AFFIRMED. Accordingly, the Civil
Registrar of Iligan City is DIRECTED to make the
following corrections in the birth record of
respondent Chule Y. Lim, to wit:
1. Her family name from "YO" to "YU";
2. Her father's name from "YO DIU TO (CO TIAN)"
to "YU DIOTO (CO TIAN)";
3. Her status from "legitimate" to "illegitimate" by
changing "YES" to "NO" in answer to the question
"LEGITIMATE?"; and,

4. Her citizenship from "Chinese" to "Filipino".


CAIaHS
SO ORDERED.
Davide, Jr., C.J., Panganiban, Carpio and Azcuna,
JJ., concur.

(Altarejos v. COMELEC, G.R. No. 163256,


November 10, 2004)
EN BANC
[G.R. No. 163256. November 10, 2004.]
CICERON P. ALTAREJOS, petitioner, vs.
COMMISSION ON ELECTIONS, JOSE ALMIE and
VERNON VERSOZA, respondents.
DECISION

Certificate of Repatriation by the Special


Committee on Naturalization, after he filed a
petition for repatriation pursuant to Republic Act
No. 8171. Thus, petitioner claimed that his
Filipino citizenship was already restored, and he
was qualified to run as mayor in the May 10,
2004 elections. Petitioner sought the dismissal of
the petition. cEaTHD
On the date of the hearing, the parties were
required to submit their Memoranda within three
days.
Private
respondents
filed
their
Memorandum, while petitioner did not file one
within the required period. 4 Petitioner, however,
filed a Reply Memorandum 5 subsequently.
Atty. Zacarias C. Zaragoza, Jr., regional election
director for Region V and hearing officer of this
case, recommended that petitioner Altarejos be
disqualified from being a candidate for the
position of mayor of San Jacinto, Masbate in the
May 10, 2004 national and local elections. He
found, thus:

AZCUNA, J p:

xxx xxx xxx

This is a petition for certiorari, with prayer for the


issuance of a temporary restraining order and/or
a writ of prohibitory and mandatory injunction, to
set aside the Resolution promulgated by the
Commission on Elections (COMELEC), First
Division, on March 22, 2004 disqualifying
petitioner Ciceron P. Altarejos from running as
mayor of San Jacinto, Masbate, and another
resolution of the COMELEC en banc promulgated
on May 7, 2004 denying petitioner's motion for
reconsideration.

The provisions of law governing the qualifications


and disqualifications of elective local officials are
found in Sections 39 and 40 of Republic Act No.
7160 otherwise known as The Local Government
Code of 1991, which provide as follows:

The factual antecedents are as follows:


Petitioner Altarejos was a candidate for mayor in
the Municipality of San Jacinto, Masbate in the
May 10, 2004 national and local elections.

SEC. 39. Qualifications. (a) An elective local


official must be a citizen of the Philippines; a
registered voter in the barangay, municipality,
city or province or, in the case of member of the
sangguniang
panlalawigan,
sangguniang
panlungsod, or sangguniang bayan, the district
where he intends to be elected; a resident therein
for at least one (1) year immediately preceding
the day of the election; and able to read and
write Filipino or any other local language or
dialect.

On January 15, 2004, private respondents Jose


Almie Altiche and Vernon Versoza, registered
voters of San Jacinto, Masbate, filed with the
COMELEC, a petition to disqualify and to deny
due course or cancel the certificate of candidacy
of petitioner on the ground that he is not a
Filipino citizen and that he made a false
representation in his certificate of candidacy that
"[he] was not a permanent resident of or
immigrant to a foreign country."

xxx xxx xxx.


(c) Candidates for the position of mayor or vicemayor
of
independent
component
cities,
component cities or municipalities must be at
least twenty-one (21) years of age on election
day.

Private respondents alleged that based on a letter


1 from the Bureau of Immigration dated June 25,
2001, petitioner was a holder of a permanent U.S.
resident visa, an Alien Certificate of Registration
No. E139507 issued on November 3, 1997, and
an Immigration Certificate of Residence No.
320846 issued on November 3, 1997 by the
Bureau of Immigration. 2

xxx xxx xxx.


(d) Those with dual citizenship.

On January 26, 2004, petitioner filed an Answer 3


stating, among others, that he did not commit
false representation in his application for
candidacy as mayor because as early as
December 17, 1997, he was already issued a

Under the terms of the above quoted statutory


provisions, it is required that an elective local
official must be a citizen of the Philippines, and
he must not have a dual citizenship; must not be

105 | C O N S T I 2 _ A r t i c l e I V _ C I T I Z E N S H I P

[SEC. 40. Disqualifications. The following


persons are disqualified from running for any
elective position:]

xxx xxx xxx.


(f) Permanent residents in a foreign country or
those who have acquired the right to reside
abroad and continue to avail of the same right
after the effectivity of this Code; . . .

a permanent resident in a foreign country or must


not have acquired the right to reside abroad.
In the present case, it has been established by
clear and convincing evidence that respondent is
a citizen of the United States of America. Such
fact is proven by his Alien Certificate of
Registration (ACR) No. E139507 issued on 3
November 1997 and Immigration Certificate of
Residence (ICR) with No. 320846 issued on 3
November 1997 by the Alien Registration
Division, Bureau of Immigration and Deportation.
This was further confirmed in a letter dated 25
June 2001 of then Commissioner ANDREA D.
DOMINGO of the Bureau of Immigration and
Deportation.
Although respondent had petitioned for his
repatriation as a Filipino citizen under Republic
Act No. 8171 on 17 December 1997, this did not
restore to respondent his Filipino citizenship,
because Section 2 of the aforecited Republic Act
No. 8171 specifically provides that "repatriation
shall be effected by taking the necessary oath of
allegiance to the Republic of the Philippines and
registration in the proper civil registry and in the
Bureau of Immigration." SDAcaT
It appears from the records of this case that
respondent failed to prove that he has fully
complied with requirements of the above-quoted
Section 2 of Republic Act 8171 to perfect his
repatriation and reacquire his Filipino citizenship.
Respondent has not submitted any document to
prove that he has taken his oath of allegiance to
the Republic of the Philippines and that he has
registered his fact of repatriation in the proper
civil registry and in the Bureau of Immigration. In
fact, in a letter date 25 June 2001, Commissioner
ANDREA DOMINGO stated that RESPONDENT is
still a holder of visa under Section 13 (g) of the
Philippine Immigration Act of 1940 as amended,
with an indefinite authorized stay in the
Philippines, implying that respondent did not
register his supposed Certificate of Repatriation
with the Bureau of Immigration otherwise his
Alien Visa would have already been cancelled.
The rule is that in case of doubt concerning the
grant of citizenship, such doubt should be
resolved in favor of the State and against the
applicant (Cheng vs. Republic, L-16999, 22 June
1965).
xxx xxx xxx
Not having been able to prove that he has fully
reacquired his Filipino citizenship after being
naturalized as a citizen of the United States, it is
clear that respondent is not qualified to be
candidate for the position of Mayor of San Jacinto,
Masbate, in the 10 May 2004 National and Local
Elections, pursuant to the aforequoted Sections
39 and 40 of the Local Government Code of 1991.
As a further consequence of his not being a
Filipino citizen, respondent has also committed
false representation in his certificate of candidacy
by stating therein that he is a natural-born Filipino
citizen, when in fact, he has not yet even
106 | C O N S T I 2 _ A r t i c l e I V _ C I T I Z E N S H I P

perfected the reacquisition of Filipino citizenship.


Such false representation constitutes a material
misrepresentation as it relates to his qualification
as a candidate for public office, which could be a
valid ground for the cancellation of his certificate
of candidacy under Section 78 of the Omnibus
Election Code . . . 6
In its Resolution promulgated on March 22, 2004,
the COMELEC, First Division, adopted the findings
and recommendation of Director Zaragoza. The
dispositive portion of said Resolution stated, thus:
WHEREFORE, premises considered, respondent
CICERON PEREZ ALTAREJOS is hereby disqualified
to run as Mayor of San Jacinto, Masbate.
Accordingly, his certificate of candidacy for the
position of Municipal Mayor of San Jacinto,
Masbate is denied due course and cancelled and
his name deleted from the certified list of
candidates for the May 10, 2004 elections. 7
On March 25, 2004, petitioner filed a motion for
reconsideration and attached the following
documents to prove that he had completed all
the requirements for repatriation which thus
entitled him to run for an elective office, viz:
(1) Oath of Allegiance dated December 17, 1997;
(2) Identification Certificate No. 116543 issued by
the Bureau of Immigration on March 1, 2004;
(3) Certification from the City Civil Registration
Office, Makati City, that the Certificate of
Repatriation and Oath of Allegiance of petitioner
was received by said office and registered, with
the corresponding fee paid, on February 18,
2004;
(4) A letter dated December 17, 1997 from the
Special Committee on Naturalization to the
Bureau on Immigration and Deportation that it
was furnishing said office with the Oath of
Allegiance and Certificate of Repatriation of
petitioner for the cancellation of petitioner's
registration in said office as an alien, and the
issuance
to
him
of
the
corresponding
Identification Card as Filipino citizen;
(5) A letter dated December 17, 1997 from the
Special Committee on Naturalization to the Local
Registrar of San Jacinto, Masbate that it was
sending petitioner's Oath of Allegiance and
Certificate of Repatriation for registration in their
records and for petitioner's reacquisition of his
former Philippine citizenship.
On May 7, 2004, the COMELEC en banc
promulgated a resolution denying the motion for
reconsideration, the dispositive portion of which
reads:
WHEREFORE,
premises
considered,
the
Commission (En Banc) RESOLVED as it hereby
RESOLVES
to
DENY
the
Motion
for
Reconsideration for UTTER LACK OF MERIT and
AFFIRMS the Resolution of the First Division. 8

The Comelec en banc held, thus:


The Comelec Rules of Procedure provides that
insufficiency of evidence to justify the decision is
a ground for a motion for reconsideration (Rule
19, Section 1). The evidence referred to in the
above provision and to be considered in the
Motion for Reconsideration are those which were
submitted during the hearing and attached to the
respective Memoranda of the parties which are
already part of the records of the case. In this
regard, the evidence of the respondent were not
able to overcome the evidence of the petitioners.
IcDHaT

When the entire records of the case was


forwarded to the Commission (First Division) the
respondent's only evidence was his Certificate of
Repatriation dated 17 December 1977 and
marked as Annex 1 of his answer. This piece of
evidence was not enough to controvert the
evidence of the petitioners which consist of the
letter of the then Bureau of Immigration
Commissioner Andrea Domingo dated 25 June
2001 which stated that as of the even date
respondent is a holder of permanent resident visa
(page 15 of the records) and the certification of
Josephine C. Camata dated 28 January 2004
certifying, that the name of the respondent could
not be found in the records of repatriation. (page
42 of the records) The questioned resolution, is
therefore, in order as the evidence submitted by
the respondent were insufficient to rebut the
evidence of the petitioner.
Now, the respondent, in his Motion for
Reconsideration, attempted to introduce to the
record new pieces of evidence, which introduction
is not anymore allowed in a Motion for
Reconsideration. These are the following a) Annex
"2" Oath of Allegiance; b) Annex "3" Bureau
of Immigration Identification Certificate; c) Annex
"4" Certification of the City Civil Registrar of
Makati City; d) Annex "5" Letter addressed to
the Local Civil Registrar of San Jacinto, Masbate
by Aurora P. Cortes of Special Committee on
Naturalization; and e) Annex "6" Letter
addressed to the Bureau of Immigration and
Deportation by Aurora P. Cortes of Special
Committee on Naturalization.
Assuming that the new evidence of the
respondent are admitted, with more reason
should we cancel his certificate of candidacy for
his act of [misrepresenting] himself as a Filipino
citizen when at the time he filed his certificate of
candidacy, he has not yet perfected the process
of repatriation. He failed to comply with the
requirements under Section 2 of [Republic Act
No.] 8171 which provides that repatriation shall
be effected by taking the necessary oath of
allegiance to the Republic of the Philippines and
registration in the proper civil registry and in the
Bureau of Immigration.
The certification was issued by the same Ms.
Josephine C. Camata, City Civil Registrar, dated
107 | C O N S T I 2 _ A r t i c l e I V _ C I T I Z E N S H I P

February 18, 2004. This time, she certifies that


Ciceron Perez Altarejos was registered under
Registry No. 1, Page 19, Book No. 1, Series of
2004 and paid under OR nos. 88325/8833256
dated February 18, 2004. (page 65 of the
records). Obviously, he was able to register in the
proper civil registry only on February 18, 2004.
The respondent was able to register with the
Bureau of Immigration only on March 1, 2004 as
evidenced by the Bureau of Immigration
Identification Certificate attached to the Motion
as Annex "3."
This fact confirms the finding of the Commission
(First Division) that at the time respondent filed
his certificate of candidacy he is yet to complete
the requirement under section two (2) of RA
8171.
As a consequence of not being a Filipino citizen,
he has committed false representation in his
certificate of candidacy. Such false representation
constitutes a material misrepresentation as it
relates to his qualification as a candidate. As such
the certificate of candidacy may be cancelled on
such ground. (Ycain vs. Caneja, 18 Phil. 778) 9
On May 10, 2004, the election day itself,
petitioner filed this petition praying that: (1) The
petition be given due course and a temporary
restraining order and/or writ of preliminary
injunction be issued ex parte restraining the
respondents and all persons acting on their
behalf, from fully implementing the questioned
COMELEC Resolutions promulgated on March 22,
2004 and May 7, 2004; (2) a writ of preliminary
mandatory injunction be issued ordering the
COMELEC and all persons acting on its behalf to
allow petitioner to run as Mayor of San Jacinto,
Masbate in the May 10, 2004 elections, and to
count and canvass the votes cast in his favor and
to proclaim him as the winning mayor of San
Jacinto,
Masbate;
and
(3)
after
proper
proceedings, judgment be rendered declaring null
and void and setting aside the COMELEC
Resolutions promulgated on March 22, 2004 and
May 7, 2004 and other related Orders of the
COMELEC or its representatives which have the
effect of illegally preventing petitioner from
running as Mayor of San Jacinto, Masbate. cCAIaD
In its Comment, 10 the Office of the Solicitor
General stated that, based on the information
relayed to it by the COMELEC, petitioner's name,
as a mayoralty candidate in San Jacinto, Masbate,
was retained in the list of candidates voted upon
by the electorate in the said municipality. Hence,
the cancellation of petitioner's certificate of
candidacy was never implemented. The COMELEC
also informed the Office of the Solicitor General
that petitioner's opponent, Dr. Emilio Aris V.
Espinosa, was already proclaimed duly elected
Mayor of San Jacinto, Masbate.
The Office of the Solicitor General contends that
said supervening event has rendered the instant
petition moot and academic, and it prayed for the
dismissal of the petition.

In his Reply, 11 petitioner opposed the dismissal


of his petition. He claims that the COMELEC
resolutions disqualifying him from running as a
mayoralty candidate adversely affected his
candidacy, since his supporters were made to
believe that his votes would not be counted.
Moreover, he stated that said COMELEC
resolutions cast a doubt on his Philippine
citizenship.
Petitioner points out that he took his Oath of
Allegiance to the Republic of the Philippines on
December 17, 1997. In view thereof, he ran and
was even elected as Mayor of San Jacinto,
Masbate during the 1998 elections. He argues
that if there was delay in the registration of his
Certificate of Repatriation with the Bureau of
Immigration and with the proper civil registry, the
same was brought about by the inaction on the
part of said offices since the records of the
Special Committee on Naturalization show that
his Certificate of Repatriation and Oath of
Allegiance have long been transmitted to said
offices.
Petitioner also asserts that the subsequent
registration of his Certificate of Repatriation with
the Bureau of Immigration and with the Civil
Registry of Makati City prior to the May 10, 2004
elections has the effect of curing the defect, if
any, in the reacquisition of his Filipino citizenship
as his repatriation retroacted to the date of his
application for repatriation as held in Frivaldo v.
Comelec.
The pertinent issues raised are the following: (1)
Is the registration of petitioner's repatriation with
the proper civil registry and with the Bureau of
Immigration
a
prerequisite
in
effecting
repatriation; and (2) whether or not the COMELEC
en banc committed grave abuse of discretion
amounting to excess or lack of jurisdiction in
affirming the Resolution of the COMELEC, First
Division.
As stated by the Office of the Solicitor General,
where the issues have become moot and
academic, there is no justiciable controversy,
thereby rendering the resolution of the same of
no practical use or value. 12 Nonetheless, courts
will decide a question otherwise moot and
academic if it is capable of repetition, yet evading
review. 13
First Issue: Is the registration of petitioner's
repatriation
with the proper civil registry and with the Bureau
of
Immigration
a
prerequisite
in
effecting
repatriation?
The provision of law applicable in this case is
Section 2 of Republic Act No. 8171, 14 thus:
SEC. 2. Repatriation shall be effected by taking
the necessary oath of allegiance to the Republic
of the Philippines and registration in the proper
civil registry and in the Bureau of Immigration.
The Bureau of Immigration shall thereupon cancel
108 | C O N S T I 2 _ A r t i c l e I V _ C I T I Z E N S H I P

the pertinent alien certificate of registration and


issue the certificate of identification as Filipino
citizen to the repatriated citizen.
The law is clear that repatriation is effected "by
taking the oath of allegiance to the Republic of
the Philippines and registration in the proper civil
registry and in the Bureau of Immigration."
Hence, in addition to taking the Oath of
Allegiance to the Republic of the Philippines, the
registration of the Certificate of Repatriation in
the proper civil registry and the Bureau of
Immigration is a prerequisite in effecting the
repatriation of a citizen. aSTcCE
In this case, petitioner took his Oath of Allegiance
on December 17, 1997, but his Certificate of
Repatriation was registered with the Civil Registry
of Makati City only after six years or on February
18, 2004, and with the Bureau of Immigration on
March 1, 2004. Petitioner, therefore, completed
all the requirements of repatriation only after he
filed his certificate of candidacy for a mayoralty
position, but before the elections.
When does the citizenship qualification of a
candidate for an elective office apply?
In Frivaldo v. Commission on Elections, 15 the
Court ruled that the citizenship qualification must
be construed as "applying to the time of
proclamation of the elected official and at the
start of his term." The Court, through Justice
Artemio V. Panganiban, discussed, thus:
Under Sec. 39 of the Local Government Code,
"(a)n elective local official must be:
* a citizen of the Philippines;
* a registered voter in the barangay, municipality,
city, or province . . . where he intends to be
elected;
* a resident therein for at least one (1) year
immediately preceding the day of the election;
* able to read and write Filipino or any other local
language or dialect."
* In addition, "candidates for the position of
governor . . . must be at least twenty-three (23)
years of age on election day."
From the above, it will be noted that the law does
not specify any particular date or time when the
candidate must possess citizenship, unlike that
for residence (which must consist of at least one
year's residency immediately preceding the day
of election) and age (at least twenty three years
of age on election day).

Philippine
citizenship
is
an
indispensable
requirement for holding an elective public office,
and the purpose of the citizenship qualification is
none other than to ensure that no alien, i.e., no
person owing allegiance to another nation, shall

govern our people and our country or a unit of


territory thereof. Now, an official begins to govern
or to discharge his functions only upon his
proclamation and on the day the law mandates
his term of office to begin. Since Frivaldo reassumed his citizenship on June 30, 1995 the
very day the term of office of governor (and other
elective officials) began he was therefore
already qualified to be proclaimed, to hold such
office and to discharge the functions and
responsibilities thereof as of said date. In short, at
that time, he was already qualified to govern his
native Sorsogon. This is the liberal interpretation
that should give spirit, life and meaning to our
law on qualifications consistent with the purpose
for which such law was enacted. . . . Paraphrasing
this Court's ruling in Vasquez v. Giap and Li Seng
Giap & Sons, if the purpose of the citizenship
requirement is to ensure that our people and
country do not end up being governed by aliens,
i.e., persons owing allegiance to another nation,
that aim or purpose would not be thwarted but
instead achieved by construing the citizenship
qualification as applying to the time of
proclamation of the elected official and at the
start of his term. 16 (Emphasis supplied.)
Moreover, in the case of Frivaldo v. Commission
on Elections, the Court ruled that "the
repatriation of Frivaldo RETROACTED to the date
of the filing of his application." In said case, the
repatriation of Frivaldo was by virtue of
Presidential Decree No. 725, which took effect on
June 5, 1975. The Court therein declared that
Presidential Decree No. 725 was a curative
statute, which is retroactive in nature. The
retroactivity of Frivaldo's repatriation to the date
of filing of his application was justified by the
Court, thus:
xxx xxx xxx
. . . The reason for this is simply that if, as in this
case, it was the intent of the legislative authority
that the law should apply to past events i.e.,
situations and transactions existing even before
the law came into being in order to benefit the
greatest number of former Filipinos possible
thereby enabling them to enjoy and exercise the
constitutionally guaranteed right of citizenship,
and such legislative intention is to be given the
fullest effect and expression, then there is all the
more reason to have the law apply in a
retroactive or retrospective manner to situations,
events and transactions subsequent to the
passage of such law. That is, the repatriation
granted to Frivaldo . . . can and should be made
to take effect as of date of his application. As
earlier mentioned, there is nothing in the law that
would bar this or would show a contrary intention
on the part of the legislative authority; and there
is no showing that damage or prejudice to
anyone, or anything unjust or injurious would
result from giving retroactivity to his repatriation.
Neither has Lee shown that there will result the
impairment of any contractual obligation,
disturbance of any vested right or breach of some
constitutional guaranty. IHCacT
109 | C O N S T I 2 _ A r t i c l e I V _ C I T I Z E N S H I P

xxx xxx xxx


Another argument for retroactivity to the date of
filing is that it would prevent prejudice to
applicants. If P.D. 725 were not to be given
retroactive effect, and the Special Committee
decides not to act, i.e., to delay the processing of
applications for any substantial length of time,
then the former Filipinos who may be stateless,
as Frivaldo having already renounced his
American citizenship was, may be prejudiced
for causes outside their control. This should not
be. In case of doubt in the interpretation or
application of laws, it is to be presumed that the
law-making body intended right and justice to
prevail. 17
Republic Act No. 8171 18 has impliedly repealed
Presidential Decree No. 725. They cover the same
subject matter: Providing for the repatriation of
Filipino women who have lost their Philippine
citizenship by marriage to aliens and of naturalborn Filipinos. The Court's ruling in Frivaldo v.
Commission on Elections that repatriation
retroacts to the date of filing of one's application
for repatriation subsists for the same reasons
quoted above.
Accordingly, petitioner's repatriation retroacted to
the date he filed his application in 1997.
Petitioner was, therefore, qualified to run for a
mayoralty position in the government in the May
10, 2004 elections. Apparently, the COMELEC was
cognizant of this fact since it did not implement
the assailed Resolutions disqualifying petitioner
to run as mayor of San Jacinto, Masbate.
Second Issue: Whether or not the COMELEC en
banc
gravely abused its discretion in affirming the
Resolution of the COMELEC, First Division?
The Court cannot fault the COMELEC en banc for
affirming the decision of the COMELEC, First
Division, considering that petitioner failed to
prove before the COMELEC that he had complied
with the requirements of repatriation. Petitioner
submitted the necessary documents proving
compliance with the requirements of repatriation
only during his motion for reconsideration, when
the COMELEC en banc could no longer consider
said evidence. As the COMELEC en banc correctly
stated:
The Comelec Rules of Procedure provides that
insufficiency of evidence to justify the decision is
a ground for a motion for reconsideration (Rule
19, Section 1). The evidence referred to in the
above provision and to be considered in the
Motion for Reconsideration are those which were
submitted during the hearing and attached to the
respective Memoranda of the parties which are
already part of the records of the case. In this
regard, the evidence of the respondent were not
able to overcome the evidence of the petitioners.
19
It is, therefore, incumbent upon candidates for an
elective office, who are repatriated citizens, to be
ready with sufficient evidence of their repatriation

in case their Filipino citizenship is questioned to


prevent a repetition of this case.
WHEREFORE, the petition seeking the nullification
of the Resolution of the COMELEC en banc of May
7, 2004, affirming the Resolution of its First
Division dated March 22, 2004, is hereby DENIED.
No costs.
SO ORDERED.
Davide, Jr., C .J ., Panganiban, Quisumbing,
Ynares-Santiago,
Sandoval-Gutierrez,
Carpio,
Austria-Martinez, Carpio Morales, Callejo, Sr.,
Chico-Nazario and Garcia, JJ ., concur.
Puno and Tinga, JJ ., are on official leave.
Corona, J ., is on leave.
(Reyes v. COMELEC, G.R. No. 207264, June
25, 2013)
EN BANC
[G.R. No. 207264. June 25, 2013.]
REGINA ONGSIAKO REYES, petitioner, vs.
COMMISSION ON ELECTIONS and JOSEPH
SOCORRO B. TAN, respondents.
RESOLUTION
PEREZ, J p:
Before the Court is a Petition for Certiorari with
Prayer for Temporary Restraining Order and/or
Preliminary Injunction and/or Status Quo Ante
Order dated 7 June 2013 filed by petitioner
Regina Ongsiako Reyes, assailing the Resolutions
dated 27 March 2013 and 14 May 2013 issued by
public respondent Commission on Elections
(COMELEC) in SPA No. 13-053. The assailed
Resolutions ordered the cancellation of the
Certificate of Candidacy of petitioner for the
position of Representative of the lone district of
Marinduque.
On 31 October 2012, respondent Joseph Socorro
Tan, a registered voter and resident of the
Municipality of Torrijos, Marinduque, filed before
the COMELEC an Amended Petition to Deny Due
Course or to Cancel the Certificate of Candidacy
(COC) of petitioner on the ground that it
contained
material
misrepresentations,
specifically: (1) that she is single when she is
married to Congressman Herminaldo I. Mandanas
of Batangas; 1 (2) that she is a resident of Brgy.
Lupac, Boac, Marinduque when she is a resident
of Bauan, Batangas which is the residence of her
husband, and at the same time, when she is also
a resident of 135 J.P. Rizal, Brgy. Milagrosa,
Quezon City as admitted in the Directory of
Congressional Spouses of the House of
Representatives; 2 (3) that her date of birth is 3
July 1964 when other documents show that her
birthdate is either 8 July 1959 or 3 July 1960; 3
(4) that she is not a permanent resident of
another country when she is a permanent
110 | C O N S T I 2 _ A r t i c l e I V _ C I T I Z E N S H I P

resident or an immigrant 4 of the United States of


America; 5 and (5) that she is a Filipino citizen
when she is, in fact, an American citizen. 6
TaSEHC
In her Answer, petitioner countered that, while
she is publicly known to be the wife of
Congressman
Herminaldo
I.
Mandanas
(Congressman Mandanas), there is no valid and
binding marriage between them. According to
petitioner,
although
her
marriage
with
Congressman Mandanas was solemnized in a
religious rite, it did not comply with certain formal
requirements prescribed by the Family Code,
rendering it void ab initio. 7 Consequently,
petitioner argues that as she is not duty-bound to
live with Congressman Mandanas, then his
residence cannot be attributed to her. 8 As to her
date of birth, the Certificate of Live Birth issued
by the National Statistics Office shows that it was
on 3 July 1964. 9 Lastly, petitioner notes that the
allegation that she is a permanent resident
and/or a citizen of the United States of America is
not supported by evidence. 10
During the course of the proceedings, on 8
February 2013, respondent filed a "Manifestation
with Motion to Admit Newly Discovered Evidence
and Amended List of Exhibits" 11 consisting of,
among others: (1) a copy of an article published
on the internet on 8 January 2013 entitled
"Seeking and Finding the Truth about Regina O.
Reyes" with an Affidavit of Identification and
Authenticity of Document executed by its author
Eliseo J. Obligacion, which provides a database
record of the Bureau of Immigration indicating
that petitioner is an American citizen and a holder
of a U.S. passport; (2) a Certification of Travel
Records of petitioner, issued by Simeon Sanchez,
Acting Chief, Verification and Certification Unit of
the Bureau of Immigration which indicates that
petitioner used a U.S. Passport in her various
travels abroad.
On 27 March 2013, the COMELEC First Division
issued a Resolution 12 cancelling petitioner's
COC, to wit:
WHEREFORE, in view of the foregoing, the instant
Petition is GRANTED. Accordingly, the Certificate
of Candidacy of respondent REGINA ONGSIAKO
REYES is hereby CANCELLED. EHaCTA
The COMELEC First Division found that, contrary
to the declarations that she made in her COC,
petitioner is not a citizen of the Philippines
because of her failure to comply with the
requirements of Republic Act (R.A.) No. 9225 or
the Citizenship Retention and Re-acquisition Act
of 2003, namely: (1) to take an oath of allegiance
to the Republic of the Philippines; and (2) to make
a personal and sworn renunciation of her
American citizenship before any public officer
authorized to administer an oath. In addition, the
COMELEC First Division ruled that she did not
have the one-year residency requirement under
Section 6, Article VI of the 1987 Constitution. 13
Thus, she is ineligible to run for the position of

Representative
Marinduque.

for

the

lone

district

of

petitioner the opportunity to question and


present controverting evidence, in violation of
Petitioner's right to due process of law.

Not agreeing with the Resolution of the COMELEC


First Division, petitioner filed a Motion for
Reconsideration 14 on 8 April 2013 claiming that
she is a natural-born Filipino citizen and that she
has not lost such status by simply obtaining and
using an American passport. Additionally,
petitioner surmised that the COMELEC First
Division relied on the fact of her marriage to an
American citizen in concluding that she is a
naturalized American citizen. Petitioner averred,
however, that such marriage only resulted into
dual citizenship, thus there is no need for her to
fulfill the twin requirements under R.A. No. 9225.
Still, petitioner attached an Affidavit of
Renunciation of Foreign Citizenship sworn to
before a Notary Public on 24 September 2012. As
to her alleged lack of the one-year residency
requirement prescribed by the Constitution, she
averred that, as she never became a naturalized
citizen, she never lost her domicile of origin,
which is Boac, Marinduque.

33)Whether
or
not
Respondent
Comelec
committed grave abuse of discretion amounting
to lack or excess of jurisdiction when it declared
that Petitioner is not a Filipino citizen and did not
meet the residency requirement for the position
of Member of the House of Representatives.

On 14 May 2013, the COMELEC En Banc,


promulgated a Resolution 15 denying petitioner's
Motion for Reconsideration for lack of merit.
Four days thereafter or on 18 May 2013,
petitioner was proclaimed winner of the 13 May
2013 Elections.
On 5 June 2013, the COMELEC En Banc issued a
Certificate of Finality 16 declaring the 14 May
2013 Resolution of the COMELEC En Banc final
and executory, considering that more than
twenty-one (21) days have elapsed from the date
of promulgation with no order issued by this
Court restraining its execution. 17
On same day, petitioner took her oath of office 18
before Feliciano R. Belmonte Jr., Speaker of the
House of Representatives.
Petitioner has yet to assume office, the term of
which officially starts at noon of 30 June 2013.
In the present Petition for Certiorari with Prayer
for
Temporary
Restraining
Order
and/or
Preliminary Injunction and/or Status Quo Ante
Order, petitioner raises the following issues: 19
31)Whether or not Respondent Comelec is
without jurisdiction over Petitioner who is a duly
proclaimed winner and who has already taken her
oath of office for the position of Member of the
House
of
Representatives
for
the
lone
congressional district of Marinduque. aEIADT
32)Whether
or
not
Respondent
Comelec
committed grave abuse of discretion amounting
to lack or excess of jurisdiction when it took
cognizance of Respondent Tan's alleged "newlydiscovered evidence" without the same having
been testified on and offered and admitted in
evidence which became the basis for its
Resolution of the case without giving the
111 | C O N S T I 2 _ A r t i c l e I V _ C I T I Z E N S H I P

34)Whether or not Respondent Commission on


Elections committed grave abuse of discretion
amounting to lack or excess of jurisdiction when,
by enforcing the provisions of Republic Act No.
9225, it imposed additional qualifications to the
qualifications of a Member of the House of
Representatives as enumerated in Section 6 of
Article VI of the 1987 Constitution of the
Philippines.
The petition must fail.
At the outset, it is observed that the issue of
jurisdiction of respondent COMELEC vis-a-vis that
of House of Representatives Electoral Tribunal
(HRET) appears to be a non-issue. Petitioner is
taking an inconsistent, if not confusing, stance for
while she seeks remedy before this Court, she is
asserting that it is the HRET which has jurisdiction
over her. Thus, she posits that the issue on her
eligibility and qualifications to be a Member of
the House of Representatives is best discussed in
another tribunal of competent jurisdiction. It
appears then that petitioner's recourse to this
Court was made only in an attempt to enjoin the
COMELEC from implementing its final and
executory judgment in SPA No. 13-053.
Nevertheless, we pay due regard to the petition,
and consider each of the issues raised by
petitioner. The need to do so, and at once, was
highlighted during the discussion En Banc on 25
June 2013 where and when it was emphasized
that the term of office of the Members of the
House of Representatives begins on the thirtieth
day of June next following their election.
According to petitioner, the COMELEC was ousted
of its jurisdiction when she was duly proclaimed
20 because pursuant to Section 17, Article VI of
the 1987 Constitution, the HRET has the exclusive
jurisdiction to be the "sole judge of all contests
relating
to
the
election,
returns
and
qualifications" of the Members of the House of
Representatives.
Contrary to petitioner's claim, however, the
COMELEC retains jurisdiction for the following
reasons:
First, the HRET does not acquire jurisdiction over
the issue of petitioner's qualifications, as well as
over the assailed COMELEC Resolutions, unless a
petition is duly filed with said tribunal. Petitioner
has not averred that she has filed such action.
HcTIDC

Second, the jurisdiction of the HRET begins only


after the candidate is considered a Member of the
House of Representatives, as stated in Section
17, Article VI of the 1987 Constitution:
Section 17.The Senate and the House of
Representatives shall each have an Electoral
Tribunal which shall be the sole judge of all
contests relating to the election, returns, and
qualifications of their respective Members. . . .
As held in Marcos v. COMELEC, 21 the HRET does
not have jurisdiction over a candidate who is not
a member of the House of Representatives, to
wit:
As to the House of Representatives Electoral
Tribunal's supposed assumption of jurisdiction
over the issue of petitioner's qualifications after
the May 8, 1995 elections, suffice it to say that
HRET's jurisdiction as the sole judge of all
contests relating to the elections, returns and
qualifications of members of Congress begins
only after a candidate has become a member of
the House of Representatives. Petitioner not
being a member of the House of Representatives,
it is obvious that the HRET at this point has no
jurisdiction over the
question.
(Emphasis
supplied.)
The next inquiry, then, is when is a candidate
considered a Member of the House of
Representatives?
In Vinzons-Chato v. COMELEC, 22 citing Aggabao
v. COMELEC 23 and Guerrero v. COMELEC, 24 the
Court ruled that:
The Court has invariably held that once a winning
candidate has been proclaimed, taken his oath,
and assumed office as a Member of the House of
Representatives, the COMELEC's jurisdiction over
election contests relating to his election, returns,
and qualifications ends, and the HRET's own
jurisdiction begins. (Emphasis supplied.) TaSEHD
This pronouncement was reiterated in the case of
Limkaichong v. COMELEC, 25 wherein the Court,
referring to the jurisdiction of the COMELEC vis-avis the HRET, held that:
The Court has invariably held that once a winning
candidate has been proclaimed, taken his oath,
and assumed office as a Member of the House of
Representatives, the COMELEC's jurisdiction over
election contests relating to his election, returns,
and qualifications ends, and the HRET's own
jurisdiction begins. (Emphasis supplied.)
This was again affirmed in Gonzalez v. COMELEC,
26 to wit:
After
proclamation,
taking
of
oath
and
assumption of office by Gonzalez, jurisdiction
over the matter of his qualifications, as well as
questions regarding the conduct of election and
contested returns were transferred to the HRET
as the constitutional body created to pass upon
the same. (Emphasis supplied.)
112 | C O N S T I 2 _ A r t i c l e I V _ C I T I Z E N S H I P

From the foregoing, it is then clear that to be


considered a Member of the House of
Representatives, there must be a concurrence of
the following requisites: (1) a valid proclamation,
(2) a proper oath, and (3) assumption of office.
Indeed, in some cases, this Court has made the
pronouncement that once a proclamation has
been made, COMELEC's jurisdiction is already lost
and, thus, its jurisdiction over contests relating to
elections, returns, and qualifications ends, and
the HRET's own jurisdiction begins. However, it
must be noted that in these cases, the doctrinal
pronouncement was made in the context of a
proclaimed candidate who had not only taken an
oath of office, but who had also assumed office.
DIETcC
For instance, in the case of Dimaporo v.
COMELEC, 27 the Court upheld the jurisdiction of
the HRET against that of the COMELEC only after
the candidate had been proclaimed, taken his
oath of office before the Speaker of the House,
and assumed the duties of a Congressman on 26
September 2007, or after the start of his term on
30 June 2007, to wit:
On October 8, 2007, private respondent Belmonte
filed his comment in which he brought to Our
attention that on September 26, 2007, even
before the issuance of the status quo ante order
of the Court, he had already been proclaimed by
the PBOC as the duly elected Member of the
House
of
Representatives
of
the
First
Congressional District of Lanao del Norte. On that
very same day, he had taken his oath before
Speaker of the House Jose de Venecia, Jr. and
assumed his duties accordingly.
In light of this development, jurisdiction over this
case has already been transferred to the House of
Representatives
Electoral
Tribunal
(HRET).
(Emphasis supplied.)
Apparently, the earlier cases were decided after
the questioned candidate had already assumed
office, and hence, was already considered a
Member of the House of Representatives, unlike
in the present case.
Here, the petitioner cannot be considered a
Member of the House of Representatives
because, primarily, she has not yet assumed
office. To repeat what has earlier been said, the
term of office of a Member of the House of
Representatives begins only "at noon on the
thirtieth day of June next following their election."
28 Thus, until such time, the COMELEC retains
jurisdiction.
In her attempt to comply with the second
requirement, petitioner attached a purported
Oath of Office taken before Hon. Feliciano
Belmonte Jr. on 5 June 2013. However, this is not
the oath of office which confers membership to
the House of Representatives. ATHCDa

Section 6, Rule II (Membership) of the Rules of the


House of Representatives provides:
Section 6.Oath or Affirmation of Members.
Members shall take their oath or affirmation
either collectively or individually before the
Speaker in open session.
Consequently, before there is a valid or official
taking of the oath it must be made (1) before the
Speaker of the House of Representatives, and (2)
in open session. Here, although she made the
oath before Speaker Belmonte, there is no
indication that it was made during plenary or in
open session and, thus, it remains unclear
whether the required oath of office was indeed
complied with.
More importantly, we cannot disregard a fact
basic in this controversy that before the
proclamation of petitioner on 18 May 2013, the
COMELEC En Banc had already finally disposed of
the issue of petitioner's lack of Filipino citizenship
and residency via its Resolution dated 14 May
2013. After 14 May 2013, there was, before the
COMELEC, no longer any pending case on
petitioner's qualifications to run for the position of
Member of the House of Representative. We will
inexcusably disregard this fact if we accept the
argument of the petitioner that the COMELEC was
ousted of jurisdiction when she was proclaimed,
which was four days after the COMELEC En Banc
decision. The Board of Canvasser which
proclaimed petitioner cannot by such act be
allowed to render nugatory a decision of the
COMELEC En Banc which affirmed a decision of
the COMELEC First Division.
Indeed, the assailed Resolution of the COMELEC
First Division which was promulgated on 27 March
2013, and the assailed Resolution of the
COMELEC En Banc which was promulgated on 14
May 2013, became final and executory on 19 May
2013 based on Section 3, Rule 37 of the
COMELEC Rules of Procedure which provides:
SECIcT
Section 3.Decisions Final after five days.
Decisions in pre-proclamation cases and petitions
to deny due course to or cancel certificates of
candidacy, to declare nuisance candidate or to
disqualify a candidate, and to postpone or
suspend elections shall become final and
executory after the lapse of five (5) days from
their promulgation unless restrained by the
Supreme Court.
To prevent the assailed Resolution dated 14 May
2013 from becoming final and executory,
petitioner should have availed herself of Section
1, Rule 37 29 of the COMELEC Rules of Procedure
or Rule 64 30 of the Rules of Court by filing a
petition before this Court within the 5-day period,
but she failed to do so. She would file the present
last hour petition on 10 June 2013. Hence, on 5
June 2013, respondent COMELEC rightly issued a
Certificate of Finality.

113 | C O N S T I 2 _ A r t i c l e I V _ C I T I Z E N S H I P

As to the issue of whether petitioner failed to


prove her Filipino citizenship, as well as her oneyear residency in Marinduque, suffice it to say
that the COMELEC committed no grave abuse of
discretion in finding her ineligible for the position
of Member of the House of Representatives.
Petitioner alleges that the COMELEC gravely
abused its discretion when it took cognizance of
"newly-discovered evidence" without the same
having been testified on and offered and
admitted in evidence. She assails the admission
of the blog article of Eli Obligacion as hearsay
and the photocopy of the Certification from the
Bureau of Immigration. She likewise contends
that there was a violation of her right to due
process of law because she was not given the
opportunity to question and present controverting
evidence.
Her contentions are incorrect.
It must be emphasized that the COMELEC is not
bound to strictly adhere to the technical rules of
procedure in the presentation of evidence. Under
Section 2 of Rule I, the COMELEC Rules of
Procedure "shall be liberally construed in
order . . . to achieve just, expeditious and
inexpensive determination and disposition of
every action and proceeding brought before the
Commission." In view of the fact that the
proceedings in a petition to deny due course or to
cancel certificate of candidacy are summary in
nature, then the "newly discovered evidence" was
properly admitted by respondent COMELEC.
Furthermore, there was no denial of due process
in the case at bar as petitioner was given every
opportunity to argue her case before the
COMELEC. From 10 October 2012 when Tan's
petition was filed up to 27 March 2013 when the
First Division rendered its resolution, petitioner
had a period of five (5) months to adduce
evidence. Unfortunately, she did not avail herself
of the opportunity given her.
Also, in administrative proceedings, procedural
due process only requires that the party be given
the opportunity or right to be heard. As held in
the case of Sahali v. COMELEC: 31 IDTHcA
The petitioners should be reminded that due
process does not necessarily mean or require a
hearing, but simply an opportunity or right to be
heard. One may be heard, not solely by verbal
presentation but also, and perhaps many times
more creditably and predictable than oral
argument, through pleadings. In administrative
proceedings moreover, technical
rules of
procedure and evidence are not strictly applied;
administrative process cannot be fully equated
with due process in its strict judicial sense.
Indeed, deprivation of due process cannot be
successfully invoked where a party was given the
chance to be heard on his motion for
reconsideration. (Emphasis supplied)

As to the ruling that petitioner is ineligible to run


for office on the ground of citizenship, the
COMELEC First Division, discoursed as follows:
". . . for respondent to reacquire her Filipino
citizenship and become eligible for public office,
the
law
requires that she
must have
accomplished the following acts: (1) take the oath
of allegiance to the Republic of the Philippines
before the Consul-General of the Philippine
Consulate in the USA; and (2) make a personal
and sworn renunciation of her American
citizenship before any public officer authorized to
administer an oath.
In the case at bar, there is no showing that
respondent
complied
with
the
aforesaid
requirements. Early on in the proceeding,
respondent hammered on petitioner's lack of
proof regarding her American citizenship,
contending that it is petitioner's burden to
present a case. She, however, specifically denied
that she has become either a permanent resident
or naturalized citizen of the USA.
Due to petitioner's submission of newlydiscovered evidence thru a Manifestation dated
February 7, 2013, however, establishing the fact
that respondent is a holder of an American
passport which she continues to use until June 30,
2012, petitioner was able to substantiate his
allegations. The burden now shifts to respondent
to present substantial evidence to prove
otherwise. This, the respondent utterly failed to
do, leading to the conclusion inevitable that
respondent falsely misrepresented in her COC
that she is a natural-born Filipino citizen. Unless
and until she can establish that she had availed
of the privileges of RA 9225 by becoming a dual
Filipino-American citizen, and thereafter, made a
valid sworn renunciation of her American
citizenship, she remains to be an American
citizen and is, therefore, ineligible to run for and
hold any elective public office in the Philippines."
32 (Emphasis supplied.) cAHIaE
Let us look into the events that led to this
petition: In moving for the cancellation of
petitioner's COC, respondent submitted records of
the Bureau of Immigration showing that petitioner
is a holder of a US passport, and that her status is
that of a "balikbayan." At this point, the burden of
proof shifted to petitioner, imposing upon her the
duty to prove that she is a natural-born Filipino
citizen and has not lost the same, or that she has
re-acquired such status in accordance with the
provisions of R.A. No. 9225. Aside from the bare
allegation that she is a natural-born citizen,
however, petitioner submitted no proof to support
such contention. Neither did she submit any proof
as to the inapplicability of R.A. No. 9225 to her.
Notably, in her Motion for Reconsideration before
the COMELEC En Banc, petitioner admitted that
she is a holder of a US passport, but she averred
that she is only a dual Filipino-American citizen,
thus the requirements of R.A. No. 9225 do not
apply to her. 33 Still, attached to the said motion
is an Affidavit of Renunciation of Foreign
114 | C O N S T I 2 _ A r t i c l e I V _ C I T I Z E N S H I P

Citizenship dated 24 September 2012. 34


Petitioner explains that she attached said
Affidavit "if only to show her desire and zeal to
serve the people and to comply with rules, even
as a superfluity." 35 We cannot, however,
subscribe to petitioner's explanation. If petitioner
executed said Affidavit "if only to comply with the
rules," then it is an admission that R.A. No. 9225
applies to her. Petitioner cannot claim that she
executed it to address the observations by the
COMELEC as the assailed Resolutions were
promulgated only in 2013, while the Affidavit was
executed in September 2012. HDTcEI
Moreover, in the present petition, petitioner
added a footnote to her oath of office as
Provincial Administrator, to this effect: "This does
not mean that Petitioner did not, prior to her
taking her oath of office as Provincial
Administrator, take her oath of allegiance for
purposes of re-acquisition of natural-born Filipino
status, which she reserves to present in the
proper proceeding. The reference to the taking of
oath of office is in order to make reference to
what is already part of the records and evidence
in the present case and to avoid injecting into the
records evidence on matters of fact that was not
previously
passed
upon
by
Respondent
COMELEC." 36 This statement raises a lot of
questions Did petitioner execute an oath of
allegiance for re-acquisition of natural-born
Filipino status? If she did, why did she not present
it at the earliest opportunity before the
COMELEC? And is this an admission that she has
indeed lost her natural-born Filipino status?
To cover-up her apparent lack of an oath of
allegiance as required by R.A. No. 9225,
petitioner contends that, since she took her oath
of allegiance in connection with her appointment
as Provincial Administrator of Marinduque, she is
deemed to have reacquired her status as a
natural-born Filipino citizen.
This contention is misplaced. For one, this issue is
being presented for the first time before this
Court, as it was never raised before the
COMELEC. For another, said oath of allegiance
cannot be considered compliance with Sec. 3 of
R.A. No. 9225 as certain requirements have to be
met as prescribed by Memorandum Circular No.
AFF-04-01, otherwise known as the Rules
Governing Philippine Citizenship under R.A. No.
9225 and Memorandum Circular No. AFF-05-002
(Revised Rules) and Administrative Order No. 91,
Series of 2004 issued by the Bureau of
Immigration. Thus, petitioner's oath of office as
Provincial Administrator cannot be considered as
the oath of allegiance in compliance with R.A. No.
9225.
These circumstances, taken together, show that a
doubt was clearly cast on petitioner's citizenship.
Petitioner, however, failed to clear such doubt.
As to the issue of residency, proceeding from the
finding that petitioner has lost her natural-born
status, we quote with approval the ruling of the

COMELEC First Division that petitioner cannot be


considered a resident of Marinduque:
"Thus, a Filipino citizen who becomes naturalized
elsewhere effectively abandons his domicile of
origin. Upon re-acquisition of Filipino citizenship
pursuant to RA 9225, he must still show that he
chose to establish his domicile in the Philippines
through positive acts, and the period of his
residency shall be counted from the time he
made it his domicile of choice.
In this case, there is no showing whatsoever that
[petitioner] had already re-acquired her Filipino
citizenship pursuant to RA 9225 so as to conclude
that she has regained her domicile in the
Philippines. There being no proof that [petitioner]
had renounced her American citizenship, it
follows that she has not abandoned her domicile
of choice in the USA.
The only proof presented by [petitioner] to show
that she has met the one-year residency
requirement of the law and never abandoned her
domicile of origin in Boac, Marinduque is her
claim that she served as Provincial Administrator
of the province from January 18, 2011 to July 13,
2011. But such fact alone is not sufficient to
prove her one-year residency. For, [petitioner] has
never regained her domicile in Marinduque as she
remains to be an American citizen. No amount of
her stay in the said locality can substitute the fact
that she has not abandoned her domicile of
choice in the USA." 37 (Emphasis supplied.)
ESAHca
All in all, considering that the petition for denial
and cancellation of the COC is summary in
nature, the COMELEC is given much discretion in
the evaluation and admission of evidence
pursuant to its principal objective of determining
of whether or not the COC should be cancelled.
We held in Mastura v. COMELEC: 38
The rule that factual findings of administrative
bodies will not be disturbed by courts of justice
except when there is absolutely no evidence or
no substantial evidence in support of such
findings should be applied with greater force
when it concerns the COMELEC, as the framers of
the Constitution intended to place the COMELEC
created and explicitly made independent by
the Constitution itself on a level higher than
statutory administrative organs. The COMELEC
has broad powers to ascertain the true results of
the election by means available to it. For the
attainment of that end, it is not strictly bound by
the rules of evidence.
Time and again, We emphasize that the "grave
abuse of discretion" which warrants this Court's
exercise of certiorari jurisdiction has a welldefined meaning. Guidance is found in Beluso v.
Commission on Elections 39 where the Court
held:
. . . A petition for certiorari will prosper only if
grave abuse of discretion is alleged and proved to
exist. "Grave abuse of discretion," under Rule 65,
115 | C O N S T I 2 _ A r t i c l e I V _ C I T I Z E N S H I P

has a specific meaning. It is the arbitrary or


despotic exercise of power due to passion,
prejudice or personal hostility; or the whimsical,
arbitrary, or capricious exercise of power that
amounts to an evasion or refusal to perform a
positive duty enjoined by law or to act at all in
contemplation of law. For an act to be struck
down as having been done with grave abuse of
discretion, the abuse of discretion must be patent
and gross. (Emphasis supplied.) SDIaCT
Here, this Court finds that petitioner failed to
adequately and substantially show that grave
abuse of discretion exists.
Lastly, anent the proposition of petitioner that the
act of the COMELEC in enforcing the provisions of
R.A. No. 9225, insofar as it adds to the
qualifications of Members of the House of
Representatives other than those enumerated in
the Constitution, is unconstitutional, We find the
same meritless.
The COMELEC did not impose additional
qualifications on candidates for the House of
Representatives who have acquired foreign
citizenship. It merely applied the qualifications
prescribed by Section 6, Article VI of the 1987
Constitution that the candidate must be a
natural-born citizen of the Philippines and must
have one-year residency prior to the date of
elections. Such being the case, the COMELEC did
not err when it inquired into the compliance by
petitioner of Sections 3 and 5 of R.A. No. 9225 to
determine if she reacquired her status as a
natural-born Filipino citizen. It simply applied the
constitutional provision and nothing more.
IN VIEW OF THE FOREGOING, the instant petition
is DISMISSED, finding no grave abuse of
discretion on the part of the Commission on
Elections. The 14 May 2013 Resolution of the
COMELEC En Banc affirming the 27 March 2013
Resolution of the COMELEC First Division is
upheld.
SO ORDERED. SHECcD
Sereno, C.J., Leonardo-de Castro, Bersamin, Del
Castillo, Abad and Reyes, JJ., concur.
Carpio, Villarama, Jr. and Leonen, JJ., join the
dissent of J. Brion.
Velasco, Jr. and Mendoza, JJ., took no part.
Brion, J., see dissent.
Peralta, J., is on official leave.
Perlas-Bernabe, J., took no part due to voluntary
inhibition.
(Republic v. Li Ching Chung, G.R. No.
197450, March 20, 2013)
THIRD DIVISION
[G.R. No. 197450. March 20, 2013.]
REPUBLIC OF THE PHILIPPINES, petitioner, vs. LI
CHING CHUNG, a.k.a. BERNABE LUNA LI, a.k.a.
STEPHEN LEE KENG, respondent.
DECISION
MENDOZA, J p:

This Petition for Review on Certiorari 1 under Rule


45 of the 1997 Rules of Civil Procedure filed by
the Republic of the Philippines, represented by
the Office of the Solicitor General (OSG),
challenges the June 30, 2011 Decision 2 of the
Court of Appeals (CA) in CA-G.R. CV No. 93374,
which affirmed the June 3, 2009 Decision 3 of the
Regional Trial Court, Branch 49, Manila (RTC),
granting the petition for naturalization of
respondent Li Ching Chung (respondent). ATaDHC
On August 22, 2007, respondent, otherwise
known as Bernabe Luna Li or Stephen Lee Keng, a
Chinese national, filed his Declaration of Intention
to Become a Citizen of the Philippines before the
OSG. 4
On March 12, 2008 or almost seven months after
filing his declaration of intention, respondent filed
his Petition for Naturalization before the RTC,
docketed as Civil Case No. 08-118905. 5 On April
5, 2008, respondent filed his Amended Petition
for Naturalization, 6 wherein he alleged that he
was born on November 29, 1963 in Fujian
Province, People's Republic of China, which
granted the same privilege of naturalization to
Filipinos; that he came to the Philippines on
March 15, 1988 via Philippine Airlines Flight PR
311 landing at the Ninoy Aquino International
Airport; that on November 19, 1989, he married
Cindy Sze Mei Ngar, a British national, with whom
he had four (4) children, all born in Manila; that
he had been continuously and permanently
residing in the country since his arrival and is
currently a resident of Manila with prior residence
in Malabon; that he could speak and write in
English and Tagalog; that he was entitled to the
benefit of Section 3 of Commonwealth Act (CA)
No. 473 reducing to five (5) years the
requirement under Section 2 of ten years of
continuous residence, because he knew English
and Filipino having obtained his education from
St. Stephen's High School of Manila; and that he
had successfully established a trading general
merchandise business operating under the name
of "VS Marketing Corporation." 7 As an
entrepreneur, he derives income more than
sufficient to be able to buy a condominium unit
and vehicles, send his children to private schools
and adequately provide for his family. 8
In support of his application, he attached his
barangay certificate, 9 police clearance, 10 alien
certification of registration, 11 immigration
certificate of residence, 12 marriage contract, 13
authenticated birth certificates of his children, 14
affidavits of his character witnesses, 15 passport,
16 2006 annual income tax return, 17 declaration
of intention to become a citizen of the Philippines
18 and a certification 19 from the Bureau of
Immigration with a list of his travel records from
January 30, 1994. 20
Consequently, the petition was set for initial
hearing on April 3, 2009 and its notice 21 was
posted in a conspicuous place at the Manila City
Hall and was published in the Official Gazette on
June 30, 2008, 22 July 7, 2008 23 and July 14,
2008, 24 and in the Manila Times, 25 a
116 | C O N S T I 2 _ A r t i c l e I V _ C I T I Z E N S H I P

newspaper of general circulation, on May 30,


2008, 26 June 6, 2008 27 and June 13, 2008. 28
Thereafter, respondent filed the Motion for Early
Setting 29 praying that the hearing be moved
from April 3, 2009 to July 31, 2008 so he could
acquire real estate properties. The OSG filed its
Opposition, 30 dated August 6, 2008, arguing
that the said motion for early setting was a "clear
violation of Section 1, RA 530, which provides
that hearing on the petition should be held not
earlier than six (6) months from the date of last
publication of the notice." 31 The opposition was
already late as the RTC, in its July 31, 2008 Order,
32 denied respondent's motion and decreed that
since the last publication in the newspaper of
general circulation was on June 13, 2008, the
earliest setting could only be scheduled six (6)
months later or on December 15, 2008.
On December 15, 2008, the OSG reiterated, in
open court, its opposition to the early setting of
the hearing and other grounds that would merit
the dismissal of the petition. Accordingly, the RTC
ordered
the
suspension
of
the
judicial
proceedings until all the requirements of the
statute of limitation would be completed. 33
The OSG filed a motion to dismiss, 34 but the RTC
denied the same in its Order, 35 dated March 10,
2009, and reinstated the original hearing date on
April 3, 2009, as previously indicated in the
notice.
Thereafter, respondent testified and presented
two character witnesses, Emelita V. Roleda and
Gaudencio Abalayan Manimtim, who personally
knew him since 1984 and 1998, respectively, to
vouch that he was a person of good moral
character and had conducted himself in a proper
and irreproachable manner during his period of
residency in the country.
On June 3, 2009, the RTC granted respondent's
application for naturalization as a Filipino citizen.
36 The decretal portion reads:
WHEREFORE, petitioner LI CHING CHUNG a.k.a.
BERNABE LUNA LI a.k.a STEPHEN LEE KENG is
hereby
declared
a
Filipino
citizen
by
naturalization and admitted as such.
However, pursuant to Section 1 of Republic Act
No. 530, this Decision shall not become executory
until after two (2) years from its promulgation and
after the Court, on proper hearing, with the
attendance of the Solicitor General or his
representative, is satisfied, and so finds, that
during the intervening time the applicant has: (1)
not left the Philippines; (2) has dedicated himself
continuously to a lawful calling or profession; (3)
has not been convicted of any offense or violation
of Government promulgated rules; (4) or
committed any act prejudicial to the interest of
the nation or contrary to any Government
announced policies. SDHAcI
As soon as this decision shall have become
executory, as provided under Section 1 of

Republic Act No. 530, the Clerk of Court of this


Branch is hereby directed to issue to the
Petitioner a Naturalization Certificate, after the
Petitioner shall have subscribed to an Oath, in
accordance with Section 12 of Commonwealth
Act No. 472, as amended.
The Local Civil Registrar of the City of Manila is,
likewise directed to register the Naturalization
Certificate in the proper Civil Registry.
SO ORDERED. 37
The OSG appealed the RTC decision to the CA. 38
On June 30, 2011, the CA affirmed the RTC
decision. 39 The CA held that although the
petition for naturalization was filed less than one
(1) year from the time of the declaration of intent
before the OSG, this defect was not fatal.
Moreover, contrary to the allegation of the OSG
that respondent did not present his Certificate of
Arrival, the fact of his arrival could be easily
confirmed from the Certification, dated August
21, 2007, issued by the Bureau of Immigration,
and from the stamp in the passport of respondent
indicating his arrival on January 26, 1981. 40 The
CA further stated that "the Republic participated
in every stage of the proceedings below. It was
accorded due process which it vigorously
exercised from beginning to end. Whatever
procedural defects, if at all they existed, did not
taint the proceedings, let alone the Republic's
meaningful exercise of its right to due process."
41
Moreover, the CA noted that the OSG did not in
any way question respondent's qualifications and
his lack of disqualifications to be admitted as
citizen of this country. Indeed, the CA was
convinced that respondent was truly deserving of
this privilege. 42
Hence, this petition. 43
To bolster its claim for the reversal of the assailed
ruling, the OSG advances this pivotal issue of:
. . . whether the respondent should be admitted
as a Filipino citizen despite his undisputed failure
to comply with the requirements provided for in
CA No. 473, as amended which are mandatory
and jurisdictional in character particularly: (i)
the filing of his petition for naturalization within
the one (1) year proscribed period from the date
he filed his declaration of intention to become a
Filipino citizen; (ii) the failure to attach to the
petition his certificate of arrival; and (iii) the
failure to comply with the publication and posting
requirements prescribed by CA No. 473. 44
The OSG argues that "the petition for
naturalization should not be granted in view of its
patent
jurisdictional
infirmities,
particularly
because: 1) it was filed within the one (1) year
proscribed period from the filing of declaration of
intention; 2) no certificate of arrival, which is
indispensable to the validity of the Declaration of
117 | C O N S T I 2 _ A r t i c l e I V _ C I T I Z E N S H I P

Intention, was attached to the petition; and 3)


respondent's failure to comply with the
publication and posting requirements set under
CA 473." 45 In particular, the OSG points out that
the publication and posting requirements were
not strictly followed, specifically citing that: "(a)
the hearing of the petition on 15 December 2008
was set ahead of the scheduled date of hearing
on 3 April 2009; (b) the order moving the date of
hearing (Order dated 31 July 2008) was not
published; and, (c) the petition was heard within
six (6) months (15 December 2008) from the last
publication (on 14 July 2008)." 46
The petition is meritorious.
Section 5 of CA No. 473, 47 as amended, 48
expressly states:
Section 5. Declaration of intention. One year
prior to the filing of his petition for admission to
Philippine citizenship, the applicant for Philippine
citizenship shall file with the Bureau of Justice
(now Office of the Solicitor General) a declaration
under oath that it is bona fide his intention to
become a citizen of the Philippines. Such
declaration shall set forth name, age, occupation,
personal description, place of birth, last foreign
residence and allegiance, the date of arrival, the
name of the vessel or aircraft, if any, in which he
came to the Philippines, and the place of
residence in the Philippines at the time of making
the declaration. No declaration shall be valid until
lawful entry for permanent residence has been
established and a certificate showing the date,
place, and manner of his arrival has been issued.
The declarant must also state that he has
enrolled his minor children, if any, in any of the
public schools or private schools recognized by
the Office of Private Education of the Philippines,
where Philippine history, government, and civics
are taught or prescribed as part of the school
curriculum, during the entire period of the
residence in the Philippines required of him prior
to the hearing of his petition for naturalization as
Philippine citizen. Each declarant must furnish
two photographs of himself. (Emphasis supplied)
ADEHTS
As held in Tan v. Republic, 49 "the period of one
year required therein is the time fixed for the
State to make inquiries as to the qualifications of
the applicant. If this period of time is not given to
it, the State will have no sufficient opportunity to
investigate the qualifications of the applicants
and gather evidence thereon. An applicant may
then impose upon the courts, as the State would
have no opportunity to gather evidence that it
may present to contradict whatever evidence
that the applicant may adduce on behalf of his
petition." The period is designed to give the
government ample time to screen and examine
the qualifications of an applicant and to measure
the latter's good intention and sincerity of
purpose. 50 Stated otherwise, the waiting period
will unmask the true intentions of those who seek
Philippine citizenship for selfish reasons alone,
such as, but not limited to, those who are merely
interested in protecting their wealth, as

distinguished from those who have truly come to


love the Philippines and its culture and who wish
to become genuine partners in nation building.
The law is explicit that the declaration of
intention must be filed one year prior to the filing
of the petition for naturalization. Republic v. Go
Bon Lee 51 likewise decreed that substantial
compliance with the requirement is inadequate.
In that case, Go filed his declaration of intention
to become a citizen of the Philippines on May 23,
1940. After eleven months, he filed his petition
for naturalization on April 18, 1941. In denying his
petition, the Court wrote:
The language of the law on the matter being
express and explicit, it is beyond the province of
the courts to take into account questions of
expediency, good faith and other similar reasons
in the construction of its provisions (De los Santos
vs. Mallare, 87 Phil., 289; 48 Off. Gaz., 1787).
Were we to accept the view of the lower court on
this matter, there would be no good reason why a
petition for naturalization cannot be filed one
week after or simultaneously with the filing of the
required declaration of intention as long as the
hearing is delayed to a date after the expiration
of the period of one year. The ruling of the lower
court amounts, in our opinion, to a substantial
change in the law, something which courts can
not do, their duty being to apply the law and not
tamper with it. 52
The only exception to the mandatory filing of a
declaration of intention is specifically stated in
Section 6 of CA No. 473, to wit:
Section 6. Persons exempt from requirement to
make a declaration of intention. Persons born
in the Philippines and have received their primary
and secondary education in public schools or
those recognized by the Government and not
limited to any race or nationality, and those who
have resided continuously in the Philippines for a
period of thirty years or more before filing their
application, may be naturalized without having to
make a declaration of intention upon complying
with the other requirements of this Act. To such
requirements shall be added that which
establishes that the applicant has given primary
and secondary education to all his children in the
public schools or in private schools recognized by
the Government and not limited to any race or
nationality. The same shall be understood
applicable with respect to the widow and minor
children of an alien who has declared his
intention to become a citizen of the Philippines,
and dies before he is actually naturalized.
(Emphases supplied)
Unquestionably, respondent does not fall into the
category of such exempt individuals that would
excuse him from filing a declaration of intention
one year prior to the filing of a petition for
naturalization. Contrary to the CA finding,
respondent's premature filing of his petition for
naturalization before the expiration of the oneyear period is fatal. 53
118 | C O N S T I 2 _ A r t i c l e I V _ C I T I Z E N S H I P

Consequently, the citation of the CA of the ruling


in Tam Tan v. Republic 54 is misplaced. In that
case, the Court did not excuse the noncompliance with the one-year period, but
reiterated that the waiting period of one (1) year
is mandatory. In reversing the grant of
naturalization to Tam Tan, the Court wrote:
The appeal is predicated on the fact that the
petition for naturalization was filed (26 October
1950) before the lapse of one year from and after
the filing of a verified declaration of his bona fide
intention to become a citizen (4 April 1950), in
violation of Section 5 of Commonwealth Act No.
473, as amended.
The position of the Government is well taken,
because no petition for naturalization may be
filed and heard and hence no decree may be
issued granting it under the provisions of
Commonwealth Act No. 473, as amended, before
the expiration of one year from and after the date
of the filing of a verified declaration of his bona
fide intention to become a citizen of the
Philippines. This is mandatory. 55 Failure to raise
in the lower court the question of non-compliance
therewith does not preclude the Government
from raising it on appeal. 56 aHESCT
Nevertheless, after the one-year period, the
applicant
may
renew
his
petition
for
naturalization and the evidence already taken or
heard may be offered anew without the necessity
of bringing to court the witnesses who had
testified. And the Government may introduce
evidence in support of its position. 57
The decree granting the petition for naturalization
is set aside, without costs.
In naturalization proceedings, the burden of proof
is upon the applicant to show full and complete
compliance with the requirements of the law. 58
The opportunity of a foreigner to become a
citizen by naturalization is a mere matter of
grace, favor or privilege extended to him by the
State; the applicant does not possess any natural,
inherent, existing or vested right to be admitted
to Philippine citizenship. The only right that a
foreigner has, to be given the chance to become
a Filipino citizen, is that which the statute confers
upon him; and to acquire such right, he must
strictly comply with all the statutory conditions
and requirements. 59 The absence of one
jurisdictional requirement is fatal to the petition
as this necessarily results in the dismissal or
severance of the naturalization process.
Hence, all other issues need not be discussed
further as respondent failed to strictly follow the
requirement mandated by the statute.
It should be emphasized that "a naturalization
proceeding is so infused with public interest that
it has been differently categorized and given
special treatment. . . . [U]nlike in ordinary judicial
contest, the granting of a petition for
naturalization does not preclude the reopening of
that case and giving the government another

opportunity to present new evidence. A decision


or order granting citizenship will not even
constitute res judicata to any matter or reason
supporting a subsequent judgment cancelling the
certification of naturalization already granted, on
the ground that it had been illegally or
fraudulently procured. For the same reason,
issues even if not raised in the lower court may
be entertained on appeal. As the matters brought
to the attention of this Court . . . involve facts
contained in the disputed decision of the lower
court and admitted by the parties in their
pleadings, the present proceeding may be
considered adequate for the purpose of
determining the correctness or incorrectness of
said decision, in the light of the law and extant
jurisprudence." 60
Ultimately, respondent failed to prove full and
complete compliance with the requirements of

119 | C O N S T I 2 _ A r t i c l e I V _ C I T I Z E N S H I P

the Naturalization Law. As such, his petition for


naturalization must be denied without prejudice
to his right to re-file his application.
WHEREFORE, the petition is GRANTED. The June
30, 2011 Decision of the Court of Appeals in CAG.R. CV No. 93374 is REVERSED and SET ASIDE.
The petition for naturalization of respondent Li
Ching Chung, otherwise known as Bernabe Luna
Li or Stephen Lee Keng, docketed as Civil Case
No. 08-118905 before the Regional Trial Court,
Branch 49, Manila, is DISMISSED, without
prejudice.
SO ORDERED.
Velasco, Jr., Peralta, Abad and Leonen, JJ., concur.

Вам также может понравиться